NCLEX PN ARCHER REVIEW

Pataasin ang iyong marka sa homework at exams ngayon gamit ang Quizwiz!

The nurse is reviewing leadership and management concepts with a student nurse. It would require further teaching if the student nurse made which of the following statements? A. "The Laissez-faire leadership style is a passive leadership approach." B. "A Registered Nurse (RN) may delegate accountability to a Licensed Practical Nurse (LPN)." C. "The rights of delegation include task, circumstance, person, direction, supervision." D. "The State Nurse Practice Act defines roles and responsibilities of nursing professionals."

"A Registered Nurse (RN) may delegate accountability to a Licensed Practical Nurse (LPN)." Rationale: An RN may delegate specific responsibilities to an LPN but cannot delegate accountability. The RN retains accountability when delegating client assignments and tasks but maintains accountability

The licensed/practical vocational nurse (LPN/VN) has attended a staff education program about osteomyelitis. Which of the following statements by the nurse would indicate effective understanding? A. "IV antibiotic therapy is typically given for seven to fourteen days." B. "The most common cause of acute osteomyelitis is a virus." C. "A fever is present with temperatures typically greater than 101° F (38.3° C)." D. "Petechiae on the affected extremity is a common finding."

"A fever is present with temperatures typically greater than 101° F (38.3° C)." Rationale: Acute osteomyelitis is manifested by localized bone pain, a fever, and swelling to the affected extremity.

The practical nurse is reinforcing prior teaching given to parents about antepartum testing. Which statements would be appropriate for the practical nurse to include? Select all that apply. "Oral glucose tolerance testing will measure fetal activity at certain intervals." "A nonstress test may be used to measures fetal heart rate." "Amniocentesis may be used to assess if you have preeclampsia." "Chorionic villus sampling may be done to assess for neural tube defects." Group B streptococcus (GBS) screening between 35 and 37 weeks

"A nonstress test may be used to measures fetal heart rate." Group B streptococcus (GBS) screening between 35 and 37 weeks Rationale: A nonstress test is performed in the third trimester if the client has indications such as a high-risk pregnancy that may result in a stillbirth or complications such as fetal hypoxia. Group B streptococcus testing is typically done during the third trimester of pregnancy, usually between 35 and 37 weeks. This is because GBS bacteria can be present in the vagina and rectum of some pregnant women and can be transmitted to the baby during childbirth, potentially causing serious infections.

The nurse is providing education to a group of pregnant women regarding the prevention of postpartum thrombophlebitis. Which statement by one of the expecting mothers would indicate a correct understanding of the teaching? Select all that apply. "After we give birth, we are at an increased risk of clots for 6 to 8 weeks." "We shouldn't go on car rides longer than 4 hours for a few weeks after giving birth." "After delivery, we should get up and walk as soon as we are able." "Avoiding crossing our legs will help prevent clots from forming." "Having a vaginal delivery will increase the risk for postpartum clots"

"After we give birth, we are at an increased risk of clots for 6 to 8 weeks." "We shouldn't go on car rides longer than 4 hours for a few weeks after giving birth." "After delivery, we should get up and walk as soon as we are able." "Avoiding crossing our legs will help prevent clots from forming." Rationale: Mothers are at an increased risk for clots for about 6 to 8 weeks after delivery. This is due to a natural increase in clotting factors in the body at this time. When there are increased clotting factors, clots form more readily. Mothers should avoid going on car rides longer than 4 hours for a few weeks after they give birth, as the increase in clotting factors after birth puts them at higher risks for clots. Sitting still in for longer than 4 hours could be dangerous due to the likelihood of developing a clot. This is excellent advice to share with expecting mothers. One of the essential ways to prevent postpartum thrombophlebitis is early ambulation. By encouraging them to get up and walk as soon as they are able, the likelihood of them developing clots will decrease. When legs are crossed for a prolonged period, increased pressure and immobility can lead to clot development. These mothers should be encouraged to avoid crossing legs and ambulate as soon and often as they are able.

The nurse is doing follow-up teaching with a client who has diabetes mellitus. Which of the following information should the nurse include? Select all that apply. "You will need yearly hemoglobin A1C lab tests." "Your diet should consist mostly of simple carbohydrates." "Annual visual examinations are recommended." "You should take more insulin before exercising." "Your liver enzymes will be monitored closely." "Check your blood sugar if you start to feel shaky."

"Annual visual examinations are recommended." "Check your blood sugar if you start to feel shaky." Rationale: A client with diabetes mellitus should be advised to have annual eye examinations because of the risk of diabetic retinopathy. Finally, the nurse should emphasize the client checks their blood glucose for any symptoms of HYPOglycemia such as palpitations, drowsiness, and feeling shaky.

*NGN* 72-year-old male presents to the emergency department Item 2 of 6 Based on these findings, it would be essential for the nurse to make which statement? "Would you tell me more about your bedtime routine?" "Could you describe the severity and quality of your pain?" "Are you having any thoughts of harming yourself?" "Have you ever received mental health services before?"

"Are you having any thoughts of harming yourself?" Rationale: It is essential that the nurse recognize the client's cues as a concern for suicide. It is appropriate for the nurse to ask the client directly if he is having any thoughts of harming himself. The nurse should not ask leading questions such as "You are not thinking about harming yourself, are you?" This type of question discourages the client from being honest. Assessing the client's bedtime rituals, pain level, or history of obtaining mental health services does not address the core concern of the client's safety.

The son of a client with early Alzheimer's disease states, "I'm so tired of hearing Dad talk about the past all the time." What is the nurse's best response? A. "You should have more patience with your father and accepting of his disease." B. "He is quite anxious at this stage. Reliving the past helps him become calm again." C. "He has lost his short-term memory but can still remember events from long ago." D. "Just remind him when he repeats himself and that will reinforce better behavior."

"He has lost his short-term memory but can still remember events from long ago." Rationale: Family members can become frustrated when clients with Alzheimer's disease lose short-term memory. The nurse should explain to the family member that it is the "short-term memory" that is declining and encourage the client to talk about things that he/she can remember.

The nurse is caring for a 3-year-old diagnosed with bronchitis when the mother asks the nurse what this diagnosis means, which of the following responses correctly explains bronchitis? A. "Bronchitis occurs when an infection causes inflammation in the large airways. These include the trachea and bronchi, which are in the lower part of the respiratory tract." B. "Bronchitis occurs when an infection causes inflammation in the small airways. These include the trachea and bronchi, which are in the upper part of the respiratory tract." C. "Bronchitis occurs when an infection causes inflammation in the large airways. These include the trachea and bronchi, which are in the upper part of the respiratory tract." D. "Bronchitis occurs when an infection causes inflammation in the small airways. These include the trachea and bronchi, which are in the lower part of the respiratory tract."

"Bronchitis occurs when an infection causes inflammation in the large airways. These include the trachea and bronchi, which are in the lower part of the respiratory tract." Rationale: This statement correctly describes bronchitis to the mother. Bronchitis occurs when an infection causes inflammation in the large airways. These include the trachea and bronchi, which are in the lower part of the respiratory tract.

The nurse is preparing a prenatal class focused on nutrition. Which of the following statements should the nurse reinforce? A. "Raw sushi is okay to consume during pregnancy because of its high protein level." B. "Caffeine intake should be limited during pregnancy to less than 200 mg/day." C. "To increase your iron intake, choose citrus fruits or strawberries." D. "Sodium intake is restricted to 1 gram per day during all three trimesters."

"Caffeine intake should be limited during pregnancy to less than 200 mg/day." Rationale: Caffeine intake should be less than 200 mg/day. Excessive caffeine intake has been linked to miscarriage or preterm birth. An 8-oz (240 mL) cup of brewed coffee contains approximately 137 mg of caffeine.

The licensed practical/vocational nurse (LPN/VN) reinforces teaching to a client prescribed tamsulosin. Which of the following statements should the nurse include? A. "This medication may turn your urine reddish/orange." B. "You will urinate more often with this medication." C. "Change positions slowly while you take this medication." D. "Avoid calcium-containing foods while on this medication."

"Change positions slowly while you take this medication." Rationale: Tamsulosin is an alpha-1 antagonist medication indicated in the treatment of benign prostatic hypertrophy. This medication causes vasodilation, and orthostatic hypotension is the most significant side effect. The nurse should educate the client to change positions slowly while taking this medication to reduce the risk of orthostasis.

You are the nurse performing education for a client with AIDS at the community clinic. Which of the following statements is an example of appropriate teaching? A. "Do not wash your dishes with your roommate's dishes." B. "Clean all utensils and dishes before reusing them." C. "Do not use the same shower or toilet as your roommate." D. "Hand sanitizer is not necessary unless you plan on touching someone else."

"Clean all utensils and dishes before reusing them." Rationale: Stagnant water and food particles can be a breeding ground for pathogenic microorganisms. A client with an AIDS diagnosis is susceptible to contracting illness/infections more quickly due to the deficiency in his/her immune system. The focus of education should include measures to protect the client from contracting illnesses from others.

The nurse has attended a staff education program about managing clients with peripheral arterial disease. Which of the following statements by the nurse would require follow-up? A. "The client should engage in a daily exercise regimen." B. "Smoking cessation is an essential treatment goal for clients who smoke." C. "Resting in a recliner with the legs dependent should be recommended." D. "Devices that elevate the legs above the heart should be provided at discharge."

"Devices that elevate the legs above the heart should be provided at discharge." Rationale: Peripheral arterial disease is caused by conditions such as hypertension, hyperlipidemia, and diabetes mellitus which cause atherosclerosis of the peripheral arteries. This impeded blood flow may cause the client to experience intermittent claudication (pain with ambulation that is relieved with resting). The client should be educated on self-management strategies, including sleeping or resting with the legs dependent (below the heart) to facilitate blood flow and not wearing constrictive clothing that may further impede blood flow. This statement requires follow-up because the client's legs should be below the heart to facilitate blood flow.

Which of the following statements made by a TB client being prepared for discharge indicate his understanding of the education provided? Select all that apply. "Everyone in my family needs to go and see the doctor for TB testing." "I will continue to take the isoniazid until I am feeling completely well." "I will cover my mouth and nose when I sneeze or cough and put my used tissues in a plastic bag." "I will change my diet to include more foods rich in iron, protein, and vitamin C."

"Everyone in my family needs to go and see the doctor for TB testing." "I will cover my mouth and nose when I sneeze or cough and put my used tissues in a plastic bag." "I will change my diet to include more foods rich in iron, protein, and vitamin C." Rationale: Family members should be tested because of their repeated exposure to the client. Respiratory hygiene practices, such as covering the mouth and nose when coughing or sneezing and properly disposing of used tissues, are essential to prevent the spread of TB bacteria to others. Good nutrition, including foods rich in iron, protein, and vitamin C, can support the client's immune system and overall health during TB treatment. Adequate nutrition is important for the body's ability to fight off the infection and promote healing.

What question would be most important to ask a male client who is in for a digital rectal examination? A. "Have you noticed a change in the force of urine when you empty your bladder?" B. "Have you noticed a change in tolerance of certain foods in your diet?" C. "Do you notice polyuria in the AM?" D. "Do you notice any burning with urination or any odor to the urine?"

"Have you noticed a change in the force of urine when you empty your bladder?" Rationale: A change in the urinary stream force could indicate a complication associated with BPH (benign prostatic hypertrophy).

The patient with COPD reports to the nurse that she has trouble sleeping at night. Which question is most important for the nurse to ask next? A. "What do you eat before you go to bed?" B. "How many pillows do you sleep on at night?" C. "Have you always been a light sleeper?" D. "Is your partner snoring and keeping you awake?"

"How many pillows do you sleep on at night?" Rationale: Orthopnea is shortness of breath that occurs when lying flat, causing the person to have to sleep propped up in bed or sitting in a chair. Asking the patient how many pillows she uses to sleep on is a way to assess if the patient has been educated about preventing orthopnea. COPD causes blocked or narrowed airways that make breathing more difficult. Patients may experience symptoms like wheezing, coughing, mucus production, and tightness in the chest. Smoking or exposure to harmful chemicals can cause COPD. Orthopnea is a common symptom of COPD patients.

The nurse is caring for a client who is receiving prescribed acamprosate. Which of the following statements, if made by the client, would indicate a therapeutic response? A. "I no longer hear voices." B. "I have more motivation during the day." C. "I am not drinking alcohol anymore." D. "My anxiety has lessened in public."

"I am not drinking alcohol anymore." Rationale: Acamprosate is a medication intended to treat alcohol use disorder. This medication may be combined with naltrexone to increase the chance of sobriety.

The nurse is caring for a client who is receiving prescribed varenicline. Which of the following statements, if made by the client, would indicate a therapeutic response? A. "I am not smoking cigarettes anymore." B. "My depression has gotten better." C. "I am sleeping eight hours a night." D. "I can focus on one task at a time."

"I am not smoking cigarettes anymore." Rationale: Varenicline is a medication intended to reduce nicotine withdrawal symptoms and cravings. Following the initiation of varenicline, the client's comments that they are not smoking cigarettes anymore indicate varenicline has been therapeutically effective. Clients go back to resuming tobacco smoking if the withdrawal symptoms are not appropriately treated.

The nurse at a health fair is talking with a client who is in perimenopause. Which of the following client statements requires follow-up by the nurse? Select all that apply. "I am using oil-based vaginal lubricants to lessen the discomfort during intercourse." "My doctor recommended I start taking Black cohosh to help with the loss of bone density" "I stopped exercising to reduce my hot flashes." "I recently started increasing my water intake to help prevent urinary tract infections." "Doing Kegel exercises will increase muscle tone around my vagina and urinary meatus."

"I am using oil-based vaginal lubricants to lessen the discomfort during intercourse." "My doctor recommended I start taking Black cohosh to help with the loss of bone density" "I stopped exercising to reduce my hot flashes." Rationale: Oil-based lubricants should not be used because they adhere to the mucous membrane for long periods and provide a medium for bacterial growth. Water-soluble lubricants are recommended. This client statement requires follow-up by the nurse. Black cohosh has demonstrated efficacy in reducing the vasomotor symptoms of menopause, specifically hot flashes. Black cohosh does not help with osteoporosis, whereas calcium, vitamin D, and weight-bearing exercises help increase bone density. This client statement requires follow-up by the nurse. The client should not stop exercising because menopause increases cardiovascular disease risk. In fact, most women who have a myocardial infarction are post-menopausal. Exercise should be encouraged, and exercise does not cause more hot flashes. This client statement requires follow-up by the nurse.

A 12-year-old client with chronic asthma exacerbations has decided to try guided imagery as a way to manage anxiety contributing to asthma attacks. Which of the following statements by the client indicates an understanding of this stress-reduction technique? A. "I can do this anytime and anywhere when I feel anxious." B. "I must be lying down to practice guided imagery." C. "My mom will have to be with me any time I try this." D. "I will play music every time I do my guided imagery to make sure it works."

"I can do this anytime and anywhere when I feel anxious." Rationale: Guided imagery is a stress-reduction technique that can be done in any place at any time. In fact, this is one of the biggest advantages of this technique. Anytime the client begins to feel anxious, they can practice guided imagery.

The nurse is collecting data on a client who has a suspected retinal detachment. Which of the following client statements would be consistent with this diagnosis? A. "My vision has a cloudy appearance." B. "I have intense pain above my eyebrow." C. "I am having trouble with my peripheral vision." D. "I can see bright flashes of light."

"I can see bright flashes of light." Rationale: A retinal detachment is a medical emergency as it may become progressive and give the client blindness in the affected eye. The client may experience a loss of vision that appears as if a curtain is closing, or they may experience bright flashes of light.

The nurse has provided medication instructions to a client who has been prescribed venlafaxine. Which of the following statements, if made by the client, would indicate a correct understanding of the teaching? Select all that apply. "I may not notice an improvement in my mood right away." "This medication may lower my blood pressure." "If I have thoughts of harming myself, I should call 911." "I will need to have weekly laboratory tests." "I may continue taking St. John's Wort."

"I may not notice an improvement in my mood right away." "If I have thoughts of harming myself, I should call 911." Rationale: Venlafaxine is a serotonin-norepinephrine reuptake inhibitor (SNRI). This medication is used to treat depression and anxiety. Like most serotonergic drugs, the client may not experience an effect for two to four weeks. If no effect is achieved within six weeks, the prescriber may change the medication. Venlafaxine may increase thoughts of suicidal ideation, and the client should be educated to seek help if these thoughts should occur.

*NGN* The nurse is caring for a 29-year-old female in the physician's office Item 6 of 6 The nurse is discussing pernicious anemia with the client Which of the following client statements would indicate effective understanding? "I should increase my fluid intake because this medication may cause constipation." "I may notice black tarry stools with this medication." "I may require additional doses of this medication." "This medication will help my blood coagulate."

"I may require additional doses of this medication." Rationale: Vitamin B12 treatment for pernicious anemia secondary to gastric surgery may require indefinite treatment. Vitamin B12 is water-soluble and thus is readily excreted from the body. Repeated treatments are expected until the B12 levels normalize. Gastrointestinal manifestations (constipation, black tarry stools) are side effects associated with iron. Vitamin K is essential in blood clotting, and when Vitamin K is given, it assists with blood coagulation.

The licensed practical/vocational nurse (LPN/VN) has reinforced medication instructions to a client who has been prescribed a fentanyl transdermal patch. Which of the following statements, if made by the client, would indicate a correct understanding of the instructions? Select all that apply. "I may still need pain medication while this patch is applied." "If the patch comes loose, I may reinforce it with a piece of tape." "I can apply heat to the patch site to increase the pain relief." "I should remove this patch while I am sleeping." "The patch will need to be changed every 72 hours."

"I may still need pain medication while this patch is applied." "If the patch comes loose, I may reinforce it with a piece of tape." "The patch will need to be changed every 72 hours." Rationale: A variety of routes, including transdermal patches, can deliver fentanyl. This patch is effective for around-the-clock pain control, but the client may still experience breakthrough pain requiring a more immediate release type of pain control. The client may reinforce the patch with tape if it starts to loosen. The fentanyl patch should be changed every 72 hours, with a new patch applied to a new site.

The nurse is visiting a client who is 82 years old, has impaired vision, and lives alone. It would be necessary for the nurse to follow up if the client states which of the following? Select all that apply. "I secured my throw rugs to the floor with tape." "I switched to using an electric shaver instead of a razor." "I usually sit in a recliner while I listen to the television." "I use different shaped containers with lids to organize my medications." "I use the upstairs bathroom instead of the one downstairs."

"I secured my throw rugs to the floor with tape." "I use the upstairs bathroom instead of the one downstairs." Rationale: An older client with impaired vision that lives alone has significant risk factors for falls. The nurse should follow up if the client states that they secured the scattered rugs with tape. The client should not have any scattered rugs. Finally, a client climbing the stairs to use the bathroom increases the risk of falls. The nurse should advise the client to use the closest bathroom.

The nurse reinforces discharge instructions to a client who underwent left eye cataract surgery with a lens implant. Which statement by the client would indicate a correct understanding of the teaching? A. "I should avoid getting water in the eye for 3 to 7 days after surgery." B. "It is okay for me to resume normal chores such as vacuuming." C. "It is okay for me to have green or yellow, thick drainage from the eye." D. "I may take aspirin for any pain I may experience."

"I should avoid getting water in the eye for 3 to 7 days after surgery." Rationale: This statement indicates effective teaching by the nurse. Following cataract surgery, the client should not get any water in the affected eye for three to seven days. This measure will reduce the potential for infection.

The nurse is teaching a caregiver how to administer an injection of enoxaparin. Which statement, if made by the caregiver, would require further teaching? A. "I will give this injection in the abdomen." B. "I should expel the air bubble before administering." C. "Green leafy vegetables are allowed while taking this medication." D. "This medication may increase the risk for bleeding."

"I should expel the air bubble before administering." Rationale: This statement is incorrect and requires follow-up. Enoxaparin comes in prefilled syringes that are administered to the client subcutaneously. Since this medication comes in prefilled syringes, the air bubble should not be expelled. This is designed to remain next to the plunger to ensure the full dose is administered.

The nurse is reinforcing education to parents of a child who plans on riding their bicycle. Which statements, if made by the parents, indicate effective understanding? A. "I should tell my child to ride against the traffic pattern." B. "I should instruct my child to walk their bike through busy intersections." C. "Wearing a helmet is only necessary when my child is riding near a busy intersection." D. "My child can ride their bike barefoot as long as it's short distances."

"I should instruct my child to walk their bike through busy intersections." Rationale: This statement indicates effective understanding. Parents should teach their child to dismount and walk their bike when crossing busy intersections. Walking the bike ensures better visibility and safety, reducing the risk of being hit by an automobile.

A client with a peptic ulcer is prescribed sucralfate. Which statement by the client indicates an understanding of the medication? A. "I should take sucralfate at least 1 hour before meals and at bedtime." B. "I will avoid taking antacids completely while I'm on sucralfate." C. "I should take sucralfate right before meals and at bedtime." D. "I can expect immediate relief of my ulcer symptoms after taking sucralfate."

"I should take sucralfate at least 1 hour before meals and at bedtime." Rationale: Sucralfate is a locally acting agent that combines with hydrochloric acid in the stomach in an acidic environment (pH less than 4) to make a cross-linking, thick compound that forms a barrier over the ulcer. Sucralfate is most effective when taken on an empty stomach to enhance its adherence to the ulcer site. It should be taken at least an hour before meals and at bedtime.

The nurse is reinforcing education for a client with Graves' disease. Which of the following client statements indicates effective understanding? A. "I will take my pulse daily and report a rate less than 60 beats/minute." B. "I am going to add hot yoga to my exercise routine." C. "I will increase the amount of fiber in my diet." D. "I should tell my physician if my blood pressure's top number exceeds 140."

"I should tell my physician if my blood pressure's top number exceeds 140." Rationale: Grave's disease is the most common form of hyperthyroidism. A significant complication of this condition is the potential for a thyroid storm. A thyroid storm is caused by a surge in thyroid hormone in the bloodstream, which causes the client to experience tachycardia, fever, hypertension, diaphoresis, and tachydysrhythmias.

The nurse is caring for a client who was recently diagnosed with human immunodeficiency virus (HIV). Which of the following statements, if made by the client, would indicate a knowledge deficit? Select all that apply

"I started researching ways to tell my family that I have AIDS." "I recently stopped sharing household utensils and towels." "I will not be able to continue my job as a phlebotomist." Rationale: All of these client statements are false and indicate a knowledge deficit. The nurse should correct the client and remind them they have HIV - not AIDS. AIDS is when the client is HIV positive and has either a CD4+ T-cell count of fewer than 200 cells/mm 3 (0.2 × 10 9 /L) or less than 14% (even if the total CD4+ count is above 200 cells/mm 3 [0.2 × 10 9 /L]) or an opportunistic infection. HIV and AIDS is not transmitted through household utensils, towels, or toilets. The client will be able to continue their job as a phlebotomist because standard precautions are utilized, which will inhibit the transmission of HIV. The client will not have to take any additional precautions.

The nurse is caring for a client who was recently prescribed antihypertensive medications. Which statement, if made by the client, requires follow-up? A. "My pulse decreases after taking my metoprolol." B. "I started taking my furosemide right before I went to sleep." C. "I am seasoning my foods without salt while on lisinopril." D. "I wear my clonidine patch for seven days."

"I started taking my furosemide right before I went to sleep." Rationale: Furosemide is a loop diuretic that should be dosed early in the day. This prevents the client from experiencing nocturia. This also reduces the risk of falls by the client as they will not have to wake up at night when there is reduced lighting.

The nurse prepares a client for a scheduled percutaneous coronary intervention (PCI). Which client statement should be reported to the primary healthcare provider (PHCP)? A. "I took my metformin this morning." B. "I get anxious when I am in closed spaces." C. "I am allergic to shellfish." D. "I may feel a warm sensation during the procedure."

"I took my metformin this morning." Rationale: This procedure involves intravenous (IV) contrast and a small chance of acute kidney injury may occur when IV contrast is given within 48 hours of metformin. Thus, the PHCP needs to be notified. Exposure to metformin prior to this procedure is not a contraindication but requires IV fluids to decrease the negative effects on the kidneys.

The nurse is reinforcing education to a pregnant client with the hepatitis C virus (HCV). Which of the following statements by the client would require follow-up? Select all that apply.

"I will be unable to breastfeed my baby." "I will have to deliver my baby by cesarean to decrease the risk of transmission." "I can continue my antiviral drugs while I am pregnant." "My baby will need to be isolated while in the hospital." Rationale: These client statements require follow-up by the nurse because they are not accurate. Hepatitis C is a blood-borne pathogen and is not transmitted in breast milk. It is safe for a woman to breastfeed if she has the hepatitis C virus. If her nipples should crack and start to bleed, breastfeeding should be halted. Vaginal delivery is approved for a client with HCV. Cesarean delivery is not advised because of the increased risk of blood exposure unless indicated for other reasons. Unlike HIV infection, antiviral drugs for HCV are unsafe during pregnancy and must be suspended during the pregnancy. Isolation for an individual with hepatitis C is unnecessary (standard precautions are used).

The LPN is reinforcing discharge instructions to a client regarding his newly initiated digoxin. Which of the following statements by the client indicates that he correctly understood the instructions? Select all that apply. "If I note color vision changes, I will call my eye doctor right away." "I will check my pulse before each dose and if the pulse is less than 60 bpm, I will hold digoxin and call my doctor." "I will increase my calcium intake significantly." "I will make sure I get enough potassium in my daily diet." "The water pills that I am on may increase the risk of side effects with digoxin." "I should avoid medications that have licorice extract."

"I will check my pulse before each dose and if the pulse is less than 60 bpm, I will hold digoxin and call my doctor." "I will make sure I get enough potassium in my daily diet." "The water pills that I am on may increase the risk of side effects with digoxin." "I should avoid medications that have licorice extract." Rationale: A nurse should understand the mechanism of action of digoxin to understand its uses, side effects, monitoring responsibilities, and client education elements. Digoxin is a cardiac glycoside that acts via the sodium-potassium pump in the myocardium. It has inotropic (influences contractility), chronotropic (influences heart rate), and dromotropic (influences conduction speed) effects. Digoxin is a positive inotrope (increases the power of heart contraction) and a negative chronotrope (decreases heart rate by its effect on sinus node) and a negative dromotrope (reduces the speed of conduction by acting at the atrioventricular node level). Due to these cardiac effects of digoxin, it is often used in clients with congestive heart failure and arrhythmias like atrial fibrillation. However, Digoxin has a narrow therapeutic index, which means it can cause significant side effects, such as cardiac arrhythmias (e.g. bradycardia, heart block, ventricular arrhythmias), even at plasma concentrations only twice the therapeutic plasma concentration range. Therapeutic serum digoxin levels range from 0.5 to 2 ng/mL. A level higher than two ng/mL is considered toxic.

The nurse is providing education for a diabetic client who is given a terbinafine prescription for onychomycosis. Which statement(s) by the client demonstrates a good understanding regarding treatment with terbinafine? Select all that apply.

"I will have to take terbinafine for 3 to 6 months." "I will need liver function tests before starting terbinafine." "It may cause taste or vision changes and, I will report vision changes to my doctor." "Dark urine, pale stools, and persistent nausea may indicate a serious side effect Rationale: Onychomycosis, also known as Tinea unguium, is a fungus infection of the nails (fingernails, toenails) that causes the nails to look thick, discolored, opaque, and crumbling. Dermatophytes cause 90% of these toenail infections. Remaining 10% are caused by non-dermatophytes (Saprophytes), and yeast (Candida). Treatment involves topical antifungals and systemic antifungals (Terbinafine, Lamisil).

The licensed practical/vocational nurse (LPN/VN) reinforces teaching a client about newly prescribed isoniazid (INH) for pulmonary tuberculosis. Which of the following statements by the client would require follow up? A. "I will have to take this medication for three months." B. "I will need to have my blood drawn periodically to see if I am having an adverse effect to this medication." C. "I will not be considered infectious if I have three consecutive negative sputum samples." D. "This medication may make my hands and feet have numbness and tingling sensations."

"I will have to take this medication for three months." Rationale: This statement requires follow-up because it is not accurate. Antitubculin medications must be taken for six to nine months to complete treatment and prevent resistance. If the client is at risk of poor treatment adherence, they may be ordered directly observed therapy where an individual supervises the client to take their medication.

The nurse has reinforced medication instruction to a client who has been prescribed methadone for opioid use disorder. Which of the following statements, if made by the client, would indicate a correct understanding of the instructions? Select all that apply. "I will need counseling while taking this medication." "I will need periodic blood tests while on this medication." "This medication may lower my risk for Hepatitis C." "This medication will send me into opioid withdrawals." "I may get drowsy while taking this medication."

"I will need counseling while taking this medication." "I will need periodic blood tests while on this medication." "This medication may lower my risk for Hepatitis C." "I may get drowsy while taking this medication." Rationale: Methadone is an efficacious medication used in the treatment of opioid use disorder, chronic pain, and in the treatment of neonatal abstinence syndrome. This medication requires close monitoring and counseling for opioid use disorder. Periodic blood tests are required as this medication may be hepatotoxic. This medication has been shown to decrease the transmission of blood-borne pathogens such as HIV and Hepatitis C. This is because the reduction of intravenous drug use decreases the risk of the transmission of these pathogens. Drowsiness is a common side effect of this medication as it is an opioid agonist.

The nurse is reinforcing teaching regarding prescribed risperidone. Which statement, if made by the client, requires follow-up? A. "I should report any abnormal movements that I develop." B. "I will need to have weekly tests to monitor my white blood cells." C. "If I get muscle stiffness, I should notify my physician." D. "I will need to chew sugarless gum if I develop a dry mouth."

"I will need to have weekly tests to monitor my white blood cells." Rationale: Risperidone is a second-generation antipsychotic used in delirium, schizophrenia, and some childhood disorders. Weekly white blood cell tests are not required with risperidone as this is appropriate teaching for an individual receiving clozapine. Clozapine may cause neutropenia.

*NGN* The home health nurse visits a client with chronic diabetes insipidus Item 1 of 1 Nurses' Note 1415 - Home health visit completed because the client was admitted to the hospital twice in the past six weeks for treatment nonadherence related to diabetes insipidus. Extensive teaching provided and reviewed education on prescribed desmopressin intranasal, maintenance of fluids, daily weight, intake and output, and when to seek emergency care. Which four (4) client statements would indicate a correct understanding of the teaching? "I should limit the amount of fluids that I drink after 5:00 PM." "I will need to weigh myself at the same time every day." "I should put both doses of the desmopressin in one nostril." "I need to keep a log of my fluid intake and urine output." "I may need an additional dose if I keep urinating a lot." "If I develop confusion with this medication, I should call 911."

"I will need to weigh myself at the same time every day." "I need to keep a log of my fluid intake and urine output." "I may need an additional dose if I keep urinating a lot." "If I develop confusion with this medication, I should call 911." Rationale: Clients with permanent diabetes insipidus will need lifelong treatment that will involve desmopressin, which can be dosed intranasally or by mouth. Diabetes insipidus requires teaching encompassing the prescribed therapy, fluid maintenance, daily weight, intake and output, and when the client should seek emergency care. It will indicate correct understanding if the client verbalizes that they need to weigh themselves at the same time of day while wearing a similar amount of clothing. Additionally, the client should log their fluid intake and urine output as they should drink fluids in an amount equal to urine output. Further, a client may take an additional dose of their prescribed desmopressin if they keep experiencing polyuria and polydipsia. An adverse effect of desmopressin is that it may cause water toxicity (persistent headache, acute confusion, nausea, vomiting) a medical emergency.

The nurse has reinforced medication instruction to a client who has been prescribed enalapril. Which of the following statements, if made by the client, would indicate a correct understanding of the teaching? A. "I will notify my prescriber if I develop swelling of the face." B. "I will need to weigh myself every day while taking this medication." C. "I should eat foods high in potassium while I am taking this medication." D. "I will need lab work done every so often to evaluate my liver function."

"I will notify my prescriber if I develop swelling of the face." Rationale: Angioedema is a life-threatening adverse effect that is seen with ACE inhibitors such as enalapril. Angioedema may cause swelling anywhere in the body but swelling in the face, lips, and eyes can be life-threatening.

The nurse has reinforced education to a client diagnosed with obstructive sleep apnea (OSA). Which client statement would indicate a correct understanding of the teaching? A. "I should use an antiseptic mouthwash immediately before going to bed." B. "I will plan on exercising at least 150 minutes a week." C. "I have been reading about the potential for me needing supplemental oxygen overnight." D. "I will sleep flat on my bed without any pillows."

"I will plan on exercising at least 150 minutes a week." Rationale: A crucial part of mitigating the symptoms of obstructive sleep apnea is for a client to lose weight. Weight reduction is a pivotal part of the treatment plan for an individual with OSA, as being overweight or obese causes fat deposits in the upper airways. Reducing these fat deposits improves muscle activity and allows for better ventilation. The client stated that they plan on exercising 150 minutes a week is a favorable response because that is the national recommendation.

The nurse has reinforced discharge instructions to the parents of an infant with a newly applied Pavlik harness. Which of the following statements by the parents would indicate the need for additional reinforcement? Select all that apply. "I will remove the harness during feedings." "I will check for red areas under the straps and at the skin folds." "I will apply moisturizing lotion under the straps." "I will place the diaper over the straps." "I will gently massage the skin under the straps to stimulate circulation."

"I will remove the harness during feedings." "I will apply moisturizing lotion under the straps." "I will place the diaper over the straps." Rationale: These statements require follow-up because they are not correct. The harness should not be removed during feedings. The harness should be applied as directed by the prescriber, but generally, it is worn 24 hours a day. The healthcare provider should remove, adjust, and apply the harness. Lotion and powders should not be placed under the straps because that may cause dermatitis. The diaper is placed under the straps to avoid soiling the harness.

What is the normal level of creatinine in a healthy adult male? A. 0.4 to 0.8 mg/dL B. 0.1-0.4 mg/dL C. 0.6-1.2 mg/dL D. 1.5-2.0 mg/dL

0.6-1.2 mg/dL Rationale: Creatinine values reflect both the amount of muscle a person has and their amount of kidney function. Hence, the levels are slightly lower in women due to lesser muscle mass. Most men with normal kidney function have 0.6 to 1.2 milligrams/deciliters (mg/dL) of creatinine. Most women with normal kidney function have between 0.5 to 1.1 mg/dL of creatinine.

The nurse has taught a client about a scheduled intravenous (IV) urography (pyelogram). Which of the following statements by the client would indicate a correct understanding of the teaching?

"I will take a laxative the night before to clear my bowels." Rationale: An IV urography (pyelogram) is a diagnostic test used to gather urinary tract imaging that views the collecting ducts and renal pelvis and outlines the ureters, bladder, and urethra. The client must perform a bowel cleansing the night before to ensure adequate visualization of the urinary tract. During this procedure, the client will empty their bladder, and then an intravenous injection of contrast medium is given, and a series of x-ray films and fluoroscopy is used to observe the passage of urine from the renal pelvis to the bladder. The use of this test has decreased because of computed tomography scans of the urinary tract.

The nurse is teaching a client about newly prescribed insulin glargine. The nurse recognizes the need for further instruction when the client makes the following statement? Select all that apply. "I will take this insulin right before my meals." "I should roll this vial of insulin before removing it with the syringe." "This insulin will help control my glucose for 24 hours." "I can only inject this insulin into my abdomen." "I'm glad to know I can mix this with my regular insulin."

"I will take this insulin right before my meals." "I should roll this vial of insulin before removing it with the syringe." "I can only inject this insulin into my abdomen." "I'm glad to know I can mix this with my regular insulin." Rationale: These statements are incorrect and require follow-up. Insulin glargine is a long-acting insulin that has no peak effect. Thus, it is not taken with meals. It is dosed once a day to provide glucose control for 24 hours. Insulin glargine is not a suspension; thus, it does not need to be rolled like NPH. This insulin is not mixed with any other insulin. Insulin glargine does not have to only be injected into the abdomen.

The nurse is reinforcing teaching to a client who is breastfeeding about ways to prevent mastitis. Which statement, if made by the client, would require follow-up? A. "I will wear a tight fitting bra to provide compression" B. "I must wash my hands thoroughly before breastfeeding." C. "I must change nursing pads as soon as they become wet." D. "I need to feed my baby every 2-3 hours."

"I will wear a tight fitting bra to provide compression" Rationale: This statement is incorrect and requires follow-up. Tight bras can cause pressure on the breasts, making milk expression more difficult and leading to mastitis.

The nurse is caring for a client who is struggling with severe depression. Which of the following statements would demonstrate effective therapeutic communication with this client? Select all that apply. "Great work today in group therapy Steve, you were really talkative today!" "I'd like to just sit with you for a while Steve." "Tell me how you're feeling Steve. I'd like to understand." "Why are you feeling depressed today Steve?" "I know exactly how you feel. I've been through the same thing."

"I'd like to just sit with you for a while Steve." "Tell me how you're feeling Steve. I'd like to understand." Rationale: In this scenario, the nurse offers to sit in silence with the client. Therapeutic silence with individuals struggling with depression can create a safe and reflective space, foster deeper communication, encourage self-reflection and emotional processing, reduce pressure and anxiety, enhance active listening, and promote self-expression. Effective therapeutic communication aims to establish trust, provide support, and encourage the client's expression of thoughts and feelings. Asking open-ended questions in a supportive, non-judgmental way offers support to the depressed client.

Which of the following statements indicates body image distortion in a client with anorexia nervosa?

"I'm so overweight." Rationale: Clients with anorexia perceive themselves to look differently than they do. Despite being too thin, this client will not eat in hopes of getting the perfect body.

The licensed practical/vocational nurse (LPN/VN) is taking vital signs on a baby diagnosed with Total Anomalous Pulmonary Venous Return (TAPVR) when the mother starts crying. Which of the below statements by the nurse is most therapeutic? A. "Don't cry, your baby will be fine!" B. "I'm so sorry you are going through this, would you like to talk?" C. "I'm sure this is hard, but your baby is doing so well!" D. "You think this is bad, you should see some of the other babies here."

"I'm so sorry you are going through this, would you like to talk?" Rationale: This is a good example of therapeutic communication. The nurse has validated the mother's feelings and encouraged further dialogue to understand what the mother is upset about.

The nurse is caring for a client who recently had a cast placed on their right lower extremity. Which of the following statements by the client would be the most alarming to the nurse? A. "I've been having pain in my right calf." B. "My right leg feels really itchy." C. "I didn't keep my leg elevated as the doctor asked me to." D. "When I put weight on my crutches, it makes arthritis in my wrists worse."

"I've been having pain in my right calf." Rationale: Pain in the casted extremity could indicate neurovascular compromise. Clients who have been in a cast should be educated on safety measures and signs of complications before discharge from care. It is not uncommon for the skin inside a cast to itch. However, any signs of neurovascular compromise should be immediately reported. If a client reports pain in a casted extremity, this is a sign of compromise and requires immediate assessment/intervention. Clients should be instructed to report pain, tingling, or edema in the extremity that is greater than before the cast was applied or if the cast feels too loose.

Which of the following statements by a patient who has recently been in a cast on the right lower extremity would be the most alarming to the nurse? A. "I've been having pain in my right calf." B. "My right leg feels really itchy." C. "I didn't keep my leg elevated as the doctor asked me to." D. "When I put weight on my crutches, it makes arthritis in my wrists worse."

"I've been having pain in my right calf." Rationale: Pain in the casted extremity could indicate neurovascular compromise. Patients who have been in a cast should be educated on safety measures and signs of complications before discharge from care. It is not uncommon for the skin inside a cast to itch. However, any signs of neurovascular compromise should be immediately reported. If a patient reports pain in a casted extremity, this is a sign of compromise and requires immediate assessment/intervention. Patients should be instructed to report pain, tingling, edema in the extremity that is greater than before the cast was applied or if the cast feels too loose.

The nurse is working with an advocacy group to raise awareness about cystic fibrosis. Which of the following statements does she know best explains the condition? A. "It is an inherited disease that causes inflammation and hypersensitivity of the airway." B. "It is an infectious disease causing inflammation and fluid accumulation in the alveoli of the lungs." C. "It is an inherited disease causing excessive, thick mucus to build up in the body and cause blockages." D. "It is an acquired disease that causes inflammation and swelling of the epiglottis."

"It is an inherited disease causing excessive, thick mucus to build up in the body and cause blockages." Rationale: This statement correctly describes cystic fibrosis as an inherited disease causing excessive, thick mucus to build up in the body and cause blockages.

Which of the following should the nurse include in the education provided to a client who is taking lisinopril? Select all that apply. "It may take several months for your blood pressure to return to normal." "You must have your potassium monitored from time to time." "This medication may change your vision at times." "You may notice a change in your sensation of taste."

"It may take several months for your blood pressure to return to normal." "You must have your potassium monitored from time to time." "You may notice a change in your sensation of taste." Rationale: Lisinopril may require 2-3 weeks of adjustment to reach maximum effectiveness. Several months of therapy may be needed for a client's functional status to return to normal. High potassium levels may occur during therapy. The use of potassium supplements or potassium-sparing diuretics should be avoided. Electrolyte levels should be monitored periodically. Other side effects associated with lisinopril include cough, taste disturbances, and hypotension.

The nurse is teaching a client about peptic ulcer disease. Which of the following statements should the nurse include? A. "You should take aspirin if you have mild aches or pains." B. "You will need to consume liquids one hour after each meal." C. "It will be important to reduce the stress in your life." D. "Take your prescribed omeprazole with food."

"It will be important to reduce the stress in your life." Rationale: A client with peptic ulcer disease will need to reduce the amount of stress in their life to mitigate some of the symptoms. Ulcers can be caused by excessive use of non-steroidal anti-inflammatory drugs, alcoholism, and stress.

The nurse is reinforcing teaching a client about prescribed lisinopril for hypertension. Which of the following information should the nurse include? Select all that apply. "Limit your intake of salt in your diet and season your foods naturally." "You must have your potassium monitored from time to time." "Stay hydrated because this medication will cause you to urinate more often." "You may notice a change in your sensation of taste." "You will need to take your pulse for one minute before this taking this medication."

"Limit your intake of salt in your diet and season your foods naturally." "You must have your potassium monitored from time to time." "You may notice a change in your sensation of taste." Rationale: Sodium intake should be limited to no more than 2 grams daily for a hypertension client. Increased sodium causes fluid retention, therefore, raising blood pressure. Salt substitutes should be avoided for a client taking an ACE inhibitor because they are potassium based and would increase the serum potassium. ACE-I's raise potassium, and this combination may lead to life-threatening hyperkalemia. Potassium and renal function tests are commonly monitored while a client receives this medication because both may become elevated. ACE-I's may cause an altered taste sensation which is expected.

The nurse observes a parent swaddling their infant with an unrepaired omphalocele. Which of the following statements would be appropriate? A. "Stop, you will kill your baby." B. "That is a nice, tight swaddle. It will help soothe your new baby." C. "May I help you? We will need to be careful with their intestines and we do not want the swaddle to push them back inside." D. "Swaddling is not allowed for these babies. Please stop."

"May I help you? We will need to be careful with their intestines and we do not want the swaddle to push them back inside." Rationale: This statement is a correct example of therapeutic communication. It validates the fear and emotions that the parent is experiencing and provides education about the infant's treatment and prognosis. This opens up the conversation for dialogue and further information about what the parent can expect.

The nurse is caring for a client receiving buspirone. Which of the following client statements would indicate a therapeutic response? A. "I am less depressed and able to spend time with my friends." B. "My anxiety has lessened, and I have started going out more." C. "I noticed an improvement in my concentration." D. "I have been able to fall asleep without any problem."

"My anxiety has lessened, and I have started going out more." Rationale: Buspirone is a serotonergic medication indicated for individuals with generalized anxiety disorder. The client reporting decreased anxiety would indicate a therapeutic effect.

The nurse is caring for a client who is receiving prescribed mirtazapine. Which of the following statements, if made by the client, would indicate a therapeutic response? Select all that apply. "I am not smoking cigarettes anymore." "My blood glucose has decreased." "My depression has gotten better." "I am sleeping eight hours a night." "My blood pressure is back to normal."

"My depression has gotten better." "I am sleeping eight hours a night." Rationale: Mirtazapine is a tetracyclic antidepressant that causes an increase in serotonin and norepinephrine. This medication is used for depressive and anxiety disorders. Mirtazapine is quite sedating and is often used for insomnia associated with depressive disorders.

The nurse is teaching a group of students about drug toxicity. The nurse is correct in stating which of the following? Select all that apply. "Naloxone is the treatment for opioid toxicity." "Magnesium is the treatment for lead toxicity." "N-acetylcysteine is the treatment for naproxen toxicity." "Calcium is the treatment for magnesium toxicity." "Flumazenil is the treatment for benzodiazepine toxicity."

"Naloxone is the treatment for opioid toxicity." "Calcium is the treatment for magnesium toxicity." "Flumazenil is the treatment for benzodiazepine toxicity." Rationale: The antidote for opioid toxicity is naloxone which may be given IV, IM, Intranasal, or SubQ. Magnesium toxicity is treated with calcium gluconate. Flumazenil is indicated for benzodiazepine toxicity. High levels of lead would be treated with succimer, which is indicated for heavy metals and lead. N-acetylcysteine is utilized in the treatment of acetaminophen toxicity - not naproxen which is an NSAID.

The LPN is reinforcing discharge instructions to the parents of a 6-year-old girl with chickenpox. The LPN knows that the parents understand when they make which of the following statements? A. "Once she has been without a fever for a day, she can go back to school." B. "She will still be infectious for 14 days, so we should let the school know she will be out for 2 weeks." C. "After antibiotics have been started, she can go back to school in 48 hours." D. "Once all of her sores are crusted over, it will be safe for her to go back to school."

"Once all of her sores are crusted over, it will be safe for her to go back to school." Rationale: This statement demonstrates understanding by the parents. Chickenpox (or varicella) is considered infectious until all of the lesions have crusted over. For most children, this occurs 6-7 days after the last of the rash presents but can take up to 10 days.

The licensed practical/vocational nurse (LPN/VN) is reinforcing teaching for a client newly diagnosed with multiple sclerosis. Which of the following statements by the client would indicate a correct understanding of the teaching? A. "If I experience double-vision, I should put an eye patch on both eyes for a few hours." B. "Planning my activities should help manage the fatigue." C. "I should plan to take a hot bath if my muscles spasm." D. "This disease may cause me to have an increased sensitivity to pain."

"Planning my activities should help manage the fatigue." Rationale: Fatigue is a significant clinical feature associated with MS. Strategies to mitigate fatigue and maximize functioning include spacing activities out, planning them in a planner or whiteboard, and taking frequent breaks. Fatigue is often worsened during elevations in temperature. Thus, activities may be best performed early morning or late evening when temperatures are not as high.

Which statement from the 13-year-old girl with acute lymphoblastic leukemia (ALL) shows the LPN that she comprehends the side effects of steroid medications? Select all that apply. "Steroids will help me grow taller and stronger." "I can stop taking my steroids once I start feeling better." "Steroids will help cure my leukemia." "Steroids will make me gain weight, but I can easily lose it." "Steroids can weaken my immune system and increase my risk of infections."

"Steroids can weaken my immune system and increase my risk of infections." Rationale: "Steroids can weaken my immune system and increase my risk of infections." This statement demonstrates comprehension of a common side effect of steroid medications in suppressing the immune system, increasing susceptibility to infections.

Which of these would be the most appropriate way to document a client's refusal of medication? A. "The client refused the heparin injection when I tried to administer it. She yelled at me, saying 'I do not want that injection right now,' and told me to leave the room. I explained the risks of not taking the medication. She seemed very annoyed that I tried to give it at that time. I will attempt again later in my shift." B. "Subcutaneous Heparin injection was attempted to be given to the client per the physician's order. The client refused, stating, 'I do not want that injection.' Potential risks for refusing the medication were reviewed with the client, and the client verbalized understanding." C. "Client stated she did not want the SQ heparin injection at this time. The risks of not taking this med was reviewed with the client. Client verbalized understanding." D. "Heparin refused during the shift, risks reviewe

"Subcutaneous Heparin injection was attempted to be given to the client per the physician's order. The client refused, stating, 'I do not want that injection.' Potential risks for refusing the medication were reviewed with the client, and the client verbalized understanding." Rationale: Documentation in healthcare should be objective, and thorough, but direct. It should be articulate, with correct grammar and spelling. The health record will be scrutinized by legal experts if a dispute about a client's care arises. In court, the health record is legal evidence of the care given to a client and is used to judge whether the interventions were timely and appropriate. Expert reviewers look for documentation of the client's baseline status, changes in status, interpretation of the changes, interventions implemented, and the client's responses to those interventions. The client has the right to refuse a medication regardless of her reasons and regardless of the consequences, except under certain circumstances (e.g. incompetency). It is up to the nurse to document thoroughly and accurately any client's refusal.

The nurse is speaking with her patient, who is undergoing chemotherapy treatment. The patient states, "My friend beat cancer using complementary therapies; I think I should try that too." Which of the following responses from the nurse is most appropriate? A. "Complementary therapies are not safe with your chemotherapy." B. "I would be desperate if I had cancer too." C. "Let us go get your healthcare provider so that we may discuss it with him." D. "Tell me more about what you mean when you say complementary therapies."

"Tell me more about what you mean when you say complementary therapies." Rationale: This is the most therapeutic statement. Effective communication always begins with an open-ended statement. It addresses the question asked by the client and will lead to further discussion. The nurse should explore what therapies the client is interested in talking about first, so that she may better help the client when discussing the therapies with the healthcare provider.

The LPN is caring for a family who just found out that their newborn has tetralogy of Fallot. They state, "we can't believe our baby is going to die!" Which of the following statements by the LPN is most appropriate? A. "Yes, that is so sad. What can I do to help you?" B. "Your baby will be fine! This is not so serious." C. "Tetralogy of Fallot can be surgically repaired. Let's talk more about what you can expect." D. "Well, at least you get to spend time with your baby now. Some people don't even get that."

"Tetralogy of Fallot can be surgically repaired. Let's talk more about what you can expect." Rationale: This statement does not support that the baby will die, but provides factual information about the treatment plan for the defect and leads into a more detailed conversation about what the parents can expect. It is clear that they do not fully understand tetralogy of Fallot (TOF) and the treatment options, so education is very important for these parents.

A nurse has attended a continuing education conference about seasonal influenza. Which of the following statements would indicate a correct understanding of the conference? A. "Oseltamivir therapy should be started 96 hours after the onset of symptoms." B. "The live attenuated vaccine (LAV) is for healthy non-pregnant individuals, 2 through 49 years old." C. "Individuals who are pregnant should not receive the inactivated influenza vaccine." D. "Visitors should be provided face shields when entering a client's room."

"The live attenuated vaccine (LAV) is for healthy non-pregnant individuals, 2 through 49 years old." Rationale: This statement is correct. The LAIV is a nasal spray flu vaccine approved for use in healthy non-pregnant people, 2 through 49 years old. Individuals who are pregnant, immunocompromised, younger than 2, or older than 49 should not receive this vaccine. The LAIV contains weakened influenza viruses that are cold-adapted, which means they are designed to only multiply at the cooler temperatures found within the nose and not the lungs or other areas where warmer temperatures exist. No influenza vaccine causes influenza. The LAIV has demonstrated a more robust immune response when compared to the IIV.

While caring for a client who is suspected of having appendicitis, the nurse overhears his conversation with a loved one. Which of the following statements would prompt immediate intervention? A. "The pain doesn't feel as bad now, I think it was just a stomach ache." B. "Would you mind getting me an ice pack?" C. "I know I'm not supposed to eat anything right now, but I'm hungry." D. "I wonder if I can play in the basketball game on Monday."

"The pain doesn't feel as bad now, I think it was just a stomach ache." Rationale: A client suspected of having appendicitis that suddenly feels better has likely experienced a rupture of the appendix. This situation warrants immediate attention since surgery will be necessary.

Which of the following statements should the nurse use to best describe a very low-calorie diet? A. "This is a long-term treatment measure that assists obese people who can't lose weight." B. "A VLCD contains very little protein." C. "This diet can be used only when there is close medical supervision." D. "This diet consists of solid food that is pureed to facilitate digestion and absorption."

"This diet can be used only when there is close medical supervision." Rationale: Very Low-Calorie Diets (VLCD) are used in the clinical treatment of obesity under close medical supervision. The diet is low in calories, high in quality proteins, and has a minimum of carbohydrates to spare protein and prevent ketosis. Very low-calorie diets, generally providing fewer than 800 kcal per day, became widely available for use in treating adult obesity in the 1980s. These diets, sometimes called protein-sparing modified fasts, were associated with significant medical risks (electrolyte abnormalities, arrhythmias, and sudden death).

The nurse has provided medication instruction to a patient who has been prescribed metformin. Which of the following statements, if made by the patient, would indicate a correct understanding of the teaching? A. "This medication may cause me to have bloating or loose stools." B. "I will need to check my blood glucose prior to taking this medication." C. "If I eat fewer carbohydrates in a day, I should skip a dose." D. "The goal of this medication is to increase my hemoglobin A1C."

"This medication may cause me to have bloating or loose stools." Rationale: The most common side-effect associated with Metformin is gastrointestinal upset. This side-effect typically occurs at the start of the therapy and subsides over time. To minimize these effects, the patient should take this medication with meals.

The licensed practical/vocational nurse (LPN/VN) reinforces teaching to a client newly prescribed methotrexate (MTX). Which of the following statements should the nurse include? Select all that apply.

"This medication may cause you to bruise more easily." "You will need to take folic acid with this medication." "Avoid large crowds and wash your hands frequently." Rationale: Methotrexate (MTX) is indicated for a variety of autoimmune conditions. This medication carries serious adverse effects such as pancytopenia (low red blood cells, white blood cells, and platelets). Thus, this may cause a client to bruise more easily and be at a higher risk of infection. MTX antagonizes folic acid, and while a client is taking MTX, folic acid supplementation is typically prescribed.

The licensed practical/vocational nurse (LPN/VN) reinforces teaching to a client newly prescribed methotrexate (MTX). Which of the following statements should the nurse include? Select all that apply. "This medication may cause you to bruise more easily." "You will need to take folic acid with this medication." "You will need to remain upright for thirty minutes after taking a dose." "This medication may cause your gums to get enlarged." "Avoid large crowds and wash your hands frequently."

"This medication may cause you to bruise more easily." "You will need to take folic acid with this medication." "Avoid large crowds and wash your hands frequently." Rationale: Methotrexate (MTX) is indicated for a variety of autoimmune conditions. This medication carries serious adverse effects such as pancytopenia (low red blood cells, white blood cells, and platelets). Thus, this may cause a client to bruise more easily and be at a higher risk of infection. MTX antagonizes folic acid, and while a client is taking MTX, folic acid supplementation is typically prescribed.

The nurse is caring for a teenager who is recovering from a tonsillectomy. The nurse walks in the room, and sees the client eating chips and salsa from a Mexican restaurant. Which of the following responses is most appropriate? A. "I love that restaurant! Their chips are so good." B. "You cannot eat anything yet, I am sorry" C. "Chips are not a good choice right now because you need a high protein diet after your surgery." D. "Those chips are really hard on the back of your throat where you had your surgery. I'm worried they could cause you to bleed if they damage your incision site. Let's get something softer for you to eat right now."

"Those chips are really hard on the back of your throat where you had your surgery. I'm worried they could cause you to bleed if they damage your incision site. Let's get something softer for you to eat right now." Rationale: This is the most appropriate response by the nurse. She correctly explains to the client that the sharp tortilla chips would be really hard on the client's surgical site after a tonsillectomy. Allowing client's to eat foods like chips or popcorn after surgery in the back of the throat would put them at risk for damage to the incision and subsequent hemorrhage. Offering the client something soft, such as gelatin or soup, is the most appropriate.

When teaching medication safety to a toddler's parent, which statement by the parent would be a cause for concern?

"To get her to take her medicine, we tell her it's candy." Rationale: Children should never be told that medication is candy.

You are reinforcing education with the parents of a toddler suffering from gastroesophageal reflux disease (GERD). You know they understand your teaching when they make which of the following statements? Select all that apply. "We should feed him 6 small meals a day instead of a few big ones." "Making sure he is sitting upright while eating may help the reflux." "He should try to sleep on his left side so that his stomach can empty more easily." "There are no medications that can help with this disease, so we will have to make lifestyle changes." This is a chronic condition. There is no chance he will grow out of it.

"We should feed him 6 small meals a day instead of a few big ones." "Making sure he is sitting upright while eating may help the reflux." Rationale: Small, frequent meals are an excellent recommendation to help alleviate GERD symptoms. This will ensure the stomach does not overfill and help decrease the amount of reflux the client is experiencing. The upright position is essential for GERD clients while they are eating. This is a useful educational point. Upright positioning will help to prevent or decrease the passage of gastric contents into the esophagus.

The licensed practical/vocational (LPN/VN) assists a registered nurse (RN) in planning a community health course about preventing Lyme disease. Which of the following information should be included? A. "You should try limiting your outdoor activities between 10 a.m. and 4 p.m." B. "Wear sunglasses that wrap around and block UVA and UVB rays." C. "Wear long-sleeved clothing when in heavily wooded areas." D. "Apply sunscreen with at least an SPF of 30."

"Wear long-sleeved clothing when in heavily wooded areas." Rationale: Lyme disease is spread by a deer tick commonly found in heavily wooded areas. Wearing long-sleeved clothing, applying tick repellent. and showering after hikes in the woods is an effective strategy in preventing being bitten by a tick and further infected with the bacteria.

The nurse is collecting data on a client newly admitted to the medical-surgical unit. Which question would be appropriate for the nurse to ask regarding the client's spirituality? A. "What are your sources of hope and strength?" B. "Have you considered arranging a visit from members of your church?" C. "Do you attend worship any specific day of the week?" D. "Can you tell me about your dietary preferences and any restrictions?"

"What are your sources of hope and strength?" Rationale: Spirituality and religion are not synonymous. Spirituality refers to an interpretation of a client's beliefs regarding their own would or spirit and a connection bigger than oneself. Spirituality is fluid and often evolves. Religion is structured and may have a specific God or gods. Religion is often ritualistic and may involve readings from a well-regarded book or text. Spirituality is fluid and is purely internal to one's ability to feel connected and look inside for hope and strength. Religion often looks to an external source, such as the Bible, for guidance. This question is appropriate to ask the client to inquire about their spirituality.

The nurse is caring for a client who is in psychosis. The client states, "You all are trying to kill me!" Which of the following responses would be most appropriate for the nurse to make? A. "What your experiencing is not real." B. "Are you hearing voices?" C. "You are safe here, please be calm." D. "What makes you think we are trying to kill you?"

"What makes you think we are trying to kill you?" Rationale: A client experiencing psychosis does not exhibit a rational thought process and may have impaired reality testing. If the client is paranoid, the nurse should attempt to understand the paranoia as the client has likely misconstrued an action.

The LPN is reinforcing teaching regarding nutrition for toddlers. Which of the following information should they include? Select all that apply. "Whole milk can be offered to provide calcium and vitamin D." "Using food as a reward can be a positive incentive." "Good iron-rich food choices include melon and strawberries." "Roasted vegetables are preferred over fried vegetables." "Vitamin A rich foods should be given with iron to increase its absorption."

"Whole milk can be offered to provide calcium and vitamin D." "Roasted vegetables are preferred over fried vegetables." Rationale: Plain cow milk (whole milk) or fortified unsweetened soy beverage can be offered at 12 months of age to help meet calcium, potassium, vitamin D, and protein needs. Flavored milk should be avoided until 24 months because they contain excessive sugars. Vegetables are an excellent source of fiber and are dense in vitamins. The healthy benefits of vegetables can be negated if they are fried. Roasting the vegetables are preferred.

The nurse is reinforcing teaching with a client who is scheduled for a contraction stress test (CST). Which of the following statements should the nurse include? A. "You will need to consume a liquid with 50 grams of glucose." B. "You may need to stimulate your nipples during this test." C. "A positive result means your baby has had no late decelerations." D. "A negative result means your baby has had variable decelerations."

"You may need to stimulate your nipples during this test." Rationale: A CST is indicated for clients who are high-risk and are in the third trimester. It requires the client to have contractions either through oxytocin administration or nipple stimulation.

The nurse is discharging a client following knee arthroplasty. Which of the following information should the nurse reinforce in the discharge teaching? A. "After this procedure, you will use a wheelchair to get around." B. "You will need to resume your anticoagulants." C. "Placing a pillow under your knee will help with the pain." D. "You may ice the site for one hour at a time."

"You will need to resume your anticoagulants." Rationale: Following a knee arthroplasty, the cliets will not be as mobile and require crutches. This may increase their risk of developing venous thromboembolism (VTE). Thus, the nurse should reiterate that the clients resume their prescribed anticoagulants or initiate them if they are newly prescribed. Evidence-based guidelines recommend that clients undergoing total hip or total knee arthroplasty receive anticoagulant prophylaxis for a minimum of 14 days.

You are reinforcing counseling for two parents that are preparing for the birth of their first child. The mother has sickle cell anemia. So the father has decided to undergo genetic testing to determine if he is a carrier or not. He finds out that he is not a carrier. What is the chance of their baby having sickle cell anemia? A. 25% B. 50% C. 75% D. 0%

0% Rationale: The baby has no chance or 0% chance of having sickle cell anemia. Instead, the baby will be a carrier. Since the mother has the disease, she is ss. Since the father has tested that he is not a carrier, he is SS. This means that the only combination possible for the baby is Ss or a carrier.

The nurse in the emergency department (ED) is caring for a client with heat stroke. Which prescription should the nurse anticipate from the primary healthcare provider (PHCP)? A. Acetaminophen B. Naproxen C. Ketorolac D. 0.9% saline

0.9% saline Rationale: 0.9% saline infusion is necessary to administer to a client with heat stroke because this condition induces a significant fluid deficit. Additionally, 0.9% saline may be cooled to provide an adjunctive cooling measure.

The nurse is discussing alcohol withdrawal with a group of nursing students. The nurse knows that the following components of alcohol withdrawal would be expected to go in what order? 1. Mild tremors 2. Delirium Tremens 3. Hyperthermia 4. Hallucinations

1. Mild tremors 2. Hyperthermia 3. Hallucinations 4. Delirium Tremens Rationale: 1. Mild tremors are something you would expect to see at the beginning of alcohol withdrawal in the first stage. This typically starts about 4-12 hours after the client has stopped drinking and will be accompanied by other symptoms such as nausea, insomnia, and headaches. At this point, the client is still alert and oriented. 2. Hyperthermia will follow the mild tremors. The withdrawal is entering into the second phase or moderate withdrawal. At this point, clients start to get confused and hyperactive. Everything increases, they present with hyperthermia, hyperactivity, hypertension, tachypnea, and tachycardia. 3. Hallucinations will begin after the client becomes hyperthermic and hyperactive. This is a severe withdrawal. Hallucinations and illusions can be auditory or visual and are very scary for the client. It is essential to stay with the client at this phase of the withdrawal. 4. The last symptom you would expect to see in this progression is delirium tremens. This is the most severe and dangerous phase. Delirium Tremens (DTs) is considered a medical emergency. These clients have an altered mental status and sympathetic overdrive, which can progress to cardiovascular collapse. They continue to experience hallucinations and illusions and can have tonic-clonic seizures.

*NGN* The nurse cares for a client with disseminated intravascular coagulation (DIC) and prescribed multiple blood products Item 1 of 1 Nurses' Note Blood consent was completed and obtained. Type and screen and type and cross results noted. 20-gauge peripheral vascular access device started in the right antecubital space. Flushed with no resistance and positive blood return. Educated the client on the purpose of the transfusion and signs and symptoms of a transfusion reaction. Orders Two units of Packed Red Blood Cells (PRBCs) Two units of Platelets One unit of Fresh Frozen Plasma (FFP) Laboratory Hemoglobin: 5.4 g/dL (Male: 14-18; Female: 12-16) Hematocrit: 16.20% (Male: 42-52%; Female: 37-47%) Platelets: 19,000 mm3 (150,000-400,000 mm3) INR: 3.4 seconds (0.9-1.2 seconds) Select the most likely option for the missing information in the table below by choosing from the list of options. Each option may only be us

1. Packed Red Blood Cells (PRBCs) 2. Administer over 15-30 minutes 3. Give through short tubing and small filter Rationale: Packed Red Blood Cells are indicated for hemoglobin of 7 g/dL or less. Additionally, the transfusion time for PRBCs is 2-4 hours. The blood product should be type-specific, but if not possible, O negative may be administered as it is the universal donor. When infusing PRBC's, 0.9% saline should be spiked with the blood product using y-type tubing. Fresh Frozen Plasma is indicated for clotting factor replacement and volume expansion. FFP must be type specific and is administered over 15-30 minutes. To determine efficacy, the nurse should reassess the PT/INR after the transfusion. Platelets are used to treat platelet dysfunction and thrombocytopenia. Platelet transfusions are indicated once the platelet count reaches 20,000-25,000 mm3. Platelets do not have to be type specific as they are pooled from as many as ten donors. The infusion time is 15-30 minutes.

The nurse is caring for a female client who is incontinent of urine. The MD orders an indwelling Foley catheter to be placed. Place the following actions in the order that the nurse takes to correctly insert the Foley catheter:

1. Perform hand hygiene, identify patient, explain procedure, and wear sterile gloves 2. Spread the labia and hold them open 3. Cleanse the meatus from front to back on the right side, then left side, and down the center 4. Insert the catheter and inflate the balloon 5. Secure the catheter to the client, and initial the securement device with the date and time Rationale: First, the nurse should perform hand hygiene, identify the client using 2 client identifiers, explain the procedure to the client, and wear sterile gloves. Second the nurse uses her non-dominant hand to spread the labia and hold them open. Third the nurse uses her dominant hand to cleanse the meatus from front to back on the right side, then left side, then down the center. Fourth the nurse will insert the catheter and inflate the balloon. Lastly, the nurse will secure the catheter to the client, and place their initials, date, and time on the securement device.

The nurse is triaging phone calls in the prenatal clinic. The nurse should initially follow-up on the client who is A. 16 weeks of gestation and reports a fluttering sensation B. 30 weeks of gestation and reports perianal itching and bright red blood in the stool C. 28 weeks of gestation and reports intermittent leg cramping with swelling in her feet D. 38 weeks of gestation and reports lower back pain that increases with walking

38 weeks of gestation and reports lower back pain that increases with walking Rationale: These manifestations indicate that the client may be beginning labor. True labor manifestations include contraction patterns of increasing frequency, duration, and intensity. The contractions tend to increase with walking and may start in the lower back and gradually sweep around to the lower abdomen. Considering the client's gestational age and manifestations, this client requires immediate follow-up.

*NGN* The nurse cares for a 22-year-old female with an eating disorder Item 1 of 1 History And Physical A 22-year-old female was admitted voluntarily to the inpatient unit following a need to 'get help with her eating habits.' Reportedly, the client admits to eating a large amount of food and feeling 'disgusted' afterward, which triggers self-induced vomiting. She has done this multiple times and reports it has 'gotten out of control.' She says she is always worried about her appearance 'not being good enough.' She denies using any laxatives or diuretics; however, her mother reports she found two empty boxes of laxatives in her apartment. On exam, the client is alert and completely oriented. She is cooperative during the exam and has an anxious and worried affect. She has a slender appearance and a current body mass index (BMI) of 20. Scars were observed on both index fingers. She says physically, her only complain

1. Provide small, frequent meals 2. Recommend inpatient group therapy 3. Promote positive reframing Rationale: This client is demonstrating signs of bulimia nervosa. To address the imbalanced nutrition nursing diagnosis, the nurse should provide small, frequent meals. This decreases the interval between large meals, decreasing the likelihood of purging. The client should not eat meals alone as they risk purging. They should be supervised thirty minutes following meal consumption. Group therapy is recommended for a client with a disturbed body image. This allows the client to engage with others and develop rapport and self-esteem. Promoting positive reframing is always an effective strategy for anxiety nursing diagnosis. This helps decondition this cognitive and behavioral disconnect.

The nurse is reinforcing education to a client about newly prescribed aspart insulin. The nurse should instruct the client to self-administer this insulin A. 30-45 minutes before a meal. B. one hour after a meal. C. 20-30 minutes before a meal. D. 10-15 minutes before a meal.

10-15 minutes before a meal. Rationale: Aspart insulin is a rapid actin insulin that should be administered to the client no greater than 10-15 minutes prior to the meal or while the client is actively eating. Prior to the administration of this insulin, the client's blood glucose should be obtained.

The primary healthcare provider (PHCP) prescribes 250 mL of 0.9% saline to infuse over 75 minutes. How many mL per hour will be administered to the client?

200 mL/hr To solve this problem, the formula of volume / time (hours) will be used. First, convert the minutes to hours 75 minutes / 60 minutes = 1.25 hrs Next, divide the prescribed total volume by the infusion time 250 mL / 1.25 hours = 200 mL/hr

The nurse is preparing to administer an intramuscular (IM) injection to a neonate. Which gauge and size needle should the nurse use to administer the medication? A. 19 gauge, 1 1/2" (3.8 cm) needle B. 18 gauge, 1" (2.5 cm) needle C. 20 gauge, 1" (2.5 cm) needle D. 25 gauge, 5/8" (1.6 cm) needle

25 gauge, 5/8" (1.6 cm) needle Rationale: This needle size and gauge are appropriate for a neonate. When administering IM medications to a neonate or young child, the vastus lateralis is the preferred site. For the volume to be administered in an IM, it is recommended to be 0.5 mL or less for infants; up to 2 mL for children.

You are reinforcing counseling for two parents that are preparing for the birth of their first child. They decided to undergo genetic testing and find out that they are both carriers of sickle cell anemia. What is the chance of their baby having sickle cell anemia? A. 25% B. 50% C. 75% D. 100%

25% Rationale: Their baby has a 25% chance of having sickle cell anemia. The father and the mother are Ss because they are carriers.

The licensed practical/vocational nurse (LPN/VN) is working with a registered nurse caring for a child experiencing an adrenal crisis. The RN has established a peripheral vascular access device and the LPN/VN anticipates the RN to administer intravenous Select all that apply. potassium chloride. 5% dextrose with 0.9% saline. hydrocortisone. levothyroxine. desmopressin. propranolol.

5% dextrose with 0.9% saline. hydrocortisone. Rationale: An adrenal crisis is a medical emergency for both an adult and a child. Remember, you need to add the treatment in an adrenal crisis (Addisonian crisis). The immediate treatment for a client in an adrenal crisis is replacing the corticosteroid via intravenous (IV) hydrocortisone. The treatment goal of administering IV hydrocortisone is to increase the low glucose levels and retain some of the fluid and sodium. The second essential treatment is administering IV fluids of 5% dextrose with 0.9% saline. The 5% dextrose with 0.9% saline will raise the glucose (D5) and circulating volume (0.9% saline). Giving D5W alone would be detrimental as the water will lower serum sodium levels.

Hemophilia is an X-linked recessive disorder - unaffected man and woman who is a carrier have a baby, what percent of their male offspring would be expected to have hemophilia?

50%

You are reinforcing counseling for two parents who are preparing for the birth of their first child. The father has sickle cell anemia and the mother is a carrier. You tell them that their baby has what chance of having sickle cell anemia? A. 25% B. 50% C. 75% D. 100%

50% Rationale: Their baby has a 50% chance of having sickle cell anemia. The father is ss because he has the disease, and the mother is Ss since she is a carrier.

You are working in the intensive care nursery and are assigned to take care of an infant withdrawing from cocaine. At your first assessment, you appreciate the following: a high-pitched cry, no tremors, increased muscle tone, sleeping for 3 hours in between feedings, no congestion, RR = 42, excessive sucking on the pacifier, poor feeding, no vomiting, and no loose stools. What is the NAS score for this client? A. 7 B. 12 C. 2 D. 8

7 Rationale: Each of these observations gives you a NAS score for this infant: High pitched cry = 2 points. No tremors = 0 points Increased muscle tone = 2 points. Sleeping for 3 hours in between feedings =0 points No congestion = 0 points RR = 42 = 0 points Excessive sucking on the pacifier = 1 point Poor feeding = 2 points No vomiting = 0 points No loose stools = 0 points This adds up to a total of 7 points for the NAS score for this client. A score of 7 is in the moderate range. Less than 5 is mild, 5-8 is moderate, 8-12 is severe, and greater than 12 is very severe. For a score of 7, a breakthrough dose of morphine may not be appropriate; the infant may not be ready to wean down on their methadone further.

You are taking care of a client who is postoperative day one following a mitral valve replacement. He is getting ready to ambulate for the first time. His heart rate is 102 beats/minute, and the stroke volume based on the echocardiogram is 72 mL. What is his cardiac output (CO)? A. 7.344 L/min B. 30 L/min C. 55% D. 73.444 mL/min

7.344 L/min Rationale: To answer this question correctly, you must know the right formula for Cardiac Output (CO). CO = Heart Rate (HR) x Stroke Volume (SV). Heart rate is measured in beats per minute, whereas stroke volume is measured in milliliters (mL). The HR is simply the number of times the heart beats per minute. Stroke volume (SV) is the mL of blood that the heart pumps out with each contraction. By multiplying the two together, you get how many milliliters of blood the heart is pumping out each minute. This is the cardiac output. Cardiac output is usually reported in liters/min, and the average is about 5 L/min, but it varies greatly depending on the client's size. A decreased cardiac output (low-output failure) is seen in congestive heart failure. An increased cardiac output (high-output failure) may be seen in hyperthyroidism, thiamine deficiency, and severe uncorrected anemia. A high cardiac output state refers to a resting cardiac output of more than 8 L /min. For this problem: Cardiac Output (CO) = 102 beats per minute (HR) x 72 mL (SV) = 7,344 mL/min or 7.344 L/min.

The LPN is caring for the following clients. The nurse should identify which client is at the highest risk for falling? A. 88-year-old admitted with a chest tube secondary to pneumothorax and has a history of dementia. B. 44-year-old admitted with heart failure, has a peripheral IV, and receiving IV furosemide. C. 33-year-old admitted with cholecystitis, has a peripheral IV, and is receiving IV hydromorphone. D. 28-year-old admitted with bacteremia, is receiving intravenous fluids via central line, and is diaphoretic.

88-year-old admitted with a chest tube secondary to pneumothorax and has a history of dementia. Rationale: This client has advanced age, has a medical device that impedes their mobility, and has cognitive impairments. Thus, all these risk factors put this client at a very high risk of falling.

The nurse, while on duty in an adult cardiac telemetry unit, encounters the following rhythms on the monitor. Which of the following clients should the nurse promptly attend to and immediately notify the RN about? See image

A Rationale: This electrocardiogram (ECG) shows a third-degree heart block, otherwise known as a complete heart block. This rhythm is fatal without intervention. If you notice this, you must notify the RN immediately so appropriate interventions can be implemented. A complete heart block may lead to fatal symptomatic bradycardia with a heart rate of less than 40/min, hypotension, seizures, cerebral ischemia, cardiac arrest, or sudden cardiac death. Therefore, it is considered a fatal rhythm, and hence, this client is a top priority.

Which of these would be an appropriate assignment for the LPN/LVN? A. A 17-year-old client with a femur fracture that is just returning to the floor from the recovery unit. B. A 21-year-old client with cystic fibrosis that needs an early morning sputum sample collection. C. An 82-year-old client 2 days post-op after knee replacement surgery who needs help ambulating. D. A 31-year-old client who is suffering from an acute asthma attack.

A 21-year-old client with cystic fibrosis that needs an early morning sputum sample collection. Rationale: Collecting sputum samples on stable clients are within the scope of practice for an LPN/LVN. While an LPN/LVN may feel comfortable providing care for clients with varying degrees of illness, that does not mean that all attention should be delegated to an LPN/LVN. As an LPN/LVN, it is your responsibility to know which clients you should and should not care for and why. This is important because if an LPN/LVN accepts an assignment that is not within their scope of practice/competency, it could result in legal ramifications.

Which of the following patient assignments would be appropriate for unlicensed assistive personnel? A. A 65-year-old male requiring sterile dressing changes. B. A 26-year-old female requiring a one-person assist in ambulating to the restroom. C. An 80-year-old male who is receiving enteral feedings continuously through an NG tube. D. A 16-year-old female who is 4 hours post-cardiac catheterization.

A 26-year-old female requiring a one-person assist in ambulating to the restroom. Rationale: A 26-year-old female requiring a one-person assist in ambulating to the restroom would be an appropriate assignment for unlicensed assistive personnel (UAP). UAP are skilled in assisting clients with ambulation and this is within their scope of practice.

Which of the following clients should the nurse assess first when preparing to do initial rounds? A. The client with diabetes that is being discharged today. B. A 32-year-old female with a tracheostomy that is experiencing copious secretions. C. A 16-year-old client that is scheduled for physical therapy this morning. D. An 80-year-old male with a decubitus ulcer that needs a dressing change.

A 32-year-old female with a tracheostomy that is experiencing copious secretions. Rationale: The client with potential airway compromise should always be given the highest priority. Remember "ABCs" (Airway, Breathing, Circulation). Copious secretions in a client with a tracheostomy can be a sign of airway obstruction or respiratory distress, which is considered a critical and potentially life-threatening situation.

The nurse is conducting presurgical assessments on clients at an ambulatory care center. In which scenario should the nurse report to the surgeon, as it may necessitate postponing surgery? A. A 20-year-old client who is a vegan. B. An elderly client who intakes daily nutritional drinks. C. A 40-year-old client who takes ginkgo biloba and aspirin daily. D. An infant who is breastfeeding.

A 40-year-old client who takes ginkgo biloba and aspirin daily. Rationale: Ginkgo biloba (herbal), aspirin, and vitamin E all have anticoagulant properties. It is vital to notify the surgeon about these medications to decrease the client's risk of excessive bleeding.

Which of the following clients is at greatest risk for impaired vascular perfusion? A. An 80-year-old female with a history of alcoholism. B. A 75-year-old male with a history of radon gas exposure. C. A 59-year-old male with a history of cigarette smoking. D. A 60-year-old male with a diagnosis of hypertension.

A 60-year-old male with a diagnosis of hypertension. Rationale: Many conditions can disrupt the exchange of oxygen and carbon dioxide. Still, diabetes, obesity, anemia, high blood pressure, and coronary artery disease are some of the more common risk factors that can cause ineffective tissue perfusion. Other risk factors include immobility, hypoxia, decreased cardiac output, diabetes, and hypoventilation.

The nurse has received an assignment of four clients on the Medical-Surgical floor. Which client should she/he check on first? A. A 61-year-old male client that is one-day post-op from hernia repair, complaining of pain at the incision site. B. A 68-year-old female client with type II diabetes, complaining of stomach discomfort. C. A 72-year-old male client with emphysema and a history of uncontrolled hypertension, complaining of a headache. D. A 70-year-old female patient who is two days post-op from ankle surgery, complaining of some shortness of breath.

A 70-year-old female patient who is two days post-op from ankle surgery, complaining of some shortness of breath. Rationale: The ABCs identify the airway, breathing, and cardiovascular status of the client as the highest priority in that sequential order. Maslow's Hierarchy of Needs identifies the physiological or biological needs, including the ABCs, the safety/psychological/emotional needs, the need for love and belonging, the requirements for self-esteem, and others' esteem, and self-actualization needs in that order of priority.

The nurse is caring for clients in a long term care facility, which client is at the greatest risk for developing a decubitus ulcer? A. An incontinent client who had 3 diarrhea stools. B. An 80-year-old ambulatory diabetic client. C. A 79-year-old malnourished client on bed rest. D. An obese client who uses a wheelchair.

A 79-year-old malnourished client on bed rest. Rationale: Prolonged inadequate nutrition causes weight loss, muscle atrophy, and the loss of subcutaneous tissue. These three conditions reduce the amount of padding between the skin and bones, thus increasing the risk of pressure ulcer development. More specifically, inadequate protein intake, carbohydrates, fluids, zinc, and vitamin C contribute to pressure ulcer formation. Several factors contribute to the formation of pressure ulcers: friction and shearing, immobility, inadequate nutrition, fecal and urinary incontinence, decreased mental status, diminished sensation, excessive body heat, advanced age, and the presence of certain chronic conditions.

Which of the following clients fall within the scope of practice of a licensed practical/vocational nurse (LPN/VN)? Select all that apply. A client requiring an assessment of their current medications A client needing a nasogastric tube (NGT) for enteral feedings A client with an insulin pump who is unsure of how to load the insulin A client requesting additional teaching on discharge prescriptions A client requiring airborne isolation and bronchodilators via an inhaler

A client needing a nasogastric tube (NGT) for enteral feedings A client requiring airborne isolation and bronchodilators via an inhaler Rationale: Practical/vocational nurses should get the most stable client assignment. Skills such as the insertion of an NGT are within the scope of an LPN/VN. Further, LPN/VN's may care for a client in isolation and administer bronchodilators via an inhaler.

Which of the following clients would be most appropriate to assign to the LPN? Select all that apply. A client receiving antibiotics for lower extremity cellulitis. A client newly admitted with an exacerbation of myasthenia gravis. A client with a chest tube receiving mechanical ventilation. A client requiring a referral for an outpatient support group. A client needing to receive intramuscular RhoGAM. A patient needing scheduled tube feedings and colostomy irrigations.

A client receiving antibiotics for lower extremity cellulitis. A patient needing scheduled tube feedings and colostomy irrigations. Rationale: When making client assignments, the LPN should be assigned to the stable client with a predictable outcome. A client receiving antibiotics for lower extremity cellulitis is a low acuity illness and may be cared for by the LPN. Scheduled tube feedings and colostomy irrigations are within the scope of an LPN, and this can be delegated.

The nurse is caring for the following assigned clients. Which client should the nurse follow up with first?

A client requesting diphenhydramine after starting an intravenous antibiotic. Rationale: A client requesting diphenhydramine following the initiation of an antibiotic requires immediate follow-up because the client could be experiencing an allergic reaction ranging from mild to severe. Thus, the nurse should quickly follow-up with this client.

While working in the emergency department's resuscitation area, EMS notifies you that a 7-year-old male with an avulsion fracture to the left tibia is 20 minutes out. You know to expect which of the following? A. A fracture that pulls a part of the bone from the tendon or ligament. B. A fracture with which the whole cross section of the bone is fractured. C. A fracture that results from an underlying disease or disorder, not physical trauma or stressors. D. A fracture that affects only one side of the bone.

A fracture that pulls a part of the bone from the tendon or ligament. Rationale: An avulsion fracture pulls a part of the bone from the tendon or ligament. Fractures are a common occurrence, and clients often present to the emergency department. A nurse should be able to recognize different types of bone fractures and plan for appropriate nursing intervention.

The nurse is caring for assigned clients. The nurse should recognize that the client at greatest risk for compartment syndrome is the client who has which of the following? A. A left tibial fracture that was recently placed in a cast B. Swelling in the ankles and is wearing compression stockings C. Chronic osteomyelitis of the right femur D. Skin traction following a left hip fracture

A left tibial fracture that was recently placed in a cast Rationale: A client who recently had a fracture and cast is at high risk of compartment syndrome. The recent fracture causes swelling, which can be enclosed by the cast. Orthopedic fractures are a significant risk factor for compartment syndrome.

Which of the following clients should the LPN/LVN attend to first? A. A client who is newly diagnosed with Hepatitis A that is reporting stomach pain and itchy skin. B. A patient in an arm cast that is 2 days post-op and reports feeling numbness and tingling in his affected arm. C. A post-op prostatectomy patient complaining of bladder spasms and blood in his foley bag. D. A patient with a newly placed NG tube complaining of pain around the face and a "plugged" nose.

A patient in an arm cast that is 2 days post-op and reports feeling numbness and tingling in his affected arm. Rationale: Numbness and tingling hours or days after a cast is applied may indicate compartment syndrome and should be reported to a doctor right away.

What consideration should the nurse keep in mind regarding the use of side rails for a confused client? A. They prevent confused clients from wandering. B. A history of a previous fall from bed with raised side rails is insignificant. C. A person of small stature is at increased risk for injury from entrapment. D. Alternative measures are ineffective to prevent wandering.

A person of small stature is at increased risk for injury from entrapment. Rationale: Studies of restraint-related deaths have shown that people of small stature are more likely to slip through or between the side rails.

The licensed practical/vocational nurse (LPN/VN) is assisting the triage nurse. Which of the following clients in the emergency department (ED) triage requires the most rapid action to protect other clients in the ED from infection? A. A travel blogger who needs tuberculosis testing after exposure to a person with TB during his trip. B. An older woman who has a history of a methicillin-resistant Staphylococcus aureus (MRSA) leg wound infection. C. A pregnant woman with a blister-like rash on the face that is possibly having varicella. D. An infant with a runny nose whose older brother has pertussis

A pregnant woman with a blister-like rash on the face that is possibly having varicella. Rationale: The primary responsibility of the triage nurse is to conduct an initial nursing assessment and determine which clients require immediate care or isolation. The triage nurse should be capable of identifying clients who may pose a potential risk to others by being familiar with commonly occurring illnesses and infections. Chickenpox (Varicella) is transmitted through the air and can easily spread to other clients in the emergency unit. Varicella is highly contagious, and pregnant women are at risk of severe complications. It's crucial to promptly isolate and manage this client to prevent potential outbreaks and protect others in the ED from exposure to the virus. This client should be placed in a negative-pressure room.

The licensed practical/vocational nurse (LPN/VN) is reviewing incident reporting. Which of the following situations would require an incident report? Select all that apply. A visitor refusing to wear personal protective equipment (PPE). A visitor activating a patient's patient-controlled analgesia (PCA) device. A visitor requesting that their family member get pain medication. A visitor assisting their family member with brushing their teeth. A visitor stating that they fell while using the bathroom.

A visitor refusing to wear personal protective equipment (PPE). A visitor activating a patient's patient-controlled analgesia (PCA) device. A visitor stating that they fell while using the bathroom. Rationale: Incident (sometimes termed occurrence or event) reporting is required when any activity deviates from the norm. Incident reporting may be completed for visitors. Events that would warrant reporting would include the refusal to wear PPE, activating a client's PCA device, and starting that they fell while they used the bathroom.

The nurse is assessing a client diagnosed with stage IV nephroblastoma. Which of the following actions is contraindicated in this client? A. Checking capillary refill B. Auscultating heart sounds C. Abdominal palpation D. Assessing urine color

Abdominal palpation Rationale: Abdominal palpation is contraindicated in the client with a nephroblastoma. This is because the tumor is located in the abdominal cavity, and by palpating it, you can disturb the tissue and cause cells from the tumor to break loose and spread to other parts of the body. The nurse should not perform abdominal palpation on a client with a nephroblastoma.

The nurse from an adult-only unit has been pulled to a floor that has clients of all ages. What would be an appropriate action for this nurse? A. Accept the assignment, but ask to be paired with a more experienced LPN. B. Accept the assignment, but explain the situation to the charge nurse and ask for a quick orientation before starting. C. Take the assignment but tell the charge nurse she will only care for adult clients. D. Refuse to take the assignment, as caring for the infant and child population is not within her practice scope.

Accept the assignment, but explain the situation to the charge nurse and ask for a quick orientation before starting. Rationale: The nurse should take the assignment, but explain the situation to the charge nurse and ask for a quick orientation before starting. While the LPN may specialize in a specific type of nursing or feel more comfortable caring for a particular client population, she should be able to use her skillset to safely and independently care for other people as well. However, the LPN should let the charge nurse know her background before beginning her shift to familiarize herself with new equipment, ask questions, and identify resources.

When collecting data about a client's pain, the first step in pain assessment is for the nurse to: A. Accept the client's report of pain. B. Get the description of the location and intensity of the pain. C. Have the client identify coping methods. D. Determine the client's status of pain.

Accept the client's report of pain. Rationale: Assessment of pain is a vital part of any nursing assessment. In fact, pain is often called the "fifth vital sign". If a client does not immediately report pain, it is still the nurse's responsibility to ask if he/she feels any pain. If the client reports pain (under any circumstance), the nurse should validate the client's concern by acknowledging he/she is feeling discomfort and then assess more thoroughly to find out the location and intensity, identify coping mechanisms, and follow up with their pain status.

The nurse is performing a physical assessment on a client with Cushing's disease. Which assessment findings should the nurse expect? Select all that apply. Hypotension Acne Hirsutism Buffalo hump Truncal obesity

Acne Hirsutism Buffalo hump Truncal obesity Rationale: Acne is an expected symptom of Cushing's disease. This is due to increased sex hormones, such as testosterone, estrogen, and progesterone. Excessive levels of these hormones cause oily skin to build up, often leading to acne. Hirsutism is defined as excessive body hair in either men or women in places where the hair usually is absent, such as the chin or cheeks of the face. Hirsutism is caused by increased androgens, or male sex hormones such as testosterone, in the body. A buffalo hump is the classic sign of Cushing's disease. This refers to a lump of fat that develops between the shoulder blades on the top of the back. It is due to the excessive amount of glucocorticoids that clients with Cushing's disease have. Glucocorticoids cause the breakdown of fats; when there are too many, they can cause fat redistribution. This often leads to fat in odd places, such as a buffalo hump. This excessive fat storage causes an individual to develop truncal obesity.

The nurse cares for a 41-year-old female in the emergency department (ED) Item 6 of 6

Activated Partial Thromboplastin Time (aPTT): 110 seconds Rationale: While a client receives a heparin infusion, the goal is to prolong the control (baseline) aPTT 1.5 to 2.5 times. 110 seconds is too prolonged and requires the nurse to review the heparin protocol to hold the infusion for a specified period or reduce the rate. The other labs are within normal limits. The client was prescribed warfarin, whose INR is now 2.7, which is therapeutic as the goal is to have the INR between 2-3.

The nurse collects data on a client with chronic obstructive pulmonary disease (COPD). Based on the client's vital signs, the nurse should take which action? See the exhibit. A. Administer acetaminophen (APAP) B. Provide the client with warm blankets C. Apply oxygen at 2 liters via nasal cannula D. Obtain an arterial blood gas (ABG)

Administer acetaminophen (APAP) Rationale: The client has a fever, and the treatment for a fever includes fluids and antipyretics such as acetaminophen or ibuprofen.

The nurse is caring for a 1 year old male diagnosed with acute otitis media. He is experiencing otalgia, has been febrile for 24 hours, and is pulling at his left ear. Which of the following interventions is the priority nursing action? A. Position the child on his left side B. Administer antibiotic ear drops C. Administer acetaminophen as prescribed D. Apply a heat pack to the left ear

Administer acetaminophen as prescribed Rationale: Administering acetaminophen is the priority nursing action in this scenario. The question states that the patient has been febrile for 24 hours. It is the priority to address this concern, and the nurse can do so through the administration of the antipyretic, acetaminophen.

A nurse is caring for a client with heart failure. Which of the following interventions should the nurse implement to manage fluid volume overload? Select all that apply. Administer diuretics as prescribed Monitor daily weights Restrict fluid intake to 500 mL per day Collect data on the client's lung sounds for crackles Encourage high-sodium diet Collect data on the client's lung sounds for rhonchi

Administer diuretics as prescribed Monitor daily weights Collect data on the client's lung sounds for crackles Rationale: Several interventions may be necessary to manage fluid volume overload in a client with heart failure. Administering diuretics helps promote the excretion of excess fluid and decrease fluid volume. Monitoring daily weights is crucial in identifying changes in fluid status. Auscultating lung sounds for crackles is important to detect pulmonary congestion, a sign of fluid overload.

The nurse is teaching a new group of nurses about insulin administration for a client with type I diabetes mellitus. Which of the following points should be included? Select all that apply. Administer insulin subcutaneously, not intramuscularly. Regular insulin can be administered intravenously in emergency situations. Rotate injection sites to prevent lipohypertrophy. Long-acting insulin should not be mixed with other types of insulin It's safe to administer cold insulin directly from the refrigerator.

Administer insulin subcutaneously, not intramuscularly. Regular insulin can be administered intravenously in emergency situations. Rotate injection sites to prevent lipohypertrophy. Long-acting insulin should not be mixed with other types of insulin Rationale: Insulin is typically administered subcutaneously. Intramuscular injection can lead to unpredictable absorption rates and potential for hypoglycemia. Regular insulin is the only type of insulin that can be administered intravenously, which may be done in emergency situations such as diabetic ketoacidosis, where rapid control of blood glucose is needed. Regular rotation of insulin injection sites within the same general area (like the abdomen or thighs) is recommended to prevent lipohypertrophy, a thickening or lump under the skin caused by repeated injections at the same site, which can affect insulin absorption. Long-acting insulins (glargine and detemir) should not be mixed with other insulins in the same syringe due to their pH or potential for precipitation, which could affect their action.

A nurse has received orders to administer a RhoGAM injection IM to a postpartum client. Which situation is NOT a contraindication for the administration of this injection? A. Administration to a client who has a history of a systemic allergic reaction to preparations containing human immunoglobulins. B. Administration of the injection within 72 hours after delivery. C. Administration to an Rh-positive female client. D. Administration to a client with an elevated temperature.

Administration of the injection within 72 hours after delivery. Rationale: The injection should be given within 72 hours after birth. RhoGAM is administered intramuscularly within 72 hours after birth to prevent sensitization to the Rh factor in an Rh-negative woman with an Rh-positive infant. This injection will prevent hemolytic disease in subsequent pregnancies. Each vial of RhoGAM is cross-matched to a specific woman. The nurse must do all appropriate checks for client identification to avoid an error in administration.

The nurse is planning care for a child who has had a near drowning at a local swimming pool. It would be a priority for the nurse to have which item at the bedside? A. Warm isotonic fluids B. Advanced airway C. Rectal thermometer D. Defibrillator

Advanced airway Rationale: Drowning may cause a catastrophic pulmonary injury, and maintaining the client's airway to deliver warm, humidified oxygen is essential. While thermoregulation needs to be attained (drowning results in a decrease in body temperature), the essential item to have is airway equipment. Restoring a client's airway will prevent a serious complication of an anoxic brain injury.

The nurse prepares a client for a computed tomography (CT) scan of their abdomen and pelvis with intravenous (IV) contrast. The nurse should take which action prior to the client's exam? A. Remove any medicated patches before the exam B. Instruct the client to empty their bladder right before the test C. Advise the client that they may experience a flushing sensation during the exam D. Assess the client for an implantable pacemaker

Advise the client that they may experience a flushing sensation during the exam Rationale: Flushing of the face is a response to the intravenous administration of contrast dye commonly seen in clients. Often, clients experience a warm sensation throughout the body once the intravenous contrast dye begins infusing. Usually, if the client does experience this warm sensation, clients will typically report the sensation occurring initially in the face and neck region. Shortly after, clients will often state they feel the warmth in their pelvic area.

The nurse is assisting in the placement of an indwelling Foley catheter in a male client. The nurse knows to inflate the balloon on the catheter at which of the following points in the procedure? A. Upon meeting resistance B. As soon as urine is observed in the tubing C. After advancing to the point of bifurcation D. After fully advancing the length of the catheter

After advancing to the point of bifurcation Rationale: The nurse should inflate the balloon on the catheter once she reaches the point of bifurcation. This is achieved by slowly advancing the catheter, observing the tubing for urine to appear, and then continuing to advance to the point of bifurcation after urine is observed. This will ensure the balloon is in the bladder before the nurse inflates it.

The nurse is caring for a client with the following clinical data. Which medication would the nurse clarify with the primary healthcare provider (PHCP) before administration based on the vital signs? See the image below. A. Metoprolol 50 mg PO Daily B. Lisinopril 40 mg PO Daily C. Albuterol 2.5 mg via nebulizer Daily D. Diltiazem XR 120 mg PO Daily

Albuterol 2.5 mg via nebulizer Daily Rationale: The vital signs (VS) are all within normal limits except the pulse, which is 123 bpm, and the blood pressure is slightly elevated. This should cause the nurse to clarify the prescription of albuterol with the PHCP as this medication increases heart rate. This would foreseeably worsen the tachycardia that the client is already experiencing.

The nurse reviews prescriptions for assigned clients. Which prescription should the nurse clarify with the primary healthcare provider (PHCP)? A. Albuterol via nebulizer for a patient with hypokalemia. B. Clozapine for a patient with severe schizophrenia. C. Lisinopril for a patient with congestive heart failure. D. Verapamil for a patient with migraine headaches.

Albuterol via nebulizer for a patient with hypokalemia. Rationale: Albuterol is a bronchodilator that is used for asthma exacerbations. Adversely, this medication may lower serum potassium levels. The nurse should question this order as this medication may decrease the potassium further.

The nurse knows which of the following healthcare providers are responsible for documenting care provided to a client? A. The LPNs should document the care they provided and the care given by unlicensed assistive staff. B. The registered nurse must document all of the care that is provided by the nursing assistants because they are accountable for all care. C. All staff members should document all of the care that they have provided. D. All staff should document all of the care that they have provided, but the registered nurse is the only independent practitioner that signs it.

All staff members should document all of the care that they have provided. Rationale: All staff members, including unlicensed assistive staff like nursing assistants, document and sign all of the care that they have personally provided. For example, the nursing assistants will document the vital signs that they have taken, the licensed practical nurses will document all of the treatments and medications that they have given to the client, and the registered nurse will document nursing diagnoses and assessments that they have completed. There is an old saying among healthcare professionals that has been passed on to new generations. The saying is, "I don't care what you did; if you didn't document it, you didn't do it." Documentation is an essential part of client care.

The nurse is assisting a client of the Orthodox Jewish faith while serving lunch. A kosher meal has been delivered to the client. What is the next appropriate action to perform with this client? A. Substitute plastic utensils with metal utensils. B. Unwrap the eating utensils for the client. C. Carefully transfer the food from a styrofoam tray to a ceramic plate. D. Allow the client to unwrap the utensils and prepare their own meal.

Allow the client to unwrap the utensils and prepare their own meal. Rationale: A person of the Orthodox faith should be able to unwrap the utensils and prepare their own meal.

The licensed practical/vocational nurse (LPN/VN) is reinforcing teaching to a group of students about medications and fall prevention. The nurse would be correct to identify which of the following medications can increase the risk for falls? Select all that apply. Naproxen Alprazolam Bumetanide Verapamil Allopurinol Thiamine

Alprazolam Bumetanide Verapamil Rationale: Medications that may hasten the risk for falls include benzodiazepines such as alprazolam. This medication causes drowsiness and may impair judgment. Bumetanide is a loop diuretic; this medication may cause a client to experience orthostatic hypotension and the urgency to use the bathroom. Both of which pose a fall hazard. Verapamil is a calcium channel blocker and is utilized in the management of migraines and hypertension. This medication causes vasodilation; therefore, it will allow the client to become orthostatic if they do not shift positions slowly.

The licensed practical/vocational nurse (LPN/VN) collects data on a client with irritable bowel syndrome. Which of the following findings would be expected? A. Unexplained weight loss B. Epigastric pain and nausea C. Alternating constipation and diarrhea D. Low-grade fever and fatigue

Alternating constipation and diarrhea Rationale: Alternating constipation and diarrhea are the hallmark manifestations associated with irritable bowel syndrome (IBS). IBS is a functional gastrointestinal disorder that commonly presents with abdominal pain or discomfort associated with changes in bowel habits, including alternating periods of constipation and diarrhea. This is a key diagnostic feature of IBS.

The nurse receives a prescription for donepezil. The nurse understands that this medication is used to treat A. Guillain Barré syndrome B. Parkinson's disease C. Alzheimer's disease D. Meniere's disease

Alzheimer's disease Rationale: Alzheimer's disease is the most common form of dementia. Treatment options are limited but may include donepezil which is approved to treat mild, moderate, or severe Alzheimer's disease. This medication is an acetylcholinesterase Inhibitor.

The nurse is caring for a client with Helicobacter pylori. The nurse should anticipate a prescription for which of the following medications? A. Dicyclomine B. Metoclopramide C. Valacyclovir D. Amoxicillin

Amoxicillin Rationale: - Amoxicillin is an antibiotic that is commonly used to treat Helicobacter pylori infections. - Dicyclomine is an antispasmodic medication used to treat gastrointestinal spasms, which is common in individuals with irritable bowel syndrome. Metoclopramide is a medication that causes gastric emptying and is used for nausea and vomiting. Valacyclovir is an antiviral indicated for herpes infections.

The licensed practical/vocational nurse (LPN/VN) cares for the assigned clients. Which of the following clients would be appropriate for the nurse to recommend to the registered nurse (RN) for a referral for an interdisciplinary conference? Select all that apply. Pulmonary tuberculosis with multiple prescriptions An ischemic stroke who has left-sided hemiplegia Hyperthyroidism and is scheduled for a thyroidectomy Stage one Alzheimer's disease who lives with family A fractured tibia and fibula and is homeless End-stage-renal disease who refuses dialysis

An ischemic stroke who has left-sided hemiplegia A fractured tibia and fibula and is homeless Rationale: A client with an ischemic stroke with hemiplegia will require interdisciplinary care such as occupational and physical therapy. Further, the client may require subacute rehabilitation provided by nursing. A client with a fractured tibia and fibula will require physical therapy and social services consultation to assist the client with housing.

The nurse is collecting data on a client with schizophrenia. Which of the following would be an expected finding? Select all that apply. Apraxia Anhedonia Avolition Delusions Bradykinesia

Anhedonia Avolition Delusions Rationale: Clinical features of schizophrenia include positive and negative symptoms. Anhedonia (lack of pleasure), avolition (reduced motivation), and delusions (false, fixated belief) are all associated with this psychiatric disorder.

The nurse is reinforcing education to a client with glaucoma. Which of the following classifications of medications should the nurse instruct the client to avoid? A. Osmotic diuretics B. Beta-adrenergic blockers C. Anticholinergics D. Alpha 2-adrenergic blockers

Anticholinergics Rationale: Anticholinergic medications can increase the IOP and worsen clients' condition with glaucoma. Anticholinergic agents also can produce central side effects, such as confusion, unsteady gait, or drowsiness in adults. Children may become restless or spastic. Glaucoma is one of the leading causes of blindness in the United States. In some cases, it is genetic. In others, it may occur due to eye injury or disease. Some medications may contribute to glaucoma development, such as long-term use of topical glucocorticoids, antihypertensives (some), antihistamines, and antidepressants. The primary risk factor associated with glaucoma includes high blood pressure.

The nurse is caring for a client who has developed Malignant Hyperthermia. Which of the following actions should the nurse take? Select all that apply.

Apply a cooling blanket Insert indwelling urinary catheter Administer prescribed dantrolene Rationale: Malignant hyperthermia is a medical emergency and requires the nurse to intervene by applying a cooling blanket and ice to the axilla and groin. The nurse should also monitor the client's urinary output by inserting an indwelling urinary catheter. The nurse should be prepared to administer Dantrolene as this skeletal muscle relaxant is an effective treatment.

The nurse is caring for a client with the following clinical data. Based on the vital signs, which medications would the nurse clarify with the primary healthcare provider (PHCP) before administration? See the exhibit. Select all that apply.

Atenolol 50 mg PO Daily Spironolactone 50 mg PO Daily Fentanyl 50 mcg IV Push q 6 hours PRN Pain Rationale: The vital signs show hypotension (90/60 mm Hg). The nurse should clarify the prescriptions of atenolol, spironolactone, and fentanyl. All these medications decrease blood pressure, and considering how low the client's blood pressure is; it would be highly detrimental.

During a physical assessment, the nurse inspects the client's abdomen. What assessment technique would the nurse perform next? A. Percussion B. Palpation C. Auscultation D. Whichever is most comfortable for the patient

Auscultation

The nurse is caring for a 70-year-old client in a clinic who has complaints of perineal irritation due to frequent incontinence. Which of the following measures, suggested to the client by the nurse, are most appropriate? Select all that apply. Use extra-large briefs to provide for air movement. Apply a generous amount of barrier cream. Gently cleanse the perineum 2 to 3 times per day with warm water and pat dry. Apply Bacitracin cream to the perineum. Ambulate the client to the bathroom every two hours.

Apply a generous amount of barrier cream. Gently cleanse the perineum 2 to 3 times per day with warm water and pat dry. Rationale: Barrier cream is appropriate for preventing and managing perineal irritation because it creates a protective layer on the skin, helping to prevent contact with irritants such as urine and feces. This can reduce the risk of skin breakdown and discomfort. Proper perineal hygiene is crucial for maintaining skin integrity and preventing irritation in individuals with incontinence. Regular cleansing with warm water helps remove irritants and bacteria while patting the area dry minimizes friction and moisture, both of which can contribute to skin irritation.

The nurse is caring for a seven-year-old client brought to the clinic by her parents Item 4 of 6 The nurse creates a plan of care for the client Click to specify if each intervention is indicated or not indicated Apply hydrogen peroxide solution to the affected area (Indicated or Not Indicated) Educate the client on appropriate hand hygiene (Indicated or Not Indicated) Education on warm baths with baking soda for itching (Indicated or Not Indicated) Administration of prescribed acetaminophen (Indicated or Not Indicated) Collection of blood cultures (Indicated or Not Indicated)

Apply hydrogen peroxide solution to the affected area (Not Indicated) Educate the client on appropriate hand hygiene (Indicated) Education on warm baths with baking soda for itching (Indicated) Administration of prescribed acetaminophen (Indicated) Collection of blood cultures (Not Indicated) Rationale: Interventions that are indicated include appropriate hand hygiene. Frequent hand sanitation will decrease the likelihood of cellulitis if the skin is broken by itching. Educating on comfort measures such as warm baths with baking soda or oats is indicated to soothe itching. Acetaminophen would be helpful if the client has a fever.

Which of the following statements accurately describes behaviors that place juveniles at increased risk for injury? Select all that apply. Approximately 5000 individuals under the age of 21 die from alcohol-related accidents annually. 1 in 3 high school students report using some type of tobacco product. The CDC lists motor vehicle accidents as the number one cause of death among adolescents The use of OTC and prescription drugs among teens is at an all-time high. Homicide rates for adolescents are high. As many as 30% of school-aged children are bullied.

Approximately 5000 individuals under the age of 21 die from alcohol-related accidents annually. The CDC lists motor vehicle accidents as the number one cause of death among adolescents The use of OTC and prescription drugs among teens is at an all-time high. Rationale: Each year, underage drinking claims the lives of approximately 5,000 people under 21. The CDC lists motor vehicle accidents as the number one cause of adolescent death. Marijuana use among teenagers has been on the rise. The abuse of prescription medications and OTC drugs has remained at a high level. Much of the adolescent's time is spent away from home with their peer group. They are particularly at risk for motor vehicle accidents because they spend so much time in automobiles and are prone to distracted driving and driving under the influence of substances.

*NGN* The nurse is caring for a client in the mental health unit experiencing psychosis Item 1 of 1 Nurses' Notes 1300 - Client was banging their food tray and shouting at other clients. De-escalated the situation by escorting the client back to their room. Once back to their room, the client kept shouting 'they are after me!' at the roommate. The client stopped the shouting but resumed several minutes later. The client refused the scheduled by-mouth (PO) olanzapine, stating, 'they don't want me to take that.' Medications Olanzapine 10 mg PO Daily Medical History Schizophrenia Vitamin D deficiency Hyperlipidemia Select two (2) actions the nurse should take Provide therapeutic touch Limit interaction with the client Place the client in seclusion Ask if the client hears any voices Crush the olanzapine in the client's food Reassign the client to a private room

Ask if the client hears any voices Reassign the client to a private room Rationale: This client is experiencing overt paranoia, and the nurse should ask if the client hears any voices. Auditory hallucinations are the most common perceptual disturbance associated with schizophrenia and may contribute to a client's paranoia. If auditory hallucinations are evident, this allows the nurse to intervene by exploring the content of the hallucination and reorientating the client. Reassigning the client to a private room is appropriate. The client is in the mental health unit, and milieu therapy provides an organized, structured, and safe environment for all clients. The client's condition is disrupting this type of therapy, and for the safety of the other clients, the client should be reassigned to a private room close to the nursing station.

A primigravida client begins labor and is visibly upset that her family is unavailable. Which is the most appropriate approach for the nurse to take to help meet the client's needs at this time? A. Assure her that the nursing triage team will stay with her at all times. B. Encourage the client regarding her own abilities to cope and maintain a sense of control. C. Ask the client if there is someone else who wants to be her support person. D. Tell the client she will try to locate her family.

Ask the client if there is someone else who wants to be her support person. Rationale: Allow the client to select another individual to give support. This allows her to have someone with her until her family can be with her. Women and families have different expectations during childbearing. These expectations are shaped by their experiences, knowledge, belief systems, social, and family backgrounds. In most cases, a childbirth companion (or social support during birth) has improved the whole birth experience.

Which of the following parenting styles is highly controlling, expecting always to be obeyed, and inflexible with the rules? A. Authentic B. Permissive C. Authoritarian D. Indifferent

Authoritarian Rationale: The parenting style described is authoritarian. This parent is often described as the rigid disciplinarian. They are highly controlling, expect to always be obeyed, and are inflexible with the rules. Though these parents may have their child's best interests at heart, they do not support their growing autonomy. Instead, they expect to be obeyed without reason.

A client with a crush injury to her left arm calls the nurse's station and requests pain medication. An hour after administration, the client is still complaining of intense pain. What is the next nursing action? A. Ask the client to describe the pain in quality and intensity. B. Offer the client a distraction, such as a book or television. C. Tell the client she can have more medication in three hours. D. Tell the client that crush injury victims should expect intense pain.

Ask the client to describe the pain in quality and intensity. Rationale: A crush wound is a wound caused by a force that leads to compression or disruption of tissues. It is often associated with fractures. Usually, there is minimal to no break in the skin. While other external symptoms, such as bruising or edema, may be visible, nurses should also rely on subjective symptoms reported by the client. Unrelieved pain is an indication of a complication. Clients who experience a crush injury are at risk for developing compartment syndrome. Therefore, asking the client to be specific about the quality and intensity of pain is vital in re-evaluating the client's status.

A client has acute bone pain related to metastases of cancer. The best way for a hospice nurse to assess the client's level of pain is: A. Check vital signs after giving pain medication. B. Note observations about the client's behavior. C. Evaluate verbal and non-verbal actions. D. Ask the client to rate his pain on a scale from 0-10.

Ask the client to rate his pain on a scale from 0-10. Rationale: Only the client can report on their pain level; it is a subjective perception that should not be judged or dismissed. Asking him to rate his pain on a scale of 0-10 should be the guide for managing his care and pain relief. Three aspects regarding the definitions of pain have essential implications for nurses. First, pain is a physical and emotional experience, not all in the body or all in the mind. Second, pain is a response to actual or potential tissue damage, so laboratory or radiographic reports may not be abnormal despite the real pain. Finally, emotional responses to pain, such as anxiety, depression, anger, and demoralization, may be described in the place of pain.

The nurse has received the following prescriptions for newly admitted clients. It would be a priority for the nurse to administer which prescription? A. Aspirin to a client experiencing an acute myocardial infarction B. Lisinopril to a client with essential hypertension C. Risperidone to a client with schizophrenia D. Levodopa-carbidopa to a client with Parkinson's disease

Aspirin to a client experiencing an acute myocardial infarction Rationale: A client experiencing a myocardial infarction is an acute emergency that requires immediate intervention. The standard treatment includes (in no order) morphine, oxygen, nitroglycerin, and aspirin.

The LPN enters her 5 year old patients room and finds him lying on the floor. His fall was unwitnessed. What is her priority nursing action? A. File an incident report B. Assist the child back to bed C. Call for help D. Assess the child for any injuries

Assess the child for any injuries Rationale: The priority nursing action is to assess the patient!! Before doing anything else, the nurse needs to know if the child was injured during the fall. This will determine what her next steps are.

The nurse is caring for a client with Borderline Personality Disorder. Which of the following actions should the nurse take? Select all that apply. Assess the patient for suicide Encourage independent decision-making Establish therapeutic boundaries Refer the patient for therapy Encourage social relationships

Assess the patient for suicide Establish therapeutic boundaries Refer the patient for therapy Rationale: Individuals with Borderline Personality Disorder (BPD) often engage in self-harm/parasuicide behaviors in which the intent is not death. These gestures may be superficial cutting, etc. All clients should be assessed for suicide regardless of their diagnosis. Therapeutic boundaries should be established as a characteristic of this personality disorder is polarizing individuals and splitting. Referring the client for therapy is one of the cornerstone treatments for BPD.

The nurse is assessing a client who just returned from surgery. The nurse checks preoperative vital signs at 8:30 AM to compare them with the current vital signs at 10:30 AM. What action should the nurse take? See the exhibit. A. Assess the surgical wound B. Collect blood cultures C. Administer oxygen at 2 L/minute D. Encourage by-mouth (PO) fluids

Assess the surgical wound Rationale: The client's 10:30 AM vital signs show signs of shock. Considering this client is in the immediate postoperative period, the nurse should assess the surgical wound for signs of hemorrhage. The nurse should reinforce the dressing if this is the source of the bleeding. The nurse should notify the primary healthcare physician (PHCP) of the client's change in condition.

The nurse manager reviews client assignments. Which client assignment would be inappropriate for a licensed practical/vocational (LPN/VN) nurse? A. Obtaining an occult blood sample from a 15-year-old client with ulcerative colitis. B. Assessing a 35-year-old client newly admitted for chest pain. C. Reinforcing education to a 25-year-old first-time mother on how to properly care for her new baby. D. Providing pin care and data collecting on neurovascular status for a client in cervical traction.

Assessing a 35-year-old client newly admitted for chest pain. Rationale: LPN/VNs should be assigned the most stable client with a predictable outcome. A client newly admitted for chest pain would not be stable or predictable and, thus, require intervention.

The licensed practical/vocational (LPN/VN) nurse is caring for a client with cardiac arrest and assisting the registered nurse (RN) in identifying priorities and delegation. As the LPN/VN assists with assigning roles to other staff, this is demonstrating which leadership style? A. Authoritative B. Situational C. Democratic D. Laissez-faire

Authoritative Rationale: Authoritative leadership is demonstrated in this situation as the nurse assists with the delegation and provides directives to accomplish tasks. This is an appropriate leadership style in an emergency where one individual assigns tasks/roles to decrease confusion and chaos in an emergent situation.

While teaching a new graduate nurse about the psychosocial development of your 2-year old client, you educate her that your client is in which stage of development according to Erikson's stages of psychosocial development? A. Initiative vs. Guilt B. Autonomy vs. Shame and Doubt C. Industry vs. Inferiority D. Trust vs. Mistrust

Autonomy vs. Shame and Doubt Rationale: Autonomy vs. Shame and Doubt is the typical development stage for early childhood, which lasts from ages 2 to 3 years, so this is what the nurse would expect for her 2-year-old client. In Autonomy vs. Shame and Doubt, children seek to develop a sense of personal control over physical skills and a sense of independence. When they are successful, for example, with a task like toilet training, they feel independent, leading to a sense of autonomy. When they are not successful, they feel they are a failure, resulting in shame and self-doubt.

The nurse is caring for a 2-year-old client. Among Erickson's Stages of Development, which of the following development stages would the nurse expect the client to be in? A. Autonomy vs. Shame and Doubt B. Industry vs. Inferiority C. Trust vs. Mistrust D. Initiative vs. Guilt

Autonomy vs. Shame and Doubt Rationale: Autonomy vs. Shame and Doubt is the typical development stage for early childhood, which lasts from ages 2 to 3 years, so this is what the nurse would expect for her 2-year-old client. In Autonomy vs. Shame and Doubt, children seek to develop a sense of personal control over physical skills and independence knowledge. When they are successful, for example, with a task like toilet training, they feel independent, leading to a sense of autonomy. When they are not successful, they think they are a failure, resulting in shame and self-doubt

The nurse is validating a client's understanding of the management of low back pain. Which of the following statements should the nurse reinforce? A. Avoid bending at the waist and lifting heavy objects. B. Weight-bearing exercises are recommended. C. Increase your sun exposure and calcium intake. D. Lay on your stomach four times a day and flex your legs.

Avoid bending at the waist and lifting heavy objects. Rationale: Low back pain symptoms may be mitigated using activity restriction as well as medications such as nonsteroidal anti-inflammatory drugs (NSAIDs). Bending at the waist and lifting heavy objects would not be recommended for a client with low back pain.

The nurse is reinforcing teaching with the parents of a child diagnosed with eczema. Which of the following information should the nurse include? Select all that apply. Avoid harsh soaps and detergents. Wash the affected areas 4-5 times per day. Apply lotion immediately after bathing. Keep nails short. "Baths with hot water are preferred."

Avoid harsh soaps and detergents. Apply lotion immediately after bathing. Keep nails short. Rationale: It is appropriate education to teach your client to avoid using harsh soaps and detergents. This can irritate the skin and exacerbate eczema. Common irritants include soaps, detergents, perfumes, cosmetics, jewelry, and fragrances. You should educate the client to avoid these harsh chemicals. This is an appropriate education. Applying lotion immediately after a bath will help the client keep moisture over the affected areas and decrease the itching and irritation associated with eczema. Keeping nails short is a critical piece of education for a child diagnosed with eczema. Their skin will be very itchy, but we need to teach them how to control this itching and avoid irritation from scratching. We also don't want any papules to be broke open or compromise tissue integrity due to prolonged itching from sharp nails. Young children that aren't able to stop scratching may need to wear mittens.

The nurse is reinforcing teaching to a client about newly prescribed carbamazepine. Which of the following information should the nurse include? A. This medication will require weekly dosage adjustments B. Avoid taking this medication with grapefruit C. This medication may make your emotions more intense D. Take this medication if you feel like you are going to have a seizure

Avoid taking this medication with grapefruit Rationale: Carbamazepine is an antiepileptic drug, and mood stabilizer. This medication is indicated in the management of epilepsy and bipolar disorder. This medication should not be taken with grapefruit products because cytochrome P-450 3A4 (liver enzyme) is inhibited by the ingredients in grapefruit, leading to increased serum concentrations of carbamazepine.

Which advice is most appropriate for a client on neutropenic precautions who wants to learn ways to prevent infection? A. Only brush your teeth once a day or every other day. B. Avoid the use of tampons for menstrual periods. C. Do not let visitors within 10 feet. D. Wash hands after handling pets.

Avoid the use of tampons for menstrual periods. Rationale: Tampons may cause tears in the vagina that could lead to infection. Tampons can introduce harmful bacteria into the vaginal canal, especially during insertion and removal. This risk is significant for neutropenic clients, as their weakened immune system may not be able to fight off infections effectively.

*NGN* The clinic nurse is caring for a 38-year-old male Item 2 of 6 The nurse reviews the nurses' notes and vital signs Select the additional diagnostic data necessary to help interpret the findings Basic metabolic panel (BMP) (Necessary or Not Necessary) Lipid panel (Necessary or Not Necessary) Colonoscopy (Necessary or Not Necessary) Thyroid panel (Necessary or Not Necessary) Prostate-specific antigen (PSA) (Necessary or Not Necessary) 24-hour urine collection (Necessary or Not Necessary)

Basic metabolic panel (BMP) (Necessary) Lipid panel (Necessary) Colonoscopy (Not Necessary) Thyroid panel (Necessary) Prostate-specific antigen (PSA) (Not Necessary) 24-hour urine collection (Not Necessary) Rationale: The client has an elevated BMI, abdominal obesity, and acanthosis nigricans, suggesting some metabolic abnormalities. A BMP is appropriate and necessary to help potentially support the diagnosis of either diabetes, hyperlipidemia, or metabolic syndrome. A lipid panel is essential considering the client's choice of ultra-processed foods, which is a direct cause of hyperlipidemia. A thyroid panel is recommended for this client. The client's symptoms of constipation, decreased sexual drive, weight gain, and reduced activity tolerance could be explained by a dysfunction in the thyroid. Limited evidence may suggest that thyroid dysfunction is familial, but this could be another weak supporting factor that could help explain his symptoms.

Which actions are recommended guidelines when providing foot care for residents in a long-term care facility? Select all that apply. Bathe the feet thoroughly in mild soap and tepid water solution. Soak the feet in warm water and bath oil. Dry feet thoroughly, including the area between the toes. Use an alcohol rub if the feet are dry. Use an antifungal foot powder if necessary to prevent fungal infections. Cut the toenails at the lateral corners when trimming the nails.

Bathe the feet thoroughly in mild soap and tepid water solution. Dry feet thoroughly, including the area between the toes. Use an antifungal foot powder if necessary to prevent fungal infections.

When instructing a patient with type 1 diabetes about exercise guidelines, which of the following instructions are most appropriate for the nurse to give to the patient? Select all that apply. "Be sure to eat a simple carbohydrate snack before you exercise." "Do not administer insulin immediately before and after exercise." "It is best to eat a more complex carbohydrate before you exercise so that you don't bottom out." "You may want to leave an energy drink with electrolytes in your gym locker in case you need it." "It is smart to alert your gym that you have type 1 diabetes."

Be sure to eat a simple carb snack before you exercise Do not administer insulin immediately before and after exercise It is smart to alert your gym that you have type 1 diabetes Rationale: Eating a simple carbohydrate before exercise can help boost blood sugar. Patients should not administer insulin immediately before or after training, as this may cause blood sugar levels to drop too much. Notifying the gym that the patient has diabetes will be helpful in the event of an emergency. If the patient experiences an emergency like hypoglycemia, responders can be alerted and can take appropriate precautions.

Which form of therapy would most likely be used to treat a group of clients affected by phobias? A. Behavioral psychotherapy B. Cognitive-behavioral psychotherapy C. Psychoanalysis D. Cognitive psychotherapy

Behavioral psychotherapy Rationale: Behavioral psychotherapy is useful for clients who are adversely affected by phobias, addictive disorders, and substance-related disorders. Some of the techniques used with behavioral therapy include operant conditioning as put forth by Skinner, aversion therapy, desensitization therapy, modeling, as well as complementary and alternative stress management techniques.

The practical nurse is caring for a client who has a chloride level of 115 mEq/L(98-106 mEq/L). Which of the following orders would the nurse expect for this patient? Select all that apply. Bicarbonate IV normal saline IV lactated ringers Furosemide Carbonic anhydrase inhibitors

Bicarbonate IV lactated ringers Rationale: Bicarbonate is a medication commonly used to decrease the chloride level. This client has hyperchloremia, as their chloride level is 115 mEq/L, which is above the normal range of 96-108 mEq/L. It is, therefore, appropriate to administer bicarbonate to lower the chloride level in this client. IV lactated ringers solution is the appropriate choice for IV fluids in a client with hyperchloremia. Normal saline should be avoided to prevent increasing the chloride level further. Hydration is a very important component in treating hyperchloremia, so providing IVF for hydration is appropriate, it just needs to be the correct fluid.

The nurse cares for a client with polycystic kidney disease (PKD). Which of the following would indicate the client is achieving treatment goals? A. Blood Pressure 128/63 mmHg B. Creatinine 2.3 mg/dl C. Proteinuria D. Sodium 132 mEq/L

Blood Pressure 128/63 mmHg Rationale: Treatment goals for a client with polycystic kidney disease (PKD) include maintaining normotension, the glomerular filtration rate (GFR), and the prevention of sodium wasting, which is evidence of a decline in renal function. Hypertension is a cardinal finding in PKD, and if a client is achieving the treatment goals, they will maintain regulated blood pressure.

A 25-year-old is found unconscious with fever and a noticeable rash. Which of the following tests will most likely be a priority order? A. Blood sugar check B. CT scan C. Blood cultures D. Arterial blood gases

Blood cultures Rationale: Blood cultures would be ordered to investigate the source of fever and rash.

The nurse is caring for a child with cystic fibrosis at the clinic. The nurse anticipates that the primary healthcare provider (PHCP) will order which routine laboratory test? A. Blood glucose B. Total cholesterol C. 24-hour urine D. Blood cultures

Blood glucose Rationale: Diabetes mellitus is a common co-morbidity associated with cystic fibrosis (CF). The damage that CF may cause to the pancreas may induce diabetes. Thus, random blood glucose levels and quarterly hemoglobin A1C levels are commonly ordered throughout the course of the illness. A random blood glucose level greater than 200 mg/dL may suggest the presence of diabetes.

This nurse is caring for a client who is receiving prescribed hydralazine. Which of the following findings would indicate a therapeutic response? A. Blood pressure 130/70 mm Hg B. Pulse (P) 67/minute C. Total cholesterol 185 mg/dL D. aPTT 45 second

Blood pressure 130/70 mm Hg Rationale: Hydralazine is a vasodilator and is intended to treat hypertension. The client's blood pressure of 130/70 mm Hg is within normal limits and indicates a therapeutic effect.

A nurse is preparing the plan of care for a client with stage 2 ovarian cancer who is a Jehovah's witness. The client has been told that surgery is necessary. Taking into consideration the client's religious preferences in developing the plan of care, the nurse documents which of the following? A. Religious sacraments and traditions are unimportant. B. Medication administration is not allowed for this group. C. Surgery is strictly prohibited in this religious group. D. Blood transfusion or the administration of blood and blood products is forbidden for this group.

Blood transfusion or the administration of blood and blood products is forbidden for this group. Rationale: For Jehovah's witnesses, surgery is allowed, but the administration of blood and blood products is forbidden.

An 8-year-old boy diagnosed with hemophilia A is brought into the urgent care clinic for a prolonged episode of hematemesis. Which of the following describes this symptom? A. Bleeding into the joints B. Bleeding from the nose C. Dark, black, tar-like stools D. Bloody vomit

Bloody vomit Rationale: Hematemesis is bloody vomit. This symptom is common with hemophilia and can lead to severe complications if not treated promptly.

A 4-year-old boy is recovering from abdominal surgery at the pediatric unit. As the nurse caring for the child, which of the following activities do you recommend that he prioritize? A. Blowing bubbles B. Peek-a-boo C. Building blocks D. Playing with clay

Blowing bubbles Rationale: Letting the child blow bubbles will stimulate lung expansion, preventing respiratory problems arising from surgery. Following the abdominal surgery, respirations are not as efficient because anesthesia hampers it, and it hurts to breathe. Consequently, mucus builds up, and the lung may collapse fully or partially ( atelectasis). Pneumonia may follow. The collapsed lung may result in dyspnea and respiratory failure and complicate the post-operative recovery. Therefore, primary health care providers ( PHCP) order incentive spirometers to reduce the risk of respiratory problems after surgery. However, if the child is under the age of five or is unable to use the incentive spirometer for another reason, they should blow bubbles for two to three minutes every hour. Blowing bubbles will serve as an alternative to incentive spirometry in these children.

The nurse is collecting data on a client with Paget's disease. Which of the following would be an expected finding? A. Bone deformities B. Berry aneurysm C. Heberden's nodes D. Janeway lesions

Bone deformities Rationale: Paget's disease is a disease caused by a bone becoming weakened and remodeled, which may result in deformities. The most common area this inappropriate bone remodeling affects is the skull, pelvis, and spine.

When caring for an infant during cardiac arrest, which pulse must be palpated to determine cardiac function? A. Carotid B. Brachial C. Pedal D. Radial

Brachial Rationale: The brachial pulse is the most accessible pulse on an infant, thus it is the site of choice. Accurate assessment of heart rate, breathing, and color is an essential part of infant resuscitation. The guidelines state that heart rate may be assessed using a stethoscope or palpating the umbilical, brachial, or femoral pulse.

Which of the following are fine motor skills the nurse should expect a 5-month-old to have developed? Select all that apply. Palmar grasp Bringing objects up to their mouth Pincer grasp Transferring objects from hand to hand Plays with fingers in midline Opens and shuts hands

Bringing objects up to their mouth Plays with fingers in midline Opens and shuts hands Rationale: The ability to bring objects up to the mouth is an excellent motor skill developed by four months of age. The nurse would expect that the five-month-old infant can bring objects up to their mouth. If they have not yet met this milestone by five months of age, follow-up is warranted to evaluate the infant further. They may be missing other signs as well and need help, such as physical therapy. Playing with fingers in the midline is a fine motor skill that infants are expected to develop at four months of age. If the nurse does not observe this skill in the 5-month-old, they would need to follow up. Opening and shutting hands is another fine motor skill that the five months old should be able to demonstrate. This fine motor skill should be acquired by two months of age, so if the nurse does not see this ability in the 5-month-old, follow-up is required.

The nurse is reviewing dietary teaching with a client who has hypercalcemia. Which of the following foods are high in calcium and should be avoided in the dietary recommendations? Select all that apply. Broccoli 2% milk Whole wheat pasta Bananas Salmon

Broccoli 2% milk Salmon Rationale: Broccoli and milk is rich in calcium. It should therefore be avoided in clients with hypercalcemia. Fish such as sardines and salmon are also high in calcium and should be limited on a low calcium diet.

25-year-old female reports intermittent abdominal pain, bloating, and flatulence that has lasted for several months. Which of the following would the nurse tell the client to avoid? A. Fiber B. Broccoli C. Yogurt D. Simple carbohydrates

Broccoli Rationale: Broccoli forms gas in the stomach and should be avoided for this client.

Which of the following would the nurse consider to be a warning sign of the presence of physical abuse? Select all that apply. Upper respiratory infections Bruises and broken bones Unintended pregnancies Repetitive strain injuries Alcoholism Depression

Bruises and broken bones Unintended pregnancies Alcoholism Depression Rationale: Health issues related to domestic violence include physical injuries from the assault, such as bruises and broken bones. Families experiencing domestic violence have more unintended pregnancies, miscarriages, abortions, and low-birth-weight babies. Families experiencing domestic violence have higher rates of substance abuse and depression.

The nurse knows it is important to monitor a client's use of complementary and alternative medicine (CAM) because? A. Clients should be warned that most CAM therapies are potentially dangerous. B. Additional treatment may not be needed if the client is using CAM. C. CAM therapy could interact with prescription and over-the-counter medications. D. Most CAM therapies are essentially ineffective.

CAM therapy could interact with prescription and over-the-counter medications. Rationale: One key concept to remember when dealing with alternative therapies is that natural does not always mean "better or safe." Some herbal products contain ingredients that may interact with prescription drugs. For example, clients taking medications with potentially dangerous adverse effects, such as insulin, warfarin, or digoxin, should be warned to never take dietary supplements without first discussing their needs with a physician. Complementary and alternative medicine (CAM) comprises an incredibly diverse set of therapies and healing systems. CAM is considered to be outside the mainstream of healthcare. From a therapeutic perspective, much of the value of CAM therapies is their ability to reduce medication needs.

The nurse is reinforcing teaching to a client who is taking phenytoin. Which over-the-counter (OTC) medication should the nurse advise the client to avoid taking at the same time? A. Acetaminophen B. Ibuprofen C. Calcium carbonate D. Guaifenesin

Calcium carbonate Rationale: Calcium carbonate (Tums) should not be taken simultaneously with phenytoin because taking them together can decrease the effects of phenytoin. Antacids containing calcium carbonate reduce the bioavailability of phenytoin by reducing both the rate of absorption and the amount of intake. Phenytoin is an anticonvulsant, therefore not getting a therapeutic dose may result in the client having a recurrent seizure. Clients should be cautioned against the concomitant use of antacids containing calcium carbonate and phenytoin. If the client needs calcium carbonate, he should be instructed to separate the intake of calcium carbonate and phenytoin by at least two to three hours.

The LPN enters the room of a client with Tetralogy of Fallot. They immediately note that the infant is cyanotic and their SpO2 is 32%. Which of the following is the priority nursing action? A. Administer propranolol B. Administer sodium bicarbonate C. Calm the infant D. Notify the healthcare provider

Calm the infant Rationale: Immediately calming the infant is the nursing priority during a tet spell. While the infant is crying, their pulmonary vascular resistance is increasing leading to decreased oxygenated blood and more cyanosis. By calming them down you will immediately be decreasing their pulmonary vascular resistance so that blood can flow to the lungs and provide oxygen to the body. This is the first action that the nurse should take.

The nurse reviews clinical data for a client 24 hours postpartum following a vaginal delivery. Which of the following findings would require follow-up by the nurse? A. White blood cell count 14,000 mm3 B. BUN 18 mg/dL C. Capillary blood glucose 258 mg/dL D. Urinary output 60 mL/hr

Capillary blood glucose 258 mg/dL Rationale: This blood glucose is greater than 250 mg/dL and is clinical hyperglycemia. Regardless if the client has a history of diabetes mellitus, this CBG requires follow-up because it is the only abnormal clinical data.

The licensed practical/vocational nurse (LPN/VN) is collecting data on a client who just received one unit of packed red blood cells (PRBCs). Which of the following findings would indicate a therapeutic response? A. Bounding peripheral pulses B. Hematuria C. Oral temperature 100.4°F (38°C) D. Capillary refill < 3 seconds

Capillary refill < 3 seconds Rationale: A capillary refill of < 3 seconds is an optimal physical assessment finding and would be desired following the administration of PRBCs. PRBCs are indicated for clients with anemia (hemoglobin less than 7 g/dL). Giving a client, PRBCs improves tissue perfusion and may reflect the capillary refill finding.

Which of the following medications would the nurse expect to administer to her patient presenting with an intussusception? Select all that apply. Cefazolin Lactated Ringers Metoprolol Ranitidine Antiemetics Pain medication

Cefazolin Lactated Ringers Antiemetics Pain medication Rationale: Cefazolin is a broad-spectrum antibiotic commonly used before or after surgeries as a prophylactic antibiotic. It is a cephalosporin antibiotic. It is used when a surgical repair of intussusception is performed to prevent infection. Lactated Ringers is an isotonic crystalloid solution used for maintenance IV hydration in a patient with intussusception. Nausea and vomiting can be caused by several factors, including the mechanical obstruction of the bowel, irritation of the bowel lining, and stimulation of the vomiting center in the brain. Antiemetic medications work by blocking the signals in the brain that cause nausea and vomiting or by reducing the irritation of the bowel lining, which can help to reduce the symptoms. The pain associated with intussusception can be quite severe and can interfere with the client's ability to tolerate oral intake, participate in activities of daily living, and rest comfortably. Pain medications can help to relieve the discomfort associated with the condition, improving the client's overall comfort and well-being.

The nurse should expect to administer which of the following medications to the infant diagnosed with omphalocele? Select all that apply. Ceftriaxone D5W Albumin 25% Sodium bicarbonate Pain medicine

Ceftriaxone D5W Pain medicine Rationale: Ceftriaxone is a cephalosporin antibiotic. It is appropriate to use in an infant with omphalocele for prophylactic infection prevention. Infants with omphalocele are at increased risk for infection due to their intestines being exposed to the open air. D5W is an isotonic crystalloid solution that would be appropriate for IV maintenance fluid administration in an infant with an omphalocele. These infants will have higher insensible fluid losses due to evaporation from their exposed intestines so maintenance fluid will be a necessary part of their preoperative treatment. Surgery is the mainstay of treatment for omphalocele. After surgery, medications may be given to manage pain and prevent infection. Pain management options for infants can include acetaminophen or opioids, depending on the severity of the pain.

The nurse is caring for a seven-year-old client brought to the clinic by her parents Item 3 of 6 The nurse determines that if the client's itching is not controlled, which complication may develop? Cellulitis Pneumonia Encephalitis Desquamation

Cellulitis Rationale: The most common complication of varicella infections is secondary bacterial skin infections such as cellulitis. This is caused when bacteria enter from a break in the skin.

Which procedures necessitate the use of surgical asepsis techniques? Select all that apply. Intramuscular medication administration. Central line intravenous medication administration. Donning gloves in the operating room. Neonatal bathing. Foley catheter insertion. Emptying a urinary drainage bag.

Central line intravenous medication administration. Donning gloves in the operating room. Foley catheter insertion. Rationale: Surgical asepsis is used when managing central line intravenous medication administration, when donning sterile gloves in the operating room and when inserting an indwelling Foley catheter. Surgical asepsis, or sterile technique, refers to those practices that keep an area or object free of all microorganisms; it includes practices that destroy all microorganisms and spores. Surgical asepsis is used for all procedures involving the sterile areas of the body.

The nurse performs data collection on an older adult client and observes apnea alternates with periods of rapid breathing. The nurse should document this respiratory pattern as A. Cheyne-Stokes. B. Kussmaul's. C. agonal. D. tachypnea.

Cheyne-Stokes. Rationale: Cheyne-Stokes respirations are characterized when the respiratory rate and depth are irregular and alternate with periods of apnea and hyperventilation. The pattern then reverses, and the breathing slows and becomes shallow, concluding as apnea before respiration resumes. This may occur in older adults without any underlying pathology. This also could be characteristic of the client's critical condition following a neurological injury.

The nurse is anticipating a client arriving at the emergency department (ED) exposed to inhalation anthrax. The nurse anticipates that the primary healthcare provider (PHCP) will prescribe which medication? A. Acyclovir B. Zidovudine C. Ciprofloxacin D. Oseltamivir

Ciprofloxacin Rationale: Anthrax is a bacterial infection treated with antibiotics such as penicillin, doxycycline, and ciprofloxacin. Inhaled anthrax is most effectively treated with a combination of ciprofloxacin and another antibiotic (i.e. penicillin, clindamycin, chloramphenicol). Antibiotics are usually given for 60 days because it takes that long for spores to germinate. Acyclovir is an antiviral used to treat herpes. Antiviral medications do not affect anthrax, which is a bacterial infection. Zidovudine is an anti-retroviral medication that is used for the treatment of HIV. Antiviral drugs do not affect anthrax, which is a bacterial infection. Oseltamivir is an antiviral drug used to treat influenza. Antiviral medications do not affect anthrax, which is a bacterial infection.

The nurse is caring for a client who has just been diagnosed with peritonitis. Which of the following medications should the nurse anticipate the primary health care provider (PHCP) will prescribe? A. Pantoprazole B. Ciprofloxacin C. Lactulose D. Loperamide

Ciprofloxacin Rationale: Peritonitis is an intra-abdominal severe infection that has a significant mortality rate. Peritonitis may originate from perforation (appendix, intestine, etc.), which causes a significant amount of fluid and bacteria to shift into the peritoneum. The priority treatment in peritonitis is administering prescribed antibiotics such as ciprofloxacin, metronidazole, or ceftriaxone.

The nurse is caring for a child in the emergency department (ED) who sustained a bite by a rabid animal. The nurse should take which initial action? A. Assess the wound's length and width B. Cleanse the wound with soap and water C. Obtain a prescription for an antibiotic D. Report the bite to animal control

Cleanse the wound with soap and water Rationale: Cleansing the wound inflicted by a rabid animal with soap and water is essential to prevent a rabies infection. Aggressive wound cleaning minimizes the exposure to this infection which can be fatal without the appropriate cleaning and post-exposure prophylaxis.

*NGN* The nurse is caring for a 47-year-old male in the outpatient clinic Orders Discharge home Schedule a follow-up appointment in four weeks Sertraline 50 mg PO Daily Clonidine 0.1 mg PO Daily Zolpidem 5 mg PO, PRN insomnia The nurse reviews the orders and formulates a teaching plan for the newly prescribed medications For each medication, select the appropriate option for drug classification and client teaching that should be reinforced

Clonidine - alpha2-adrenergic agonist This medication may cause you to become dizzy or tired. Sertraline - selective serotonin reuptake inhibitor Diarrhea is a common side effect of this medication. Zolpidem - Hypnotic Do not take this medication with alcohol Rationale: - Clonidine is indicated in the treatment of hypertension. The medication may be administered as a pill or transdermal patch for seven days. It should not be abruptly discontinued because of the risk of rebound hypertension due to a catecholamine surge. Clonidine has a sedative effect, and the client should not take this medication with alcohol or while driving/performing tasks requiring a high degree of concentration. - Sertraline is a potent, selective serotonin reuptake inhibitor. Sertraline is indicated in treating anxiety, obsessive-compulsive, and depressive disorders. SSRIs typically cause gastrointestinal distress once they are started and may be lessened by taking the medication with food. - Zolpidem is a non-benzodiazepine sedative-hypnotic indicated in the treatment of insomnia. This medication should not be taken with alcohol or other CNS depressants because of the risk of respiratory depression.

As an LPN, you are reinforcing discharge instructions with the family of a 5-year-old child recently diagnosed with hemophilia B. Which clotting factor is the child missing? Select all that apply. Clotting factor VIII Clotting factor IX Clotting factor VII Clotting factor X Clotting factor XI

Clotting factor IX Rationale: Clotting factor IX: This is the correct option. Hemophilia B, also known as Christmas disease, is caused by a deficiency of clotting factor IX.

While working in the emergency department the nurse assesses a 3 day old infant brought in by his mother. She states "he is always so sweaty and hot, and just doesn't want to eat! I think something is wrong." The nurse is unable to palpate a femoral pulse, but notes +3 brachial pulses. Which congenital heart defect does the nurse suspect? A. Hypoplastic left heart syndrome B. Patent ductus arteriosus C. Transposition of the great arteries D. Coarctation of the aorta

Coarctation of the aorta Rationale: The nurse suspects that this infant has coarctation of the aorta. In this defect, there is a stricture in the aorta preventing blood flow out of the left ventricle. It usually occurs beyond the blood vessels that branch off to your upper body and before the blood vessels that lead to your lower body. So blood flow to the upper body is abundant, but hardly any can make it to the lower part of the body. Therefore, there are decreased lower extremity pulses and increased upper extremity pulses.

What interventions should the nurse anticipate when providing care for a patient who has undergone repair of an anorectal malformation? Select all that apply. Imodium administration Initiation of a high-calorie diet Colace administration Initiation of a high-fiber diet Monitor the client for signs of infection

Colace administration Initiation of a high-fiber diet Monitor the client for signs of infection Rationale: The nurse would expect to administer colace (Docusate) after the client has a repair of an anorectal malformation. Colace is a stool softener that will help to pull water into the intestines and soften the stool. This will make bowel movements easier for the client after surgery. Initiation of a high fiber diet after surgery for an anorectal malformation is an expected intervention. A high fiber diet will assist the client in passing stool more quickly, which is vital after repairing an anorectal malformation. Monitor the patient for signs of infection, such as fever, increased pain, or redness and swelling around the surgical site.

The nurse administered prescribed six units of regular insulin. Which data collection finding requires follow-up? A. Rapid, labored breathing B. Increase appetite C. Cold sweats D. Increased urination

Cold sweats Rationale: Regular insulin may be given subcutaneously and peaks within two to four hours after administration. The peak effects of the medication raise the client's risk for hypoglycemia. Cold sweats are a clinical feature of hypoglycemia.

Which of the following would the nurse expect to be administered to a newborn with respiratory distress syndrome (RDS)? A. Theophylline B. Colfosceril C. Dexamethasone D. Albuterol

Colfosceril Rationale: RDS (Respiratory Distress Syndrome) is a type of neonatal respiratory disease most often caused by a lack of surfactant in the lungs. The fetal lungs start making surfactant during the third trimester of pregnancy, or around 26 weeks gestation through labor and delivery. Surfactant coats the alveoli's insides, reducing the surface tension of fluid in the lungs and making the alveoli more stable. This keeps the lungs from collapsing when the newborn exhales.

When performing skin assessments on clients, nurses must pay attention to cleanliness, color, temperature, texture, moisture, sensation, vascularity, and lesions. Which guidelines should the nurse follow when performing these assessments? Select all that apply. Compare bilateral body parts for symmetry. Proceed in a toe-to-head systematic manner. Use standard terminology to report and record findings. Do not allow data from the nursing history to direct the assessment. Document only skin abnormalities on the patient record. Perform the appropriate skin assessment when risk factors are identified.

Compare bilateral body parts for symmetry. Use standard terminology to report and record findings. Perform the appropriate skin assessment when risk factors are identified. Rationale: When performing a skin assessment, the nurse should compare bilateral parts for symmetry, use standard terminology to report and record findings, and perform the appropriate skin assessment when risk factors are identified.

While caring for a 6-week-old child, the LPN measures the client's temperature as 38.7 degrees Celsius. Which diagnostic tests does the LPN anticipate the provider to order? Select all that apply. Complete blood count (CBC) Blood cultures Urinalysis X-ray of the chest Electrocardiogram (ECG)

Complete blood count (CBC) Blood cultures Urinalysis Rationale: Complete blood count (CBC): A high temperature in a 6-week-old child may prompt the provider to order a CBC to evaluate for signs of infection, such as an elevated white blood cell count. Blood cultures: Blood cultures may be ordered to identify the presence of bacteria or other pathogens in the bloodstream, especially if there are concerns about a systemic infection. Urinalysis: A urinalysis may be requested to assess for urinary tract infections (UTIs) as a potential cause of the fever in the 6-week-old child.

*NGN* The nurse is caring for a 29-year-old female in the physician's office Item 4 of 6 The nurse plans care for this client For each potential order, click to specify whether the potential order is indicated or not indicated for the client Complete blood count (CBC) (Indicated or Not Indicated) Cobalamin (Vitamin B12) level (Indicated or Not Indicated) Computed tomography (CT) scan of the head and neck (Indicated or Not Indicated) Urine analysis (Indicated or Not Indicated)

Complete blood count (CBC) (Indicated) Cobalamin (Vitamin B12) level (Indicated) Computed tomography (CT) scan of the head and neck (Not Indicated) Urine analysis (Not Indicated) Rationale: The nurse anticipates that the physician will order a complete blood count (CBC) and a cobalamin (Vitamin 12) level to confirm a pernicious anemia diagnosis. If the client does indeed have pernicious anemia, the CBC will show megaloblastic anemia. Specifically, the macrocytic red blood cells (increased mean corpuscular volume) and low reticulocyte count (immature red blood count). The cobalamin (Vitamin 12) level would be low. A head and neck CT scan is unnecessary or used in diagnosing pernicious anemia. This is also true for an unnecessary urine analysis

The nurse is in an elevator and observes two staff members discussing a client's condition. The nurse understands that this conversation may potentially violate which ethical principle? A. Beneficence B. Confidentiality C. Autonomy D. Veracity

Confidentiality Rationale: This conversation being observed by the nurse may violate the client's confidentiality. Conversations about a client's personal medical information (PMI) should be kept private and involve only those involved in the client's care. This is considered the right to know, which mandates that information be safeguarded and limited in how it is shared. This discussion observed by the nurse does not potentially violate these ethical principles. Beneficence refers to taking positive actions to help others over themselves. Autonomy refers to protecting and promoting client independence. Veracity is an ethical principle referring to telling the truth. Honesty supports a trusting nurse-client relationship.

Which of the following conditions may cause an increased cortisol level in a client? A. Addison's disease B. Congestive heart failure C. Renal failure D. Cushing's disease

Cushing's disease Rationale: Cushing's syndrome produces elevated cortisol levels. Cortisol is best known for helping support the body's natural "fight-or-flight" instinct in a crisis. It also plays a vital role in several other body functions, including managing the use of carbohydrates, fats, and proteins, regulating blood pressure, increasing blood sugar levels, controlling the sleep/wake cycle, and boosting energy to help manage stress and restore balance.

The nurse is setting up the room for a patient newly diagnosed with Celiac disease. She knows to place the patient on which of the following precautions? A. Droplet precautions B. Contact precautions C. Standard precautions D. Neutropenic precautions

Contact precautions Rationale: Celiac disease requires standard precautions. It is not an infectious disease and is not transmitted from person to person; therefore, there is no reason to initiate any additional precautions.

You are taking care of a 5-year-old girl on a pediatrics floor at the hospital. While engaging her in conversation, you note that she uses 4-5 words in complete sentences. She can tell you what color her stuffed animals are, and she tells you stories about what the stuffed animals have done today. Knowing the appropriate language development milestones, the nurse should do which of the following? A. Consult the speech-language pathologist for evaluation. B. Notify the health care provider. C. Continue with the assessment. D. Engage the child's mother with questions about how the child communicates at home.

Continue with the assessment Rationale: The nurse should continue with her assessment. The nurse has observed several milestones of language development that are normal for a 5-year-old. She should take note of this and continue to assess the child. Other language development milestones that she would expect include: a vocabulary of about 2,100 words, correctly naming objects and people, and knowing their own name and address.

You are caring for an elderly woman who is a practicing Orthodox Judaism. Which meal would you most likely offer this client? A. Cottage cheese and fruit B. Beef lasagna C. Hamburger and milk D. Pork cutlet parmigiana

Cottage cheese and fruit Rationale: You would offer this client a meal consisting of cottage cheese and fruit because Orthodox Jewish people are not permitted to have dairy products and meat in one meal.

Which of the following interventions should the nurse anticipate for an infant with an omphalocele awaiting surgical repair? Select all that apply. Cover the intestines with a sterile gauze soaked in saline Prone positioning Radiant warmer for thermoregulation Trophic feedings Assessing the infant's bowel function

Cover the intestines with a sterile gauze soaked in saline Radiant warmer for thermoregulation Assessing the infant's bowel function Rationale: It would be appropriate to cover the intestines with sterile gauze soaked in saline. This will do two things: it will keep the intestines moist, and it will prevent infection. It is a priority nursing intervention to prevent infection in these clients because their abdominal contents are exposed to the environment putting them at risk of infection. It is also a priority to keep the intestines from drying out, as this will severely decrease their ability to function after the surgical repair. Using a radiant warmer for thermoregulation is appropriate. These infants will struggle to maintain their body temperatures, as their intestines are open to the air. Additionally, they cannot be tightly swaddled as this would put pressure on the exposed intestines. Using a radiant warmer can help with thermoregulation in these infants without compromising the intestines. Assessing the infant's bowel function, including the frequency and consistency of stools, is an important part of preoperative care.

The nurse is caring for a client who experienced a myocardial infarction (MI) 24 hours ago. It would be necessary for the nurse to immediately notify the primary health care provider (PHCP) if the client has which of the following? A. An elevated troponin level B. A white blood cell (WBC) count of 13,000 mm3 C. Apprehension about attending cardiac rehabilitation D. Crackles auscultated to the midline of the lung fields

Crackles auscultated to the midline of the lung fields Rationale: Following a myocardial infarction (MI), the client is at risk for developing pulmonary edema. This is caused by the heart's inability to eject blood, consequently caused by an insult to the myocardium. When caring for a client with an MI, the nurse should monitor the client for life-threatening ventricular arrhythmias as well as pulmonary edema. If a client is experiencing this complication, they will develop crackles in the lung fields, tachypnea, and hypoxia.

The nurse is caring for a client diagnosed with pernicious anemia. The nurse should anticipate a prescription for which medication? A. Thiamine B. Cyanocobalamin C. Iron dextran D. Folic acid

Cyanocobalamin Rationale: Pernicious anemia is characterized by the inability of the body to utilize Vitamin B12. This results in a decrease in hemoglobin, giving the patient anemia. The nurse should anticipate a prescription for cyanocobalamin, which may be administered parenterally for the greatest benefit.

The nurse is caring for a client exhibiting signs of poor muscle coordination, stooped posture, and slow movements. Which medication is most likely to cause these symptoms?

Haloperidol Rationale: Haloperidol is a typical antipsychotic that may adversely cause extrapyramidal side effects (EPS). These effects include akathisia, dystonia, pseudo-parkinsonism, and/or tardive dyskinesia. Tardive dyskinesia is an adverse effect that occurs with antipsychotics and has an onset of months to years while on the medication.

The nurse is working with a client who suffers from obsessive-compulsive disorder (OCD). The client has an obsession with the dangers of germs and performs compulsive hand washing hundreds of times per day. Which of the following should the nurse include in this client's treatment plan? Select all that apply. Create a schedule for the hand washing ritual. Teach about the dangers of over washing their hands. Incorporate meditation into their daily schedule. Block the sink so the client is not tempted to wash their hands. Administer fluoxetine as ordered.

Create a schedule for the hand washing ritual. Incorporate meditation into their daily schedule. Administer fluoxetine as ordered. Rationale: Creating a schedule is one of the most critical aspects of treatment for clients with OCD. In this schedule, it is essential to allow time for the ritual - not allowing time for the compulsive activity will dramatically increase anxiety. Adding time for mediation into the daily schedule is an appropriate intervention. Meditation is an excellent coping mechanism and can be added to replace some of the handwashing. Selective serotonin reuptake inhibitor (SSRI) medications are commonly used to help clients manage compulsive behaviors. Examples of SSRIs include fluoxetine, fluvoxamine, and sertraline.

The nurse is caring for a client who has sickle cell disease (SCD). Which of the following laboratory findings would require follow-up? A. Hemoglobin 11.2 mg/dL B. Creatinine 2.5 mg/dL C. BUN 19 mg/dL D. Platelet count 150,000 mm3

Creatinine 2.5 mg/dL Rationale: One of the many complications associated with sickle cell disease is renal injury. The significantly elevated creatinine requires follow-up because this is evidence of significant renal insufficiency.

The nurse is reviewing the laboratory results of assigned clients. Which of the following results would require immediate follow-up? Select all that apply. Creatinine 2.7 mg/dl for a patient receiving vancomycin. Hemoglobin A1C of 6.9% for a patient with diabetes mellitus. Platelet count of 152,000 mm3 for a patient receiving methotrexate. Potassium 3.1 mEq/dl for a patient receiving bumetanide. Calcium 10.8 mg/dl for a patient receiving hydrochlorothiazide.

Creatinine 2.7 mg/dl for a patient receiving vancomycin. Potassium 3.1 mEq/dl for a patient receiving bumetanide. Calcium 10.8 mg/dl for a patient receiving hydrochlorothiazide. Rationale: Vancomycin is a nephrotoxic antibiotic, and the nurse must monitor the client's creatinine. This creatinine is elevated (normal 0.6-1.2 mg/dl). A client receiving the loop diuretic, bumetanide, must be monitored for hypokalemia (normal 3.5 - 5.0 mEq/dl). Finally, the client receiving hydrochlorothiazide has elevated calcium which adversely occurs with this medication (normal 9.0 - 10.5 mg/dl).

he nurse is caring for a client with polycystic kidney disease (PKD). The nurse should prioritize which of these lab results to determine renal function in a client with a background of PKD? A. BUN 90 mg/dL [Both Sexes: 10-20 mg/dL, 3.6 to 7.1 mmol/L] B. Serum potassium 7.0 MEq/L [Both Sexes: 3.5-5 mEq/L] C. Uric acid 7.5 [3.4-7.0mg/dL for men & 2.4-6.0 mg/dL for women] D. Creatinine 8.7 mg/dL [Male: 0.6-1.2 mg/dL Female: 0.5-1.1 mg/dL, Male: 53-106 μmol/L, Female:44-97 μmol/L]

Creatinine 8.7 mg/dL [Male: 0.6-1.2 mg/dL Female: 0.5-1.1 mg/dL, Male: 53-106 μmol/L, Female:44-97 μmol/L] Rationale: Creatinine is a specific indicator of renal function/failure. Polycystic kidney disease (PKD) is a genetic disorder that causes fluid-filled cysts to grow inside the kidneys. Unlike simple kidney cysts that may develop later in life, PKD cysts can change the shape of organs and alter their function. Several tests can evaluate renal functioning. A creatinine level of 8.7 mg/dL is significantly higher than the normal reference range for both sexes. Creatinine is a waste product that is filtered by the kidneys, and elevated levels indicate reduced kidney function and impaired Glomerular Filtration Rate (GFR). This result is a strong indicator of kidney dysfunction in the context of PKD.

The nurse is conducting a health screening at a local health fair. Which of the following should the nurse recognize as a risk factor for developing testicular cancer? Select all that apply. Cryptorchidism Human immunodeficiency virus (HIV) Vasectomy Family history Herpes simplex virus (HSV)

Cryptorchidism Human immunodeficiency virus (HIV) Family history Rationale: Risk factors for testicular cancer include cryptorchidism, human immunodeficiency virus (HIV), and family history. Cryptorchidism refers to an undescended testicle where the testicle fails to descend to its normal position in the scrotum. Undescended testicles are associated with decreased fertility, testicular torsion, inguinal hernias, and an increased risk of testicular germ cell tumors. HIV-positive men have an increased risk of developing testicular cancer. A family history of testicular cancer is another risk factor, with an 8-10 times increased risk if the man has a sibling with testicular cancer.

The nurse is assisting a client with diverticulosis to select appropriate foods. Which foods should be avoided? Select all that apply. Bran Fresh peaches Cucumber salad Cooked broccoli Cabbages

Cucumber salad Cooked broccoli Cabbages Rationale: The client with diverticulitis should avoid foods with seeds. Additionally, they should avoid eating gas-forming foods that increase abdominal discomfort. When the colon must repeatedly move highly compacted fecal material, the longitudinal and circular muscles enlarge over time. This increases the force on the mucosal tissues, causing them to "balloon" out between the muscles and form pouches in which fecal matter becomes trapped. The development of these outpouchings is called diverticulosis. In some cases, these pouches become infected, causing a condition called diverticulitis. People whose diets are low in fiber or consist mainly of refined foods are at high risk for developing diverticulosis. Education about appropriate nutrition is a critical nursing intervention to help patients manage the disease.

The practical nurse cares for a patient with hypokalemia and accelerated hypertension. The physician has listed the cause as hyperaldosteronism. Which of the following endocrine disorders causes increased aldosterone? Select all that apply. Cushing's disease Addison's disease Conn's syndrome Pheochromocytoma Crohn's disease

Cushing's disease Conn's syndrome Rationale: Cushing's disease results from increased secretion of adrenocorticotropic hormone (ACTH) by the pituitary gland, which stimulates the adrenal cortex, leading to elevated cortisol and aldosterone levels. Symptoms include abdominal obesity, moon facies, neck hump, abdominal striae, high blood glucose, secondary diabetes, hypertension, hypokalemia, osteoporosis, and increased risk of infections. Clinical symptoms include abdominal obesity, moon facies, neck hump, abdominal striae, increased blood glucose, secondary diabetes, hypertension, and hypokalemia.

When a nursing student asks a nurse on her assigned floor what cyanosis means, what is the nurse's best response? A. Cyanosis means the client has been exposed to cyanide poisoning. B. Cyanosis is the blue coloring of the skin and mucous membranes in the presence of poorly oxygenated blood. C. Cyanosis is the primary indication that the client has pneumonia. D. Cyanosis is the blue coloring of the skin and mucous membranes in the presence of highly oxygenated blood.

Cyanosis is the blue coloring of the skin and mucous membranes in the presence of poorly oxygenated blood. Rationale: Cyanosis is the bluish discoloration of the skin and mucous membranes caused by decreased peripheral circulation or reduced oxygenation of the blood. It may be related to cardiac, pulmonary, or peripheral vascular problems (e.g. arteriosclerosis). In dark-skinned clients, you can best see cyanosis by examining the conjunctiva, tongue, buccal mucosa, palms, and soles for a dull dark color.

*NGN* The emergency department (ED) nurse is caring for a 42-year-old reporting flank pain Item 2 of 6 The nurse is analyzing the client's findings For each finding below, click to specify if the findings are consistent with the disease process of cystitis or urolithiasis. Each finding may support more than one disease process. Note: Each column must have at least one response option selected

Cystitis: dysuria and increased urinary frequency Nephrolithiasis: Dysuria, flank pain, increased urinary frequency, and nausea and vomiting

A client with a history of congestive heart failure is admitted with acute exacerbation. While collecting data on the client's vital signs which finding would indicate a potential complication of heart failure? A. Decrease in blood pressure B. Increase in temperature C. Decrease in respiratory rate D. Increase in blood pressure

Decrease in blood pressure Rationale: Congestive heart failure (CHF) is characterized by the heart's inability to pump sufficient blood to meet the body's demands. It can result in fluid overload and increased pressure in the blood vessels. A potential complication in a client with CHF is decreased cardiac output, which can lead to decreased blood pressure. Initially, the blood pressure may increase. However, a complication is decompensation which may cause the compensatory mechanisms to fail.

The nurse is collecting data on a client with hypothyroidism. Which of the following findings would be expected? Select all that apply. Decreased libido Bradycardia Heat intolerance Fatigue Constipation

Decreased libido Bradycardia Fatigue Constipation Rationale: Hypothyroidism causes an overall slowing of metabolic processes that contribute to clients developing constipation, fatigue, weight gain, bradycardia, periorbital edema, and decreased libido.

This nurse is caring for a client who is receiving prescribed sitagliptin. The nurse understands that this medication is intended to treat which condition? A. Hyperlipidemia B. Diabetes mellitus C. Hypothyroidism D. Hypertension

Diabetes mellitus Rationale: Sitagliptin is a DPP-4 Inhibitor used in managing diabetes mellitus type II. This medication reduces blood glucose levels by delaying gastric emptying and slowing the rate of nutrient absorption into the blood.

The nurse should recognize which of the following common physiological signs of aging. Select all that apply.

Decreased metabolism Reduced muscle mass and strength Changes in bone density Rationale: Decreased metabolism: As people age, their metabolic rate tends to decrease. This can result in a gradual decrease in energy levels and a potential increase in weight if dietary and exercise habits are not adjusted accordingly. Reduced muscle mass and strength: With aging, there is a gradual loss of muscle mass and strength, known as sarcopenia. This can lead to decreased mobility, balance issues, and an increased risk of falls. Changes in bone density: Bones tend to become less dense and more fragile with age, leading to an increased risk of fractures. Conditions like osteoporosis are more prevalent in older adults, especially postmenopausal women.

The nurse is caring for a client who is receiving prescribed ketorolac. Which of the following findings would indicate a therapeutic response? Select all that apply. Decreased pain Increased urinary output Decreased blood pressure Decreased temperature Increased muscle coordination

Decreased pain Decreased temperature Rationale: Ketorolac is a medication used to treat pain and pyrexia. A client exhibiting a decrease in pain and having a reduction in temperature would be a therapeutic response.

he nurse is caring for a client who is receiving prescribed methylergonovine. Which of the following findings would indicate a therapeutic response? A. Increased blood pressure B. Decreased post-partum bleeding C. Decreased uterine tone D. Increased urinary output

Decreased post-partum bleeding Rationale: Methylergonovine is an alkaloid medication used to manage postpartum hemorrhage (PPH). This medication causes vasoconstriction, therefore, decreasing postpartum bleeding.

The nurse is caring for a client with end-stage renal disease who is receiving prescribed sevelamer. Which of the following findings would indicate a therapeutic response? A. Decreased serum calcium levels B. Increased hemoglobin and hematocrit C. Decreased serum potassium levels D. Decreased serum phosphorus levels

Decreased serum phosphorus levels Rationale: Sevelamer is a phosphate binder indicated in the treatment of hyperphosphatemia associated with chronic kidney disease. This medication is purported to decrease serum phosphorus levels by binding to food. Thus, this medication is given with meals. Combined with a low phosphorus diet, the goal of this medication is to decrease serum phosphate levels.

The LPNs client assignment includes a fresh post-op cholecystectomy client. Which of the following interventions are appropriate to decrease the risk of atelectasis? Select all that apply. Deep inspiration Supine position with the head end of the bed elevated Change position every 2 hours Encourage the patient to cough at least 10 times/hr Promote early ambulation

Deep inspiration Supine position with the head end of the bed elevated Change position every 2 hours Encourage the patient to cough at least 10 times/hr Promote early ambulation Rationale: Deep inspirations and the use of incentive spirometry decrease the risk of atelectasis which is a common complication of the post-op period. The supine position with the head of the bed elevated allows for maximum thoracic expansion by lowering the abdominal pressure on the diaphragm. Changing client positions every 1-2 hours increases mobility allowing full chest expansion and increased perfusion to the lungs. Encouraging the client to deep breathe at least 10 times per hour when awake promotes alveolar expansion. Early mobility and ambulation promote breathing and increased perfusion to the lungs.

Which of the following steps is the final step used during the physical assessment of the abdomen?

Deep palpation Rationale: Deep palpation is cautiously done after light palpation when necessary because the client's responses to deep palpation may include tightening of the abdominal muscles. When this occurs, it could make light palpation less effective, particularly if an area of pain or tenderness has been palpated. A complete health assessment may be conducted starting at the head and proceeding systematically downward (head-to-toe evaluation). However, the procedure can vary according to the individual's age, the severity of the illness, the preferences of the nurse, the location of the examination, and the hospital's priorities and procedures.

The nurse is reinforcing teaching to staff about the use of warfarin. The nurse uses which medical conditions as examples of diagnosis that may lead to the use of warfarin. Select all that apply.

Deep vein thrombosis (DVT) Atrial fibrillation (AFib) Ischemic stroke Rationale: Deep vein thrombosis (DVT). This is a correct medical condition for which warfarin is commonly prescribed. Warfarin often prevents and treats blood clots, including deep vein thrombosis. It does not dissolve the clot. Atrial fibrillation (A-Fib). This is a correct medical condition for which warfarin is frequently prescribed. Warfarin helps prevent blood clots in individuals with atrial fibrillation, characterized by irregular heart rhythm. Ischemic stroke. This is a correct medical condition for which warfarin may be prescribed. Warfarin is commonly used as secondary prevention in individuals with an ischemic stroke to reduce the risk of recurrent clotting events.

The nurse is caring for a 4-year-old in respiratory distress. Which of the following complications should the nurse monitor for that frequently occur with respiratory distress? A. Ectopy B. Irritability C. Sepsis D. Dehydration

Dehydration Rationale: Dehydration is a frequent complication of respiratory distress for which the nurse must know to monitor. Tachypnea (rapid breathing) is often seen in children with respiratory distress. Additionally, mouth breathing is common in children due to nasal congestion, edema, and inflammation. As these children exhale more and more frequently, significant insensible fluid losses occur. Since children with respiratory distress do not take enough fluids by mouth, their intake is rarely enough to keep up with their insensible losses. Therefore, dehydration frequently occurs in a child experiencing respiratory distress. If the child is receiving humidified oxygen, insensible losses from tachypnea are minimal. Otherwise, it's important that the maintenance fluids include an additional 20-50% as a respiratory replacement in a child with tachypnea.

Which of the following members of the intradisciplinary team should be consulted for an infant suspected of having Celiac disease? A. Pharmacist B. Pulmonologist C. Occupational therapist D. Dietician

Dietician Rationale: Consulting with a dietician is of the utmost importance for the patient who is suspected of having Celiac disease. The dietician is the expert in this area and will provide support, education, and a dietary plan for this patient. Learning to avoid gluten can be difficult for the family, and the dietician is the best resource to help them navigate this.

The nurse is assessing a client with dependent personality disorder. Which of the following would be an expected finding? Select all that apply. Difficulty with decision-making Flamboyant behaviors Intense and unstable relationships Avoiding social relationships Feels helpless when alone

Difficulty with decision-making Feels helpless when alone Rationale: A dependent personality disorder is characterized by difficulty with making decisions, problems with expressing disagreement, and the individual often feels helpless when alone. Many individuals who are around a person with this disorder, often feel burdened because of the difficulty with trivial decision-making. The nurse should encourage decision-making but never make decisions for the client. Psychotherapy is the preferred treatment for all personality disorders.

The physician has ordered a 24-hour urine specimen. After explaining the procedure to the client, the nurse collects the first specimen. This specimen is then: A. Placed in a separate container and later added to the collection. B. Discarded, then the collection process begins. C. Tested, then discarded. D. Saved as part of the 24-hour collection.

Discarded, then the collection process begins. Rationale: The client should collect the first specimen, which is considered "old urine" or urine in the bladder before the test began. After the first discarded specimen, urine is collected for 24 hours. A 24-hour urine collection may be prescribed to evaluate some renal disorders by showing kidney function at different times of the day and night. The nurse is responsible for providing the collection container and educating the client on how to collect the specimen.

The client using over-the-counter nasal decongestant drops reports unrelieved and worsening nasal congestion. What is the appropriate instruction for this client? A. Discontinue the medication for several days. B. Use a combination of oral medications and drops for better results. C. Switch to a stronger dose of the decongestant drops. D. Increase the frequency of the nasal decongestant drops.

Discontinue the medication for several days. Rationale: Rebound congestion

After administering an insulin injection to a client on a sliding scale, the nurse realizes that the dose given was too high by mistake. Which of the following would be the best response by the charge nurse to prevent future errors? A. Discuss events preceding the error with the nurse. B. Complete an incident report and place it in the client's chart. C. Inform the client, family, and physician of the error. D. Monitor the client for adverse effects.

Discuss events preceding the error with the nurse. Rationale: Events preceding the error should be discussed with the nurse. This is the only response that focuses on preventing future errors. It would be most important to determine factors that contributed to the error, such as rushing, lack of knowledge/education, improper staffing caseload/client acuity, or communication issues.

Which of the following infection control activities should be delegated to an experienced nursing assistant? A. Asking clients about the duration of antibiotic therapy. B. Demonstrating correct handwashing techniques to the client and family. C. Disinfecting blood pressure cuffs after clients are discharged. D. Screening clients for upper respiratory tract symptoms.

Disinfecting blood pressure cuffs after clients are discharged. Rationale: Nursing assistants can follow agency protocol to disinfect items that come in contact with intact skin by cleaning with chemicals such as alcohol. Nurses must know their own scope of practice and the scope of practice of the UAP, which may vary depending on a facility's policies and procedures. Thus, the nurse must know the employer's policies and procedures for delegation, the UAP's job description, and the UAP's skill level.

The nurse reinforces teaching to parents planning to tell their children about their divorce. To promote effective coping by the children, the nurse should teach the parents to do which of the following? Select all that apply. Do not avoid telling the children about the divorce Provide physical and emotional reassurance to the children Try not to cry in front of the children Limit the amount of time discussing the divorce Children feeling anger or resentment is expected

Do not avoid telling the children about the divorce Provide physical and emotional reassurance to the children Children feeling anger or resentment is expected Rationale: Divorce or parental separation is traumatic for all ages, including adolescents. It is highly recommended that telling children and adolescents about divorce be planned and best delivered by both partners simultaneously. This is an ideal setup because it establishes a unified front of support at a time when the child or adolescent may feel lost. The parents should not avoid the discussion of divorce because it is unsettling for the child to have a sense of confusion or ambiguity regarding the parental arrangement. Addressing the divorce with the children clearly and empathetically is best. Providing physical and emotional reassurance, such as holding the child's hand, offers comfort. It is expected that children may react negatively with a sense of resentment. This resentment can be ameliorated by keeping the channels of communication open and conveying both physical and emotional support.

Questions to ask for suicidal ideations:

Do you have a plan? What is your plan? Do you have the means or methods to carry out your plan? Have you attempted suicide previously?

The nurse is assisting a client using a fracture bedpan. Which action should the nurse take? A. Position the client prone while applying the bed pan B. Raise the head-of-bed to 30 degrees C. Place the open rim of the bedpan facing toward the head of the bed D. Lower all of the side rails

Raise the head-of-bed to 30 degrees Rationale: Placing the head-of-bed at 30 to 60 degrees will facilitate comfort by preventing strain on the lumbar spinal column.

The nurse is caring for a client receiving intermittent bolus tube feeds. The nurse prepares to begin the next feed and aspirates 85 mL of residual from the nasogastric tube. Which of the following actions are appropriate? Select all that apply. Document the color, odor, consistency, and amount of the residual. Hold the next feeding. Send the residual to the lab. Administer the residual back to the client. Administer the feeding, but subtract the quantity of the residual from the feed.

Document the color, odor, consistency, and amount of the residual. Administer the residual back to the client. Rationale: This is the appropriate documentation for any fluids that the nurse observes from the client. You can remember this documentation from the mnemonic COCA: color, odor, consistency, and amount. For example, in this situation, it may be tan, formula-like residual, non-odorous, thick, 85 mL. This documentation will help identify what is normal for the client, and when there are any issues. The nurse should administer the residual back to the client. This is incredibly important for the prevention of electrolyte imbalances. The stomach contents are rich in many electrolytes, such as potassium and chloride. Removing the contents of the stomach and not returning them to the client could create dangerous electrolyte abnormalities.

Oral iron supplementation for severe iron deficiency anemia - appropriate action if a dark black stool is noted in the diaper:

Document the finding Continue with your assessment Administer the oral supplement as prescribed Rationale: Documenting this finding in the chart is an appropriate nursing action, but no further action is needed. It is an appropriate nursing action to continue with your assessment. Since the finding is expected, no other steps are necessary. It is an appropriate nursing action to administer the oral iron supplement as prescribed.

The nurse is caring for a client who is experiencing early decelerations. Which of the following actions should the nurse take?

Document the findings Rationale: Early decelerations are a reassuring finding and are caused by head compression, which is a normal part of labor.

The nurse is caring for a client diagnosed with Lyme disease. The nurse anticipates the primary healthcare provider (PHCP) prescribe which medication? A. Doxycycline B. Enalapril C. Simvastatin D. Famotidine

Doxycycline Rationale: Doxycycline is an effective treatment for Lyme disease. Lyme disease is an infectious disease caused by the Borrelia bacterium, spread by ticks. The most common sign of infection is an expanding area of redness on the skin, known as erythema migrans, that appears at the tick bite site about a week after it occurred. The rash is typically neither itchy nor painful. The rash is classically referred to as a bullseye rash.

The nurse cares for a client diagnosed with pelvic inflammatory disease (PID). The nurse anticipates the primary healthcare provider (PHCP) to prescribe which medication?

Doxycycline Rationale: Pelvic inflammatory disease (PID) is most likely caused by sexually transmitted infections or bacterial vaginosis. Doxycycline is an effective antibiotic utilized in PID.

The practical nurse is assisting in the monitoring of a client with a chest tube and notes each of the following findings. Which of the following are expected findings with the care of a chest tube? Select all that apply. Drainage system at a level below the patient's chest Vigorous bubbling in the water-seal chamber Stable water in the tube of the water-seal chamber during inhalation and exhalation. Occlusive dressing over the chest-tube Stable vital signs

Drainage system at a level below the patient's chest Occlusive dressing over the chest-tube Stable vital signs Rationale: It is expected that the drainage system will be at a level below the client's chest. This is what allows gravity to help drain fluid from the pleural space. If the drainage system was above the client's chest, the chest tube would not work properly. An occlusive dressing placed over the chest tube is appropriate. This is important to ensure that air does not enter the pleural space causing a pneumothorax. The nurse should check the dressing to ensure that it is airtight. Stable vital sign are expected sincluding heart rate, blood pressure, and respiratory rate. Any significant changes in vital signs may indicate a complication and should be reported to the healthcare provider immediately.

The nurse is preparing to administer a scheduled intramuscular injection to an apprehensive child. Which therapeutic action should the nurse take? A. Draw a "magic circle" on the area before the injection. B. Have another nurse hold down the child. C. Apply EMLA cream to the area immediately before the injection. D. Administer the medication right after the child's nap.

Draw a "magic circle" on the area before the injection. Rationale: Techniques to make an intramuscular injection less traumatizing include drawing a magic circle around the area, and after the injection, the nurse may fill in a smiley face.

When an elderly home health client suddenly develops delirium, what is the first thing the home health nurse should assess for? A. Drug intoxication B. Increased hearing loss C. Cancer metastases D. Congestive heart failure

Drug intoxication Rationale: Drug intoxication, from prescription or over-the-counter medications, is more common in the elderly due to slower metabolism and absorption. Combinations of digoxin, diuretics, analgesics, and anticholinergics should be examined. Delirium is an acute and reversible syndrome. It is characterized by changes in memory, judgment, language, mathematical calculation, abstract reasoning, and problem-solving ability. The most common causes of delirium are infection, medications, and dehydration.

Which of the following are potential complications of cleft lip and cleft palate in the infant? Select all that apply. Ear infections Feeding difficulties Weight gain Speech delay

Ear infections Feeding difficulties Speech delay Rationale: When a child has a cleft lip and cleft palate, the tissue and bone inside their mouth are not appropriately fused. This means there is a space between their upper lip and palate. Ear infections will be a frequent complication for these patients due to the dysfunction of the eustachian tube, which connects the middle ear and the throat. Feeding issues are a common complication of cleft lip and cleft palate because it is harder for these infants to eat with the abnormality in their palate. The space in the roof of the mouth makes it very hard to suck and get a good seal around the bottle or nipple. Speech delays and language delays are both common complications of cleft lip and cleft palate. This is because the roof of the mouth and lip have spaces where they should not, which decreases muscle function and leads to delayed or abnormal speech. Many of these infants will require consultation with a speech-language pathologist.

While assessing a laboring mother during a contraction. The LPN notes a decrease in fetal heart rate from 150 to 120 bpm. The heart rate slows for about 10 seconds and increases back to 150 bpm as the contraction ends. Which of the following correctly classifies this observation? A. Late deceleration B. Moderate variability C. Early deceleration D. Marked variability

Early deceleration Rationale: Early decelerations occur when the fetal heart rate decreases at the same time as a contraction. In this question, the nurse noted a decrease from 150 to 120 bpm with the diminution, and then a return to baseline. This occurs due to the pressure of the head of the fetus on the pelvis or soft tissue, and no intervention is required by the nurse after an early deceleration.

The nurse is planning a staff development conference about bioterrorism. Which of the following information should the nurse include? Select all that apply. Ebola virus disease (EVD) requires contact and droplet precautions Early treatment with prescribed ciprofloxacin is essential in inhalation anthrax A client with inhalation anthrax should be assigned to a room with monitored negative air pressure The plague is spread by infected bird droppings The plague produces a "bull's eye" rash at the site of infection

Ebola virus disease (EVD) requires contact and droplet precautions Early treatment with prescribed ciprofloxacin is essential in inhalation anthrax Rationale: Ebola virus disease (EVD) can be fatal if not treated. The transmission of EVD includes person-to-person, primarily through unprotected contact with blood and body fluids. Additionally, the nurse should institute droplet precautions while wearing a face shield to prevent splashing of infected body fluids. Inhalation anthrax is not spread from person to person and is treated with aggressive dosing of antibiotics such as levofloxacin, ciprofloxacin, or doxycycline.

The emergency department (ED) nurse is triaging a client who reports recent international travel to West Africa and has signs and symptoms of conjunctival infection, fever, rash, vomiting, and blood in their stool. The nurse is concerned that this client may have A. pulmonary tuberculosis. B. encephalitis. C. Ebola virus disease. D. inhalation anthrax.

Ebola virus disease. Rationale: West Africa was a site of a recent Ebola virus disease (EVD) epidemic. The manifestations of ebola include conjunctival injection, fever, rash, vomiting, and blood in their stool. This information makes it reasonable to raise the suspicion that this client may have EVD.

Which of the following interventions would be appropriate for a practical nurse caring for a toddler diagnosed with phenylketonuria? Select all that apply.

Elimination of dairy, meat, and eggs from the diet Strict avoidance of aspartame Foods must be measured to provide the prescribed amount of phenylalanine. Rationale: Elimination of dairy, meat, and eggs from the diet is an essential intervention for a toddler with phenylketonuria (PKU). In phenylketonuria, there is impaired metabolism of an essential amino acid named phenylalanine. When patients eat foods that contain this amino acid, they cannot break it down, and levels of this amino acid can then become toxic to the patient. Dairy, meat, and eggs are high in protein and therefore have a large amount of the amino acid phenylalanine. Therefore, eliminating these items from the diet is essential for children with PKU. Strict avoidance of aspartame is an essential intervention for a toddler with PKU. In phenylketonuria (PKU), there is impaired metabolism of an essential amino acid named phenylalanine. When patients eat foods that contain this amino acid, they cannot break it down, and levels of this amino acid can then become toxic to the patient. The artificial sweetener aspartame has a large amount of the amino acid phenylalanine. Therefore, eliminating it from the diet is essential for children with PKU. Foods must be measured to provide the prescribed amount of phenylalanine. The phenylalanine requirement must be recalculated if the toddler has a decreased appetite or refusal to eat.

The nurse is caring for a 2-day old infant with fetal alcohol spectrum disorder (FASD) and is preparing the family for discharge. Which of the following educational points are essential to include? Select all that apply. Emphasize the importance of avoiding alcohol consumption during future pregnancies. Educate the family about the potential long-term developmental and cognitive challenges associated with FASD. Teach the family strategies for managing behavioral issues that may arise in the child with FASD. Discuss the need for regular follow-up with a pediatrician or specialist to monitor the child's growth and development. Provide information on community resources and support groups for families affected by FASD.

Emphasize the importance of avoiding alcohol consumption during future pregnancies. Educate the family about the potential long-term developmental and cognitive challenges associated with FASD. Teach the family strategies for managing behavioral issues that may arise in the child with FASD. Discuss the need for regular follow-up with a pediatrician or specialist to monitor the child's growth and development. Provide information on community resources and support groups for families affected by FASD. Rationale: Emphasizing the importance of avoiding alcohol consumption during future pregnancies is crucial for the family to understand. By educating them about the risks of alcohol consumption during pregnancy, they can make informed decisions to prevent FASD in future pregnancies. The family needs to be aware of the potential challenges their child may face in the long term. Understanding the developmental and cognitive impacts of FASD can help them provide appropriate support and interventions. Behavioral issues are common in children with FASD. Equipping the family with strategies and techniques to manage and address these challenges can enhance their ability to support the child's well-being and development. Regular follow-up with a healthcare provider is vital to monitor the child's growth, development, and overall health. It allows for early intervention and timely adjustments to the child's care plan, optimizing outcomes. Connecting the family with community resources and support groups can provide them with a network of individuals who understand and can relate to their experiences. These resources can offer guidance, emotional support, and access to specialized services for children with FASD.

Strategies to promote effective sleep - recommendations:

Empty bladder before bed Plan to vigorous exercise earlier in the day Rationale: Strategic ways for a client to get a restful night's sleep include emptying their bladder before bed and engaging in vigorous exercise in the early morning and afternoon. Vigorous exercise within two hours of bed may increase the client's arousal level and decrease sleep induction.

Medication to control hypertension associated with nephroblastoma:

Enalapril Rationale: Enalapril is an ACE inhibitor used to lower blood pressure. Since clients with nephroblastoma are hypertensive due to increased renin levels, this medication is commonly prescribed to decrease their blood pressure. ACE inhibitors reduce blood pressure by inhibiting angiotensin II formation in the RAAS system, so they are an excellent choice for treating hypertension caused by nephroblastoma.

The nurse is caring for a client experiencing prolonged labor with hypotonic contractions. Which of the following actions should the nurse take? Select all that apply. Encourage frequent voiding. Maintain strict bedrest. Prepare for a prescribed infusion of oxytocin. Encourage frequent repositioning. Prepare for an infusion of intravenous (IV) fluids.

Encourage frequent voiding. Prepare for a prescribed infusion of oxytocin. Encourage frequent repositioning. Prepare for an infusion of intravenous (IV) fluids. Rationale: Prolonged labor with hypotonic contractions is classified as labor dystocia. Labor dystocia is a broad term that indicates that labor is not progressing. Key interventions for a client experiencing labor dystocia include encouraging the client to void frequently (when she feels the urge) because a full bladder will impede uterine contractions. A potential infusion of oxytocin to augment uterine contractions is a plausible prescription to be anticipated from the primary healthcare provider (PHCP). Frequent maternal repositioning is a key and noninvasive intervention that helps with fetal descent and effective contractions. The nurse should keep the client upright and encourage frequent repositioning. Fluid and electrolyte imbalances may be a cause of stunted labor. The nurse should be prepared to administer parenteral fluids because fluid and electrolyte abnormalities may cause labor dystocia.

The nurse is caring for a client with suspected meningitis. Which priority action should the nurse take immediately following a lumbar puncture procedure? A. Test for gag reflex return B. Elevate the head of the bed to 30 degrees C. Encourage oral fluid intake D. Determine if the client is positive for the Brudzinski sign

Encourage oral fluid intake Rationale: A lumbar puncture (or spinal tap) procedure is used to obtain cerebrospinal fluid (CSF) to diagnose meningitis and identify the cause. Following this procedure, the nurse would encourage oral fluid intake to replace CSF volume and reduce the risk of spinal headaches. Additionally, clients with suspected meningitis have a fever, and encouraging fluid intake would help reduce the fever and prevent dehydration.

A nurse is caring for a client diagnosed with chronic obstructive pulmonary disease (COPD). Which of the following interventions should the nurse implement for this client? Select all that apply. Administer a long-acting bronchodilator for acute dyspnea. Encourage smoking cessation. Provide oxygen therapy at a rate of 4 liters per minute. Instruct the client to lie down in a supine position. Reinforce education regarding pursed-lip breathing technique.

Encourage smoking cessation Reinforce education regarding pursed-lip breathing technique Rationale: Smoking is a major risk factor for the development and progression of COPD. Encouraging smoking cessation is an essential intervention to prevent further lung damage and improve the client's respiratory status. Pursed-lip breathing is a breathing technique that can help clients with COPD improve their breathing efficiency and control dyspnea

The nurse is caring for a post-operative client at risk for a pressure ulcer. Which intervention should the nurse include in the plan of care? A. Apply sequential compression devices B. Apply an extra sheet to the bed C. Position the client on a donut pillow D. Encourage the consumption of high-protein foods

Encourage the consumption of high-protein foods Rationale: High protein foods are encouraged because they promote wound healing and prevent fluid shifting, which may lead to a pressure ulcer. Optimal protein intake is key to preventing (and healing) a pressure ulcer.

Which of the following nonpharmacological interventions are appropriate for a 2-month-old with a fever of 39.2 degrees Celsius? Select all that apply.

Encouraging adequate fluid intake Dressing the child in lightweight clothing Using cooling techniques Providing a cool and comfortable environment Rationale: It is crucial to ensure that the 2-month-old with a fever stays well-hydrated. Offering fluids, such as breast milk or formula, helps prevent dehydration and supports the body's healing process. Dressing the baby in light, breathable clothing helps facilitate heat dissipation and prevents overheating. Avoiding excessive layers or heavy clothing promotes comfort and helps regulate body temperature. Employing cooling techniques like lukewarm sponge baths or placing cool, damp cloths on the baby's forehead and underarms can aid in reducing body temperature. These measures should be done gently to prevent chilling the baby. Creating a cool and comfortable environment by adjusting the room temperature, using a fan, or providing good air circulation can help lower the baby's body temperature and promote comfort.

The nurse is caring for a postoperative client at risk for venous thromboembolism (VTE). The nurse anticipates that the primary healthcare provider (PHCP) will prescribe which medication? A. Enoxaparin B. Verapamil C. Tranexamic acid D. Ropinirole

Enoxaparin Rationale: Enoxaparin is a low molecular weight-based heparin (LMWH) indicated for VTE prophylaxis following surgery. This medication is only given subcutaneously in the abdomen.

The nurse knows that which of the following scenarios are common reasons for disciplinary action against practical nurses by the State Board of Nursing? Select all that apply. Failure to administer medications as prescribed. Violation of client confidentiality. Poor attendance record. Engaging in personal relationships with clients. Failing to maintain continuing education requirements.

Failure to administer medications as prescribed. Violation of client confidentiality. Engaging in personal relationships with clients. Failing to maintain continuing education requirements.

The nurse reinforces teaching to a client with hypertension about the newly prescribed furosemide. Which of the following should the nurse include in the teaching?

Take this medication in the early part of the day Rationale: Furosemide is a loop diuretic and may be indicated for conditions such as heart failure or hypertension. The client should be instructed to take this medication in the earlier part of the day to avoid nocturia.

The nurse is discussion health-care-associated infections with a group of students. The nurse understand that which of the following actions is most effective at reducing the incidence of health-care-associated infections? A. Screen all newly admitted clients for colonization or infection with MRSA. B. Develop policies that automatically start antibiotic therapy for clients colonized by multi-drug resistant organisms. C. Ensure that dispensers for alcohol-based hand rubs are readily available in all client care areas of the hospital. D. Require nursing staff to don gowns to change wound dressings for all clients.

Ensure that dispensers for alcohol-based hand rubs are readily available in all client care areas of the hospital. Rationale: Since healthcare workers' hands are the most common means of transmission of infection from one client to another, the most effective method of preventing disease spread is to make supplies for hand hygiene readily available for staff to use. Reducing the risk of healthcare-associated infections is the responsibility of every healthcare worker. Following standard precautions for all clients is the easiest and most effective way of preventing disease spread.

Which action taken by the school nurse will have the most impact on the school's incidence of infectious disease? A. Ensure that students are immunized according to national guidelines. B. Provide written information about infection control to all patients. C. Make soap and water readily available in the classrooms. D. Teach students how to cover their mouths when coughing.

Ensure that students are immunized according to national guidelines Rationale: The incidence of once-common infectious diseases such as measles, chickenpox, and mumps have been most effectively reduced by immunization of all school-aged children. School-aged children are at risk for problems such as exposure to viruses, respiratory infections, and parasitic infections (such as scabies or lice). Vaccination protects children from severe illness and complications of vaccine-preventable diseases, which can include amputation of an arm or leg, paralysis of limbs, hearing loss, convulsions, brain damage, and death.

The process of absorbing drugs before elimination after they have been excreted into bile and delivered to the intestines is called:

Enterohepatic cycling

The nurse is assessing a 2 year old with the following symptoms: excessive drooling, stridor, difficulty swallowing, and difficulty speaking. Which of the following conditions does the nurse suspect? A. Croup B. Epiglottitis C. Laryngotracheal bronchitis D. Bronchiolitis

Epiglottitis Rationale: The cardinal signs of epiglottitis are 'the 4 D's' - drooling, dysphonia, dysphagia, and distress. Difficulty swallowing is dysphagia and difficulty speaking is dysphonia. Stridor is a high-pitched wheezing sound caused by disrupted airflow - hence the distress. This child is presenting with all of those cardinal symptoms and is therefore highly suspicious of epiglottitis.

The nurse assists a registered nurse in developing a care plan for a client with psychosis associated with schizophrenia. Which of the following interventions would be appropriate? A. Provide therapeutic touch B. Set limits on splitting behavior C. Establish a trusting, non-threatening relationship D. Immediately restrain the client for verbal aggression

Establish a trusting, non-threatening relationship Rationale: The most crucial goal for a person with schizophrenia is establishing a trusting relationship. This therapeutic rapport may help clients decrease their paranoia, which is commonly found in individuals with schizophrenia.

Upon entering a client's room, the nurse finds the client lying on the floor. What is the first action the nurse should implement? A. Call for help to get the client back in bed. B. Assist the client back to bed. C. Establish if the client is responsive. D. Ask the client for details about what happened.

Establish if the client is responsive. Rationale: Assessing if the client is responsive is the primary concern of the nurse in this example. Consciousness should be assessed first, and the nurse should move on to the subsequent action of moving/assisting the client.

Increased levels of which of the following hormones is related to hyperemesis gravidarum? A. Testosterone B. Progesterone C. Aldosterone D. Estrogen

Estrogen Rationale: The cause of hyperemesis is related to high estrogen and human chorionic gonadotropin (hCG) levels. Nausea and vomiting, also known as morning sickness, are common during the first trimester of pregnancy for many women. If nausea and vomiting interfere with an inadequate intake of fluid/food and persists past 20 weeks of gestation, it is termed hyperemesis gravidarum. The cause is unknown, but elevated hormone levels and the relaxation of smooth muscles result in delayed gastric emptying, which is believed to contribute to this condition. Hyperemesis can cause problems for the mother and fetus. Severe hyperemesis gravidarum can result in preterm labor. The dehydration that occurs may lead to reduced placental perfusion and inadequate oxygenation to the fetus. Fetal growth can be compromised, leading to an infant who is small for gestational age. Women with hyperemesis gravidarum in the second trimester have an increased risk for preterm labor, pre-eclampsia (i.e. an increase in blood pressure, protein in the urine, and edema), and placental abruption.

When orienting an older client to his hospital room's safety measures, what is the admission routine's priority component? A. Explain how to use the telephone. B. Introduce the client to her roommate. C. Review the hospital policy on visiting hours. D. Explain how to operate the call light.

Explain how to operate the call light. Rationale: Knowing how to use the call light is a safety priority.

The nurse is caring for a client who is recovering from abdominal surgery. There is a noticeable pinkish fluid oozing from the incision site. Which phase of the inflammatory response does this represent? A. Vascular response B. Cellular response C. Exudate formation D. Healing

Exudate formation Rationale: The fluid and white blood cells that leak from blood vessels in response to injury/inflammation are exudates. Exudates are present in the wounds as they heal. The nature and quantity of exudate depend on the severity of the damage and the tissues involved. For example, a surgical incision may ooze clear or pinkish (serous or serosanguinous) exudate for a day or two. If an exudate becomes purulent (thick, tan, green, or yellow), it is not normal and may suggest infection. In such cases, the nurse should immediately notify the health care provider.

The nurse is caring for a client who is newly prescribed cimetidine. The nurse understands that this medication is prescribed to treat which condition? A. Cystic fibrosis B. Clostridium difficile C. H. pylori D. Crohn's disease

H. pylori Rationale: Cimetidine is a H2 receptor antagonist indicated in treating peptic ulcer disease, gastric esophageal reflux disease, or H. pylori infections. This older medication has widely been replaced with newer H2 receptor antagonists because this medication is known to cause significant drowsiness.

The nurse knows that which of the following scenarios are common reasons for disciplinary action against practical nurses by the State Board of Nursing? Select all that apply. Failure to administer medications as prescribed. Violation of client confidentiality. Poor attendance record. Engaging in personal relationships with clients. Failing to maintain continuing education requirements.

Failure to administer medications as prescribed. Violation of client confidentiality. Engaging in personal relationships with clients. Failing to maintain continuing education requirements. Rationale: Failure to administer medications as prescribed. This is a valid reason for disciplinary action against practical nurses. Failing to administer medications as prescribed can harm clients and violate nursing standards and client safety protocols. Violation of client confidentiality. This is a valid reason for disciplinary action against practical nurses. Violating client confidentiality by sharing client information without proper authorization or justification breaches client privacy and can lead to legal and ethical consequences. Engaging in personal relationships with clients. This is a valid reason for disciplinary action against practical nurses. Engaging in personal relationships with clients is considered unprofessional and can compromise client care, boundaries, and ethical standards. Failing to maintain continuing education requirements. This is a valid reason for disciplinary action against practical nurses. Nursing boards often require practical nurses to participate in continuing education programs to ensure ongoing competency and keep up-to-date with advancements in the field. Failing to meet these requirements can result in disciplinary action, which indicates a lack of commitment to professional growth and acquiring the necessary knowledge and skills.

*NGN* 72-year-old male presents to the emergency department Item 1 of 6 Nurses' Notes 1430 - A 72-year-old white male stated he hasn't felt good lately and feels sad much of the time. He becomes tearful when telling you about the loss of his wife eight months ago. He states he feels lonely and hopeless. The client also stated that the osteoarthritis he was diagnosed with five years ago has worsened. He stated that lately, he gets tired easily but has difficulty falling and staying asleep. He reported that the only activity he has maintained is attending church services. Which four (4) assessment findings require further investigation by the nurse? Feelings of hopelessness Worsening osteoarthritis Only attending church services Feelings of loneliness Loss of his wife eight months ago Sleep disturbances

Feelings of hopelessness Feelings of loneliness Loss of his wife eight months ago Sleep disturbances Rationale: The client is exhibiting depressive symptoms that are a concern for suicidality. His hopelessness, loneliness, the recent loss of his wife, and sleep disturbances were all quite concerning. Hopelessness is a very significant risk factor for suicide because it inhibits forward-thinking by the client. Consequently, the client's loneliness from losing his wife is also a risk factor for suicide. This significant disruption in a support system likely stems from the client's dysphoria. Insomnia is a risk factor for suicide ideations and further characterizes the client's depression.

The nurse is taking vital signs for a client who has a chest tube in place. While counting respirations, the nurse notes that the water in the water seal chamber is fluctuating. Which of the following actions are appropriate based on this finding? A. Finish counting the client's respirations B. Empty the water-seal chamber C. Assist the client with incentive spirometry D. Notify the RN

Finish counting the client's respirations Rationale: It is appropriate for the nurse to finish counting the client's respirations and continue to monitor them. Fluctuations of the water in the water-seal chamber with inspiration and expiration is a sign that the drainage system is patent. Normally, the water level will increase when the client breathes in, and decrease when they breathe out. This is due to changes in intrathoracic pressures.

A nurse is preparing to administer gentamycin to a child. The order is for 3 mg/kg IV daily in three divided doses. The client weighs 97 lbs. How many milligrams should the nurse administer per dose? Fill in the blank. Round your answer to the nearest whole number.

First, convert the client's weight from pounds to kilograms 97 lbs = 44.1 kg Next, determine the total daily dose for this child 3 mg x 44.1 = 132.3 mg Next, determine the individual dose. Divide the daily dose by 3. 132.3 mg/day / 3 doses/day = 44.1 mg Finally, round the dose to the nearest whole number 44.1 mg = 44 mg

The nurse cares for a client receiving 1300 units/hr of heparin. The bag is labeled 25,000 units in 500 mL of dextrose 5% in water. How many mL should the nurse record that the client received in eight hours?

First, determine how many mL/hr the client is receiving (dose ordered / dose on hand x volume) 1300 units / 25000 units x 500 mL = 26 mL Next, take the mL/hr the client is receiving and multiply it by 8 26 mL x 8 hours = 208 mL

The nurse is preparing to administer penicillin V potassium to a child with pneumonia. The child weighs 18.5 kg. The prescription is for 50 mg/kg/day PO divided doses every six hours. How many milligrams should the child receive with each dose? Round your answer to the nearest whole number. Fill in the blank.

First, determine the total daily dose for this child 50 mg x 18.5 = 925 mg Next, determine the individual dose. The drug is given every six hours, so the child will receive four individual doses each day. Divide the daily dose by the frequency of dosing 925 mg/day / 4 doses/day = 231.25 Finally, round the dose to the nearest whole number 231.25 = 231 mg

The nurse is caring for a 16-year-old child who was brought to the emergency department by their mother after the child experienced a tonic-clonic seizure for which they are on seizure medications for. Which of the following drugs are used for long-term control of tonic-clonic seizures? Select all that apply. Gabapentin Ethosuximide Diazepam Carbamazepine Methylphenidate Alprazolam

Gabapentin Carbamazepine Rationale: Gabapentin is an antiepileptic drug commonly used for the treatment of seizures, including tonic-clonic seizures. It works by affecting calcium channels in the brain to reduce excitability, making it a suitable option for long-term seizure control. Carbamazepine is an antiepileptic drug commonly used for the long-term control of various types of seizures, including tonic-clonic seizures. It works by reducing the excessive electrical activity in the brain and is a suitable option for long-term use.

The nurse is observing unlicensed assistive personnel (UAP) care for assigned clients. Which of the following actions by the UAP would require the nurse to intervene? Select all that apply. Flexes and extends the patient's elbow for an active range of motion. Obtains orthostatic blood pressure by having the patient stand first. Places the cane on the unaffected side of a patient who had a stroke. Provides a hot foot soak for a patient with diabetes mellitus. Obtains a urine specimen from an indwelling catheter bag.

Flexes and extends the patient's elbow for an active range of motion. Obtains orthostatic blood pressure by having the patient stand first. Provides a hot foot soak for a patient with diabetes mellitus. Obtains a urine specimen from an indwelling catheter bag. Rationale: When supervising a UAP, the nurse should intervene if they are flexing and extending the client's elbow as that is not an active range of motion. The UAP doing the exercise for the client would be considered a passive range of motion. When obtaining orthostatic blood pressure, the correct sequence is supine, sitting, and standing. The UAP starting the orthostatic vital signs with the client standing is inappropriate. A client with diabetes mellitus should not have feet soaked in hot water as this could impair their skin integrity and cause ulceration. Finally, UAPs may not perform any tasks involving sterility. This includes aspirating urine from an indwelling catheter's tubing using a sterile syringe.

After reporting to her usual adult medical-surgical floor, the LPN is told she must float to the mother-baby unit. The LPN has never cared for this patient population before. Which of the following actions is most appropriate? A. Refuse the assignment. B. Float to the mother-baby unit and identify tasks within her training that she can safely perform. C. Call the nurse manager. D. Float to the mother-baby unit and ensure no one knows her inexperienc

Float to the mother-baby unit and identify tasks within her training that she can safely perform. Rationale: Floating to the mother-baby unit and identifying tasks within her training that she can safely perform is the correct action. This promotes patient safety and benefits both the nurse and the unit.

The nurse is caring for a client diagnosed with trichotillomania. The nurse anticipates a prescription for which medication from the primary healthcare provider (PHCP)? A. Fluoxetine B. Amphetamine C. Haloperidol D. Bupropion

Fluoxetine Rationale: Trichotillomania is a syndrome that causes a client to engage in hair-pulling. This disorder is categorized as an obsessive-compulsive disorder. Common sites for hair pulling include the eyebrows, scalp hair, and chin. Selective serotonin reuptake inhibitors (SSRIs) combined with psychotherapy are effective treatments for this disorder. Medications that may be used include fluoxetine, citalopram, or paroxetine.

The practical nurse prepares to administer a cycled tube feed to a client through their NG tube. Before initiating the feed, which of the following actions does the nurse correctly take? Select all that apply. Flush the nasogastric tube with saline. Verify placement of the nasogastric tube. Elevate the head of the bed. Ask the patient to remain in bed during the tube feeding. It is not necessary to check for residual with a cycled tube feed.

Flush the nasogastric tube with saline. Verify placement of the nasogastric tube. Elevate the head of the bed. Rationale: Before beginning a cycled tube feed, it is appropriate for the nurse to flush the nasogastric tube with saline. This allows the nurse to verify that the tube is patent and that the formula will freely flow into the client's stomach during the feed. It is incredibly important to always verify the placement of the nasogastric tube before putting anything in it. If the tube has moved and the tip of it is no longer in the stomach, the feeding could be aspirated causing serious problems such as pneumonia. The gold-standard to verification of tube placement is visualization on an x-ray. After the placement has been initially verified, the nurse may mark where the tube is located at the nare of the client to check that the tube has not moved and remains in the stomach prior to each feed. It is appropriate to elevate the head of the bed prior to any tube feeding. This allows gravity to help the tube feeding flow into the stomach and prevent reflux. For clients receiving a bolus feed a high fowler's position is preferred, and for clients receiving a cycled or continuous feed a semi-fowler's position is preferred.

During a busy shift, the nurse appropriately delegates tasks to the unlicensed assistive personnel (UAP) working with her. After delegating, which is the nurse's primary responsibility? A. Document the completion of the task. B. Make a list of tasks not yet completed to pass on to the next shift. C. Observe the UAP for the duration of the task. D. Follow-up with the UAP to ensure completion of the task by evaluating the outcome.

Follow-up with the UAP to ensure completion of the task by evaluating the outcome. Rationale: The nurse should follow-up with the unlicensed assistive personnel (UAP) to ensure completion of the task, evaluating the outcome. The ultimate responsibility for any task will always remain with the person who delegated it. Therefore, after delegating a task, the nurse's primary responsibility will be to follow up with the UAP.

While working in a maternity clinic, the nurse recognizes which of the following as presumptive signs of pregnancy? Select all that apply.

Frequent urination Darkening of the areola Rationale: Pregnancy signs are classified into three categories: Presumptive, Probable, and Positive. Presumptive signs include those subjective signs (experienced by the mother) that suggest but do not positively indicate pregnancy. Such signs may also be seen in conditions other than pregnancy. Probable signs are strong indicators of pregnancy but need confirmation. These are objective signs observed by the examiner. Some of these signs may be seen in other conditions such as pelvic malignancies and pelvic congestion. Finally, positive signs of pregnancy are those absolute signs of fetal presence that can be explained only by pregnancy.

The emergency department (ED) nurse cares for a client receiving prescribed warfarin and reports dizziness, black tarry stools, and bloody gums. The international normalized ratio (INR) returns at 5. The nurse anticipates the primary healthcare provider (PHCP) will prescribe which blood product? A. Packed red blood cells (PRBCs) B. Platelets C. Granulocytes D. Fresh frozen plasma (FFP)

Fresh frozen plasma (FFP) Rationale: FFP would be prescribed because this client is experiencing bleeding related to the prescribed warfarin. The client's INR is grossly elevated (therapeutic for VTE prophylaxis is 2-3), and FFP includes the Vitamin K-dependent clotting factors (factors II, VII, IX, X, proteins C, and S) that need to be replaced to stop the bleeding. Vitamin K may be prescribed, but the efficacy takes six to eight hours. FFP can treat the bleeding almost immediately.

The LPN is conducting a health screening at a local health fair. Which of the following should the LPN recognize as a risk factor for developing type II diabetes mellitus? Select all that apply. Gestational diabetes Metabolic syndrome Chronic corticosteroid use Gastric bypass surgery Obesity

Gestational diabetes Metabolic syndrome Chronic corticosteroid use Obesity Rationale: Type two diabetes mellitus is the most common type of diabetes worldwide. Risk factors for diabetes mellitus include gestational diabetes, metabolic syndrome, chronic corticosteroid use, and obesity. Screening for type two diabetes mellitus typically starts at age 45.

The nurse is caring for a client who has type 2 diabetes mellitus and hypertension. The client is nothing by mouth status (NPO) before a scheduled surgery. Which of the following prescribed medications should the nurse question?

Glipizide Rationale: Glipizide is a sulfonylurea and is given to the client with meals to manage blood glucose. This medication will lower blood glucose and could potentially cause hypoglycemia. The client is NPO and will not receive any food. Thus, the nurse should question the administration of this medication to prevent the client from developing hypoglycemia.

When preparing to change the linens on the bed of a client who has a draining sacral wound infected by MRSA, which PPE should the nurse plan to use? Select all that apply. Gloves N95 respirator Surgical mask Goggles Gown

Gloves Gown Rationale: A gown and gloves should be used when coming in contact with linens that may be contaminated by wound secretions. Approximately half of all MRSA infections are acquired in the hospital. One-fourth is associated with having received health care, but onset is in the community; the remainder is considered community-acquired. Due to aggressive health care emphasis on preventing MRSA transmission using standard and contact precautions, rates have decreased but are still unacceptably high. More Americans die each year from MRSA than from AIDS.

The nurse evaluates a student's ability to remove personal protective equipment (PPE) appropriately. It would indicate effective teaching if the student removes the PPE in which order? Place the steps in the appropriate order.

Gloves Goggles Gown Mask Rationale: The nurse can prevent disease transmission by exercising meticulous hand hygiene, educating others, and correcting inappropriate actions. The appropriate donning sequence of PPE is the gown, mask or respirator, goggles or face shield, and gloves. The appropriate sequence for doffing PPE is gloves, face shield or goggles, gown, and mask or respirator.

A client has been diagnosed with disseminated herpes zoster. Which personal protective equipment (PPE) will you need to put on when preparing to assess the client? Select all that apply. Goggles Gown Gloves Shoe covers N95 respirator Surgical face mask

Gown Gloves N95 respirator Rationale: Since herpes zoster is spread through airborne means and direct contact with the lesions, contact and airborne precautions should be followed. This means the nurse should wear an N95 respirator or high-efficiency particulate air filter respirator, a gown, and gloves. Herpes zoster, also known as shingles, is caused by the reactivation of the varicella-zoster virus (VZV), the same virus that causes varicella (chickenpox). Primary infection with VZV causes varicella. Once the illness resolves, the virus remains latent in the dorsal root ganglia. VZV can be reactive later in a person's life and create a painful, maculopapular rash called herpes zoster. Active herpes zoster lesions are infectious through direct contact with vesicular fluid until they dry and crust over. People with active herpes zoster lesions should cover their injuries and avoid contact with susceptible people in their household and occupational settings until their wounds are dry and crusted.

The nurse is caring for a client that is prescribed a calcium channel blocker (CCB) for hypertension. When reinforcing education about the medication, the nurse knows that which of these foods should avoided? A. Eggs B. Milk C. Grapefruit D. Bananas

Grapefruit Rationale: Grapefruit and its juice contain furanocoumarins, which block the enzymes involved in metabolizing many drugs, including calcium channel blockers. Grapefruit can interfere with other drugs too, including statins (atorvastatin, lovastatin, simvastatin), some antibiotics, and some cancer drugs. Medication blood levels can increase, resulting in toxicity. The calcium channel blockers' levels are increased when grapefruit or grapefruit juice is consumed, potentially causing hypotension.

When looking for trends in a postoperative client's vital signs, which documents would the nurse review first? A. Admission sheet B. Admission nursing assessment C. Activity flow sheet D. Graphic record

Graphic record Rationale: The graphic record should be reviewed first. The graphic record is a document that displays the client's vital signs data over a specific period. The graphical representation allows for easy visualization of any patterns or fluctuations in the vital signs, which could indicate improvement, stability, or potential signs of complications.

The nurse is assisting a client with food choices appropriate for Celiac disease. Which food items would be appropriate to select? Select all that apply. Grilled chicken Wheat pasta Scrambled eggs Oatmeal Beef patties

Grilled chicken Scrambled eggs Beef patties Rationale: Celiac disease is characterized by an individual's intolerance to gluten. Choices A, C, and E all have no gluten.

Which of the following is not an example of an inspection? Select all that apply. Heart rate and rhythm irregular Abdomen tympanic Lungs clear Skin is clear

Heart rate and rhythm irregular Lungs clear Rationale: Each of these answer choices is an example of auscultation.

The nurse is reviewing laboratory data. Which laboratory data requires follow-up?

Hemoglobin A1C 7.5% Ratoionale: A hemoglobin A1C of 7.5% is elevated and requires follow-up. The normal hemoglobin A1C is any value less than 5.7%, as a hemoglobin A1C of 6.5% meets the diagnosis of diabetes. A hemoglobin A1C between 5.7% and 6.4% concerns the nurse because this indicates prediabetes.

The nurse is caring for a client with diabetes mellitus. Which of the following laboratory data requires follow-up? Select all that apply.

Hemoglobin A1C 8.5% [< 5.7%] Creatinine 1.9 mg/dL [0.6-1.2 mg/dL] BUN 25 mg/dL [10-20 mg/dL] Proteinuria Rationale: The client's hemoglobin A1C is elevated as the therapeutic goal for a client with diabetes is to attain less than 7%. This elevated level is causing the client to experience an insult to the kidneys, which is evident by the increased BUN (normal 10-20 mg/dL) and creatinine (normal 0.6-1.2 mg/dL). Finally, proteinuria is further evidence that this client is experiencing diabetic nephropathy.

The nurse is admitting a child diagnosed with epiglottitis. During her admission questions, which vaccination should the nurse specifically ask the mother about? A. Tdap B. Influenza C. Hib D. MM

Hib Rationale: Hib - Haemophilus influenzae type B - is the most common cause of the bacterial infection that causes epiglottitis. Incidence has been significantly decreased by the Hib vaccine. That is why the nurse should ask the mother about this vaccination during the admission questions.

The nurse knows that which of the following are possible causes of constipation in the pediatric client? Select all that apply. Hirschsprung disease Spina bifida Iron supplements Psychosocial factors Changes in routine

Hirschsprung disease Spina bifida Iron supplements Psychosocial factors Changes in routine Rationale: Hirschsprung disease can be a structural cause of constipation. In Hirschsprung disease, there is a lack of innervation in the colon's nerve cells, leading to an inability for the child to pass stool. Spina bifida can be a structural cause of constipation. In Spina bifida, there is a loss of tone and sensation in the bowel, making them prone to constipation. Iron supplements are a medication that commonly causes constipation. It is one of their most common side effects. Clients should be educated about this side effect and measures to prevent constipation. Sometimes a bowel regimen may be necessary. Several psychosocial factors can cause constipation in pediatric clients. For example, a fear of using the toilet in public, a change in routine, difficult experiences passing stool, or painful stooling can cause constipation. Changes in routine can contribute to constipation in children.

When assessing a client's eyes for accommodation, what actions would the nurse perform? Select all that apply. Bring a penlight from the side of the client's face and briefly shine the light on the pupil. Hold a forefinger, a pencil, or another straight object about 10 to 15 cm (4" to 6") from the bridge of the client's nose. Hold a finger about 6" to 8" from the bridge of the client's nose. Darken the room. Ask the client to look straight ahead. Ask the client to first look at a close object, then at a distant object, then back at the close object.

Hold a forefinger, a pencil, or another straight object about 10 to 15 cm (4" to 6") from the bridge of the client's nose. Ask the client to first look at a close object, then at a distant object, then back at the close object. Rationale: To test accommodation, the nurse would hold the forefinger, a pencil, or another straight object about 4-6 inches from the bridge of the client's nose. Then the nurse would ask the client to first look at a close object, then at a distant object, then back to the object being held. The pupil normally constricts when looking at a near object and dilates when looking at a distant object.

The nurse is ambulating a client who is wearing a gait belt. The client begins to fall. The nurse should take which appropriate action to minimize injury? A. Hold the gait belt, extend one leg, let the client slide against the leg, and lower the client to the floor. B. Let go of the gait belt, grab the client under each arm, and gently lower the client to the floor. C. Grasp the gait belt, and instruct the client to fall gently down to the floor in a side-lying position. D. Hold the gait belt, and lower the client to the floor by using a narrow base of support.

Hold the gait belt, extend one leg, let the client slide against the leg, and lower the client to the floor. Rationale: For a client who is ambulating, the nurse should always be slightly behind the client and positioned on the affected (weaker) side. If a client starts to fall, the nurse should hold the gait belt, try to extend one leg, let the client slide against the leg, and gently lower the client to the floor, protecting the head. The nurse can help prevent client injury by maintaining a wide base of support.

The nurse is conducting a health screening at a local health fair. Which of the following should the nurse recognize as a risk factor for developing pulmonary tuberculosis (TB)? Select all that apply. Human Immunodeficiency Virus (HIV) Organ transplant Chronic corticosteroid use Influenza vaccination Obesity

Human Immunodeficiency Virus (HIV) Organ transplant Chronic corticosteroid use Rationale: According to the Centers for Disease Control (CDC), risk factors for TB include immunosuppression, organ transplant, chronic corticosteroid use, substance use, diabetes mellitus, and residing in environments such as nursing homes, prisons, and homeless shelters.

The nurse is assessing a client with pheochromocytoma. Which of the following would be an expected finding? Select all that apply. Hyperglycemia Hypertension Ataxia Oliguria Headache

Hyperglycemia Hypertension Headache Rationale: Manifestations of pheochromocytoma include hyperglycemia, hypertension, and headache. Other features associated with this condition include weight loss, anxiety, and palpitations. Pheochromocytoma is a condition caused by a tumor that sits on the adrenal medulla. This can cause a surge in catecholamine discharge resulting in headaches, palpitations, marked hypertension, and hyperglycemia.

The nurse is caring for a client on a continuous telemetry monitor and observes a prolonged PR interval, widened QRS complex, and a tall T-wave. Which electrolyte abnormality does the nurse suspect? A. Hypermagnesemia B. Hypomagnesemia C. Hyperphosphatemia D. Hypochloremia

Hypermagnesemia Rationale: Hypermagnesemia is a dangerous electrolyte abnormality that can cause many EKG changes in severe cases. A prolonged PR interval widened QRS complex, and a tall t-wave are characteristics of hypermagnesemia.

The nurse is caring for a 10-year-old client with a tracheostomy tube. The nurse notes that the client has a large volume of secretions and prepares to suction the client. Which of the following actions should she take first? A. Hyperoxygenate the client B. Ask the client to take a deep breath C. Place the client supine D. Notify the RN

Hyperoxygenate the client Rationale: It is necessary to hyperoxygenate the client prior to taking any of the other actions. This is one of the first steps in suctioning a tracheostomy. The nurse hyperoxygenates the client to prepare them for the procedure and prevent desaturations. The nurse then inserts the suction catheter without suctioning to the pre-measured depth, applies intermittent suction and rotates the suction catheter as it is removed from the tracheostomy.

The nurse is caring for a client with the following tracing on the electrocardiogram. When reviewing the client's medical history, which condition could be causing this dysrhythmia? See the image below. Select all that apply. Hyperthyroidism Severe hypothermia Hypothyroidism Shock Increased intracranial pressure

Hyperthyroidism Shock Rationale: The tracing shows sinus tachycardia. Sinus tachycardia may be caused by various conditions,,,, including febrile illness, dehydration, shock, anemia, and hyperthyroidism. Sinus tachycardia is a heart rate greater than 100 beats-per-minute, has a p-wave before each QRS complex, and is regular. The treatment of ST is the underlying cause, for example, if the client is febrile, the treatment would be prescribed fluids and antipyretics (acetaminophen).

The practical nurse is informed in report that her client is at risk for developing hypercalcemia. The nurse should monitor for which of the following signs and symptoms of hypercalcemia. Select all that apply. Irritability Hypoactive bowel sounds Decreased deep tendon reflexes Shortened PR interval Palpitations

Hypoactive bowel sounds Decreased deep tendon reflexes Palpitations Rationale: Hypoactive bowel sounds are a sign of hypercalcemia that the nurse would monitor. Hypercalcemic clients have decreased peristalsis leading to hypoactive bowel sounds. This causes abdominal pain, nausea, and constipation. Decreased deep tendon reflexes are a sign of hypercalcemia that the nurse would monitor. In hypercalcemic clients, everything slows down. The client may experience weakness, flaccidity, and decreased deep tendon reflexes. Hypercalcemia can cause changes in heart function, including arrhythmias or even cardiac arrest. As such, an assessment of the cardiovascular system may reveal irregular heart rhythms, palpitations, or other signs of cardiac dysfunction.

A patient that has suffered a third-degree burn injury involving 27% of Total Body Surface Area (TBSA) a few hours ago has been rushed to the emergency room. Which of the following should the LPN expect to find in this patient? Select all that apply. Hyponatremia Hyperkalemia Hypotension Increased urinary output Severe hypophosphatemia Edema in burned areas

Hyponatremia Hyperkalemia Hypotension Edema in burned areas Rationale: Initial resuscitation period: This refers to the period between 0 to 36 hours from the time of burn injury. Due to the damage to the tissues and vessels in major burns, capillary/vascular permeability is significantly increased, and fluid and electrolyte shifts occur between the body compartments. Significant edema in the burn area occurs due to fluid accumulation in the burned tissues due to increased vascular permeability and increased interstitial osmotic pressure. Due to changes in cellular permeability, sodium ions enter the cellular compartment resulting in low levels of intravascular sodium (hyponatremia, Na < 135 mEq/L) Extensive tissue necrosis and cell lysis in major burns also lead to the exit of potassium ions from the cell into the intravascular compartment resulting in hyperkalemia (K > 5.1 mEq/L). Restoring sodium losses by using appropriate fluid and correcting severe hyperkalemia is necessary during this period. The body's initial response in a major burn is to shunt blood toward the brain and heart, and away from peripheral vasculature and other organs, resulting in a low circulating volume leading to hypotension

The nurse is caring for a client who is postoperative ordered incentive spirometry. The nurse understands that this device will help prevent which complication? A. Venous thromboembolism B. Obstructive sleep apnea C. Hypostatic pneumonia D. Aspiration pneumonia

Hypostatic pneumonia Rationale: Hypostatic pneumonia after surgery is best prevented through incentive spirometry and early mobilization. The purpose of incentive spirometry (IS) is to promote deep breathing to prevent or treat atelectasis in the postoperative client. Hypostatic pneumonia is caused by pulmonary congestion in the dorsal region of the lungs. This type of pneumonia is common for those who are bedridden or have restricted mobility. Hypostatic pneumonia can be prevented through early postoperative ambulation and incentive spirometry.

The nurse in the emergency department (ED) is anticipating the arrival of a client with severe hypothermia. Which of the following would be an expected finding? A. Tachycardia B. Hypotension C. Tachypnea D. Hypertension

Hypotension Rationale: Severe hypothermia is a medical emergency defined as a core temperature of less than 82.4°F (28°C). This state causes massive vasodilation that causes the client to experience hypotension, bradycardia, bradypnea, and neuromuscular weakness.

When assessing for dehydration, what will the nurse observe? A. Headache and increased urinary output. B. Weight gain and edema. C. Hypertension and decreased urinary output. D. Hypotension, headache, and dry mucous membranes.

Hypotension, headache, and dry mucous membranes. Rationale: When there is an excessive loss of fluid within the body, dehydration can occur. Dehydration may be caused by acute illness or a chronic disease process. Common symptoms include dry mucous membranes, dark urine, decreased urinary output, confusion, low blood pressure, muscle cramps, and constipation.

While reviewing the morning labs of your client, you see the following results from their thyroid panel. What diagnosis does the nurse suspect? TSH: 7 mU/L T4: 1.0 mcg/dL T3: 2.0 ng/dL

Hypothyroidism Rationale: Hypothyroidism would be manifested with an increased thyroid-stimulating hormone level and decreased T4 and T3, as shown in these labs. Because of the increased TSH level, the thyroid gland is tricked into thinking that there is enough thyroid hormone already in the body and does not secrete more. The decreased T3 and T4 levels cause hypothyroidism symptoms, such as weight gain and fatigue.

Sudden infant death syndrome (SIDS) - statement that require follow-up:

I have been keeping my baby warm with extra blankets while he sleeps Rationale: Environmental factors such as soft mattresses/bedding, blankets, bumper pads affixed to the crib, and accessories in the crib increase the risk for SIDS dramatically. The recommended sleeping position for an infant is supine, with no stuffed animals, blankets, or accessories.

The LPN is attending to a client who presents to the emergency department after suffering a burn injury. He describes the burn as extremely painful. Select the zone corresponding to the area up to which the wound could be extending. A. I B. II C. III D. IV

II Rationale: II represents the dermal layer, which is involved in a 2nd-degree burn. Second-degree burns are the most painful of all the injuries. A wound extending through both the epidermis and dermis is classified as a second-degree burn (partial thickness burn). It is different from first-degree burns because blisters do form; in addition, the skin is moist and red. These burns extend through the upper dermis and into the lower layer of the dermis where most pain sensory receptors are located. This makes second-degree burns the most painful burns.

The LPN is performing a breast exam on her 65-year-old client. If she were to find a malignant lump, where is the location she is most likely to find it? See the exhibit. A. I B. II C. III D. IV

II Rationale: Quadrant II is the outer, upper quadrant of the breast, where malignant masses are most found. The breast tissue is thicker in the exterior, upper quadrant, and sometimes extends into the tissue surrounding the armpits. The nurse should pay close attention to palpation of this region of the breast. About 50% of breast cancer is detected in the upper outer quadrant (UOQ).

The nurse is discussing infection control with a group of nursing students. Which indication would be appropriate for the nurse to use an alcohol-based sanitizer? Select all that apply. Immediately before touching a client If the hands are visibly soiled with blood or bodily fluids When changing linens for a client infected with Clostridium difficile After changing a diaper for an infant infected with norovirus Before moving from work on a soiled body site to a clean body site on the same client Clostridium difficile

Immediately before touching a client Before moving from work on a soiled body site to a clean body site on the same client Rationale: Alcohol-based hand rubs (ABHRs) are recommended over hand hygiene with soap and water because of the risk of errors associated with washing hands with soap and water. Specific criteria have been established for when a healthcare worker should use an ABHR, and these two circumstances are appropriate.

The nurse is reinforcing education regarding the development of pressure ulcers. Which intrinsic risk factor should be included? A. Shearing B. Friction C. Impaired tissue perfusion D. Pressure

Impaired tissue perfusion Rationale: Impaired tissue perfusion is an internal risk factor. Intrinsic refers to anything essential or belonging naturally. Other intrinsic risk factors associated with skin breakdown include: Poor nutritional status Incontinence Alterations in fluid balance Altered neurological status

A client with a vancomycin-resistant enterococcus (VRE) infection is admitted to the medical unit. Which action can be delegated to a nursing assistant who is assisting with the client's care? A. Implement contact precautions when handling the client. B. Educate the client and family members on ways to prevent transmission of VRE. C. Monitor the results of the laboratory culture and sensitivity test. D. Collaborate with other departments when the client is transported for an ordered test.

Implement contact precautions when handling the client. Rationale: All hospital personnel who care for the client are responsible for the correct implementation of contact precautions.

While working on the pediatric floor, you are assigned a client with impetigo. Which of the following actions do you take to prevent the spread of this disease? A. Initiate standard precautions B. Initiate contact precautions C. Initiate droplet precautions D. Initiate airborne precautions

Initiate contact precautions Rationale: Clients with impetigo need to be placed on contact precautions to prevent the spread of this highly contagious disease. According to the CDC, these precautions are "for clients who may be infected or colonized with specific infectious agents for which additional precautions are needed to prevent infection transmission. Contact precautions will be used for any disease in which direct contact with the infectious organism can cause illness. This includes impetigo and other conditions such as viral gastroenteritis, MRSA, and scabies. Contact precautions will be required before entering the room; a gown and gloves are donned.

A nurse is caring for a client with a history of seizures who is at risk for injury. Which intervention is the highest priority to reduce the client's risk of injury? A. Keeping the client's room dimly lit to minimize visual stimulation B. Administer antiepileptic medications as prescribed. C. Implement seizure precautions, including padded side rails up and the bed in the lowest position. D. Provide education to the client and family about seizure triggers and safety measures.

Implement seizure precautions, including padded side rails up and the bed in the lowest position. Rationale: Seizures pose a significant risk of injury, and implementing seizure precautions helps to create a safe environment for the client during a seizure episode. Side rails up and padded as well as the bed in the lowest position prevent the client from falling out of bed or injuring themselves on nearby furniture. These precautions can minimize the potential for head injuries, fractures, and other injuries associated with seizures. Other precautions might include ensuring that oxygen and suctioning equipment with an airway are readily available. If the client does not have IV access, insert a saline lock, especially if he or she is at significant risk for generalized tonic-clonic seizures. The saline lock provides ready access if IV drug therapy must be given to stop the seizure. Specific seizure precautions vary, depending on health care agency policy.

The nurse is reviewing labs for a patient with a serum potassium level of 3.3 mg/dL(3.5-5 mEq/L). Which action would the nurse recognize as the highest priority? A. Educate the patient on potassium-rich foods B. Implement telemetry monitoring C. Obtain an order for STAT IM KCl D. Assess the patient's neurological status

Implement telemetry monitoring Rationale: The normal range for serum potassium is between 3.5-5 mg/dL, so this patient's level is low. Hypokalemia can lead to life-threatening cardiac arrhythmias. Of the options provided, initiating telemetry monitoring would be the highest priority in order to assess the patient's heart function and monitor for any changes.

The nurse is reinforcing education to parents of a child with sickle cell anemia. Which of the following instructions should be reinforced? Select all that apply. Increase the amount of fluids throughout the day Perform vigorous exercise for 60 minutes a day Avoid areas with high altitudes Notify the primary healthcare provider (PHCP) for any illnesses Good hand hygiene after using the bathroom and before meals

Increase the amount of fluids throughout the day Avoid areas with high altitudes Notify the primary healthcare provider (PHCP) for any illnesses Good hand hygiene after using the bathroom and before meals Rationale: Hydration is an essential component of preventing a sickle cell crisis, so this is a crucial education point. The nurse should provide specific recommendations and encourage the client to consume water-rich foods such as fruit and gelatin. High altitudes may trigger a vasocclusive crisis because it allows the oxygen to dissociate from the hemoglobin quicker. This may induce hypoxia which may trigger a crisis. The parents need to call the child's primary care doctor for any sign of illness. Illnesses may trigger a crisis, and conferring with the PHCP is recommended to prevent a crisis. Since infection risk is higher in those with sickle cell anemia, the nurse should reinforce the child washing their hands frequently.

Which of the following are clinical manifestations of the aging immune system that increase the susceptibility to illness? A. Increased lymph tissue. B. Increased autoimmune responses. C. Increased circulation of lymphocytes. D. Increased T and B cell production.

Increased autoimmune responses. Rationale: The elderly experience increased autoimmune responses. This increases the risk of diseases such as rheumatoid arthritis and other collagen disorders.

The nurse cares for a 38-year-old female client recently diagnosed with Graves' disease. The client presents with a visibly enlarged thyroid gland, heat intolerance, excessive sweating, and unintentional weight loss. What additional signs or symptoms may be present in this client? Select all that apply. Increased heart rate and palpitations. Diarrhea and frequent bowel movements. Tremors, particularly in the hands and fingers. Eye changes such as exophthalmos. Intolerance to cold temperatures.

Increased heart rate and palpitations. Diarrhea and frequent bowel movements. Tremors, particularly in the hands and fingers. Eye changes such as exophthalmos. Rationale: In Graves' disease, an overproduction of thyroid hormones (hyperthyroidism) can speed up the body's metabolism, leading to symptoms like increased heart rate and palpitations. Hyperthyroidism can increase the speed at which food travels through the digestive tract, resulting in diarrhea or more frequent bowel movements. Hyperthyroidism can lead to nervous system hyperactivity, which can cause fine tremors, especially in the hands and fingers. Graves' disease can cause Graves' ophthalmopathy, leading to inflammation and other changes in the eye tissues, resulting in bulging eyes or exophthalmos.

The nurse is discussing the prevention of constipation with a pregnant client. What intervention is the most appropriate for the nurse to provide? A. Increasing the consumption of fruits and vegetables. B. Taking a mild over-the-counter laxative. C. Lying flat on the back when sleeping. D. Reduce consumption of iron by half the amount.

Increasing the consumption of fruits and vegetables. Rationale: Dietary roughage (or fiber) with sufficient fluids and exercise may help relieve constipation. Constipation in pregnant women is thought to occur due to hormones that relax the intestinal muscle and by the pressure of the expanding uterus on the intestines. Relaxation of the abdominal muscle causes food and waste to move slower through your system. Sometimes iron tablets may contribute to constipation.

The LPN works in a community health clinic with new mothers and infants. Which of the following is accurate regarding expected growth in infants? Select all that apply. Infants should triple their birth weight by 12 months of age. Infants should double their birth weight by 6 months of age. Infants should triple their birth weight by 6 months of age. Infants should double their birth weight by 12 months of age. Infants should gain one pound month until 12 months of age or weight doubles

Infants should triple their birth weight by 12 months of age. Infants should double their birth weight by 6 months of age. Rationale: Infants should triple their birth weight by 12 months of age. Infants should double their birth weight by six months of age.

The nurse is discussing infection control with a group of nursing students. It would be correct to state that droplet precautions are used for which condition? Select all that apply. Influenza Viral meningitis Pertussis Hepatitis C Lyme disease

Influenza Pertussis Rationale: Conditions requiring droplet precautions include influenza and pertussis.

Among Erickson's Stages of Development, which of the following development stages would the nurse expect for her 4-year-old client to be in? A. Trust vs. Mistrust B. Initiative vs. Guilt C. Identity vs. Role Confusion D. Industry vs. Inferiority

Initiative vs. Guilt Rationale: Initiative vs. Guilt is the typical development stage for preschool children, who are 3 to 5-year-olds, so this is correct for your 4-year-old client. In initiative vs. guilt, children start to assert control and power over their environment. Success leads to initiative when they feel a sense of purpose, but children who try to exert too much power and experience disapproval end up feeling guilty.

The nurse is reviewing endocrine disorders with a group of students. It would be correct for the nurse to identify which manifestation is associated with hyperthyroidism? Select all that apply.

Injected (red) conjunctiva. Insomnia. Increased systolic blood pressure. Diaphoresis. Rationale: Excessive thyroid hormones stimulate most body systems, causing hypermetabolism and increased sympathetic nervous system activity. This gives the client with hyperthyroidism the classic features such as tachycardia, increased body temperature, weight loss, diaphoresis, insomnia, elevated systolic blood pressure, and ocular symptoms such as an injected (reddened) conjunctiva.

The nurse is caring for a client with newly prescribed zolpidem. The nurse understands that this medication is indicated for which condition? A. Attention Deficit Hyperactivity Disorder B. Generalized Anxiety Disorder C. Narcolepsy D. Insomnia

Insomnia Rationale: Zolpidem is a non-benzodiazepine indicated in the treatment of insomnia.

The nurse plans to use a mechanical lift to transfer a client from a stretcher to a wheelchair. Which appropriate action should the nurse take? A. Keep the stretcher's side rails raised during the transfer B. Instruct the client to fold their arms over their chest C. Apply gloves and gown for this procedure D. Unlock the wheels on the stretcher and wheelchair

Instruct the client to fold their arms over their chest Rationale: During any mechanical lift transfer, the nurse should instruct the client to fold their arms over the chest, preventing injuries to the client's arms during the transfer.

The nurse is removing a nasogastric tube (NGT). The nurse should take which action? A. Deflate the balloon B. Irrigate the tube with 200 mL of water C. Instruct the client to take a deep breath and hold it D. Assess the gag reflex

Instruct the client to take a deep breath and hold it Rationale: Prior to removing the NGT, it would be appropriate for the nurse to tell the client to take a deep breath and hold it as the tube is removed. The nurse should then pinch the tube and remove the tube quickly and steadily over 3-6 seconds while the client holds their breath or during exhalation.

The nurse is educating a group of nursing students about fetal alcohol spectrum disorders (FASDs). Which of the following features are characteristic of FASDs? Select all that apply. Intellectual disabilities. Facial abnormalities. Growth delays. Motor impairments. Anxiety.

Intellectual disabilities. Growth delays. Motor impairments. Rationale: Intellectual disabilities are characteristic features of FASDs. Prenatal alcohol exposure can cause neurodevelopmental issues, leading to intellectual disabilities and cognitive impairments in children. Another characteristic feature of FASDs is facial abnormalities. Children affected by prenatal alcohol exposure may have distinctive facial features, including a smooth philtrum (the area between the upper lip and nose), a thin upper lip, and small eye openings. Growth delays are often seen in individuals with FASDs. Prenatal alcohol exposure can affect overall growth, leading to below-average height and weight in affected individuals. Motor impairments, such as poor coordination and balance difficulties, can occur in individuals with FASDs. These difficulties may affect fine motor skills and gross motor functions. Choice E is incorrect.

The licensed practical/vocational nurse (LPN/VN) is caring for a primigravida client with the following clinical data. The LPN/VN should take which action based on the result? Nonstress Test (NST) = Reactive A. Interpret this result as normal B. Prepare the client for a contraction stress test C. Anticipate an order for repeat testing D. Inquire if the patient ate prior to the test

Interpret this result as normal Rationale: A reactive NST is an expected finding and indicates fetal well-being. In a reactive NST, two or more FHR accelerations of at least 15 beats per minute, lasting at least 15 seconds from the beginning of the acceleration to the end, in association with fetal movement, for 20 minutes.

The nurse is triaging a client who reports a sexual assault. Which actions should the nurse take? Select all that apply.

Interview the client in a private room Refer the client to support groups Record verbatim statements in the medical record Rationale: When caring for a client who has been sexually assaulted, the nurse must maintain a private environment to allow for the client to be forthcoming with their answers. Support groups are quite influential in the recovery process, and it would be wise for the nurse to provide these at discharge.

You are called to assess a 4-year-old client who has suffered second and third-degree burns to her chest, abdomen, and legs. It is estimated that about 40% of her total body surface area (TBSA) is burned. Upon assessment, her vital signs are as follows: HR: 140 RR: 44 BP: 90/60 SpO2: 88% on Room Air Which of the following interventions is the priority? A. Fluid replacement B. Intubation and mechanical ventilation C. Wound debridement D. Standard precautions

Intubation and mechanical ventilation Rationale: Remember your ABCs! Airway, Breathing, Circulation. This is the correct order of priority actions! You must address the airway first! Intubation and mechanical ventilation are the priority for this client. Intubation is the A in your ABC's airway. The stem of the question states that this client has burns to her chest. You know that smoke inhalation can burn the trachea and compromise the airway, and wounds to the chest are an indication that inhalation injury has likely occurred.

The nurse is collecting data on a child with bilateral lower extremity chemical burns. The nurse suspects that the child may have been abused. The nurse should take which initial action? A. Cover the affected area with sterile dressing B. Irrigate the affected area with saline C. Report the suspected abuse D. Document the findings

Irrigate the affected area with saline Rationale: A common mnemonic to remember is "the solution to pollution is dilution." When a client has a chemical burn, the highest priority is to copiously irrigate it (dilute it) with saline or water. Prompt irrigation of the area exposed to caustic substances ( acid, alkali) dilutes the chemical, attempts to neutralize the pH change in the skin, and decreases the extent of the dermal injury. Additionally, dilution lessens the risk of the caregiver getting burned by the chemical.

The nurse is supervising a student nurse administer prescribed medications via a double-lumen nasogastric tube (NGT) with an air vent. Which action by the student requires follow-up? The student A. Irrigates the air vent before medication administration with water B. Contacts the pharmacy to obtain available medications in liquid form C. Flushes the NGT between medications with water D. Administers each medication separately through the NGT

Irrigates the air vent before medication administration with water Rationale: The air vent should not be irrigated with water or used to administer medications. The purpose of the air vent is to permit free, continuous drainage of secretions when the NGT is connected to suction. This vent is found on a Salem sump tube and is often called a blue "pigtail."

The nurse is caring for a client with hypertension. Which of the following lifestyle modifications are recommended for managing hypertension? Select all that apply. Isometric exercises Decreasing sodium intake Smoking cessation Use of herbal supplements Weight loss

Isometric exercises Decreasing sodium intake Smoking cessation Weight loss Rationale: Isometric exercise, which increases muscle tension or muscle work but does not shorten or actively move muscle, is ideal for clients who do not tolerate increased activity, such as a client who is immobilized in bed. The benefits include an increase in muscle mass, tone, and strength, thus decreasing the potential for muscle wasting; increased circulation to the involved body part; and increased osteoblastic activity. Reducing sodium intake is crucial in managing hypertension. High sodium consumption can lead to fluid retention and increased blood pressure. Smoking and exposure to secondhand smoke can significantly increase the risk of developing hypertension and cardiovascular diseases. Smoking cessation is strongly recommended to manage hypertension. Maintaining a healthy weight or achieving weight loss if overweight or obese is beneficial in managing hypertension. Losing excess weight can help lower blood pressure and reduce the strain on the cardiovascular system.

The nurse reviews a client's medication record who takes prescribed sildenafil. Which medication should the nurse clarify with the primary healthcare provider (PHCP)? A. Lisinopril B. Isosorbide C. Atorvastatin D. Losartan

Isosorbide Rationale: Isosorbide is a nitrate medication and should not be taken concurrently with phosphodiesterase inhibitors such as sildenafil. The combination of the two may result in profound hypotension.

The nurse is caring for a client who has just been diagnosed with severe acne vulgaris. Which of the following medications should the nurse anticipate the primary health care provider (PHCP) will prescribe? A. Isotretinoin B. Acyclovir C. Ketoconazole D. Ethambutol

Isotretinoin

The nurse is caring for a newborn. Which of the following actions would be appropriate for the nurse to take? A. Perform APGAR assessment at five and ten minutes B. Suctions the nose then the mouth C. Administer RhoGAM intramuscularly D. Obtain the first outfit from the warmer

Obtain the first outfit from the warmer Rationale: A newborn is at risk of cold stress during the first few hours of post-intrauterine life. To prevent cold stress, the nurse should place the newborn skin-to-skin with a parent, obtain the first outfit out of a warmer, and dry the newborn thoroughly.

The nurse is instructing a client on deep breathing and coughing. The client asks the nurse, 'Why is it best for me to be sitting for these?' What response would be best for the nurse to provide? A. It is physically more comfortable for the client. B. It helps the client to support their positioning with a pillow. C. It loosens respiratory secretions. D. It allows the client to observe their room and relax.

It helps the client to support their positioning with a pillow. Rationale: The client should be sitting when deep breathing and coughing. This position allows the client to use a pillow, providing abdominal support when coughing. It also allows the lungs to expand more fully and the diaphragm to drop. Coughing and deep breathing exercises are essential to enhance lung expansion and mobilize secretions, thereby preventing atelectasis (collapse of the alveoli) and pneumonia.

The LPN is educating a group of new graduate nurses on the cardiovascular system. Which of the following statements best describes the cardiovascular system? A. It has a heart with six chambers, strong vessels, and valves. B. It is a double-pump circulating blood out to the lungs and the body. C. It includes concepts of pre-contractility, post-contractility, and load. D. It functions with a conduction system and starts in the ventricles.

It is a double-pump circulating blood out to the lungs and the body. Rationale: This statement is correct. The heart is a double pump with four chambers, four valves, and a conduction system with a pacemaker originating in the atrium.

The nurse is reinforcing teaching with the parents of a child diagnosed with impetigo. Which of the following statements, if made by the parent, would indicate effective understanding? Select all that apply. It is the most common bacterial skin infection between the ages of 2 and 5. The torso is the area most commonly affected by impetigo. The causative agent of impetigo is Haemophilus influenzae. Impetigo is usually not a systemic infection. "I should keep my child home from swim practice until the blisters heal." "I should not share my child's linens with anyone else in the house."

It is the most common bacterial skin infection between the ages of 2 and 5. Impetigo is usually not a systemic infection. "I should keep my child home from swim practice until the blisters heal." "I should not share my child's linens with anyone else in the house." Rationale: This is a true statement. Impetigo is the most common bacterial skin infection among 2 to 5-year-olds. It is highly contagious, and precautions should be taken to prevent the disease spread among families, daycare centers, and schools. Impetigo is not usually a systemic infection. In most cases, the disease is confined only to the skin, where the sores break. The area around the nose, mouth, hands, and feet are the most commonly affected. Topical antibiotics can usually treat impetigo. The child is well appearing other than the apparent skin infection and will often be afebrile "I should keep my child home from swim practice until the blisters heal" is a true statement. Impetigo is a skin condition caused by group A Streptococcus (GAS; Streptococcus pyogenes) or Staphylococcus aureus. The condition is highly contagious and spreads by contact with the vesicles. The vesicles should remain clean and dry, and swimming is prohibited because it allows for further disease transmission. Children can return to school 24 hours after beginning antimicrobial therapy if the affected area remains covered. Draining lesions should be kept covered. "I should not share my child's linens with anyone else in the house" is another true statement. Linens are an effective way to transmit the bacteria, and they should not be shared and laundered daily to prevent reinfection.

What are the benefits of providing oral care to a client in critical care? Select all that apply. It promotes the client's sense of well-being. It prevents deterioration of the oral cavity. It contributes to a decreased incidence of aspiration pneumonia. It eliminates the need for flossing. It decreases oropharyngeal secretions. It compensates for an inadequate diet.

It promotes the client's sense of well-being. It prevents deterioration of the oral cavity. It contributes to a decreased incidence of aspiration pneumonia. Rationale: Adequate oral hygiene is essential for promoting a client's sense of well-being and preventing oral cavity deterioration. Diligent oral hygiene care can also improve oral health and limit pathogens' growth on oropharyngeal secretions, decreasing the incidence of aspiration pneumonia and other systemic diseases.

Which of the following are opportunistic infections that adversely affect patients infected with HIV/AIDS? Select all that apply. Kaposi's sarcoma Wilms' sarcoma Tuberculosis Toxoplasma gondii Visual loss Peripheral neuropathy

Kaposi's sarcoma Tuberculosis Toxoplasma gondii Rationale: In addition to Kaposi's sarcoma, tuberculosis, toxoplasma gondii, patients infected with HIV/AIDS are also at risk for developing herpes simplex, histoplasmosis, and salmonella. Opportunistic infections (OIs) occur more often or are more severe in people with weakened immune systems than in people with healthy immune systems. People with weakened immune systems include people living with HIV.

The nurse is caring for a client receiving lorazepam. Which of the following reported herbal supplements would require follow-up? Select all that apply. Kava Glucosamine Valerian Garlic Saw palmetto

Kava Valerian Rationale: Lorazepam is a CNS depressant, and the client should avoid potentiating the effects of this medication. Herbal products such as kava and valerian are CNS depressant medications that should not be given concurrently while a client receives lorazepam. Lorazepam and one of these medications may cause profound sedation.

The nurse cares for a newly admitted client with a full-thickness burn of over 25% of their total body surface area. The nurse should take the following actions? Select all that apply. Keep the client nothing-by-mouth (NPO) status Obtain a 12-lead electrocardiogram Prepare to infuse hypertonic saline Irrigate the burns with chlorhexidine Obtain an arterial blood gas (ABG)

Keep the client nothing-by-mouth (NPO) status Obtain a 12-lead electrocardiogram Obtain an arterial blood gas (ABG) Rationale: For a client admitted with a significant burn, the nurse should acutely assess and maintain airway patency. Further, the nurse should keep the client on NPO status because of the risk of oral mucosa irritation and gastrointestinal dysfunction. A twelve-lead echocardiogram is necessary because acutely, the potassium shifts extracellular leading to hyperkalemia. This may lead to cardiac dysrhythmias. Finally, the nurse should obtain an arterial blood gas to determine the client's oxygen status.

The clinic nurse is assessing a 16-year-old female client with anorexia nervosa. Which of the following would be expected assessment findings? Select all that apply. Lanugo. Heavy menstrual periods. Hypertension. Hypothermia. Hyperkalemia. Russell's sign.

Lanugo Hypothermia Rationale: Lanugo describes fine, soft hair that covers the body and limbs of a human fetus/newborn; it is abnormal for a 16-year-old to have lanugo. In a client who is severely underweight and has lost a large amount of subcutaneous fat, the body may develop lanugo as a form of insulation. Subcutaneous fat is necessary to insulate the body and regulate the temperature. Clients with anorexia nervosa lose a significant amount of subcutaneous fat due to malnourishment and weight loss. Consequently, they are prone to hypothermia.

Which of the following terms describe the soft down hairs present on the shoulders, back, and forehead of newborns? A. Milia B. Lanugo C. Vernix D. Mongolian spot

Lanugo Rationale: Lanugo is the soft hairs present on the shoulders, back, and forehead of newborns.

*NGN* The nurse cares for a 41-year-old female in the emergency department (ED) Item 1 of 6 Nurses' Note 0900 - Client reports a concern for increased swelling and pain in her left lower extremity. The onset of the symptoms was yesterday evening and when she woke up the pain and swelling had increased. Two days prior, the client sustained abrasions and bruising on the extremity while attempting to get out of a swimming pool. 2+ edema was evident in the left lower extremity; area was warm to touch. Several abrasions on leg noted which were dry and appeared to be healing. Erythema noted. Peripheral pulse and sensation were intact, toe movement was evident along with distal and sensation. Pain reported with leg movement. Pain rated a '7' on a scale 0-10. Current daily medications include ortho tri-cyclen, bupropion, and a multivitamin. She denies any alcohol or drug use. Smokes 2-3 cigarettes per day. Vital Signs Ora

Left lower leg edema Pain with movement Left lower leg temperature Rationale: The assessment findings of left lower leg edema, pain during movement, and the temperature of the affected extremity are concerning. These assessment findings are abnormal; the client's vital signs, peripheral pulses, and healing abrasions are not of concern because they are normal findings.

The nurse is caring for a client whose most recent serum sodium level was 152 mEq/L(135-145 mEq/L). Which of the following signs and symptoms could be attributed to the client's sodium level? Select all that apply. Lethargy Dry mucous membranes Tachypnea Cyanosis Restlessness and irritability

Lethargy Dry mucous membranes Restlessness and irritability Rationale: Sodium plays a very important role in the brain, and imbalances in the serum sodium level can cause major neurological changes. The client who is hypernatremic, or has a sodium level greater than 145 mEq/L is at risk for changes in their level of consciousness ranging from restlessness and agitation to lethargy, stupor, and coma. A client who has a high sodium level, greater than 145 mEq/L will often have dry mucous membranes. Hypovolemic hypernatremia is the most common form of hypernatremia. Causes include renal losses of free water (osmotic diuresis, post obstructive diuresis) or extrarenal losses (diarrhea, sweating, increased insensible losses). In such cases, the client is often dehydrated, and fluid volume deficit is manifested by dry mucous membranes. Dry mucosa may also be secondary to the relationship sodium has with water. Water follows sodium, so where there is an increased level of sodium in the extracellular space, water leaves the cells and follows the sodium into the extracellular space. This causes dry mouth and mucous membranes. Cellular dehydration can lead to changes in the nervous system, including the brain, which can cause restlessness and irritability.

The nurse is assessing a client with appendicitis. Which of the following would be an expected finding? Select all that apply. Leukocytosis Melena Fever Nausea and Vomiting Anorexia

Leukocytosis Fever Nausea and Vomiting Anorexia Rationale: Clinical manifestations of appendicitis include leukocytosis, fever, nausea and vomiting, right lower quadrant pain, and anorexia.

The licensed practical/vocational nurse has attended a staff conference on the causes and management of constipation. Which of the following medications would contribute to constipation? Select all that apply.

Levetiracetam Oxycodone Multivitamin drops Diuretics Rationale: Levetiracetam is an antiepileptic medication. One of the side effects of antiepileptics is constipation. Oxycodone is an opioid medication used for severe pain. Opioids commonly cause constipation; the nurse should know these side effects when administering them. Multivitamin with iron. One of the most common side effects of this medication is constipation. Diuretics, such as furosemide and hydrochlorothiazide, can lead to dehydration and slow down bowel movements.

The LPN is reinforcing education to a client about modifiable risk factors and risk factors that are not. Which of the following is most likely able to be corrected? A. Genetic predisposition B. Lifestyle choices C. Depression D. All of the above

Lifestyle choices Rationale: Lifestyle choices are the risk factors that are most likely able to be corrected. Poor lifestyle choices place a person at risk and they are often considered risky behaviors.

The emergency department (ED) nurse cares for a client with severe intrabdominal bleeding. The client has tachycardia, hypotension, and a thready pulse. The nurse anticipates the primary healthcare provider (PHCP) will prescribe which blood product?

Packed red blood cells (PRBCs) Rationale: The client is experiencing intraabdominal bleeding with manifestations confirming shock. The client will need to have the blood volume replaced with emergent surgery. Type-specific PRBCs would be preferred; however, if the client is critical, O-negative blood may be transfused.

The client with tuberculosis is now on isoniazid. Which laboratory test should be monitored at least monthly? A. PT and PTT B. CBC C. BUN D. Liver enzymes

Liver enzymes Rationale: Although it is rare, liver toxicity is a severe adverse effect of Isoniazid. Healthcare providers should monitor for signs of jaundice, fatigue, elevated liver enzymes, and loss of appetite. Liver enzyme tests are usually performed monthly during therapy to identify early hepatotoxicity. Isoniazid is bactericidal for actively growing organisms and bacteriostatic for dormant mycobacteria. It is selective for M. tuberculosis. Isoniazid is used alone for chemoprophylaxis or in combination with other antitubercular drugs when treating active disease.

The nurse is caring for a client prescribed a topical glucocorticoids. The nurse should monitor for all the following systemic effects of the medication except: A. Mood changes B. Osteoporosis C. Liver toxicity D. Adrenal insufficiency

Liver toxicity Rationale: Liver toxicity is not a systemic effect associated with the use of glucocorticoids. Topical glucocorticoids or corticosteroids are used in cases of dermatitis and eczema to treat symptoms of burning, itching, and inflammation. They may also be used in conjunction with other medical therapies for the treatment of psoriasis.

*NGN* The nurse performs a home safety survey for an individual with epilepsy Click to specify the findings that require intervention by the nurse Tap to highlight and select the answer. Living area and bedroom Multiple glass tables in the living room Multiple feather pillows present on the bed Relocated bedroom from the second floor to the first floor Padded covers on the edges of countertops Wall-to-wall carpeting was removed and replaced with scattered rugs on hardwood flooring Kitchen Kitchen knives were readily accessible Client reports using the microwave instead of the stove Bathroom Rubber mats in the bathtub Shower chair with hand-held shower nozzle present Locks on the bathroom door

Living area and bedroom Multiple glass tables in the living room Multiple feather pillows present on the bed Wall-to-wall carpeting was removed and replaced with scattered rugs on hardwood flooring Kitchen Kitchen knives were readily accessible Bathroom Locks on the bathroom door Rationale: Multiple glass tables in the living room: multiple glass tables are concerning because the client may fall and sustain a serious injury during a seizure. Tables should be limited; if used, the edges should be covered with padded covers to reduce head injury. Multiple feather pillows are present on the bed: seizure-safe pillows are available for sale as they reduce asphyxiation. Seizure-safe pillows should be used; the fewer pillows, the better to reduce the risk of asphyxiation. Wall-to-wall carpeting was removed and replaced with scattered rugs on hardwood flooring: wall-to-wall carpeting is highly recommended because if the client falls while having a seizure, this will reduce the injury when compared to a hardwood floor. Kitchen knives were readily accessible: Knives should not be used because a serious injury can be sustained if a seizure occurs while using a knife. A food processor should be used instead of knives. Locks on the bathroom door: Locks should not be on the door because if the client has a seizure in the bathroom, immediate access would be hampered by the lock. Instead, the client should have a placard that can be flipped over on the door indicating whether the bathroom is occupied. A shower chair with a handheld nozzle should be used instead of the client bathing in a tub (drowning).

The licensed practical/vocational nurse (LPN/VN) is caring for a client with the following clinical data. Which prescription would the LPN/VN request from the primary healthcare provider (PHCP) based on the clinical data? See the exhibit. A. Diphenhydramine B. Lorazepam C. Phenytoin D. Clozapine

Lorazepam Rationale: Lorazepam is a benzodiazepine used in the management of alcohol withdrawal symptoms. The client exhibits these symptoms, evidenced by perspiration on the forehead, nystagmus, coarse tremors, and visual hallucinations.

A pregnancy-related spinal change that can alter mobility is known as: A. Ankylosing spondylosis B. Lordosis C. Scoliosis D. Kyphosis

Lordosis Rationale: As a fetus grows, a variety of changes appear in a pregnant woman's body. The thoracic and lumbar spine curvature change, pain in the low back and pelvic region can increase; balance and gait patterns also change. Some studies report that the center of gravity of pregnant women moves towards the abdomen, resulting in an increase in lumbar lordosis, posterior tilt of the sacrum, and movement of the head to the back to compensate for the increased weight as the fetus grows.

After experiencing a traumatic amputation and related body image disturbance. The nurse documents the nursing diagnosis of body image disturbance related to changes in appearance secondary to: A. Severe trauma B. Loss of a body part C. Chronic disease D. Loss of body function

Loss of a body part Rationale: Although the amputation was related to severe trauma, being specific about what type of injury, the loss of a body part, gives precise information to other health care team members who may assume care of this client. The nursing diagnosis is "Body Image Disturbance". When referencing a nursing diagnosis secondary to a condition/experience, it is essential to be specific.

A pregnant client at 16 weeks gestation developed a pulmonary embolism and was initiated on intravenous heparin therapy two days ago. She is now getting discharged. The nurse has provided medication education and you are assigned to reinforce the medication teaching. Which of the following medications do you expect the physician to order at discharge? A. Warfarin B. Rivaroxaban C. Apixaban D. Low Molecular Weight Heparin (LMWH)

Low Molecular Weight Heparin (LMWH) Rationale: The physician will most likely order Low Molecular Weight Heparin (LMWH) to be self-administered twice daily. The dosing schedule and monitoring of LMWH are more convenient than that of unfractionated heparin. LMWH does not cross the placenta and, therefore, does not cause fetal harm. LWMH and unfractionated heparin are the anticoagulants of choice during pregnancy. LMWH will provide therapeutic anticoagulation for the rest of the pregnancy. Most venous thromboembolism events need therapeutic anticoagulation for at least 3 to 6 months. However, pregnancy is a hypercoagulable state. The client, therefore, is at risk for recurrent thromboembolism throughout her pregnancy and at least six weeks post-partum.

The nurse is helping the unlicensed assistive personnel pass meal trays. When providing a meal tray for a client diagnosed with pheochromocytoma, which dietary item should the nurse remove? A. Macaroni and cheddar cheese B. Watermelon slices C. Caffeine free cola D. Baked chicken

Macaroni and cheddar cheese Rationale: Pheochromocytoma is caused by a tumor on top of the adrenal medulla, causing a surge in catecholamines to be released, thus causing the client to experience headaches, hypertension, hyperglycemia, tremor, and unintentional weight loss. A client with pheochromocytoma is advised to modify their diet so that it does not increase blood pressure. Cheddar cheese contains tyramine and should not be included in the client's diet. Other dietary modifications include limitations of caffeinated beverages which may also raise blood pressure.

The nurse observes sparks fly from a client's bathroom light. Which action should the nurse take first? A. Obtain a fire extinguisher B. Close the bathroom door C. Remove the client from the room D. Activate the fire alarm.

Remove the client from the room Rationale: The safety of the client is the highest priority in any emergency situation. Removing the client from the room is the first and most crucial action to protect them from potential harm, such as electrical shock or fire-related injuries.

The nurse is attending a staff development conference about the causes of labor dystocia. It would be correct for the nurse to identify which of the following may cause a delayed progression during labor? Select all that apply.

Magnesium sulfate infusion Uterine overdistention Hypoglycemia Epidural analgesia Rationale: Magnesium sulfate relaxes the uterus and may decrease the intensity of uterine contractions. A decrease in intensity will decrease the progression of labor. Often when magnesium sulfate is infused, oxytocin may be used in conjunction. Uterine overdistention is a cause of labor dystocia because when the uterus is stretched, it does not contract properly. Hypoglycemia is a cause of delayed labor progression because of the maternal fatigue it induces. While epidural analgesia provides effective pain control, the decrease in sensation will also decrease the woman's drive to push and interfere with the internal rotation mechanism.

The nurse is developing a plan of care for a client admitted to the mental health unit with significant paranoia. Which of the following should the nurse include in the client's plan of care? Select all that apply.

Maintain consistent caregivers. Involve the client in decision-making. Rationale: A client experiencing paranoia may be very conspiratorial, and while it is important to reinforce reality, it would be appropriate to acknowledge their feelings. Involving the client in the decision-making process and avoiding any surprises is essential. Consistent caregivers are recommended because this cements the therapeutic relationship with staff.

The licensed practical/vocational nurse (LPN/VN) is assisting a registered nurse (RN) care for a client receiving mechanical ventilation for a traumatic brain injury. Which of the following actions by the RN requires the PN/VN to interevene? A. Suctioning the client when the high-pressure alarm sounds B. Hyperventilating with 100% FiO2 prior to suctioning C. Performs oral care with a chlorhexidine solution D. Maintaining the head of the client's bed at 90 degrees

Maintaining the head of the client's bed at 90 degrees Rationale: Maintaining a client's head of the bed at 90 degrees is detrimental for a client with a traumatic brain injury. The client should avoid hip and neck flexion as this raises ICP. Head of the bed recommendations for a client with a risk for increased ICP is 30 to 45 degrees. This is also recommended to prevent the client from developing ventilator-acquired pneumonia, as the question states the client is on mechanical ventilation.

Which of the following statements regarding mass casualty events are true? Select all that apply. Mass casualties are events that overwhelm local medical capabilities. When a mass casualty occurs, there is a need to increase the staff at the hospital. Many local agencies will collaborate to handle a mass casualty situation. An example of a mass casualty event is a fight between visitors in the intensive care unit.

Mass casualties are events that overwhelm local medical capabilities. When a mass casualty occurs, there is a need to increase the staff at the hospital. Many local agencies will collaborate to handle a mass casualty situation. Rationale: Mass casualties are events that overwhelm local medical capabilities. Another term for a mass casualty event is a disaster. When a mass casualty occurs, there is a need to increase the hospital staff to provide safe client care to everyone involved in the event. Many local agencies will collaborate to handle a mass casualty situation. Collaboration between multiple agencies and health care facilities will be necessary to provide safe care and proper handling of the crisis.

The nurse is caring for a client with appendicitis. Which of the following statements are correct regarding this condition? Select all that apply. McBurney's point tenderness is a sign of appendicitis. Appendicitis is more common among males. A low carbohydrate diet is a risk factor for appendicitis. Diagnosis of appendicitis is confirmed by endoscopic retrograde cholangiopancreatography. The client may have an elevated white blood cell count (WBC).

McBurney's point tenderness is a sign of appendicitis. Appendicitis is more common among males. The client may have an elevated white blood cell count (WBC). Rationale: McBurney's point tenderness refers to right lower quadrant pain, suggesting appendicitis. Rebound tenderness is a common finding when this area is palpated. Appendicitis is more common among males than females. A low-grade fever, abdominal pain in the right lower quadrant, and an elevated white blood cell count are common findings in appendicitis.

The home health nurse is assessing a client with suspected carbon monoxide poisoning. The nurse should take which priority action? A. Move the client outdoors B. Notify the primary healthcare provider (PHCP) C. Auscultate the client's lung sounds D. Assess the client's pulse oximetry

Move the client outdoors Rationale: Carbon monoxide poisoning is a serious emergency that is often fatal if not promptly treated. This medical emergency requires the client to be immediately relocated away from the carbon monoxide. Moving the client outside is effective because of the fresh air. Once this has been completed, the nurse should notify the PHCP or call emergency medical services (EMS) for further treatment. Another priority treatment is providing the client with 100% high-flow oxygen regardless of their pulse oximetry, lung sounds, or arterial blood gas results.

The nurse is caring for an 82-year-old with cancer. Due to the chemotherapy treatments, the client is unable to take in enough calories for his nutritional requirements. The provider orders a nasogastric (NG) tube to be inserted. Place the following actions in the order that the nurse takes to correctly insert the NG tube: Lubricate the tip of the NG tube. Insert the NG tube in the nare pushing it gently back and down toward the ear. Measure the length of the NG tube from the nose to the earlobe to the xiphoid process. Encourage the client to swallow a few sips of water and advance the NG tube past the oropharynx. Secure the NG tube at the pre-measured mark.

Measure the length of the NG tube from the nose to the earlobe to the xiphoid process. Lubricate the tip of the NG tube. Insert the NG tube in the nare pushing it gently back and down toward the ear. Encourage the client to swallow a few sips of water and advance the NG tube past the oropharynx. Secure the NG tube at the pre-measured mark.

Which of the following terms refers to the first stool passed by a newborn infant? A. Melena B. Meconium C. Diarrhea D. Hematemesis

Meconium Rationale: Meconium is defined as the first stool passed by a newborn infant. It is typically a dark black/green sticky stool.

The nurse assesses each of the following on a 12-year-old female. Which of the following growth milestones are expected for female adolescents of this age? Select all that apply.

Menarche Thelarche Rationale: Menarche is defined as the first occurrence of menstruation or the first time a girl gets her period. This is one of the most important milestones of female adolescents. It typically occurs about two years after thelarche, or the beginning of breast development. Thelarche is defined as the beginning of breast development at the onset of puberty. This is a significant milestone for female adolescents. It can occur anywhere between 8 years of age and 13 years, as there is substantial individual variation.

The LPN is reinforcing teaching to students about the causes of hypokalemia. Which of the following conditions should the nurse identify as being a cause of this electrolyte imbalance? Select all that apply. Diabetic ketoacidosis Addison's disease Metabolic alkalosis Chronic renal failure Cushing's syndrome

Metabolic alkalosis Cushing's syndrome Rationale: Hypokalemia is when the potassium level is less than 3.5 mEq/l. Conditions such as metabolic alkalosis, potassium wasting diuretics, Cushing's syndrome, and alcoholism may all contribute to hypokalemia.

ADHD medication:

Methylphenidate Rationale: ADHD may be treated by psychostimulants such as amphetamines or methylphenidate. These medications work by projecting the dopamine and norepinephrine in the front of the brain to ameliorate the symptoms of inattention, impulsivity, and hyperactivity.

Your client is experiencing severe, acute anxiety before a scheduled endoscopy procedure. Which of the following medications is most likely to be ordered by the physician? A. Oxycodone B. Midazolam C. Clonazepam D. Haloperidol

Midazolam Rationale: Midazolam is a benzodiazepine used for acute anxiety attacks. Midazolam is preferred in this setting because of its rapid onset (2 to 5 minutes after IV administration) and short duration of action (3 to 8 hours). It can be administered intravenously or orally. Given these benefits, midazolam would be the most useful for the client experiencing an acute anxiety attack prior to/during endoscopic procedures or prior to surgery.

Which action should the nurse take to most effectively reduce the incidence of hospital-associated urinary tract infections? A. Delegate perineal hygiene to unlicensed assistive personnel B. Review intake and output (I&O) charting for fluid imbalances C. Minimize the use of an indwelling urinary catheter D. Request random urine analysis (UA) from the primary healthcare provider

Minimize the use of an indwelling urinary catheter Rationale: The most effective way to reduce the incidence of UTIs is to avoid using invasive urinary catheters. These types of catheters should be limited in their use and removed as soon as possible. Studies have demonstrated that bacteria can start to colonize about 48 hours following the insertion.

Correct statements about the proper administration of polyethylene glycol prescribed for constipation:

Mix the powder with any beverage that the patient enjoys Administer at the same time every day Instruct that the medication can take 1-3 days to work Rationale: It is appropriate to mix polyethylene glycol with any beverage the patient enjoys. Soda and juice are common choices due to their ability to mask the flavor better than water. It is preferable to administer polyethylene glycol at the same time every day. This promotes a bowel regimen and routine, which maximizes the success of the medication. It is also useful to help the client remember to take their medication with a routine established. Polyethylene glycol can take 1-3 days to produce a bowel movement, and it is important to continue taking it as directed until a bowel movement occurs.

A cardiac intensive care unit nurse is caring for a client who underwent a coronary artery bypass graft (CABG) 24 hours ago. The nursing care plan indicates a nursing diagnosis of "decreased cardiac output related to alterations in cardiac contractility." Based on the formulated nursing diagnosis, which nursing intervention should be implemented in the nursing care plan? A. Monitor the client's arterial blood gas (ABG) continuously. B. Monitor the client's weight daily and calculate the change. C. Administer prescribed opioids. D. Monitor mediastinal chest tubes for hourly output.

Monitor the client's weight daily and calculate the change. Rationale: Monitoring the daily weight of the client and noting any changes provides the nurse with a picture of the client's fluid volume status, which is influenced by the client's cardiac output. Weight is the most reliable indicator of fluid gain and loss.

A client is being discharged from the hospital after being diagnosed with lupus erythematosus. The client is advised to follow up with what to monitor his condition? A. HgbA1C B. Daily blood pressure checks C. Monthly urine specimens D. Monthly CBC

Monthly urine specimens Rationale: Lupus erythematosus, particularly systemic lupus erythematosus (SLE), can affect various organs including the kidneys. One common complication is lupus nephritis, where the kidneys become inflamed. Monitoring kidney function is crucial for individuals with lupus to detect any signs of kidney involvement and assess disease activity. Regularly checking urine specimens for the presence of protein, blood, or other abnormalities can provide valuable insights into kidney function and lupus-related kidney problems.

The LPN is working on the pediatric floor caring for a 2-year-old who receiving 100% FiO2 via a nasal cannula. At the end of her shift, the hospital receives a tornado warning. Which of the following actions should the nurse take to best protect her patient? A. Clock out, her shift is over and she is not responsible. B. Remove the nasal cannula and carry the child to a tornado shelter. C. Move the patient as close to the interior of the room as possible. D. Close all of the doors.

Move the patient as close to the interior of the room as possible. Rationale: During a tornado warning, the appropriate nursing action is to move patients away from windows and as close to the interior of the room as they can safely be moved. This best protects them in the event of a tornado.

The nurse is caring for a client with streptococcal pharyngitis (tonsillitis) who has been placed on droplet precautions. Which of the following statements indicates the best understanding of this type of isolation? A. The client can be placed in a room with another client with measles (rubeola). B. A special mask (N95) should be worn when working with the client. C. Must maintain a spatial distance of 3 feet. D. Gloves should be worn only when giving direct care.

Must maintain a spatial distance of 3 feet. Rationale: A spatial distance of at least 3 feet is recommended. The most common forms of transmission of an organism in a client with tonsillitis are coughing, sneezing, and talking. Droplets can travel no more than 3 ft, so precautions should be maintained when there is a possibility of entering this distance.

The nurse is caring for a client immediately following an ultrasound-guided thoracentesis. Which client finding requires follow-up? Select all that apply. Nagging cough Trachea slanted more to the unaffected side Rapid heart rate Localized discomfort at the needle site Crackling sound made at the insertion site when palpated

Nagging cough Trachea slanted more to the unaffected side Rapid heart rate Crackling sound made at the insertion site when palpated Rationale: Complications following a thoracentesis include pneumothorax, bleeding, and infection. The biggest concern is pneumothorax. Following a thoracentesis, the client should be assessed for increased respiratory rate, elevated heart rate, nagging cough, decreased oxygen saturation, decreased breath sounds, and air that makes a popping sound near the insertion site when palpated. These are complications and warrant an immediate chest radiograph and supportive measures.

The nurse is counseling a client with opioid use disorder. The nurse understands that treatment choices for opioid use disorder include which of the following? Select all that apply. Selegiline Naltrexone Methadone Buprenorphine Bupropion

Naltrexone Methadone Buprenorphine Rationale: Naltrexone, Methadone, and Buprenorphine are three agents approved for the management of opioid use disorder. These medications have various mechanisms of action. Naltrexone is an opioid receptor antagonist and may be administered as a single-dose injection. Buprenorphine is a partial agonist and is available in preparations such as sublingual tablets or film. Methadone is a full agonist that may be used daily.

The nurse has attended a staff conference on cultural considerations. Which of the following statements would be correct regarding the Amish culture? Select all that apply. Natural and alternative treatments are common in the community Church is extremely important in this community Women and men are equal and can both make healthcare decisions Most of the Amish community choose to have health insurance Blood and blood products are commonly declined

Natural and alternative treatments are common in the community Church is extremely important in this community Rationale: Alternative medical choices and natural treatments are commonly used in this culture. The nurse should recognize this fact because standard therapies may be abandoned for treatments that may be unproven. Church and religion are fundamental in this community. If an individual in the community is ill, it is common for a religious leader to request updates about the client's condition.

While reviewing a client's medication list, the nurse understands which prescribed medication(s) is/are classified as calcium channel blockers? Select all that apply. Nifedipine. Propranolol. Verapamil. Hydralazine. Digoxin.

Nifedipine. Verapamil. Rationale: Nifedipine and verapamil are calcium channel blockers ( CCBs). Other CCBs include amlodipine, nicardipine, felodipine, and diltiazem. CCBs are broadly classified into dihydropyridine and non-dihydropyridine classes. The dihydropyridine calcium channel blockers ending with the suffix "-dipine" are more selective to the vascular system. They cause systemic vasodilation and are used to decrease blood pressure (treat hypertension). Verapamil is a non-dihydropyridine calcium channel blocker relatively selective to the myocardium. Verapamil reduces myocardial contractility (negative inotropy) and is often used to treat angina. Tachycardia is detrimental in angina because an increased heart rate increases myocardial oxygen demand. Verapamil is a preferred CCB in treating angina because it does not cause much vasodilation and minimal reflex tachycardia.

Which of these is the best example of an ethical principle in nursing? A. Fidelity - the nurse, maintained honesty with the client during any education and care. B. Veracity - the nurse followed through with any promises made to the client during his or her care. C. Beneficence - the nurse promoted the client to be respected and involved in his or her care. D. Nonmaleficence - the nurse did not cause harm to the client.

Nonmaleficence - the nurse did not cause harm to the client. Rationale: The nurse did not cause harm to the client, which is known as nonmaleficence. The ethical principles that nurses must adhere to are the principles of justice, beneficence, nonmaleficence, accountability, fidelity, autonomy, and veracity.

The LPN is providing care to a client with a tracheostomy tube and notes the client's respiratory rate and heart rate increase during suctioning. Which of the following actions are appropriate for the nurse to take? Select all that apply. Notify the RN Call a rapid response Hurry up and finish the suctioning Stop suctioning Check the oxygen saturation

Notify the RN Stop suctioning Check the oxygen saturation Rationale: Tracheal suctioning for purposes of maintaining an open airway falls within the scope of LPN practice. LPNs may also perform nasopharyngeal and oral pharyngeal suctioning. However, deep suctioning or bronchial tree suctioning is out of the scope of LPN practice. It is important for the LPN to understand the complications of tracheal suctioning. If the nurse notices a change in vital signs (tachycardia, tachypnea) while suctioning a patient, it is a sign of distress which indicates that the client is not tolerating the suctioning. The LPN should stop the suctioning, check the oxygen saturation, and notify the RN immediately. Hypoxemia is an important cause of tachycardia and cardiac arrhythmias during suctioning. Other things to monitor for would be bradycardia, changes in the heart rhythm (arrhythmias), desaturations, or cyanosis. It would be appropriate for the LPN to check the O2 saturation and provide supplemental O2 as ordered.

You are assessing a 4-year-old preschooler and note the following vital signs: Pulse: 146 RR: 42 BP: 72/48 Which of the following actions are appropriate given these vital signs? Select all that apply.

Notify the healthcare provider. Administer IVF as ordered by your health care provider. Rationale: This is an appropriate action. The health care provider should be notified immediately. The nurse has identified that the child is in shock and should immediately notify the healthcare provider. This is an appropriate action. The nurse has identified that the child is in shock due to their hypotension and tachycardia. She notifies the health care professional and is expecting an order for IV fluids. This will help to increase the blood pressure, beginning the process of addressing the shock by increasing the blood pressure to provide profusion to the vital organs. This is an appropriate action. These vital signs are not within the normal limits for a preschooler. Typical vital signs for a preschooler are: Pulse: 80-120 RR: 20-30 BP: 95/65

When assessing a client who has been ordered skeletal traction, the assessment reveals her foot is pale, cool, and her pulse is not palpable. What is the priority nursing intervention? A. Reassess the foot in twenty minutes. B. Readjust the traction. C. Administer the ordered as-needed medication. D. Notify the physician.

Notify the physician. Rationale: The symptoms indicate circulatory impairment. The physician (or practitioner) must be notified immediately.

The nurse is caring for a client with a suspected femoral artery occlusion. The nurse should take which action? A. Elevate the affected leg B. Apply a cold compress C. Notify the primary healthcare provider (PHCP) D. Perform passive range of motion to the affected leg

Notify the primary healthcare provider (PHCP) Rationale: The essential action for a client with a suspected femoral artery occlusion is to notify the PHCP or rapid response. This is a medical emergency! If untreated, this extremity may have to be amputated because of the interruption in distal perfusion.

The LPN is caring for an infant who will be going to the operating room the next day for surgical repair of his total anomalous pulmonary venous return. She has finished signing the consent paperwork for the operation. The mother states "I'm not so sure about this. What if my baby dies?!" What is the appropriate action for the LPN? A. Explain the procedure to the mother. B. Notify the surgical team and have them come back to speak with the mother. C. Reassure the mother that everything will go as planned. D. Tell the mother that because she has already signed the consent paperwork she cannot change her mind now.

Notify the surgical team and have them come back to speak with the mother. Rationale: The LPN has identified that the mother has concerns about the surgery, so it is her responsibility to notify the surgical team and have them come back to speak with the mother.

The nurse is conducting a health screening at a local health fair. Which of the following should the nurse recognize as a risk factor for developing ovarian cancer? Select all that apply. Nulliparity Advancing age Family history Herpes simplex virus (HSV) Early menarche

Nulliparity Advancing age Family history Early menarche

While monitoring a client with myocardial infarction who is receiving tissue plasminogen activator (Activase, TPA), the nurse should prioritize which of the following? A. Observe for neurological changes. B. Monitor for any signs of renal failure. C. Observe for signs of bleeding. D. Check the client's food diary.

Observe for signs of bleeding. Rationale: Bleeding is the priority concern for any client who is taking a thrombolytic medication. Bleeding is a potential adverse effect of tPA therapy, and prompt recognition and intervention are crucial to prevent complications.

The nurse is developing a plan of care for a client diagnosed with Kawasaki disease. Which of the following should the nurse include in the client's plan of care? Select all that apply. Initiate contact precautions Obtain a 12-lead electrocardiogram Offer soft foods and liquids Implement fluid restriction Administer aspirin, as prescribed

Obtain a 12-lead electrocardiogram Offer soft foods and liquids Administer aspirin, as prescribed Rationale: Kawasaki disease is an autoimmune disorder that occurs primarily in individuals younger than five. This disease process may consequently cause inflammation to the coronary arteries leading to aneurysms. Thus, an electrocardiogram should be performed along with an echocardiogram. Soft foods and liquids should be offered because of the chapping of the lips. Fluids would be encouraged because of the fever commonly associated with Kawasaki disease. Finally, treatment for this disease includes either medium to high dose aspirin or intravenous immunoglobin.

The nurse is caring for a client who sustained an electrical burn. The nurse plans to take which priority action? A. Obtain an electrocardiogram (ECG) B. Prepare for an arterial blood gas (ABG) collection C. Provide wound care D. Initiate supplemental oxygen

Obtain an electrocardiogram (ECG) Rationale: Electrical burns are serious and require the client to undergo cardiac monitoring because of the risk of dysrhythmias. The nurse's priority action is to obtain telemetry monitoring or perform a 12-lead electrocardiogram.

The nurse is caring for a client who was newly placed on a clozapine prescription. Which of the following teaching points should the nurse reinforce? A. Maintain a healthy diet because of weight gain B. Exercise regularly and maintain hydration C. Expect excessive secretions in the mouth D. Obtain follow-up laboratory work

Obtain follow-up laboratory work Rationale: Follow-up laboratory work is essential for a client taking clozapine. The medication may adversely cause neutropenia. The client will be instructed to obtain this necessary laboratory work to ensure they are not experiencing agranulocytosis, which may make the client susceptible to infection.

The nurse is taking the vital signs of a client with a diagnosis of acute lymphoblastic leukemia (ALL). His temperature is 38.7 degrees C. What is the nurse's priority? A. Place cool washcloths on the client's head. B. Continue with the assessment. C. Obtain intravenous access on the client. D. Assess the client's perfusion.

Obtain intravenous access on the client. Rationale: It is the priority action to establish intravenous access to this client. This client has a diagnosis of acute lymphoblastic leukemia (ALL), so the nurse knows that the client is immunocompromised. The client is very susceptible to infections, and with a fever of 38.7 degrees C, there should be a high index of suspicion for disease. Broad-spectrum IV antibiotics will need to be started right away. Therefore it is the priority for the nurse to start an IV.

The nurse is visiting the home of a client with Clostridium difficile. Which infection control measure should the nurse include? A. Ask the client to wear a surgical mask during the visit. B. Obtain vital signs with a disposable blood pressure cuff. C. Interview the client while maintaining 3 feet distance. D. Use sterile gloves when performing venipuncture.

Obtain vital signs with a disposable blood pressure cuff. Rationale: C. diff is a spore-producing bacterium that allows it to be transmitted between clients, environmental surfaces, and contaminated hands. Obtaining vital signs with disposable equipment is recommended to prevent the transmission of this pathogen.

The nurse is participating in a committee with the objective of promoting healthcare justice in the community. Which of the following recommendations should the nurse make to achieve the goal? A. Establishing interdisciplinary collaboration between nursing and nutritional services B. Providing more confidential waste containers at local drug stores C. Offering free telehealth offerings in underserved areas of the community D. Offering inpatient clients the ability to select their meal times

Offering free telehealth offerings in underserved areas of the community Rationale: The premise of social justice is expanding access to affordable healthcare for all individuals. The nurse recommending health services for underserved areas is a way to improve health inequalities in the community. Another example would be the nurse endorsing expanding health services and eligibility for Medicaid.

The nurse is caring for an infant who presents with a congenital abnormality where their abdominal contents come through the umbilicus while remaining in the peritoneal sac. The nurse knows the infant will be diagnosed with which of the following? A. Intussusception B. Gastroschisis C. Omphalocele D. Hirschsprung's disease

Omphalocele Rationale: This infant has an omphalocele. An omphalocele is a congenital abnormality where the abdominal contents come through the umbilicus while remaining in the peritoneal sac.

The LPN is reinforcing education regarding advance directives with the client. Which of the following statements are not true regarding advance directives? Select all that apply.

Only one physician must determine when a client is unable to make medical decisions for himself. Advance directives must be reviewed and re-signed every ten years to remain valid. An advance directive is legally valid in every state, no matter which state it was initially created. Rationale: Once a client arrives at a hospital, physicians will need to evaluate the client and implement the advance directive, if necessary. Two physicians, not one are required to determine whether a client cannot make decisions for themself. Advance directives do not expire and remain in effect until they are changed. It is not true that they need to be signed every ten years to stay valid. Some states do not honor advance directives created in other states. So, if a client moves, he/she should check with his/her new state policies on the topic.

As you are bathing your client and providing nail care, you notice that the client's nails look abnormal. You would document this nail abnormality as: A. Onychomycosis B. Onychomadesis C. Onychorrhexis D. Onychia

Onychomycosis Rationale: You would document this nail abnormality as onychomycosis. Onychomycosis is a fungal infection of the nails (fingernails, toenails) that causes the nails to look thick, discolored, opaque, and crumbling. Since onychomycosis is the most common cause of nail dystrophy a nurse plays a crucial role in the diagnosis, management, and education of the clients.

A nursing assistant tells the nurse that her client with COPD did not get his annual flu shot this year and has not had a pneumonia vaccination. The CNA should report which finding to the nurse immediately A. Blood Pressure of 150/80 mm/Hg B. Respiratory rate of 26 breaths/min C. The heart rate of 92 beats/min D. Oral temperature of 101.4 degrees F

Oral temperature of 101.4 degrees F Rationale: An elevated temperature indicates some form of infection, possibly respiratory in origin. A client who did not receive pneumonia or influenza vaccine is at increased risk of developing pneumonia and influenza. Monitoring for signs/symptoms of infection is a crucial nursing intervention.

The terms used to describe the patterns of growth and development according to individual growth patterns and developmental levels include which of the following? Select all that apply. Orderly Simple Sequential Unpredictable Differentiated Integrated

Orderly Sequential Differentiated Integrated Rationale: Growth and development are orderly, sequential, continuous, and complex. All humans experience the same general growth patterns and developmental levels, but because these patterns and levels are individualized, a wide variation in biological and behavioral changes is expected. Certain milestones can be identified within each developmental level, such as when the infant rolls over, crawls, walks, or says their first words. Although growth and development occur individually for different people, certain generalizations can be made about the nature of human development for everyone.

The nurse is caring for a client who has influenza. Which of the following prescriptions may be prescribed by the primary healthcare provider (PHCP)? A. Valacyclovir B. Oseltamivir C. Azithromycin D. Omeprazole

Oseltamivir Rationale: Oseltamivir is an antiviral agent approved for the treatment of influenza. This medication should be initiated within 48 hours of symptom onset.

The nurse recognizes which of the following anticholinergics is appropriate for a client diagnosed with urinary bladder urgency and incontinence? A. Dicyclomine B. Ipratropium C. Oxybutynin D. Scopolamine

Oxybutynin Rationale: Oxybutynin is an anticholinergic drug. It is used to treat urinary bladder urgency and incontinence. It works as a bladder relaxant and has 10 times more potent antispasmodic effect than atropine. Anticholinergics are drugs that have actions opposite those of the parasympathetic branch. Their work mimics the fight-or-flight response. Dicyclomine is used to treat irritable bowel syndrome. Ipratropium is used to treat asthma. Scopolamine is used to treat irritable bowel syndrome and motion sickness.

The nurse receives the following critical laboratory result for a client with end-stage renal disease. The nurse anticipates the physician to prescribe which blood product? See the image A. Packed Red Blood Cells (PRBCs) B. Fresh Frozen Plasma (FFP) C. Albumin D. Platelets

Packed Red Blood Cells (PRBCs) Rationale: This hemoglobin and hematocrit are critically low. A transfusion of PRBCs is typically indicated once the hemoglobin is 7 g/dL or less.

*NGN* The nurse is caring for a client with a pneumothorax who has a wet suction chest tube Item 1 of 1 Nurses' Notes 1930 - Client reports pain at the chest tube insertion site 4/10, described as a 'soreness' and precipitated with movement—puffiness and crackling at the insertion site upon palpation. Tidaling was seen in the water seal chamber with intermittent bubbling. The suction control chamber had no bubbling and was filled to - 20 cm. Orders Chest tube at (-) 20 cm with wall suction For each assessment finding, click to specify if it is expected or requires follow-up Assessment Findings Pain at the insertion site (Expected or Requires Follow up) Tidaling in the water seal chamber (Expected or Requires Follow up) Intermittent bubbling in the water seal chamber (Expected or Requires Follow up) No bubbling in the suction control chamber (Expected Requires or Follow up) Puffiness and crackling at insert

Pain at the insertion site: Expected Tidaling in the water seal chamber: Expected Intermittent bubbling in the water seal chamber: Expected No bubbling in the suction control chamber: Requires Follow up Puffiness and crackling at insertion site upon palpation: Requires Follow up Rationale: The nurse must understand expected findings for a client who has a wet suction chest tube for a pneumothorax. These expected findings include pain or soreness at the insertion site. Clients are often prescribed pain medication mitigate the discomfort. The water seal chamber prevents air from re-entering the pleural space. The water in this chamber rises during inhalation and falls during exhalation; thus, tidaling is expected. Intermittent bubbling in this chamber is normal during exhalation or coughs. Findings requiring follow-up include no bubbling in the suction control chamber. This is concerning because the order states wall suction is to be added. The nurse should check the connections and suction apparatus to troubleshoot. If wall suction is not added, then no bubbling will occur. Puffiness and crackling at the insertion site upon palpation are concerning. This is indicative of a subcutaneous air leak and should be reported.

What should the Licensed Practical Nurse include in the care plan for a 2-year-old client who had surgery to remove one kidney due to stage III nephroblastoma? Select all that apply. Pain management Vital signs monitoring Incision site care Fluid intake and output monitoring Ambulation and exercise

Pain management Vital signs monitoring Incision site care Fluid intake and output monitoring Rationale: Pain management: The LPN should include measures to assess and manage the child's pain, such as administering prescribed pain medications, monitoring pain levels, and providing comfort measures. Vital signs monitoring: The LPN should regularly assess the child's vital signs, including temperature, heart rate, blood pressure, and respiratory rate. Incision site care: The LPN should include proper care of the surgical incision site, such as assessing for signs of infection, changing dressings as needed, and keeping the area clean and dry. Fluid intake and output monitoring: The LPN should track the child's fluid intake and output, including urine output, to ensure proper hydration and kidney function.

A patient who is 2-days postoperative from right femoral popliteal bypass surgery complains of worsening right leg pain. Upon assessment, the nurse notes swelling and ecchymosis at the incision sites. Which action would be the nurse's initial priority? A. Apply pressure to sites with a sandbag B. Palpate pedal pulses C. Assess for signs of claudication D. Apply a warm compress to the incision sites

Palpate pedal pulses Rationale: The most significant complications that this patient is at risk for after the revascularization procedure are thrombus, hemorrhage, infection, and arrhythmias. Mild to moderate swelling, bruising, and pain at the surgical site are expected and typically resolve over time as the leaked blood is reabsorbed. The most important action would be to assess the patient's pedal pulses (distal to incisions). If pulses are intact, the nurse would then address the patient's complaint of worsening pain.

The nurse is reinforcing education to a group of nursing students about abdominal assessment. The nurse knows that which of the following steps is the final step used during data collection of the abdomen? A. Inspection B. Palpation C. Percussion D. Auscultation

Palpation Rationale: Palpation is the final step in data collection of the abdomen. It involves gently pressing the abdomen to feel for masses, tenderness, or organs' size and shape beneath the skin surface. Palpation is performed last to avoid altering any findings from inspection, auscultation, and percussion, ensuring a comprehensive data collection of the abdomen.

The nurse in the medical-surgical unit cares for a client who is postoperative Item 1 of 1 Progress Notes Day 2 Status-post open (partial) gastrectomy. Client reports abdominal pain 6/10 on the Numerical Rating Scale. Client is still receiving patient-controlled analgesia (PCA) of fentanyl. Abdomen is taut with hypoactive bowel sounds in all quadrants. Reports passing no flatus. Endorses occasional nausea with no vomiting. Incision approximated with moderate serosanguinous drainage. Abnormal laboratory data noted: BUN 19 mg/dL (10 - 20 mg/dL); Creatinine 1.0 mg/dL (0.6 - 1.2 mg/dL); Potassium 3.3 mEq/l (3.5 - 5.0 mEq/l). Not tolerating a clear liquid diet, only consuming 10-15% of meals. No VS abnormalities. Day 3 Status-post open (partial) gastrectomy. Client reports abdominal pain 4/10 on the Numerical Rating Scale. Client is still receiving patient-controlled analgesia (PCA) of fentanyl. Abdomen is distended,

Paralytic ileus Hypokalemia Infusion of fentanyl Rationale: The client is at the highest risk for paralytic ileus related to persistent hypokalemia and the infusion of fentanyl the client is getting via PCA. Paralytic ileus is concerning for clients who have undergone any abdominal surgery. Combine this with hypokalemia and an opioid that reduces peristalsis, making the client's risk for paralytic ileus quite credible.

The nurse is preparing to admit a client following lumbar spinal fusion surgery. The nurse should instruct the unlicensed assistive personnel (UAP) to have which equipment at the bedside? A. Overhead trapeze B. Abduction pillow C. Transfer board D. Continuous passive motion (CPM)

Transfer board Rationale: Following a lumbar spinal fusion, the client will need to be log rolled. A transfer board/sheet, along with an ample amount of staff (at least three), will be necessary to facilitate the log roll.

The nurse is caring for a client with an ectopic pregnancy. Which findings does the nurse expect? Select all that apply. Pelvic pain Fever Vaginal bleeding Positive pregnancy test Dysuria

Pelvic pain Vaginal bleeding Positive pregnancy test Rationale: An ectopic pregnancy is a gynecological emergency. If not treated, a rupture may occur that leads to intraperitoneal bleeding. Classic manifestations of an ectopic pregnancy include: Presumptive pregnancy signs (nausea, breast tenderness) Vaginal bleeding Increased human chorionic gonadotropin (hCG) levels causing a positive pregnancy test Unilateral abdominal pain that may be confined to the pelvic area

When the nurse begins to irrigate a Salem Sump tube, she notices that the gastric drainage is dark brown. What is the first intervention the nurse should take? A. Perform a hemoccult test on the contents. B. Irrigate the tube and check the returns. C. Remove the tube from the suction. D. Check the pH of the gastric contents.

Perform a hemoccult test on the contents. Rationale: The presence of dark brown drainage may indicate the presence of bleeding or blood in the GI tract.The first nursing intervention is to perform a hemoccult test to determine the presence of blood.

Although you were informed that your assigned client has no special skincare needs, you observe reddened areas over bony prominences upon your assessment. What is the next appropriate action? A. Correct the initial assessment form. B. Redo the assessment and document the current findings. C. Conduct and document an emergency assessment. D. Perform and document a focused assessment of skin integrity.

Perform and document a focused assessment of skin integrity. Rationale: Performing and documenting a focused assessment of skin integrity is appropriate since this is a newly identified problem.

The nurse is obtaining vital signs for a client who has acquired immune deficiency syndrome (AIDS). Prior to entering the room, the nurse should do which of the following? A. Wear gloves and a gown. B. Perform hand hygiene. C. Review the client's viral load. D. Obtain a disposable stethoscope.

Perform hand hygiene. Rationale: When caring for a client who has AIDS, the nurse should maintain standard precautions. Applying PPE such as a gown, pair of gloves, and mask would be inappropriate. Standard precautions require appropriate hand hygiene and other PPE as needed.

Your 78-year-old client, who has been receiving antibiotics for 10 days, tells you that he has frequent watery stools. Which action will you take first? A. Place the client on contact precautions. B. Instruct the client about correct handwashing techniques. C. Obtain stool specimens for culture. D. Notify the physician about the loose stools.

Place the client on contact precautions. Rationale: The client's age, history of antibiotic therapy, and watery stools suggest that he may have Clostridium difficile infection. The initial action is to place him on contact precautions to prevent the spread of C. difficile to other clients. C. difficile is a spore-forming bacillus that infects the gastrointestinal (GI) tract following treatment of other infections with antibiotics. It is one of the few hospital-acquired infections (HAIs) increasing in frequency. C. difficile spores are transferred to clients mainly via the hands of health care personnel who have touched a contaminated surface or item.

The nurse is assisting with the care of a client with a chest tube. Which of the following interventions would be appropriate? Select all that apply. Periodically check that all connections are secure. Tape the tubing to the bed. Check the tubing frequently for kinks and dependent loops. Refill the water-seal chamber once a shift. Assist with chest tube drainage by measuring and recording the amount of drainage, checking for proper suction, and ensuring that the drainage system is functioning properly.

Periodically check that all connections are secure. Check the tubing frequently for kinks and dependent loops. Assist with chest tube drainage by measuring and recording the amount of drainage, checking for proper suction, and ensuring that the drainage system is functioning properly. Rationale: It is appropriate for the nurse to periodically check that all connections are secure when assisting with the care of a client with a chest tube. The chest tube drainage system will only function if it is a closed system, and in order for that to be true all the connections must be secure and air tight. It is appropriate for the nurse to check the tubing frequently for kinks and dependent loops when assisting with the care of a client with a chest tube. If there are kinks or dependent loops in the tubing the chest tube drainage system will be obstructed and not drain fluid from the client. Ensuring that this does not occur helps the chest tube drainage system remain patent so that the client's lung may expand to a normal state. The LPN may assist with chest tube drainage by measuring and recording the amount of drainage, checking for proper suction, and ensuring that the drainage system is functioning properly.

The nurse is caring for a client who has a serum calcium level of 13.2 mg/dL(9-10.5 mg/dL). Which of the following medications would be expected orders for this condition? Select all that apply. Phosphorus Calcitonin Vitamin D IV calcium gluconate Glucocorticoids

Phosphorus Calcitonin Glucocorticoids Rationale: The normal serum calcium level is 8.4-10.2 mg/dL. This client has a high serum calcium level (hypercalcemia). Phosphorus is a medication that the nurse would expect to administer to treat hypercalcemia. Phosphorus and calcium have an inverse relationship, so by increasing the serum level of phosphorus the nurse can decrease the serum level of calcium. Oral phosphate is the preferred method of administering phosphorus. If given IV, calcium phosphate forms and precipitates in the tissues. This "precipitation phenomenon" reduces serum calcium levels very quickly. Calcitonin is a medication that the nurse would expect to administer to treat hypercalcemia. Calcitonin is a thyroid hormone that decreases the plasma calcium level by inhibiting bone resorption and lowering the serum calcium concentration. Glucocorticoids are steroids that can be used to treat a variety of conditions, including hypercalcemia associated with certain cancers. Glucocorticoids work by inhibiting the activity of cells that break down bone and release calcium into the blood.

The nurse is reviewing a plan of care for a client who has a halo vest immobilizer (halo brace) following a cervical spine fracture. Which of the following should the nurse expect to be in the client's plan of care? Select all that apply. Pin care every shift Neck flexion and extension exercises Placing the wrench on the device Report loosening of the pins Use straws when providing liquids

Pin care every shift Placing the wrench on the device Report loosening of the pins Use straws when providing liquids Rationale: A halo vest immobilizer is used to stabilize cervical spinal cord injuries. The goal is to stabilize the spinal cord using external fixation. Pin care should be completed every shift using sterile gauze and the prescribed solution. If the client should have the wrench taped to the front of the vest or at the head of the bed in case the device needs to be taken down for emergent cardiopulmonary resuscitation. Loosening of the pins is the most common complication and should be addressed immediately. The client should use a straw for drinking as moving the neck to swallow liquids is not permitted.

The nurse is helping a client with a chest tube ambulate to the bathroom. The client turns suddenly and the chest tube becomes dislodged. Which of the following is the priority nursing action? A. Immediately re-insert the tube and call for help. B. Place your hand over the chest tube site and yell for help. C. Place a sterile dressing taped on three sides over the chest tube site and call for help. D. Monitor the client's vital signs while he finishes ambulating to the bathroom and then call for help.

Place a sterile dressing taped on three sides over the chest tube site and call for help. Rationale: Placing a sterile dressing taped on three sides over the chest tube site and calling for help is the appropriate action. By placing a sterile dressing over the site the nurse follows infection prevention. By taping the dressing on three sides, the dressing will cover the site, and prevent a tension pneumothorax by allowing exhaled air to escape the dressing. The nurse should then immediately call for help.

The nurse is caring for a client with a jejunostomy tube receiving intermittent enteral feedings. Which intervention would be the highest priority to reduce the risk of aspiration for this client? A. Flush the tubing with 20 mL water after feeding is completed. B. Position the client in a left-lying position after feedings. C. Assess blood glucose every 6 hours. D. Place the client in semi-Fowler's following feedings.

Place the client in semi-Fowler's following feedings. Rationale: The nurse should assist this client in semi-Fowler's position or lay them on the right side following feedings, as these positions will reduce the risk of leakage, gastric reflux, and aspiration.

A client who has recently traveled to another country presents to the emergency room with shortness of breath and suspected severe acute respiratory syndrome (SARS). What should the nurse's first intervention be? A. Place the client on contact and airborne precautions. B. Obtain blood, urine, and sputum for culture. C. Administer methylprednisolone 1 gram/IV. D. Infuse normal saline at 100 mL/hr.

Place the client on contact and airborne precautions. Rationale: A client who has recently traveled to another country presents to the emergency room with shortness of breath and suspected severe acute respiratory syndrome (SARS). What should the nurse's first intervention be?

The nurse is preparing to insert an indwelling urinary catheter. Which action may be delegated to the unlicensed assistive personnel (UAP)? A. Set up the sterile field B. Palpate the bladder for distention C. Explain the procedure to the client D. Place the urinary catheter kit at the bedside

Place the urinary catheter kit at the bedside Rationale: This task is appropriate to delegate to the UAP. Gathering supplies (suction, vital sign equipment, etc.) is within the scope of a UAP.

Which of the following client room assignments should the nurse question? A. Placement of an 89-year-old client with acute delirium at the end of the hallway. B. Placement of a 79-year-old client with C. difficile with a 26-year-old client with C. difficile. C. Placement of a 56-year-old client with HIV and bronchitis in a private room. D. Placement of a 39-year-old client with mild head trauma with a 40-year-old client with an arm fracture.

Placement of an 89-year-old client with acute delirium at the end of the hallway. Rationale: Clients with delirium and those at high risk for safety events should be roomed close to the nurse's station to accommodate close monitoring. In addition to the delirium, this client's age poses a risk for injury related to falls.

The nurse is caring for a client prescribed enoxaparin. Which laboratory values should the nurse monitor? A. Platelet count B. Activated Partial Thromboplastin Time (aPTT) C. International Normalized Ratio (INR) D. Troponin

Platelet count Rationale: Enoxaparin is a low molecular weight-based heparin, and the platelet count will need to be monitored if the client should develop heparin-induced thrombocytopenia (HIT). This condition is serious and results in a 50% decrease in the platelet count.

The licensed practical/vocational nurse (LPN/VN) is assisting in caring for a client who had bariatric surgery. Which of the following should be included? A. Pneumatic compression devices B. Insertion of an indwelling urinary catheter bed rest C. Strict bed rest D. Fluid restrictions

Pneumatic compression devices Rationale: Following bariatric surgery, the client faces an array of complications, including hemorrhage, wound disruption, pneumonia, and wound infection. Venous thromboembolism is a significant complication and may be mitigated using pneumatic compression devices and chemical prophylaxis.

Which of the following immunizations is a priority for a client who is 75-years-old and has a history of cerebrovascular disease? A. Hepatitis A vaccine B. Hepatitis B vaccine C. Pneumococcal vaccine D. Lyme disease vaccine

Pneumococcal vaccine Rationale: The pneumococcal vaccine is a priority immunization among elderly clients and those with chronic illnesses. This vaccine should be administered every five years.

George, age 8, is admitted with rheumatic fever. Which clinical finding indicates to the nurse that George needs to continue taking the salicylates he had received at home? A. Chorea B. Polyarthritis C. Subcutaneous nodules D. Erythema marginatum

Polyarthritis Rationale: Polyarthritis is characterized by swollen, painful, hot joints that respond to salicylates. Rheumatic fever is an inflammatory disease that can develop when strep throat or scarlet fever, caused by streptococcus bacteria, isn't adequately treated. It most often affects children between 5 and 15 years old, though it can develop in younger children and adults. Although strep throat is frequent, rheumatic fever is rare in the United States and other developed countries. However, rheumatic fever remains common in many developing nations. Rheumatic fever can cause permanent damage to the heart, including damaged heart valves and heart failure. Treatments can reduce inflammation, lessen pain and other symptoms, and prevent the recurrence of rheumatic fever.

While reinforcing education with a family who has a 1-year-old son diagnosed with phenylketonuria (PKU), the mother states, "I think we should just stick to the formula. I am too scared to feed him other foods." The LPN should review which of the following foods for this family to avoid? Select all that apply. Pork tenderloin Green beans Cheese omelets Pears Peas

Pork tenderloin Cheese omelets Peas Rationale: It is appropriate to inform the family to avoid pork tenderloin. Since pork tenderloin is high in protein and phenylalanine, the nurse should advise the family to avoid this food. It is appropriate to inform the family to avoid foods with eggs, such as cheese omelets. Foods high in protein have this amino acid and should be avoided. It is appropriate to inform the family to avoid dairy products, including milk, cheese, yogurt, and ice cream. The nurse should recommend sorbet or Italian ice as an alternative.

The nurse is preparing to perform a fetal non-stress test on a patient who is 34 weeks pregnant. Which action would be most important for the nurse to perform prior to this procedure? A. Explain the possible risk of inducing early labor. B. Confirm the patient's NPO status for at least 4 hours prior to the test. C. Administer oxytocin to stimulate uterine contraction. D. Position the patient in Sims position.

Position the patient in Sims position. Rationale: The nurse should instruct the patient to void prior to the test, then assist her into the left-lying Sims position to promote optimal oxygen delivery to the fetus.

The nurse is caring for a client three hours postoperative following a laparoscopic appendectomy. Which of the following client data indicates the client is ready for discharge home? Select all that apply. Positive gag reflex Hypoactive bowel sounds Blood pressure 90/60 mm Hg Incisional pain '2' on a scale of 0 to 10 Urinary output of 240 mL since surgery

Positive gag reflex Hypoactive bowel sounds Incisional pain '2' on a scale of 0 to 10 Urinary output of 240 mL since surgery Rationale: A positive gag reflex is a reassuring finding supporting the client's readiness for discharge home. Sedation used during surgery often suppresses the gag reflex, and this is an important element to be assessed after surgery. Hypoactive bowel sounds are an expected finding. Medications used intraoperatively to sedate the client reduce peristalisis. The reduction of bowel sounds (hypoactive) is evidence of this decrease in peristalsis and is not an abnormal finding. Full colonic motility usually returns within 72 hours following surgery. The client reporting incisional pain rated as a 2/10 is an expected finding. This is regarded as mild pain and would not inhibit discharge. The client's urinary output is optimal. A client, during surgery, often receives fluids that are excreted postoperatively. Low urinary output is more likely to occur because of the anticholinergic effects of the anesthesia. It would be a concern if the client had low (or no) urinary output.

While administering digoxin to an infant in heart failure, the nurse knows to monitor which of the following electrolytes? A. Potassium B. Calcium C. Sodium D. Phosphorus

Potassium Rationale: It is important that the nurse knows to monitor potassium levels while her patient is taking digoxin. This is important because if the patient is hypokalemic, digoxin toxicity can be worsened. Whereas, if the patient is hyperkalemic, digoxin will not be as effective.

*NGN* The emergency department (ED) nurse is caring for a 54-year-old female Item 1 of 1 History And Physical 1655 - 54-year-old female presents with her husband with reports of her 'not acting right.' The husband reports that three days ago, he noticed his wife 'acting odd' and reported her experiencing intermittent abdominal cramping, sweating heavily, and irritability. Today, the client woke up completely disoriented, feeling hot, and reporting that her heart was beating out of her chest. The client's husband reports that one week ago, she had such low motivation to get out of bed that he believes she has been taking more paroxetine than prescribed. The husband called their doctor and was instructed to report to the ED. The client has a medical history of chronic back pain, vitamin D deficiency, and peptic ulcer disease. She has recently been prescribed tramadol for back pain, which the husband says is contribut

Potential Condition: serotonin syndrome Actions to take: implement seizure precaution and request a prescription for diazepam Parameters to Monitor: electrocardiogram (ECG rhythm and rate and temperature Rationale: This client is experiencing serotonin syndrome (serotonin toxicity). Evidence supporting this is that the client is hyper-alert, experiencing diarrhea, diaphoretic, tachycardia, and has a significant fever. The cause of the serotonin syndrome is the client self-dosing the paroxetine, as reported by the husband. Additionally, the tramadol the client takes directly contributes to this toxicity. This, combined with the client increasing their paroxetine, is the likely etiology of the toxic serotonin levels.

The nurse is reinforcing teaching for individuals at a health fair about the human immunodeficiency virus (HIV). It would be correct for the nurse to indicate which of the following risk factors indicate the need for HIV testing? Select all that apply. Pregnancy Engaged in sex work Have a sexually transmitted infection The use of injection drugs Men who have sex with men (MSM)

Pregnancy Engaged in sex work Have a sexually transmitted infection The use of injection drugs Men who have sex with men (MSM) Rationale: Recommendations for human immunodeficiency virus (HIV) testing Adolescents and adults aged 15 to 65 years (at least one time) Individuals with a sexually transmitted infection Pregnancy The use of injection drugs Engaging in sex work Men who have sex with Men (MSM) Housed in a correctional institution such as jail and prison

The nurse is precepting a newly hired nurse to administer an intramuscular injection to an adult. Which action by the newly hired nurse requires follow-up? A. Prepares to administer the medication in the dorsogluteal. B. Prepares to insert the needle at a 90-degree angle. C. Uses isopropyl alcohol to clean the area prior to injection. D. Washes their hands before and after the procedure.

Prepares to administer the medication in the dorsogluteal. Rationale: This action requires follow-up as the dorsogluteal site is not recommended. The dorsogluteal site is not recommended because of potential damage to nearby nerves and blood vessels.

The nurse is demonstrating the appropriate use of a car seat to a client. The nurse is demonstrating which level of prevention? A. Primary B. Secondary C. Tertiary D. Quaternary

Primary Rationale: Primary prevention is often referred to as the true level of prevention because it occurs before disease or illness. Demonstrating the appropriate use of a car seat is primary prevention because it happens before an automobile crash, a leading cause of death for those younger than 19. Secondary: early identification of the disease (screenings; health fairs) Tertiary: treats the disease and prevents further complications

The nurse understands that a positive home pregnancy test is considered which level of confirmation? A. A possible sign of pregnancy. B. Presumptive sign of pregnancy. C. Probable sign of pregnancy. D. A positive sign of pregnancy.

Probable sign of pregnancy. Rationale: Over-the-counter (OTC) pregnancy tests check for the presence of hCG, which rises when pregnancy has occurred. A positive OTC pregnancy test is a probable sign of pregnancy but requires further testing for a definite diagnosis.

Which of the following is a cause of hyponatremia? A. Profound sweating B. Dehydration C. Diabetes Insipidus D. Salt-water drowning

Profound sweating Rationale: When a client sweats excessively, sodium is lost in sweat, and serum sodium levels will decrease, leading to hyponatremia. Hyponatremia refers to decreased serum sodium levels.

The LPN is caring for a client with hemophilia. When reviewing the lab values, what should the LPN expect to find? Select all that apply. Prolonged activated partial thromboplastin time (aPTT) Decreased factor VIII or IX levels Normal platelet count Abnormal bleeding time Increased prothrombin time (PT)

Prolonged activated partial thromboplastin time (aPTT) Decreased factor VIII or IX levels Normal platelet count Abnormal bleeding time Rationale: Prolonged activated partial thromboplastin time (aPTT) - Hemophilia is a bleeding disorder characterized by a deficiency or absence of clotting factors, leading to prolonged aPTT. Decreased factor VIII or IX levels - Hemophilia is categorized into two types: hemophilia A (factor VIII deficiency) and hemophilia B (factor IX deficiency). Therefore, decreased levels of these clotting factors would be expected in a client with hemophilia. Normal platelet count - Hemophilia is not associated with abnormalities in platelet count. Platelets are responsible for primary hemostasis (initial clot formation), while hemophilia affects secondary hemostasis (coagulation cascade). Abnormal bleeding time - Hemophilia primarily affects the coagulation cascade, so bleeding time is typically normal. It measures the time it takes for a small skin incision to stop bleeding and primarily reflects platelet function.

The nurse plans care for a client with moderate Alzheimer's disease (AD). Which of the following interventions should the nurse include? A. Provide a low-stimulation environment with adequate lighting B. Quiz the client with orientation questions C. Change assigned staff to avoid burnout D. Provide a broad range of choices

Provide a low-stimulation environment with adequate lighting Rationale: When caring for a client with moderate Alzheimer's disease, the nurse should provide a low-stimulated environment with adequate lighting. The rationale for giving a low-stimulated environment is to allow the client to cognitively process one task at a time and not become distracted.

A 40-year-old client who is blind and deaf has been admitted to the medical floor. What is the nurse's primary responsibility for this client? A. Make others aware of the client's deficits. B. Communicate with the nursing supervisor any client safety concerns. C. Continuously update the client on the social environment. D. Provide a safe and secure environment.

Provide a safe and secure environment. Rationale: The nurse's primary responsibility is client safety. For this deaf and blind client, it is critical to provide a secure environment.

The nurse is caring for a patient who is experiencing acute mania. Which of the following actions should be prioritized by the nurse? A. Plan structured solitary activities B. Redirect the patient's speech and ideas C. Provide high-calorie, small, frequent meals D. Initiate a psychiatry referral

Provide high-calorie, small, frequent meals Rationale: A client experiencing acute mania manifests symptoms such as inflated self-esteem, flight of ideas, psychomotor agitation, and an expansive effect. The client experiencing mania often has difficulty sleeping and exerts excessive physical energy. Thus, the nurse needs to focus on ensuring that the client's need for nutrition is met by offering high-calorie, small, frequent meals. This is the priority based on Maslow's Hierarchy of Needs.

The nurse is caring for a child diagnosed with a coarctation of the aorta who is scheduled for a repair tomorrow morning. When she auscultates his lung sounds, she notes crackles and rales. The nurse knows this is a sign of which of the following? A. Pulmonary congestion B. Foreign body aspiration C. Pneumonia D. Systemic congestion

Pulmonary congestion Rationale: Crackles and rales are indicative of pulmonary congestion. Since this child has coarctation of the aorta, there is too much blood backing up in the lungs. It is impossible for the left side of the heart to move sufficient blood forward working against the coarctation. This causes the back up of blood in the lungs, and therefore the crackles and rales indicative of pulmonary congestion.

The nurse is caring for a client who is receiving prescribed doxorubicin. Which of the following findings would indicate the client is having an adverse effect? A. Urine discoloration B. Pulmonary congestion C. Hirsutism D. Pruritus

Pulmonary congestion Rationale: Doxorubicin is an antineoplastic that is indicated for a variety of cancers. Doxorubicin is highly cardiotoxic, which may cause cardiomyopathy. The cardiotoxicity may cause a decrease in left ventricular ejection fraction, therefore, causing pulmonary congestion and, at worse life-threatening pulmonary edema. The client may present with leg edema, cough, and worsening shortness of breath.

The licensed practical/vocational nurse (LPN/VN) collects data on a client who has overdosed on aspirin. Which clinical finding would be most concerning? A. Pulmonary edema B. Tinnitus C. Nausea and vomiting D. Tachycardia

Pulmonary edema Rationale: ll of these manifestations are associated with an aspirin overdose. Pulmonary edema is the most concerning and is caused by a lung injury induced by aspirin. A treatment for aspirin overdose is an infusion of sodium bicarbonate to correct metabolic acidosis. During the infusion, the nurse must be sensitive to the potential lung injury caused by aspirin; thus, auscultating lung sounds and assessing for pulmonary edema will be essential. Manifestations of pulmonary edema include tachypnea, tachycardia, and crackles in the lung fields.

The licensed practical/vocational nurse (LPN/VN) is assisting a registered nurse in triaging a client with diabetes mellitus (type one) with diabetic ketoacidosis (DKA). Which assessment finding requires immediate follow-up by the LPN/VN? A. Pulse 112/minute B. Persistent nausea and vomiting C. Respiratory rate 21/minute D. Blood glucose 299 mg/dL

Pulse 112/minute Rationale: A complication associated with DKA is hypovolemic shock. The client having tachycardia is demonstrating early signs of this type of shock. The treatment modalities of DKA include fluid repletion and insulin administration. Considering the client's tachycardia, the nurse should initially administer the prescribed isotonic fluids to treat the significant fluid volume deficit.

Which action would be the most appropriate for a nurse to use as an alternative to restraints for an elderly client who is disoriented and tends to wander the halls of his long-term care facility? A. Sit the client in a geriatric chair near the nurse's station. B. Use bedsheets to secure the client snuggly in bed. C. Keep the client's bed in a high position, so he doesn't get out. D. Put the client's picture and a balloon on his door, so he knows which room is his.

Put the client's picture and a balloon on his door, so he knows which room is his. Rationale: Identifying the client's door with his picture and a balloon may be a helpful alternative to restraints. If safety is not an issue, the resident should be allowed to move about. Measures to help clients who experience confusion or disorientation should be initiated. Many nursing homes and assisted living facilities allow residents and family members to personalize the client's door in much the same way that a private person would decorate his/her front door of their home.

Which of the following food choices would be suitable for a client with iron deficiency anemia? Select all that apply. Quinoa Liver Spinach Baked beans Dairy product

Quinoa Liver Spinach Baked beans Rationale: Quinoa is a good source of iron and an excellent recommendation for a client with iron deficiency anemia. In one cup of quinoa, there is about 3 mg of iron. The liver is a good source of iron and an excellent recommendation for a client with iron deficiency anemia. In 3 oz of the liver, there are about 15 mg of iron. Spinach is a good source of iron and an excellent recommendation for a client with iron deficiency anemia. In 100 grams of spinach, there is about 3 mg of iron. Beans are a good source of iron and an excellent recommendation for clients with iron deficiency anemia. There is about 5 mg of iron in one cup of baked beans.

he nurse has received a telephone prescription from the primary healthcare provider (PHCP) for citalopram 10 mg PO daily. Which action is the nurse's priority after taking the telephone order? A. Verify that the medication is in stock B. Read back the prescription to the PHCP C. Inform the client of the new prescription D. Transmit the prescription to the pharmacy

Read back the prescription to the PHCP Rationale: It is essential that the nurse read back the order to the PHCP to ensure that the prescription telephone order is accurate. Any verbal or telephone order from a PHCP requires the nurse back the order to verify that the order was accurate. Telephone and verbal orders may result in medication errors because of accents, background noise, etc., that may distort communication.

Which of the following are components of the definition of critical thinking? Select all that apply. Reasoned thinking Openness to alternatives Adherence to established guidelines Ability to reflect Loyalty to traditional approaches Desire to seek the truth

Reasoned thinking Openness to alternatives Ability to reflect Desire to seek the truth Rationale: Critical thinking is a combination of reasoned thought, openness to alternatives, the ability to reflect, and a desire to seek the truth. There are many definitions of critical thinking. It is a complex concept, and people think about it in different ways. Any situation that requires critical thinking is likely to have more than one "right" answer. You do not need critical thinking to add 2 + 2 and come up with the solution. However, you do need critical thinking for problem-solving essential decisions. A crucial aspect of critical thinking is the process of identifying and checking your assumption. This is also a necessary part of the research process. Critical thinking is a combination of reasoned thought, openness to alternatives, the ability to reflect, and a desire to seek the truth.

The nurse is caring for a client who is prescribed enoxaparin. Which of the following findings in the medical history would require follow-up with the primary healthcare physician (PHCP)? A. Recent spinal surgery B. Diabetes mellitus C. Osteoarthritis D. Venous thromboembolism

Recent spinal surgery Rationale: Enoxaparin is an anticoagulant medication used to prevent blood clots. Clients who have recently undergone spinal surgery may be at risk for spinal epidural hematomas if they receive anticoagulation therapy like enoxaparin. This condition can lead to compression of the spinal cord and neurological deficits, necessitating consultation with the primary healthcare physician (PHCP) to assess the appropriateness of enoxaparin.

The nurse is caring for a client diagnosed with diabetes mellitus, type I. Which of the following teaching points is most important for the nurse to reinforce? A. Check a hemoglobin A1C level every three months B. Rotate injection sites for insulin administration C. Examine their feet with a mirror daily D. Recognize the symptoms of hypoglycemia

Recognize the symptoms of hypoglycemia Rationale: This is the most important for the nurse to reinforce because hypoglycemia may cause significant harm. Signs and symptoms of hypoglycemia include palpitations, tachycardia, cool and clammy skin, lethargy, and coma.

Which of the following symptoms should the nurse monitor for in her patient with a suspected diagnosis of intussusception? Select all that apply

Red currant jelly stool Palpable, sausage-shaped mass in the RUQ Vomiting partially digested food Rationale: Red, currant jelly stool is a classic finding of intussusception. When the bowel telescopes into another portion of the intestine, it causes intestinal obstruction and subsequently red, currant jelly stools. A palpable, sausage-shaped mass in the RUQ is a classic finding of intussusception. This is due to the physical telescoping of the intestine, and the mass can sometimes be felt upon palpation. The child may vomit bile or partially digested food, which can be a result of the obstruction caused by the intussusception.

Place the following instructions for the use of a Metered Dose Inhaler (MDI) without a spacer in the correct order: Open your mouth and place the mouthpiece 1 to 2 inches away from the mouth. Press down firmly on the canister and breathe deeply through the mouth. Tilt your head back and breathe out fully. Wait at least 1 minute between puffs. Hold your breath for at least 10 seconds. Remove the inhaler cap and shake the inhaler.

Remove the inhaler cap and shake the inhaler. Open your mouth and place the mouthpiece 1 to 2 inches away from the mouth. Tilt your head back and breathe out fully. Press down firmly on the canister and breathe deeply through the mouth. Hold your breath for at least 10 seconds. Wait at least 1 minute between puffs.

The nurse is caring for a client that is scheduled to have a thyroidectomy. The nurse understands that the primary reason for giving Potassium iodide and Iodine solution to this client preoperatively is: A. Decrease the risk of agranulocytosis postoperatively. B. Prevent tetany while the client is under general anesthesia. C. Reduce the size and vascularity of the thyroid and prevent hemorrhage. D. Potentiate the other preoperative medication's effect so less medicine can be used while the client is under anesthesia.

Reduce the size and vascularity of the thyroid and prevent hemorrhage. Rationale: It serves to reduce the size and vascularity of the thyroid gland, thereby preventing hemorrhage. Hyperthyroidism is associated with hemodynamic variations, including increased heart rate and cardiac contractility, as well as decreased peripheral resistance due to excess serum thyroid hormones. Adequate preoperative preparation of the client is crucial to avoid intraoperative or postoperative complications and decrease the gland's vascularity. While complications are rare in the hands of experienced surgeons, even a small amount of intraoperative bleeding can compromise the visualization and preservation of surrounding nerves, vasculature, and parathyroid glands. Therefore, the administration of Potassium iodide and Iodine solution preoperatively has been a practice to limit intraoperative bleeding and associated complications resulting from thyroid gland vascularization.

The clinic nurse is caring for a 38-year-old male Item 6 of 6 The nurse performs education regarding metabolic syndrome and reminds the client to return in six weeks for a follow-up appointment Click to specify the self-management goals for this client Note: Each row must have one (1) response option selected Reduction in the HDL cholesterol (Appropriate or Not Appropriate) Moderate exercise at least 150 minutes a week (Appropriate or Not Appropriate) Reduce the consumption of complex carbohydrates (Appropriate or Not Appropriate) Increase the intake of fiber-containing foods (Appropriate or Not Appropriate) Reduction in psychosocial stressors (Appropriate or Not Appropriate)

Reduction in the HDL cholesterol (Not Appropriate) Moderate exercise at least 150 minutes a week (Appropriate) Reduce the consumption of complex carbohydrates (Not Appropriate) Increase the intake of fiber-containing foods (Appropriate) Reduction in psychosocial stressors (Appropriate) Rationale: At the six-week follow-up, the nurse should assess the following elements to determine if the client is practicing optimal self-management: The goal is for the client's HDL-C to increase (not decrease). Having the client improve their dietary fiber intake and reduce their saturated fats may increase HDL-C levels. Evidence supports exercise as a key way to raise HDL-C. Increasing fiber intake is an effective way to lower LDL-C and triglycerides. Moderate exercise of at least 150 minutes per week is recommended. Exercise has many benefits, including being a natural mood booster and improving digestion. This will help the client reduce their weight with a low sodium and glycemic diet. The client should be instructed to modify their diet to be low in sodium and simple carbohydrates. Complex carbohydrates, especially those with high fiber, are strongly recommended because they promote satiety and stabilize blood glucose levels. The client should be instructed to consume less than 1500 mg per day. This is essential to the client, considering he opts for fast food in his current diet. The client is experiencing significant stress, and modifying this risk factor will be essential in self-management. It is a goal for the client to report a reduction of stress and engage in therapeutic behaviors such as exercise and relaxation.

The nurse is assessing a patient who reports intermittent tingling and numbness in bilateral lower extremities. Which intervention by the nurse would be most important to prevent injury for this patient? A. Perform Semmes-Weinstein monofilament test B. Refer the patient for a diabetic diet consult C. Obtain an order for gabapentin D. Reinforce teaching regarding appropriate footwear use

Reinforce teaching regarding appropriate footwear use Rationale: Peripheral neuropathy puts the patient at increased risk for traumatic injury and tissue breakdown since the patient may not notice early skin damage due to altered sensation. Of the options provided, educating the patient on proper footwear is the only action that aims to prevent injury related to the patient's altered sensation in the feet.

Which of the following tasks would be appropriate for an LPN (Licensed Practical Nurse) to take? Select all that apply. Reinforcing teaching to a 24-year-old first-time mother on how to care for her new baby. Adjustment of a 68-year-old patient's cervical traction as ordered by the attending doctor. Obtaining an occult blood sample from a 16-year-old patient with ulcerative colitis. An assessment of a 36-year-old man newly admitted for chest pain.

Reinforcing teaching to a 24-year-old first-time mother on how to care for her new baby. Adjustment of a 68-year-old patient's cervical traction as ordered by the attending doctor. Obtaining an occult blood sample from a 16-year-old patient with ulcerative colitis. Rationale: Initial teaching is not within the LPN's scope of practice. However, the LPN may reinforce teaching to a client. Obtaining stool samples and following orders to adjust cervical traction are all within an LPN's scope of practice. A comprehensive admission assessment is also referred to as an initial database, nursing history, or nursing assessment. It is completed when the client is admitted to the nursing unit. These forms can be organized according to health patterns, body systems, functional abilities, health problems/risks, nursing models, or type of health care setting (e.g. labor and delivery, pediatrics, mental health). The registered nurse completes an initial assessment and generally records ongoing assessments or reassessments on flow sheets or nursing progress notes.

Which of the following is the most accurate education for injury prevention in the home of elderly clients? A. Use the handrail when going up and down the stairs, ensure robes or pants are held up if flowy, and wear comfortable slippers. B. Remove all throw rugs, remove furniture from all pathways, and wear comfortable non-skid footwear. C. Use solid chairs without armrests, keep walkways clear, use cordless phones. D. Install raised toilet seats, ensure that all sinks have throw rugs to prevent slipping on water, and use grab bars in the shower/bathroom.

Remove all throw rugs, remove furniture from all pathways, and wear comfortable non-skid footwear. Rationale: Elderly individuals are more susceptible to falls due to age-related changes in balance, muscle strength, and vision. Removing throw rugs and clearing pathways of furniture helps eliminate tripping hazards, reducing the likelihood of falls and related injuries.

The licensed practical/vocational nurse (LPN/VN) cares for a client with pulmonary tuberculosis (TB). Which infection control measure should the LPN implement? A. Restrict visitors who are pregnant B. Remove any portable fans in the room C. Wear a dosimeter badge during client care D. Place the patient further away from the nursing station

Remove any portable fans in the room Rationale: Fans should be removed from a room for a client with droplet or airborne precautions. Fans may propel the transmission of a pathogen. A client with pulmonary tuberculosis should be isolated using airborne precautions. The door should be kept closed and the room should have monitored negative air pressure.

The nurse is caring for a client with pulmonary tuberculosis. Which action should the nurse take? A. Place a box of disposable respirators inside the client's room B. Remove alcohol-based sanitizers from the client's room C. Assign the client to a private room with a positive airflow D. Remove the portable fan from the client's bedside table

Remove the portable fan from the client's bedside table Rationale: For a client on either airborne or droplet precautions, the nurse should not allow (and remove) any portable fans, as these may propel pathogens and assist in disease transmission. If the client has a fever, nonpharmacological treatment options such as a cool compress or a tepid bath should be used.

Wilms tumor is a type of cancer most common in children under the age of 5. Where do these tumor cells originate from? A. Lung cells B. Epithelial cells C. Adipose cells D. Renal cells

Renal cells Rationale: Wilms tumor, also known as nephroblastoma, is a cancer of the kidneys. Its tumor cells originate from renal cells.

The nurse is caring for a client with a serum magnesium level of 3.2 mg/dL(1.5-2.5 mEq/L). Which of the following could be a contributing factor in this abnormal electrolyte level? Select all that apply.

Renal failure Adrenal insufficiency Rationale: The normal magnesium level is 1.6-2.6 mg/dL. This client has a level of 3.2 and is experiencing hypermagnesemia. Renal failure can cause hypermagnesemia since the process that keeps the levels of magnesium in the body at normal levels does not work properly in people with kidney dysfunction. The adrenal glands play a role in regulating magnesium levels in the body. If the adrenal glands are not functioning properly, magnesium levels can increase, leading to hypermagnesemia.

The nurse is caring for a 45-year-old client who has undergone electroconvulsive treatment (ECT) for severe depression. Which of the following nursing interventions is appropriate following the treatment? Select all that apply. Position the client supine with the head of the bed at 30 degrees. Reorient the client frequently. Remain with the client at all times. Promote bedrest for 12-24 hours. Ambulate the client as soon as possible.

Reorient the client frequently. Remain with the client at all times. Rationale: It will be a critical nursing intervention to frequently reorient the client who has just received electroconvulsive therapy (ECT). This is because temporary memory loss is associated with this procedure, so they will likely be confused and disoriented. Due to this disorientation, they will probably be scared; the nurse must frequently reorient them to their place and situation to make them feel safe and secure. It will be a critical nursing intervention to remain with the client who has just received electroconvulsive therapy. A side effect of electroconvulsive treatment is temporary memory loss. They will be disoriented and confused, so the nurse must remain with them to keep them safe.

When experiencing conflict with another nurse (that is not resolvable between the parties), what is the most appropriate action for the nurse moving forward? A. Report the conflict to the director of nursing over the unit. B. Report the conflict to the nurse manager of the unit. C. Report the conflict to the assigned charge nurse of the unit. D. Discuss the conflict with another nurse to attempt resolution of the issue.

Report the conflict to the assigned charge nurse of the unit. Rationale: It is important to follow the appropriate chain of command in a professional setting and not overstep any levels when moving the issue up the ladder.

The nurse notices bruises on a client's arm and observes that the client seems afraid and does not speak much. Because these are possible signs of physical abuse, what is the nurse's most appropriate action? A. Ignore the bruises, as this is not why the client is being treated, and it is not appropriate for the nurse to address them. B. Report the suspected abuse to one of the other nurses and work together on how to handle it. C. Report the findings to the appropriate authorities based on state requirements and protocols. D. Use therapeutic communication to talk to the client and attempt to get evidence of suspected abuse.

Report the findings to the appropriate authorities based on state requirements and protocols. Rationale: It is the responsibility of any healthcare provider/team member to report any suspected abuse to the police or designated agency, per state police.

A client in the post-anesthesia care unit is semiconscious and dyspneic. What should the nurse's first action be? A. Place a pillow under the client's head. B. Remove the oropharyngeal airway. C. Administer oxygen by mask. D. Reposition the client to keep the tongue forward.

Reposition the client to keep the tongue forward. Rationale: The tongue can obstruct the airway of a semiconscious client. Repositioning in the side-lying position with the face slightly down will prevent occlusion of the pharynx and allow the drainage of mucus from the mouth.

The nurse is working at the triage desk in the emergency department when a patient arrives and begins speaking in Spanish. The nurse asks if he would like an interpreter, and he shakes his head 'no.' What is the appropriate action for the nurse to take? A. Ask around to see if anyone nearby knows Spanish. B. Call the receptionist who speaks Spanish to translate. C. Pull up Google translate on the internet. D. Request an interpreter from the hospital's interpreter service.

Request an interpreter from the hospital's interpreter service. Rationale: It is most appropriate to request an interpreter from the hospital's interpreter service. A certified medical interpreter has the proper training to quickly and accurately translate the conversation as well as protect client confidentiality. This is the appropriate action by the nurse.

What is the nurse's priority when a fire occurs in a client's room? A. Rescue the client. B. Extinguish the fire. C. Sound the alarm. D. Run for help.

Rescue the client. Rationale: Client safety is always the priority. RACE = Rescue, Alert, Contain, Extinguish/Evacuate

The nurse has attended a continuing education presentation about acid-base imbalances. It would indicate a correct understanding of the conference if the nurse identifies which of the following as a cause for metabolic acidosis? Select all that apply. Severe diarrhea Hyperventilation Starvation End-stage renal disease (ESRD) Nasogastric tube (NGT) suctioning

Severe diarrhea Starvation End-stage renal disease (ESRD) Rationale: These are all possible causes of metabolic acidosis. Metabolic acidosis starts in the kidneys instead of the lungs. It occurs when they can't eliminate enough acid or when they get rid of too much base. Severe diarrhea causes metabolic acidosis because the bicarbonate is discharged with the stool. Further, starvation may cause metabolic acidosis because it causes an accumulation of ketone bodies. Finally, ESRD contributes to metabolic acidosis because of the kidneys' inability to recirculate sodium bicarbonate.

*NGN* The nurse is performing data collection of a newborn immediately following birth Click to highlight the findings in the nursing assessment that require follow-up Nursing Assessment Vitals/Observations Respiratory rate 68 breaths per minute Apical Pulse 187 Axillary Temperature 95.7°F (35.3°C) Head Symmetrical Mouth Small, white, hard cysts on the hard palate Neck Raises head from side to side while prone Cry Vigorous Respiratory Clear lung fields Extremities Unequal thigh and gluteal creases Skin Bluish-gray marks on the sacrum

Respiratory rate 68 breaths per minute Apical Pulse 187 Axillary Temperature 95.7°F (35.3°C) Extremities Unequal thigh and gluteal creases Rationale: The findings in the physical assessment that require follow-up include the infant's apical pulse of 187, an axillary temperature of 95.7°F (35.3°C), and a respiratory rate of 68 per minute. Both vital signs suggest cold stress and require measures to warm the newborn. When a newborn experiences cold stress, tachypnea and tachycardia are all associated with this condition. These compensatory mechanisms may fail if the rewarming measures are not promptly implemented. One other finding that requires follow-up in the nursing assessment is the extremity assessment yielding unequal thigh and gluteal creases. This is a finding suggestive of developmental dysplasia of the hip.

The nurse is reinforcing education to the parents of a toddler diagnosed with bronchiolitis. She tells them that which of the following is the most likely cause of bronchiolitis? A. Haemophilus influenzae type B B. Adenovirus C. Respiratory syncytial virus (RSV) D. Rhinovirus

Respiratory syncytial virus (RSV) Rationale: Respiratory syncytial virus (RSV) is the most likely cause of bronchiolitis. It is a common virus in children that causes respiratory illness and is especially common in children less than 2 years old.

*NGN* 72-year-old male presents to the emergency department Item 4 of 6 Orders Admit to involuntary status Provide enhanced observation Regular diet Consult psychiatry Additional Nursing Note 1450 - Client endorsed suicidal ideations stating, "I am thinking about ending my life." The client verbalized no specific plan. Enhanced observation and suicide precautions were initiated based on the client's statements. Removed all sharps from the room. The client's belongings were inventoried and stored. The nurse updates the nursing note with an entry at 1450 and receives orders from the primary healthcare provider (PHCP) The nurse is developing the plan of care for this client. For each possible intervention, click to specify whether or not the intervention is appropriate

Restrict the client's visitors: Not Appropriate Open letters and packages before giving them to the client: Not Appropriate Provide information on how to obtain their medical record: Appropriate Round at frequent and regular intervals: Not Appropriate Inform the client that they may not leave the facility: Appropriate Rationale: When a client is admitted involuntarily, they still retain the client bill of rights. It would be appropriate to provide information on how a client may obtain their medical record because this is a right afforded to the client. It would also be appropriate to inform the client that leaving the facility is prohibited while being admitted involuntarily. It is not appropriate to restrict visitors or open postal packages prior to giving them to the client. The client has the right to privacy, opening postal packages, and denying the client phone privileges would violate their bill of rights. Finally, rounding the client at regular intervals is discouraged. It would be appropriate to round on the client at irregular intervals to decrease predictability and enhance client surveillance.

You are teaching a community health education session for members of the community with celiac disease. You educate on each of the following foods. What should you teach are okay to eat? Select all that apply. Rice Oats Barley Corn

Rice Corn Rationale: Rice and corn does not contain gluten, so this is a safe choice for people with celiac disease.

The nurse is assisting a client pick out food options appropriate for Dumping Syndrome. Which food items, if selected by the client, would indicate effective teaching? Select all that apply. Rice cereal Pastries Chicken breast Cola Scrambled eggs

Rice cereal Chicken breast Scrambled eggs Rationale: Dumping syndrome is characterized by rapid peristalsis that may be triggered by foods with an abundance of simple carbohydrates (refined sugars). Choices A, C, and E reflect foods that are not simple carbohydrates. Foods recommended in dumping syndrome include complex carbohydrates, high protein, and high fiber.

The nurse is assisting a registered nurse in the care of a client with Bell's palsy. Which nursing diagnosis would be the highest priority for this client? A. Risk for infection B. Risk for disturbed sensory perception C. Risk for disturbed body image D. Risk for ineffective tissue perfusion

Risk for disturbed sensory perception Rationale: Bell's palsy causes acute facial paralysis or weakness in the muscles supplied by cranial nerve VII, which can result in difficulty closing the eyelid, increased sound sensitivity, altered sense of taste, difficulty chewing/swallowing, and pain.

The nurse is caring for a client with Cushing's disease. The client asks for assistance in choosing dinner. Which meal is the best option? A. Hamburger with french fries and apple slices. B. Pork chops in cream sauce with mashed potatoes and carrots. C. Roasted chicken with corn and green beans. D. Mexican-style beef with guacamole and beans on the side.

Roasted chicken with corn and green beans. Rationale: A client with Cushing's disease needs to eat a low sodium, high protein, and low-fat diet. Roasted chicken is high in protein with low in fat. Cushing's disease is a serious condition of an excess of the steroid hormone cortisol in the blood level caused by a pituitary tumor secreting adrenocorticotropic hormone (ACTH). Since cortisol is essential for metabolic processes, hypercortisolism associated with Cushing's can significantly affect how the body processes nutrients. Clients with Cushing's disease are encouraged to reduce sodium intake (to reduce the risk of hypertension and weight gain), increase calcium and vitamin D (Cushing's disease can cause reduced bone density and osteoporosis).

The nurse is reviewing isolation precautions for patient room assignments on the nursing unit. Which patient room assignment requires follow-up? A. Room 1 B. Room 2 C. Room 3 D. Room 4

Room 1 Rationale: Lyme disease is not transmitted human-to-human. This client should be placed on standard precautions as contact precautions are inappropriate. This client requires follow-up as the contact precautions are inappropriate.

Isolation precautions - room assignment that require follow-up:

Room 1: lyme disease (contact) Rationale: Lyme disease is not transmitted human-to-human. This client should be placed on standard precautions as contact precautions are inappropriate. This client requires follow-up as the contact precautions are inappropriate.

The nurse is assessing a 7-month-old infant. At this age, which of the following reflexes would the nurse expect to no longer be present? Select all that apply. Rooting Plantar Moro Palmar Sucking. Tonic neck.

Rooting Moro Palmar Tonic neck Rationale: - The Rooting reflex should disappear by 3-4 months of age. It occurs when the infants turn their face toward stimulation (such as stroking their cheek) and make sucking (rooting) motions with the mouth. This reflex helps to ensure successful feeding. - The Moro reflex should disappear by 5-6 months of age. This reflex is a response to a sudden loss of support. When support is removed, the infant spreads out the arms and cries. - The Palmar reflex should disappear by 2-3 months of age. When an object is placed in an infant's hand, and the palm is stroked, the fingers will close reflexively. - The tonic neck reflex disappears around 4 months of age. This reflex is elicited by turning the infant's head to one side and is considered positive if the infant extends the extremities on the side that the head is turned toward, and flexes the extremities on the opposite side.

The LPN is aware that which of the following maternal infections can increase the risk of congenital heart defects in the fetus? A. Parainfluenza B. Adenovirus C. Rubella D. Measles

Rubella Rationale: Rubella is a maternal infection that is known to increase the risk that the fetus will have a congenital heart defect. All mothers should be tested for Rubella, and if found to be positive, should have a fetal echocardiogram performed to evaluate the fetal heart more closely.

The nurse is data collecting on a client with congestive heart failure. Which physical finding should the nurse expect? A. Intermittent claudication B. S3 gallop C. Venous stasis ulcers D. Widened pulse pressure

S3 gallop Rationale: An S3 gallop is an expected finding in heart failure. This is often an early manifestation of heart failure; it and this sound are best auscultated at the apex of the heart.

The nurse cares for a client with a potassium of 5.7 mEq/L(3.5-5 mEq/L). Which conditions can cause hyperkalemia? by Select all that apply. Cushing's disease. nasogastric tube suction. salt substitutes. hyperinsulinism. adrenal insufficiency

Salt substitutes Adrenal insufficiency Rationale: The client's high potassium level, 5.7 mEq/L is concerning. Salt substitutes contain potassium which makes them more palatable. Excessive intake may lead to hyperkalemia. Adrenal insufficiency causes hyperkalemia because of the insufficient amount of aldosterone, which causes potassium elimination. Less aldosterone, and less potassium elimination, equates to hyperkalemia.

The nurse is caring for a client with acute renal failure. Which of the following laboratory results should be reported to the primary healthcare provider (PHCP)? A. Blood urea nitrogen 20 mg/dL B. Serum potassium 6 mEq/L C. Venous blood pH 7.30 D. Hemoglobin of 10.3 mg/dL

Serum potassium 6 mEq/L Rationale: Acute renal failure can cause a significant imbalance in lab values. Although all of the lab results listed are abnormal, the elevated potassium level is a life-threatening finding. Although all of these findings are abnormal, elevated potassium is a life-threatening finding and must be reported immediately. The normal potassium is 3.5 - 5.0 mEq/l The BUN level should be 10 to 20 mg/dL. Venous blood pH should be 7.31 to 7.41. Normal hemoglobin levels differ based on age, sex, and general health. The normal range for hemoglobin is 14 to 17.5 grams per deciliter for men. For women, 12.0 to 16 grams per deciliter.

An LPN is working with a client who has a spinal cord injury. Which of the following symptoms would indicate autonomic dysreflexia in a client who has suffered a spinal cord injury? Select all that apply.

Severe headache Flushing or redness of the skin above the level of injury Abdominal distention Rationale: Severe headache. This is a right symptom of autonomic dysreflexia. A sudden, severe headache is a common manifestation of autonomic dysreflexia resulting from an overactive autonomic nervous system response. Flushing or redness of the skin above the level of injury. This is a correct symptom of autonomic dysreflexia. Flushing or redness of the skin above the level of injury occurs due to the dilation of blood vessels caused by the abnormal autonomic response. Abdominal distention.This is a correct symptom of autonomic dysreflexia. Abdominal distention can occur due to increased intra-abdominal pressure resulting from autonomic dysreflexia, which a full bladder or bowel impaction may cause.

The nurse is caring for a client who sustained a fractured tibia and fibula and has a cast applied to the extremity. Which of the following findings would indicate the client has developed compartment syndrome? A. The development of petechiae over the chest B. A new onset of dyspnea and chest pain C. Severe pain that is unrelieved by an opioid analgesic D. Localized bone pain with a fever

Severe pain that is unrelieved by an opioid analgesic Rationale: Early manifestations associated with compartment syndrome, including paresthesia of the affected extremity and pain unrelieved by a prescribed opioid analgesic.

A pregnant client at 37 weeks gestation arrives at the hospital with the following signs and symptoms: severe headache, blurred vision, and epigastric pain not relieved by pain medication. The nurse suspects the client is experiencing which condition? A. Preterm labor B. Ectopic pregnancy C. Severe preeclampsia D. Pre-eclampsia

Severe preeclampsia Rationale: Severe preeclampsia is characterized by high blood pressure, proteinuria (presence of protein in urine), and organ dysfunction. It is a serious condition that requires immediate medical intervention to prevent complications for both the mother and the baby. Severe preeclampsia is diagnosed If one or more of the following criteria are present: Blood pressure of ≥160 mm Hg systolic or ≥110 mm Hg diastolic or higher on two occasions at least 6 hours apart while the client is on bed rest Oliguria of <500 mL in 24 hr Cerebral or visual disturbances Pulmonary edema or cyanosis Epigastric or right upper quadrant pain Impaired liver function as indicated by abnormally elevated blood concentrations of liver enzymes (to twice normal concentration), severe persistent right upper quadrant or epigastric pain unresponsive to medication and not accounted for by alternative diagnoses or both Thrombocytopenia 8. Renal insufficiency

*NGN* The nurse is caring for a 29-year-old female in the physician's office Item 1 of 6 Progress Note 1500: Client is twelve weeks postoperative following gastric bypass surgery. The client's weight at this visit is 245 pounds (113.36 kilograms), a decrease of 6 pounds since the previous visit (2.72 kilograms) four weeks ago. She reports that she is starting to notice excess skin between her legs and upper arms. The client reports exercising regularly and eating solid foods with no reflux or nausea. Incision site appears fully healed with a minimal scar. The client reports no incisional pain. She reports feeling full much sooner compared to before the surgery. Denies abdominal pain, nausea, vomiting, and diarrhea. She reports symmetric paresthesias in both the upper and lower extremities. She endorses forgetfulness and fatigue. Physical exam normal with the exception of two oral ulcers. Medical History Diabetes m

She reports symmetric paresthesias in both the upper and lower extremities. She endorses forgetfulness and fatigue. with the exception of two oral ulcers. Rationale: The nurse should be concerned with the client reporting symmetrical paresthesias, forgetfulness, fatigue, and oral ulcers found during the physical examination. These manifestations suggest a complication following this type of surgery. The nurse is not concerned with the other findings. The client losing weight is a therapeutic finding as clients after this surgery typically loses 1 to 2 pounds a week for the first year after this procedure. The client feeling fuller sooner is the intention of the surgery to reduce the intake of calories. Excess skin is common as the client starts to lose weight. Skin removal surgeries usually occur once the client gets to an appropriate weight.

The LPN is taking care of a 9-year-old boy who is undergoing testing for acute myeloid leukemia (AML). The nurse is assisting with the positioning of this client for a lumbar puncture. Which of the following positions should the nurse place her client in? A. Prone B. Trendelenburg C. Supine D. Side-lying

Side-lying Rationale: Side-lying is the most appropriate position listed for a lumbar puncture (LP). This will allow the healthcare provider to identify the lumbar vertebrae and insert the needle into the subarachnoid space at the L3-4 or L4-5 interspace.

The nurse is assessing a 6-year-old client with asthma. Which of the following findings is of highest concern? A. Expiratory wheezing B. Silent chest C. Cough D. Head bobbing

Silent chest Rationale: Silent chest is the assessment finding of most concern. This refers to the inability to auscultate any lung sounds. There is complete obstruction of the client's airway, and therefore the inability to move air. When complete obstruction occurs, this is a medical emergency. This assessment finding is of most concern because the client has lost their airway.

Which of the following is the most appropriate position for a client getting a thoracentesis? A. Sitting up, leaning over a bedside table and feet supported on the ground or stool. B. The head of the bed flat, with the client lying on the unaffected side. C. Prone with both arms extended above the head. D. With the head of the bed elevated 45 degrees and the client lying on the affected side.

Sitting up, leaning over a bedside table and feet supported on the ground or stool. Rationale: The client should be sitting up, leaning over a bedside table with arms rested, and feet supported on the ground or stool so the needle can be inserted appropriately. Usually, only sufficient fluid to lubricate the pleura is present in the pleural cavity. However, excessive fluid can accumulate as a result of injury, infection, or other pathology. In such a case or with pneumothorax, a primary care provider may perform a thoracentesis to remove the excess fluid or air to ease breathing. Thoracentesis is also performed to introduce intrapleural chemotherapeutic drugs.

A client presents to the emergency department (ED) with a suspected ectopic pregnancy. The nurse anticipates which diagnostic test will confirm this finding? A. Nonstress testing B. Abdominal radiograph (x-ray) C. Transvaginal ultrasound D. Doppler transducer

Transvaginal ultrasound Rationale: An ectopic pregnancy (EP) is a medical emergency. The imaging of choice is a transvaginal ultrasound, as this type of ultrasound may visualize an extrauterine gestational sac with a yolk sac or embryo (with or without a heartbeat).

The nurse is assessing an infant who is 9 months old. Which of the following would be an expected age-related finding? Select all that apply. Sitting without support Rolling over Standing without support Taking their first steps Walks unsupported

Sitting without support Rolling over Rationale: Sitting without support is a gross motor skill that should be developed by 8 to 9 months. Indeed, a 9-month-old infant should already be able to sit up without support. If they have not yet met this milestone by nine months of age, follow-up is warranted to evaluate the infant further. They may miss other milestones and need help, such as physical therapy. Rolling over is a milestone that should be developed in a 9-month-old infant. Rolling completely over should be accomplished by the time the infant is six months old. If they have not met this milestone by nine months of age, follow-up is warranted to evaluate the infant further. They may miss other milestones and need help, such as physical therapy.

The nurse is providing teaching to the mother of an infant with a diagnosis of heart failure. Which of the following educational points would be helpful for optimizing feedings for this infant? Select all that apply. Small, frequent feedings. Feeding every 5 hours. Feed for a maximum of 30 minutes. Increased calorie formula. Switching from breastfeeding to bottle.

Small, frequent feedings. Feed for a maximum of 30 minutes. Increased calorie formula. Rationale: It is appropriate advice to feed an infant with heart failure in small, frequent feedings. These infants will have a difficult time feeding and are working very hard during their feeds. They will need to be paced so that they conserve their energy and do not burn too many calories while feeding. Small, frequent feeds are the best way to optimize their nutrition. It is appropriate advice to feed an infant with heart failure for only 30 minutes at a time. After 30 minutes of feeding, the infant is using too much energy to gain calories and grow due to the feeding. Conserving energy is very important for infants experiencing heart failure. It is appropriate advice to feed an infant with heart failure an increased calorie formula. This will allow them to get a maximum amount of calories for growth in as little work as possible. Infants who are breastfed may require additional supplementation to grow.

The nurse is collecting data on a client with syndrome of inappropriate antidiuretic hormone (SIADH). Which of the following laboratory test requires careful monitoring? A. Potassium B. Sodium C. Glucose D. Magnesium

Sodium Rationale: Dilutional hyponatremia may occur from SIADH from the excess water caused by the antidiuretic hormone. The hyponatremia may be so severe that it may cause neuromuscular weakness and seizure activity.

You are taking care of a 10-year-old with a gastrostomy jejunostomy (G-J) tube. Which electrolyte deficit is this client at risk for? A. Sodium B. Potassium C. Chloride D. Calcium

Sodium Rationale: There is a large amount of extracellular fluid in the peritoneal cavity, which contains a high sodium content. If this fluid is lost through the G-J tube, there will be a sodium deficit leading to hyponatremia

The nurse should understand the regulations of nursing practice as put forth by the Nurse Practice Act. Which of the following statements are correct? Select all that apply. Some other issues covered by the Nurse Practice Act include grounds for disciplinary action, licensure requirements, and the rights of the nurse licensee if disciplinary action is taken. The Nurse Practice Act defines the scope of nursing practice. All nurses have the responsibility to know the provisions of the act for the state or province in which they work. The Nurse Practice Act is a series of statutes enacted by the federal government in order to regulate the practice of nursing.

Some other issues covered by the Nurse Practice Act include grounds for disciplinary action, licensure requirements, and the rights of the nurse licensee if disciplinary action is taken. The Nurse Practice Act defines the scope of nursing practice. Rationale: Nurse practice acts (NPAs) contain a provision that creates and empowers a state board of nursing to regulate nursing practice in that state. All 50 states, the District of Columbia, and the four U.S. territories have established nursing boards. Although NPAs can vary from state to state, they all have standard components because states used ANA guidelines in developing their regulations. A state's nurse practice act usually includes the following: The authority of the board of nursing, its composition, and powers A definition of nursing and the boundaries of nursing practice Standards for the approval of nursing education programs The requirements for licensure of nurses Grounds for disciplinary action against a nurse's license

7 month old food recommended:

Soy-based yogurt Rationale: Soy-based yogurt is permitted as it does not contain added sugars and will not cause intestinal complications, unlike cow's milk products.

The nurse is observing the surgical aseptic technique of a nursing student. Which observation by the nurse requires follow-up? A. Spills sterile water onto the sterile field B. Uses sterile gloves to handle objects on a sterile field C. Has sterile gauze placed into the sterile field D. Keeps the sterile field above their waist

Spills sterile water onto the sterile field Rationale: When a sterile surface comes in contact with a liquid, the sterile object or field becomes contaminated. Even though the spilled water is sterile, it creates moisture and may disrupt the sterile protective barrier. The nurse should intervene because microorganisms travel to the sterile object if moisture leaks through the protective covering of a sterile package. This observation requires follow-up because the sterile field needs to be discarded and re-established.

The nurse cares for a client with a sodium 130 mEq/dl(135-145 mEq/L). Which of the following medications may cause this abnormality? Select all that apply. Spironolactone Hydrochlorothiazide Prednisone Sodium polystyrene Tolvaptan

Spironolactone Hydrochlorothiazide Rationale: Spironolactone is a diuretic that retains potassium but causes the loss of water and sodium. Hydrochlorothiazide is a thiazide diuretic that may contribute towards hyponatremia because while it does raise serum calcium levels, it depletes every other electrolyte.

Iron deficiency anemia findings:

Tachycardia Pica Pallor Glossitis Rationale: Anemia triggers compensatory mechanisms, including an increased heart rate, to enhance tissue oxygen delivery. Tachycardia is a common finding in clients with iron deficiency anemia. Iron deficiency anemia can develop pica, characterized by cravings for non-food substances such as ice, clay, or dirt. This unusual craving is believed to be an adaptive response aimed at obtaining necessary nutrients, including iron. Iron deficiency anemia reduces the production of red blood cells and hemoglobin, leading to decreased oxygen supply to body tissues. This can result in pallor, especially in the conjunctiva, nail beds, and mucous membranes.

The nurse cares for a 41-year-old female in the emergency department (ED) Item 4 of 6 Diagnostic Results Venous Duplex Ultrasonography: Proximal deep vein thrombosis in the left popliteal vein The nurse reviews the diagnostic results For each possible physician order/prescription, click to indicate if it is anticipated or not anticipated Splint the affected extremity (Anticipated or Not Anticipated) Clopidogrel (Anticipated or Not Anticipated) Strict bed rest (Anticipated or Not Anticipated) Elevate the affected extremity (Anticipated or Not Anticipated)

Splint the affected extremity (Not Anticipated) Clopidogrel (Not Anticipated) Strict bed rest (Not Anticipated) Elevate the affected extremity (Anticipated) Rationale: Splinting the affected extremity would not be anticipated as this is ordered for orthopedic injuries. Clopidogrel is antiplatelet and treatment for VTE is anticoagulants such as rivaroxaban, heparin, or warfarin. Strict bed rest is not indicated in the management of a VTE. No credible evidence exists that this degree of immobility decreases the risk of a pulmonary embolism. Further, strict bed rest would increase the risk of the client developing pneumonia. Elevating the extremity to promote venous return is a standard intervention in the care of a client with a VTE.

Which of the following would be a priority action for a nurse who sustained a needlestick injury while working with an AIDS client? A. Contact a social worker right away. B. Start prophylactic AZT. C. Start prophylactic Pentamidine treatment. D. Make an appointment with a psychiatrist.

Start prophylactic AZT. Rationale: AZT (Zidovudine) is the most critical intervention. It is an antiretroviral medication used to prevent and treat HIV/AIDS by reducing the virus's replication.

You are providing education to a group of parents about toilet training for their toddler-age children. Which of the following educational points should you include? Select all that apply. Most children are ready to begin toilet training between 12 and 18 months old. Stay with the child while they are trying to use the toilet. Limit sitting on the toilet to 5-8 minutes at a time. A child should be able to stay dry throughout the night before you begin toilet training. "A diet low in fiber helps supports the development and maintenance of bowel movements."

Stay with the child while they are trying to use the toilet. Limit sitting on the toilet to 5-8 minutes at a time. Rationale: Parents should stay with their children while trying to use the toilet. Toilet training may be scary for some toddlers; it is a new and unfamiliar activity when they learn about their bodies and how to control something they have not previously controlled. It is essential to their psychosocial development that the toddler feels safe and supported; therefore, providing education to stay with the child while using the toilet is a good tip. Parents should limit sitting on the toilet to 5-8 minutes. Toilet training can be frustrating for toddlers, and it is important to foster their autonomy

The nurse is changing a diaper for her 7-month-old client suspected of having Celiac disease. She notes a large, pale, oily stool that is malodorous. The nurse knows this assessment finding is known as what? A. Diarrhea B. Steatorrhea C. Hematochezia D. Melena

Steatorrhea Rationale: Steatorrhea refers to the excretion of abnormal quantities of fecal fat due to reduced fat absorption by the intestines. This produces pale, oily, malodorous stools, and is a symptom of Celiac disease.

Chemotherapy induces vomiting by:

Stimulating neuroreceptors in the medulla Rationale: Vomiting (emesis) is initiated by a nucleus of cells located in the medulla called the vomiting center. This center coordinates a complex series of events involving pharyngeal, gastrointestinal, and abdominal wall contractions that lead to the expulsion of gastric contents.

The client with a history of right mastectomy is receiving maintenance IV fluids via a peripherally inserted intravenous line in the left cephalic vein. The client complains of pain at the IV site. The nurse notes that the infusion has slowed and assesses swelling as well as erythema at the IV site. Which action should the nurse take first? A. Stop the infusion and remove the IV catheter. B. Insert a new IV in the left intermediate basilic vein. C. Prepare the client for PICC line placement. D. Elevate the right arm to reduce swelling.

Stop the infusion and remove the IV catheter. Rationale: This client's IV site shows signs of phlebitis: redness, swelling, pain, and slowed infusion rate. The first priority action is to remove the current IV catheter to reduce the risk of further complications. Localized symptoms of phlebitis typically resolve after discontinuation of the catheter.

What clinical manifestation(s) would indicate to the practical nurse that a possibility of Kawasaki disease exists in an 8-year-old client? Select all that apply. Strawberry tongue Fruity breath Drooling Bright red, swollen lips Redness and swelling of the hands and feet

Strawberry tongue Bright red, swollen lips Redness and swelling of the hands and feet Rationale: Kawasaki disease is a swelling in the walls of the arteries throughout the body. Due to this inflammation, a strawberry tongue is a common identifying symptom. Other signs and symptoms include a high fever that persists for five or more days, a rash on the torso and groin, bloodshot eyes, bright red swollen lips, and red palms and soles of the feet.

*NGN* 72-year-old male presents to the emergency department Item 3 of 6 Complete the following sentence from the list of options The greatest concern for this client is (word choices: major depressive disorder, suicide, or insomnia) evidenced by the client's (word choices: feelings of hopelessness, increasing fatigue, or sleep disturbances)

Suicide Feelings of hopelessness Rationale: Suicide is absolutely the primary concern for this client. Central to suicidality is a client feeling hopeless, and this was explicitly expressed by the client. The depressive disorder that the client is likely experiencing needs to be addressed but is not life-threatening as the thoughts of suicide.

Which of the following statements correctly outlines the proper flow of blood through the heart? A. Superior and Inferior vena cavas → Right atrium → tricuspid valve → Right ventricle → pulmonary valve → pulmonary artery → lungs → pulmonary veins → left atrium → mitral valve → left ventricle → aortic valve → aorta → systemic circulation. B. Superior and Inferior vena cavas → Right atrium → mitral valve → Right ventricle → pulmonary valve → pulmonary artery → lungs → pulmonary veins → left atrium → tricuspid valve → left ventricle → aortic valve → aorta → systemic circulation. C. Superior and Inferior vena cavas → Right atrium → tricuspid valve → Right ventricle → pulmonary valve → pulmonary veins→ lungs → pulmonary artery → left atrium → mitral valve → left ventricle → aortic valve → aorta → systemic circulation. D. Superior and Inferio

Superior and Inferior vena cavas → Right atrium → tricuspid valve → Right ventricle → pulmonary valve → pulmonary artery → lungs → pulmonary veins → left atrium → mitral valve → left ventricle → aortic valve → aorta → systemic circulation. Rationale: This is the proper flow of blood through a healthy heart with normal anatomy. The superior and inferior vena cavae are the large veins that bring back deoxygenated blood from the body to the heart's right atrium. The blood enters the right atrium, passes through the tricuspid valve into the right ventricle, and is then pumped into the lungs through the pulmonary artery. In pulmonary circulation, the deoxygenated blood drops off its carbon dioxide plus waste products and picks up fresh oxygen to deliver to the body. Blood is now oxygenated. The blood returns to the left atrium through the pulmonary veins that pass through the mitral valve to enter the left ventricle which is then pumped out to the body through the aorta. Oxygenated blood is now in the systemic circulation, where it can deliver oxygen to all the tissues of the body.

The nurse plans care for a client admitted with Haemophilus influenzae, type b Meningitis. When caring for this client, the nurse should gather which appropriate personnel protective equipment (PPE)? A. Boot (shoe) covers B. Face shield C. Surgical mask D. Gown

Surgical mask Rationale: Haemophilus influenzae, type b Meningi requires droplet precautions. Droplet precautions require the nurse to don a surgical mask upon entry to the client's room. Cohorting with droplet precautions is permitted as long as the other individual has the same pathogen. Clients who require transport or want to ambulate outside their room should don a surgical mask.

Diabetes insipidus is a potential complication of which of the following procedures? A. Surgical removal of the pituitary gland B. Reduction of mass on the thyroid gland C. Hysterectomy D. Dilation and curettage

Surgical removal of the pituitary gland Rationale: Damage to the pituitary gland or hypothalamus from surgery increases the risk for diabetes insipidus. This is because the posterior pituitary is the gland that regulates the production, storage, and release of antidiuretic hormone (ADH). A decreased amount of ADH results in diabetes insipidus.

Which of the following meals would be appropriate for a nurse to assign to a client of Orthodox Judaism faith on a kosher diet? A. Pork belly roast, rice, vegetables, mixed fruit, milk B. Crab salad on a croissant, potato salad, milk, vegetables with dip C. Sweet and sour chicken with rice and vegetables, juice, mixed fruits D. Fettuccini Alfredo with shrimp and vegetables, salad, mixed fruit, iced milk tea

Sweet and sour chicken with rice and vegetables, juice, mixed fruits Rationale: Orthodox Judaism believers adhere to kosher dietary laws; for this group, the dairy-meat combination is unacceptable. Only fish that have scales and fins are allowed. Other meats that are allowed include animals that are vegetable eaters, cloven-hoofed, and those that are ritually slaughtered.

What signs should the LPN look for when assessing a 5-year-old girl with hemophilia with a recurring episode of hemarthrosis? Swelling and warmth in the affected joint. Limited range of motion in the affected joint. Pain and tenderness in the affected joint. Ecchymosis or bruising around the affected joint. Fever and chills.

Swelling and warmth in the affected joint. Limited range of motion in the affected joint. Pain and tenderness in the affected joint. Ecchymosis or bruising around the affected joint. Rationale: Hemarthrosis is characterized by bleeding into the joint, resulting in swelling and increased warmth around the affected area. Hemarthrosis can cause stiffness and reduced mobility in the joint due to the accumulation of blood and subsequent inflammation. Hemarthrosis often leads to pain and tenderness in the joint due to the pressure caused by the bleeding and resulting inflammation. Hemophilia can cause easy bruising, and ecchymosis around the joint suggests bleeding into the area.

*NGN* The nurse cares for a 41-year-old female in the emergency department (ED) Item 2 of 6 For each client finding, click to specify if it is consistent with deep vein thrombosis, compartment syndrome, or cellulitis. Each finding may support more than one condition Pain in the affected extremity (Compartment Syndrome, Cellulitis, and/or Venous Thromboembolism) Swelling to the affected area (Compartment Syndrome, Cellulitis, and/or Venous Thromboembolism) Intact sensation (Compartment Syndrome, Cellulitis, and/or Venous Thromboembolism) Warmth and erythema to the extremity (Compartment Syndrome, Cellulitis, and/or Venous Thromboembolism)

Swelling to the affected area (Compartment Syndrome, Cellulitis, and Venous Thromboembolism) Swelling to the affected area (Compartment Syndrome, Cellulitis, and Venous Thromboembolism) Intact sensation (Cellulitis and Venous Thromboembolism) Warmth and erythema to the extremity (Cellulitis and Venous Thromboembolism) Rationale: Most of these clinical findings overlap for compartment syndrome, cellulitis, and venous thromboembolism. The three differing clinical features are the intact sensation, warmth, and erythema to the extremity. One of the earliest findings associated with compartment syndrome is paresthesia, and because of the lack of perfusion, coolness to the extremity develops. Cellulitis is an infectious process that does not cause impairment in sensation.

The LPN is reinforcing education to a group of parents after an outbreak of pediculosis capitis at the local elementary school. Which of the following points should be included? Select all that apply. Teach your children not to share hats or combs to prevent the spread of lice. It is important to apply the permethrin cream once as soon as you can. Parents will need to manually remove the lice and nits with a fine-tooth comb 2-3 times/day until there are no visible lice. Anyone can get lice, it is not indicative of a dirty house. Stress the importance of washing all bed linens and clothing that the child has recently used in hot water.

Teach your children not to share hats or combs to prevent the spread of lice. Anyone can get lice, it is not indicative of a dirty house. Stress the importance of washing all bed linens and clothing that the child has recently used in hot water. Rationale: It would help if you educated parents to teach their children not to share hats or combs to prevent the spread of lice. Pediculosis capitis, or mites, is transmitted from person to person through direct contact with the scalp or personal items. Children may not understand why sharing hats or combs is terrible, so parents must talk with them about it. Anyone can get lice; it does not indicate a dirty house or child. Parents and children often feel embarrassed over having insects in their homes and fear the reaction of their peers, friends, and family. The nurse should educate the community that anyone can contract lice and that it does not reflect how clean their home environment is. Lice and nits can survive for a short time off the scalp. Washing items in hot water can kill lice and nits that may be present on these items, helping to prevent re-infestation.

A client has been marked as "confidential" due to safety concerns. Which of these actions would be inappropriate for the nurse? A. Keep the client's name/information out of public areas such as the nurse's station. B. Tell the client's mother he is okay when she calls to ask if he is still on the unit. C. Deny that the client is on the unit when visitors come or call. D. Remove the client from confidential status when he asks to be removed.

Tell the client's mother he is okay when she calls to ask if he is still on the unit. Rationale: When a client has asked to be flagged as confidential, no medical personnel can give out any information, including verifying the client's presence, in the hospital. This option violates the principle of confidentiality.

The nurse is planning a staff developmental conference about confidentiality. Which of the following scenarios should the nurse include as a violation of client confidentiality? A. Informing a visitor of the room number of a client admitted with pneumonia B. Telling a police officer who brought a client into the emergency department (ED) the urine drug screen results C. Notifying the pharmacist that a client is HIV positive and may have a potential drug interaction D. Informing local authorities that a client is suspected of being a victim of domestic violence

Telling a police officer who brought a client into the emergency department (ED) the urine drug screen results. Rationale: The results of a UDS are confidential, and that confidentiality should not be pierced because an individual is a police officer. If the police officer requests the results, they should obtain a legal court order and present it to risk management to obtain the necessary records.

The nurse is caring for a seven-year-old client brought to the clinic by her parents Item 1 of 6 History And Physical 7-year-old female arrives with her parents with reports of a rash on her back and chest that started two days ago. The parents state that she has been itching the area incessantly and that she reports a burning pain in the area. The parents say that her appetite has decreased, and she had a temperature of 100° F (37.8° C) the day prior. The client has no medical history or hospitalizations. Her parents state it is unknown that she has been around anyone who has been sick because she does attend school and an after-school program regularly. On exam, she has erythemic papules and vesicles on her back and torso. She states they 'burn and itch' and denies any other symptoms. Her skin is hot and appears dry. All other physical exam findings were unremarkable. Vital Signs Blood Pressure 104/72 mm Hg Te

Temperature Papules and vesicles Reports of burning and itching Location of the lesions Rationale: The clinical findings that are the most significant include the client's temperature, which indicates a fever. The presence of papules and vesicles that burn and itch is a common characteristic of varicella. Additionally, the location of the lesions as varicella characteristically has a centripetal outbreak starting at the trunk and working outward.

The nurse is caring for a client who has a fractured hip and has been on bedrest for a week. Which of the following symptoms would be considered signs of complications due to immobility? Select all that apply. The area is unable to be blanched on the client's sacrum. The client's skin has a faint yellow tinge. Crackles in the bases of the client's lungs. The client complains of pain, swelling, and tenderness in the left calf. The client uses the bedpan to void. The client's blood sugar is 79.

The area is unable to be blanched on the client's sacrum. Crackles in the bases of the client's lungs. The client complains of pain, swelling, and tenderness in the left calf. Rationale: Signs of complications due to immobility in a client with a fractured hip may include several key observations. Firstly, the area on the client's sacrum is unable to be blanched, indicating potential pressure ulcers or bedsores due to prolonged immobility and pressure on specific body areas. Secondly, the presence of crackles in the bases of the client's lungs may be noted, suggesting potential respiratory complications such as atelectasis or pneumonia. These respiratory issues can develop when a client remains on prolonged bedrest without sufficient mobility and deep breathing. Thirdly, if the client complains of pain, swelling, and tenderness in the left calf, it could be indicative of deep vein thrombosis (DVT), which is a known complication of immobility. DVT is characterized by the formation of blood clots in deep veins, often occurring in the legs. These signs collectively highlight the importance of close monitoring and preventive measures when managing immobile clients with fractures.

The nurse is calculating the 12-hour intake for a client The client received 0.45% saline at 85 mL/hr One eight-ounce cup of ice chips One eight-ounce cup of coffee One eight-ounce cup of ice cream Three eight-ounce cups of water One eight-ounce cup of pureed vegetables The nurse should calculate the client's total intake as how many mL? Fill in the blank.

The client received 0.45% saline at 85 mL/hr x 12 hours → 1020 mL One eight-ounce cup of ice chips → 120 mL When determining the total mL for a cup of ice, the nurse should divide the volume by 1/2 since the ice melts One eight-ounce cup of coffee → 240 mL One eight-ounce cup of ice cream → 240 mL Three eight-ounce cups of water → 720 mL One eight-ounce cup of pureed vegetables → This is excluded from the intake calculation as pureed food(s) are not a liquid at room temperature Total → 2340 mL

Which of the following client conditions are examples of subjective data? Select all that apply. The client reports feeling nauseated. The client's feet are swollen. The client tells the nurse she is nervous about test results. The client reports an itchy rash on his leg. The client rates her pain as a 6 on a scale of 1 to 10. The client vomits twice after eating supper.

The client reports feeling nauseated. The client tells the nurse she is nervous about test results. The client reports an itchy rash on his leg. The client rates her pain as a 6 on a scale of 1 to 10. Rationale: Subjective data is information that is perceived only by the person affected. This type of data cannot be perceived or verified by another person. A few examples of subjective data include feeling nervous or nauseous, itchy, cold, or experiencing pain.

*NGN* 72-year-old male presents to the emergency department Item 6 of 6 Progress Note The client was alert and oriented. He described his mood as good and had a cheerful affect. He denied any suicidal ideations. The client stated he still isn't sleeping well and would like a prescription for a sleep aid. The client has not attended group therapy since admission. The client stated he would be staying with friends from church. He also stated that he found an outpatient therapist. Click to highlight the findings in the progress note that specify that the client is ready for discharge

The client was alert and oriented. He described his mood as good and had a cheerful affect. He denied any suicidal ideations. The client stated he would be staying with friends from church. He also stated that he found an outpatient therapist. Rationale: Client statements and behavior indicating he is ready for discharge are alert and oriented. This is a positive cognitive finding. His mood and affect appear positive and harmonious and, thus, is also another indicator for discharge. The client stating that he will be staying with friends from church is reassuring as he will have a support system after discharge. Finally, working with an outpatient therapist is a reassuring finding. Findings that indicate that the client is not ready for discharge include his reluctance to attend inpatient group therapy and not being able to sleep. His report of insomnia will need to be mitigated to help decrease his further risk of suicide.

Which of the following best describes an appropriate outcome for a 75-yr-old client with a history of Huntington's disease who has developed contractures? A. The client will monitor for signs of skin breakdown as a result of the contractures. B. The client will learn to reposition himself in his bed and chair without assistance. C. The client will participate in a range of motion exercises to reduce the effects of contractures. D. The client will verbalize the effects of contractures on activities of daily living.

The client will participate in a range of motion exercises to reduce the effects of contractures. Rationale: Performing range of motion exercises will help decrease the risk of further atrophy and should be encouraged. Huntington's disease is a progressive condition that can lead to muscle atrophy and potential contractures. The client in this situation should be given a program with range of motion exercises. The nurse can help the client to increase his range of motion and prevent worsening of contractures by improving flexibility and reducing rigidity.

The LPN is assisting the nurse in caring for a client who is receiving a continuous opioid infusion. The LPN should understand which of the following, if detected, is a concerning finding? A. The client has a respiratory rate of 10 breaths/min with normal depth. B. The client's sedation level is 4. C. The client experiences mild confusion. D. The client reports constipation.

The client's sedation level is 4. Rationale: Sedation level is more indicative of respiratory depression because a drop in level usually precedes respiratory depression. A sedation level of 4 calls for immediate action because the client has minimal or no response to stimuli.

The licensed practical/vocational nurse (LPN/VN) cares for a child admitted with severe dehydration secondary to gastroenteritis. Which assessment data would be most reliable in determining the client's response to the prescribed intravenous fluid replacement? A. The number of stools in the past shift B. The current weight compared to the admission weight C. Mucous membrane assessment D. The 24-hour urinary output

The current weight compared to the admission weight Rationale: Weight is the gold standard in determining fluid status. It provides an objective assessment of the client's overall fluid status. As a reminder, One kilogram equals 2.2 pounds, equivalent to one liter of fluid. If the client has an increase in one kilogram, compared to their admission weight, they have responded favorably to the fluid replacement.

The primary healthcare provider (PHCP) prescribes a bolus of regular insulin prior to a continuous infusion. The prescription is for 0.1 units/kg. The client weighs 256 lbs. How many units of insulin should the nurse administer to the client?

The first step is to convert the client's weight from pounds (lbs) to kilograms (kg) 256 lbs → 116.36 kg Next, multiply the prescribed dosage by the client's weight 0.1 units x 116.36 kg = 11.63 units Finally, take the answer and round it to the nearest whole number 11.63 units = 12 units

The primary health care provider (PHCP) prescribes ondansetron 0.15 mg/kg IV to a pediatric client who weighs 40 lb. The medication label reads 2 mg/mL. How many milliliters will the nurse administer to the client? Fill in the blank. Round the answer to the nearest tenth.

The formula of dose ordered / dose on hand x volume will be utilized to solve this multistep problem. First, convert the weight from pounds to kilograms. 40 lbs / 2.2 kg = 18.18 kg Next, determine the prescribed dose 0.15 mg x 18.18 kg = 2.72 mg Next, take the dose ordered / dose on hand x volume 2.72 mg / 2 mg x 1 mL = 1.36 mL Finally, round the final result to the nearest number 1.36 mL = 1.4 mL

The primary healthcare provider (PHCP) prescribes a regular insulin infusion. for a client. The prescription is for 2 units/hr. The label on the medication reads 250 mL of 0.9% saline containing 100 units of regular insulin. How many mL/hr should the client receive?

The formula of dose ordered / dose on hand x volume will be utilized to solve this problem. Divide the prescribed amount of medication by what is on hand 2 units / 100 units = 0.02 units Next, take the amount of the medication and multiply it by the volume 0.02 units x 250 mL = 5 mL/hr

The primary healthcare provider (PHCP) prescribes a regular insulin infusion. The prescription is for 4.5 units/hr. The label on the medication reads 250 mL of 0.9% saline containing 100 units of regular insulin. How many mL/hr should the client receive? Fill in the blank. Round your answer to the nearest tenth.

The formula of dose ordered / dose on hand x volume will be utilized to solve this problem. Divide the prescribed amount of medication by what is on hand 4.5 units / 100 units = 0.045 units Next, take the amount of the medication and multiply it by the volume 0.045 units x 250 mL = 11.25 mL Finally, take the answer and round it to the nearest tenth. 11.25 mL = 11.3 mL

What key educational points should the LPN reinforce with the family of a 3-year-old child newly diagnosed with acute lymphoblastic leukemia (ALL)? Select all that apply. The importance of regular follow-ups for chemotherapy sessions. Monitor for signs of infection due to a weakened immune system. Possibility of hair loss as a side effect of chemotherapy. Need for regular blood tests to monitor the child's health status. The child will outgrow ALL naturally with time, without treatment.

The importance of regular follow-ups for chemotherapy sessions. Monitor for signs of infection due to a weakened immune system. Possibility of hair loss as a side effect of chemotherapy. Need for regular blood tests to monitor the child's health status. Rationale: This is correct because regular follow-up is crucial for the timely delivery of chemotherapy, which is the primary treatment modality for Acute Lymphoblastic Leukemia (ALL). Regular follow-ups also allow the healthcare team to monitor the child's response to treatment and manage any side effects promptly. ALL and its treatment can significantly weaken the immune system, making the child more susceptible to infections. Parents should be educated on signs of infection (like fever, cough, pain, or unexplained fatigue) and advised to seek immediate medical attention if these occur. Hair loss is a common side effect of many chemotherapy drugs used to treat ALL. Families need to understand this to prepare and support their children emotionally. Regular blood tests are necessary to monitor the child's blood cell counts, liver and kidney functions, and response to treatment. It can also help detect any complications at an early stage.

A practical nurse cares for an elderly client, reflecting on their life experiences and achievements. Which of the following statements reflects the concept of ego integrity? Select all that apply. The individual experiences a sense of fulfillment and satisfaction with life. The individual feels regret and dissatisfaction with life choices. The individual reflects positively on past accomplishments and life experiences. The individual feels confused and uncertain about their life's purpose. The individual demonstrates a negative outlook on the past and future.

The individual experiences a sense of fulfillment and satisfaction with life. The individual reflects positively on past accomplishments and life experiences. Rationale: This statement accurately reflects ego integrity as it describes fulfillment and satisfaction with life. This statement aligns with ego integrity as it reflects positive reflection on past accomplishments.

While you are in the NICU, an infant is being brought to the unit who is small for gestational age. Which of the following defines this term? A. The infant's weight is less than 3,000 grams. B. The infant's weight is below the 20th percentile. C. The infant's weight is less than 2,000 grams. D. The infant's weight is below the 10th percentile.

The infant's weight is below the 10th percentile. Rationale: When an infant's weight is below the 10th percentile, it is considered small for gestational age.

The nurse is discussing a breach of the standard of care with a nursing student. The nurse uses which of the following situations as an example of negligence? A. The UAP (Unlicensed Assistive Personnel) fills a water basin with warm water while the client with depression combs her hair. B. A nurse transcribes a new medication order: Questran powder 2 oz bid with wet food or one full glass of water. C. The nurse checks the distal pulses of a client's legs two hours after they have returned from a cardiac catheterization. D. The nurse observes a UAP enter the room of a client on contact precautions wearing gloves and a gown.

The nurse checks the distal pulses of a client's legs two hours after they have returned from a cardiac catheterization. Rationale: Delaying the assessment of distal pulses for two hours after a cardiac catheterization is a breach of the standard of care. Prompt and frequent assessments are crucial to monitor for potential complications and ensure the client's safety.

Which of the following is an example of a nurse-initiated nursing action? Select all that apply.

The nurse checks the skin of bedridden patients for break down. A nurse orders a kosher meal for an orthodox Jewish patient. The nurse prepares a client for minor surgery according to facility protocol. Rationale: Nurse-initiated interventions, also known as independent nursing actions, involve carrying out nurse-prescribed interventions resulting from their assessment of client needs to be documented on the nursing care plan, as well as other actions that nurses can initiate without the direction or supervision of another healthcare personnel.

The nurse is precepting a new nurse who collects a swab specimen from an open wound of a client with suspected pseudomonas infection. Which action requires immediate intervention by the nurse? A. The nurse flushes the wound with sterile saline. B. After inserting the swab, the nurse squeezes the culture tube. C. The nurse collects purulent drainage from the deepest part of the wound. D. The nurse uses a rayon swab to collect the specimen.

The nurse collects purulent drainage from the deepest part of the wound. Rationale: Purulent drainage or "pus" can give unreliable results and should not be used as the source for the specimen. The nurse should obtain the swab specimen by pressing and rotating the swab in the cleanest, deepest part of the wound.

Which nursing action is performed correctly when providing care for a newly placed gastrostomy tube of a postoperative client? A. The nurse dips a cotton-tipped applicator into the sterile saline solution and gently cleans around the insertion site. B. The nurse wets a washcloth and washes the area around the tube with soap and water. C. The nurse adjusts the external disk every three hours to avoid crusting around the tube. D. The nurse tapes a gauze dressing over the site after cleansing it.

The nurse dips a cotton-tipped applicator into the sterile saline solution and gently cleans around the insertion site. Rationale: When caring for a new gastrostomy tube, the nurse would use a cotton-tipped applicator dipped in sterile saline to gently cleanse the area, removing any crust or drainage.

The nurse has attended a staff education program about sources of negligent lawsuits. It would indicate effective understanding if the nurse identifies which of the following is a source of a negligent lawsuit? A. The nurse documents care under another nurse's username and password B. The nurse takes pictures of a client's medical record and distributes them online C. The nurse does not notify the physician of a client's critical laboratory result D. The nurse treats their spouse in the acute care facility with prescribed medications

The nurse does not notify the physician of a client's critical laboratory result Rationale: Negligence is an unintentional tort and is defined as the failure of a person to exercise the degree of care that an ordinarily prudent person would have exercised under similar circumstances. This is deemed malpractice when the nurse is committing these infractions in a professional capacity. Malpractice is the failure of a professional person to act as other prudent professionals with the same knowledge and education would have acted under similar circumstances. The failure of the nurse to communicate with the physician a critical laboratory result in a timely manner is classic negligence/malpractice. Other examples of negligent acts include - Failure to follow standards of care Failure to use equipment in a responsible manner Failure to communicate Failure to document Failure to monitor Failure to act as a client advocate

The LPN is training a new nurse on the medical-surgical floor. Which action would warrant intervention by the experienced LPN? A. The nurse administers oral cefdinir 30 minutes early. B. The nurse places a surgical mask on a patient with influenza before transport. C. The nurse obtains green drainage from a nasogastric tube for culture. D. The nurse secures a Jackson-Pratt drain to the patient's gown with a safety pin.

The nurse obtains green drainage from a nasogastric tube for culture. Rationale: Nasogastric tubes drain gastric contents, which are typically yellow/green in color due to the presence of bile. Sending it for cultures may cause unnecessary worry for the patient and would be inappropriate since this is an expected assessment. All other actions listed are appropriate.

The nurse is caring for a patient who had a fenestrated tracheostomy tube placed one week ago. Which of the following are true regarding fenestrated tracheostomies? Select all that apply. This type of tracheostomy does not require trach care. The patient with a fenestrated tracheostomy can speak. This is the only type of tracheostomy used with mechanical ventilation. A fenestrated tracheostomy can be capped if the cuff is deflated. Risks associated with fenestrated tracheostomies include bleeding, infection, accidental decannulation (removal of the tracheostomy tube), and difficulty breathing.

The patient with a fenestrated tracheostomy can speak. A fenestrated tracheostomy can be capped if the cuff is deflated. Risks associated with fenestrated tracheostomies include bleeding, infection, accidental decannulation (removal of the tracheostomy tube), and difficulty breathing. Rationale: It is true that patients with a fenestrated tracheostomy can speak. Fenestrated tracheostomy tubes have a small opening in the outer cannula. This allows some air to escape through the larynx, which means that the patient will be able to speak with this type of tube. A fenestrated tracheostomy can be capped if the cuff is deflated. It is very important to remember to deflate the cuff if capping a fenestrated tracheostomy tube, because if the tube is capped and the cuff is still inflated the patient will not be able to breathe at all. Risks associated with fenestrated tracheostomies include bleeding, infection, accidental decannulation (removal of the tracheostomy tube), and difficulty breathing.

When providing care for a client with known IV drug use, which statement would be appropriate for the nurse to highlight this behavior's risk factors? A. The use of these drugs can increase the risk of contracting diseases due to immunosuppression. B. IV drug use can lead to skin infections at injection sites and poor health. C. The risk of contracting and/or spreading blood-borne pathogens such as HIV, which can progress to AIDS, is a huge risk factor with this activity. D. Drug use can lead to unsafe sex practices, increasing the risk of transmission of sexually transmitted diseases/infections.

The risk of contracting and/or spreading blood-borne pathogens such as HIV, which can progress to AIDS, is a huge risk factor with this activity. Rationale: IV drug use involves sharing needles, which significantly increases the risk of transmitting blood-borne pathogens such as HIV, hepatitis B, and hepatitis C. These infections can lead to severe health consequences, including the progression to AIDS for individuals infected with HIV.

When discussing the Denver II with a preschooler's parents, which of these statements would indicate that they correctly understood the teaching? A. This test will tell me whether or not my child's IQ is normal. B. This test will tell me what developmental tasks my child can do today. C. This test will measure my child's development. D. This will let me know if my child's development is normal or not.

This test will measure my child's development. Rationale: The Denver Developmental Screening Test (DDST) was devised to provide a simple method of screening for evidence of slow development in infants and preschool children. The test covers four functions: gross motor, language, fine motor-adaptive, and personal-social. It has been standardized on 1,036 presumably healthy children (two weeks to six years of age) whose families reflect the occupational and ethnic characteristics of Denver's population.

While on your first shift at a Sleep clinic, you are reviewing the stages of sleep. Place the following steps or phases of sleep in an appropriate sequential order of the sleep cycle. The stage of the sleep cycle that is characterized by a 10 to 20-minute duration. The stage of the sleep cycle that is characterized by delta waves. The stage of the sleep cycle that is characterized by vivid dreams. The stage of the sleep cycle that is characterized by a brief period of very light sleep.

The stage of the sleep cycle that is characterized by a brief period of very light sleep. The stage of the sleep cycle that is characterized by a 10 to 20-minute duration. The stage of the sleep cycle that is characterized by delta waves. The stage of the sleep cycle that is characterized by vivid dreams. Rationale: Several clients suffer from sleep-related disorders and insomnia. Knowing the sleep cycle and the stages of the sleep cycle helps the nurse to understand the sleep pattern disturbances better. Each Sleep cycle lasts 90 to 120 minutes and repeats throughout the night.

*NGN* The nurse cares for a client in the outpatient surgical center who is scheduled for a cholecystectomy Item 1 of 1 Nurses' Note 0730 - The client arrives at the preoperative area with his family. He reports that he is anxious about the procedure. The pre-operative assessment was completed at this time. 20-gauge peripheral vascular access established in the right antecubital space. + blood return and flushes without resistance. The client reports no pain at the insertion site. The nurse reviews the completed pre-operative assessment Click to highlight the findings on the assessment that require follow-up Assessment Demographics ID verified and band applied Legal The surgeon has not obtained informed consent Medications Client took his prescribed phenytoin with a sip of water this morning Diet The client reports his last meal and fluid intake was the previous day at 2200 Discharge planning The client stated h

The surgeon has not obtained informed consent The client stated he was going to drive himself home after the procedure Rationale: Assessment items requiring follow-up include the informed consent not yet obtained by the surgeon. Before further preoperative activities may continue, the nurse must ensure this is completed to avoid unnecessary diagnostic testing and intervention. Additionally, the client will not be permitted to drive themselves home after this procedure because this involves general anesthesia. Activities requiring significant concentration, operation of heavy machinery, or driving are typically prohibited 24 hours following the initiation of general anesthesia.

You are witnessing a nurse waste available morphine. You should be aware of which of the following correct legal mandates in terms of controlled substances? Select all that apply. Signatures of 2 registered nurses but not of practical nurses when a narcotic is wasted. Prohibitions against the use of a placebo for pain management. The signatures of 3 registered nurses or practical nurses when a narcotic is wasted. The verification of the narcotic count at the beginning and the end of the shift. Checking the controlled substance at least 3 times before its administration. The secure locking of controlled substances to prevent diversion and theft.

The verification of the narcotic count at the beginning and the end of the shift. The secure locking of controlled substances to prevent diversion and theft. Rationale: The verification of the narcotic count at the beginning and the end of the shift (Choice D) and the secure locking of controlled substances to prevent diversion and theft (Choice F) are legal mandates in terms of narcotics and controlled substances.

A patient presents to the emergency department with a dissecting aortic aneurysm. The patient needs immediate surgery to save his life. He is unconscious and there is no family contact information on file. Which action is appropriate for obtaining informed consent for the surgery? A. There is no need for obtained consent. Send the client to surgery. B. Call the hospital lawyer. C. Search for people who may know the patient and can provide informed consent. D. Notify the on-call nursing supervisor and request her permission to waive informed consent.

There is no need for obtained consent. Send the client to surgery. Rationale: When emergency surgery is needed, delaying the surgery to obtain informed consent may result in the patient's morbidity or death. In such urgent cases, informed consent is unnecessary. It is most appropriate to begin the surgery to save the patient's life.

The pediatric nurse is treating an 18-month-old who has tested positive for Respiratory Syncytial Virus (RSV). Which of the following signs and symptoms would the nurse expect to find? Select all that apply. Thin nasal secretions. Productive cough. Bradypnea. Nasal flaring. Crackles in lungs. Drooling.

Thin nasal secretions. Nasal flaring. Crackles in lungs. Rationale: Thin nasal secretions are an expected symptom of RSV. This is an acute viral infection that affects the bronchioles. Children experience significant upper respiratory congestion when dealing with RSV and may need frequent suctioning to keep the airway clear and lessen the work of breathing. Nasal flaring is an expected sign of RSV. This is a typical signal of respiratory distress in an infant or young child. The nares flare outward with inspiration due to use of accessory muscles and increased effort to breathe. Crackles in the lungs are an expected finding with RSV. This sound occurs due to inflammation or fluid accumulation in the alveoli which results in decreased gas exchange. Wheezing may also occur due to the thick mucus in the bronchioles that restricts air flow.

The licensed practical/vocational nurse (LPN/VN) is reinforcing teaching to a client about the newly prescribed medication, epoetin alfa. Which of the following information should the nurse include? A. This medication will decrease your risk of infection B. You may notice black, tarry stools while on this medication C. This medication may raise your blood pressure D. Take this medication with food rich in Vitamin C

This medication may raise your blood pressure Rationale: Epoetin alfa is an erythropoietic growth factor indicated to increase red blood cell production for those with chronic kidney disease. This medication expands blood plasma with the therapeutic effect of increasing hemoglobin and hematocrit. It is essential to monitor the client's blood pressure while taking this medication, as an increase in blood pressure may be seen secondary to the increased blood volume. Uncontrolled hypertension is a contraindication to this medication.

The nurse is assessing a client with diabetic ketoacidosis (DKA). Which of the following would be an expected finding? Select all that apply. Thready pulse Jugular venous distention (JVD) Coarse tremors Tachycardia Orthostatic hypotension

Thready pulse Tachycardia Orthostatic hypotension Rationale: A client presenting with DKA will have signs and symptoms of dehydration that range from mild to severe. Tachycardia is a common finding in DKA because of the fluid volume deficit. This, in turn, causes a client to have a thready pulse. Orthostatic hypotension is also a common finding because of dehydration.

The nurse is planning a staff development conference about measures to reduce medication errors. It would be appropriate for the nurse to state which actions may help reduce medication errors? Select all that apply.

Timely medication reconciliation Limit the use of verbal orders to emergent situations Rationale: Timely medication reconciliation is fundamental in the prevention of medication errors. Medication reconciliation is when the client's medications are inventoried and assessed for interactions, duplications, or omissions. Completing a medication reconciliation at admission, a transition of care, and discharge will identify potentially dangerous drug-to-drug interactions. Verbal and telephone errors are significant sources of medication errors. The nurse should limit these types of orders to emergent situations. These methods are discouraged because accents, dialects, and drug name pronunciations may skew the order being given.

The primary healthcare provider (PHCP) prescribes dopamine at 5 mcg/kg/minute. The client weighs 178 lbs. The medication label reads dopamine 800 mg in 500 mL of dextrose 5% water (D5W). How many mL per hour will be administered to the client? Fill in the blank. Round your answer to the nearest whole number.

To solve this multistep problem, the formula of dose ordered / dose on hand x volume will be used First, convert the weight to kilograms 178 lbs / 2.2 = 80.9 kg Next, determine the hourly dosage 5 mcg x 80.9 kg x 60 minutes = 24270 mcg Next, convert the micrograms to milligrams 24270 mcg / 1000 mg = 24.27 mg Next, divide the dose ordered by the amount on hand x the volume 24.27 mg / 800 mg x 500 mL = 15.16 mL/hr Finally, round the answer to the nearest whole number 15.16 mL/hr = 15 mL/hr

The primary healthcare provider (PHCP) prescribes lidocaine at 2 mg/min. The medication label reads lidocaine 1 gram in 500 mL of 0.9% saline. How many mL per hour will be administered to the client?

To solve this multistep problem, the formula of dose ordered / dose on hand x volume will be used First, determine the hourly dosage 2 mg x 60 mins = 120 milligrams Next, convert the milligrams to grams so the units align 120 milligrams / 1000 = 0.12 grams Finally, divide the dose ordered by the amount on hand x the volume 0.12 grams / 1 gram x 500 mL = 60 mL/hr

The primary healthcare provider (PHCP) prescribes 100 mL of 0.9% saline to infuse over 45 minutes. How many mL per hour will be administered to the client?

To solve this problem, the formula of volume / time (hours) will be used. First, convert the minutes to hours 45 minutes / 60 minutes = 0.75 hrs Next, divide the prescribed total volume by the infusion time 100 mL / 0.75 hours = 133.33 Finally, take the mL/hour and round to the nearest whole number 133.33 = 133 mL/hr

he primary healthcare provider (PHCP) prescribes 150 mL of sterile water to be administered over one hour. The drop factor is 15 gtts/mL. The nurse sets the flow rate at how many drops per minute?

To solve this problem, the nurse will use the formula of total volume x drop factor / time in minutes First, take the prescribed volume and multiply it by the drop factor 150 mL x 15 gtt = 2250 mL Next, divide the total volume by the minutes 2250 mL / 60 minutes = 37.5 gtts Finally, perform appropriate rounding (if needed) 37.5 gtts = 38 gtts/minute

The primary healthcare provider (PHCP) prescribes azithromycin 500 mg, daily for a client with sepsis. The medication label reads 500 mg of azithromycin mixed in 250 mL of 0.9% saline. The drop factor is 15 gtts/mL and it is to be infused over 60 minutes. The nurse sets the flow rate at how many drops per minute? Round your answer to the nearest whole number. Fill in the blank.

To solve this problem, the nurse will use the formula of total volume x drop factor / time in minutes First, take the prescribed volume and multiply it by the drop factor 250 mL x 15 gtt = 3750 mL Next, divide the total volume by the minutes 3750 mL / 60 minutes = 62.5 gtts Finally, perform appropriate rounding (if needed) 62.5 gtts = 63 gtts

As a practical nurse, what educational points should you reinforce with the parents of a toddler diagnosed with an imperforate anus? Select all that apply. Toilet training will take longer for your child. Normal bowel habits can be established for your toddler over time. Do your best to have your child toilet trained before kindergarten starts. "Bowel irrigations may help your toddler achieve normal bowel function." Teach parents how to recognize signs of complications

Toilet training will take longer for your child. Normal bowel habits can be established for your toddler over time. "Bowel irrigations may help your toddler achieve normal bowel function." Teach parents how to recognize signs of complications Rationale: Toilet training for a toddler diagnosed with imperforate anus will take longer than for children who do not have this diagnosis (Choice A). Imperforate anus is usually diagnosed in infancy, but treatment will continue into their toddler years, affecting toilet training. To achieve toilet training, they will need to establish bowel habits and bowel management programs. Regular bowel habits can be established for toddlers diagnosed with imperforate anus over time (Choice B), but they will need to establish bowel habits and bowel management programs. Bowel irrigations will help the toddler achieve normal bowel function (Choice D). They may not need them every day, but bowel irrigations will likely be needed frequently to achieve regular bowel function. Teach parents how to recognize signs of complications (Choice E), such as fever, abdominal distension, or bloody stools, and when to seek medical attention.

The nurse is caring for a client diagnosed with epilepsy. The nurse should anticipate a prescription for which of the following medications? Select all that apply. Topiramate Risperidone Prazosin Hydroxyzine Lorazepam

Topiramate Lorazepam Rationale: Epilepsy is an idiopathic condition that requires maintenance treatment by using anticonvulsants. Topiramate is an anticonvulsant that may be used in the prevention of seizures. Lorazepam is also indicated for epilepsy in the event of a client experiencing an acute tonic-clonic or complex partial seizure. Topiramate should be used for maintenance purposes, and lorazepam would be indicated for an acute seizure.

Which of the following vaccines contains a live virus? A. IPV B. DTaP C. Varicella D. Hepatitis B

Varicella Rationale: Varicella is a live virus. Currently, available live attenuated viral vaccines are measles, mumps, rubella, vaccinia, varicella, zoster (which contains the same virus as varicella vaccine but in a much higher amount), yellow fever, rotavirus, and influenza (intranasal).

Which of the following iso-enzymes is most specific to myocardial infarction? A. CPK-MB B. Troponin C. Creatinine kinase D. Myoglobin

Troponin Rationale: Troponin is the most specific iso-enzyme when evaluating a suspicion of myocardial infarction (MI). Levels of troponin will elevate within 3-4 hours of myocardial infarction and remain elevated for three weeks. This means that not only is troponin the most specific cardiac biomarker for an MI, it is also the most reliable test to run if the client does not seek care for some time after their symptoms begin.

Which stage of psychosocial development does the nurse know a 2-month-old infant will be in? A. Initiative vs. Guilt B. Autonomy vs. Shame and Self Doubt C. Trust vs. Mistrust D. Industry vs. Inferiority

Trust vs. Mistrust Rationale: Trust vs. Mistrust is the typical development stage for infancy, which lasts from birth to 18 months. This is the stage the nurse would expect for her 2-month-old client. In this stage, children develop a sense of trust when caregivers provide reliability, care, and affection. When infants do not have that, they will develop mistrust.

The LPN is reinforcing education about caring for a client with a pacemaker. Which of the following teaching topics should the nurse prioritize? Select all that apply. Understanding the indications for pacemaker placement. Recognizing signs of pacemaker malfunction. Explaining different types of pacemakers and how they function. Advising clients with pacemakers to avoid all sources of electromagnetic fields (EMFs). Discussing the importance of regular pacemaker check-ups and follow-up care. Providing guidelines on safe physical activities and exercise after pacemaker placement

Understanding the indications for pacemaker placement. Recognizing signs of pacemaker malfunction. Explaining different types of pacemakers and how they function. Discussing the importance of regular pacemaker check-ups and follow-up care. Providing guidelines on safe physical activities and exercise after pacemaker placement Rationale: Knowledge of the indications for pacemaker placement is essential for nurses caring for clients with pacemakers. By understanding the medical conditions or heart conditions that may necessitate a pacemaker, nurses can better anticipate the needs of the client. Recognizing signs and symptoms of a malfunctioning pacemaker, such as dizziness, palpitations, or shortness of breath, enables nurses to respond promptly and seek appropriate medical intervention. Identifying potential issues with the pacemaker's functioning can prevent serious complications and improve client outcomes. Providing information about the various types of pacemakers and their functionalities allows nurses to tailor care to each client's specific device. Different pacemakers, such as single-chamber, dual-chamber, or biventricular pacemakers, serve distinct purposes and address unique cardiac conditions. Regular pacemaker check-ups and follow-up care are essential for monitoring the device's functionality and the client's cardiac status. Scheduled follow-up visits with healthcare providers allow for pacemaker programming adjustments, device interrogation, and information on the client's response to the device. Reinforcing education on safe physical activities and exercise after pacemaker placement empowers them to maintain a healthy lifestyle while avoiding unnecessary risks to the pacemaker's functioning. It is essential to provide guidelines that encourage clients to stay active and engage in appropriate phys

When instructing an ambulatory, alert, and oriented female client, what instructions should the nurse provide regarding collecting a midstream urine specimen? A. Cleanse the perineal area from back to front with circular wipes. B. Cleanse the vaginal meatus from the front to back. C. Use a new antiseptic wipe with each wipe and wipe from the inner labia to the outer labia. D. Use one wipe to cleanse the inner labia and another for the outer labia.

Use a new antiseptic wipe with each wipe and wipe from the inner labia to the outer labia. Rationale: Use a new antiseptic wipe with each wipe and wipe from the inner labia to the outer labia.

Which of these would be most important to include in discharge teaching for a client with a platelet count of 40.000 mcL (40 x 10^9/L)? A. Be sure to take your aspirin with meals daily. B. You may continue to shave with a straight-edge razor. C. Use a soft toothbrush and floss gently. D. You should take a multivitamin daily.

Use a soft toothbrush and floss gently. Rationale: This client has thrombocytopenia and should be on bleeding precautions. Using a soft toothbrush and flossing gently can prevent the gum tissue from bleeding. Platelets (thrombocytes) are important for blood clotting. The normal range for platelets is 150,000-400,000 mcL (150-400 x 10^9/L). Thrombocytopenia is a lower than normal number of platelets (less than 150,000 platelets per microliter) in the blood. It can be inherited or acquired. The causes of thrombocytopenia can be classified into three groups: Diminished production: caused by viral infections, vitamin deficiencies, aplastic anemia, or drug-induced Increased destruction: caused by drugs, heparin use, idiopathic, pregnancy, or immune issues Sequestration: caused by an enlarged spleen, neonatal, gestational, pregnancy

The nurse is caring for assigned clients. The nurse should recognize that the client at greatest risk for postpartum hemorrhage (PPH) is the client who has which of the following? A. Uterine atony and delivered with the assistance of forceps B. Postpartum urinary incontinence and diuresis C. An active outbreak of genital herpes and had a cesarean section D. Gestational diabetes and has postpartum hyperglycemia

Uterine atony and delivered with the assistance of forceps Rationale: Uterine atony is the most common cause of PPH. This is when the uterus fails to contract after delivery. Additionally, delivery with the use of instruments such as forceps raises the risk of PPH because of the trauma that may be caused by the instruments.

The nurse is caring for a 14-year-old scheduled for an appendectomy. What is the nurse's role in obtaining informed consent before surgery? Select all that apply. Informing the parents that only the surgeon may withdraw the surgical consent Review the risks and benefits of the surgery with the parents Validate that the parents are competent to provide consent for the client Witness the signature on the informed consent Make sure that the consent is witnessed by two healthcare professionals

Validate that the parents are competent to provide consent for the client Witness the signature on the informed consent Rationale: Since the client is 14, they are a minor, and their parents will be responsible for signing informed consent. The nurse is accountable for validating that the parents are competent to provide consent for the client. The nurse will serve as the witness for the informed consent. This is one of the primary responsibilities of the nurse when a client is getting a procedure and signing a consent. The other primary responsibility will be to serve as the client's advocate and ensure that the parents have received sufficient information to make an informed decision. If they have not, the nurse must call the surgeon to return and speak further with the parents.

The nurse detects an elevated temperature in a client who is scheduled for surgery. The client has been afebrile and has no other symptoms of fever. What should be the first nursing action? A. Inform the charge nurse. B. Inform the surgeon. C. Validate the finding. D. Document the finding.

Validate the finding. Rationale: The nurse should first validate the finding if it is unusual, deviates from normal, and is unsupported by other data.

The nurse cares for a 41-year-old female in the emergency department (ED) Item 3 of 6 Complete the sentences below from the list of options The client is at greatest risk for (venous thromoembolism, cellulitis, or compartment syndrome). The primary healthcare provider (PHCP) will likely order a (venous duplex ultrasonography, ankle-brachial index, or radiograph (x-ray)) to confirm this diagnosis.

Venous thromboembolism Venous duplex ultrasonography Rationale: This client most likely has venous thromboembolism. The triggering event of the VTE was likely the injury to her leg. Combined with the oral contraceptives and her tobacco use, these are substantial risk factors for her developing VTE. The gold standard diagnosis for VTE is using a venous duplex ultrasonography which is a non-invasive test that may be performed at the bedside.

The nurse is reviewing tetralogy of Fallot with a nursing student. It would indicate effective teaching if the student identifies which defects with this disorder? Select all that apply. Ventricular septal defect (VSD). Overriding aorta. Pulmonary artery stenosis. Concentric right ventricular hypertrophy. Mitral valve regurgitation

Ventricular septal defect (VSD). Overriding aorta. Pulmonary artery stenosis. Concentric right ventricular hypertrophy. Rationale: Tetralogy of Fallot is a congenital heart defect composed of four errors, a ventricular septal defect (VSD) being one of them. The VSD is a hole between the right and left ventricles, allowing the oxygenated and deoxygenated blood to mix in, essentially one ventricle. An overriding aorta being one of them is another feature. This means the aorta is positioned over the VSD instead of over the left ventricle, where it should be. Pulmonary stenosis is another feature of ToF. The pulmonary arteries are narrowed and hardened, making it difficult for the right ventricle to pump blood to the lungs. Right ventricular hypertrophy is one of them. This portion of the error is actually due to another part: pulmonary stenosis. Since these vessels are narrowed and hardened, it is difficult for the right ventricle to pump blood through them and out to the lungs. This puts extra work on the heart, and after some time, the muscle of the right ventricle gets more substantial or hypertrophied due to the extra work.

Which of the following are important nursing interventions for a client before going down for a cardiac catheterization? Select all that apply. Verify if they have any allergies. Check their BUN and creatinine levels. Check their CBC for WBC level. Administer acetaminophen.

Verify if they have any allergies. Check their BUN and creatinine levels. Rationale: It is essential to verify if a client has any allergies before any procedure. We must know if there are medications that are unsafe to administer to this client. We are explicitly verifying that the client is not allergic to iodine or shellfish. This is important because an iodine-based dye is used during cardiac catheterization to visualize the vessels. Should you discover that the client is allergic to iodine or shellfish, contact the health care provider immediately, and do not administer the dye. Verifying that the client has a normal BUN and creatinine is essential before beginning a cardiac catheterization. Because there is a large amount of iodine-based dye injected during this procedure, the kidneys must be functioning well to excrete the dye. If the client has poor BUN and creatinine, it can lead to severe kidney injury.

A client with a history of falls is admitted to the medical-surgical unit. The nurse should plan to implement which intervention to reduce this client's risk of falling? A. Encouraging the client to ambulate independently to improve muscle strength. B. Verify that the bed alarm is enabled during client rounding. C. Implementing a fall risk assessment every two days D. Implementing a restrictive mobility policy to minimize the potential of falls.

Verify that the bed alarm is enabled during client rounding. Rationale: Ensuring the bed alarm is on will help notify staff if the client is trying to get up unassisted and allow the staff to intervene and reduce their risk of falling. This client has a history of falls, and it is appropriate to provide this measure.

The nurse on the medical-surgical floor is preparing to receive a newly admitted client who reports an allergy to latex. The nurse should plan to take which action? Select all that apply. Verify the allergy is documented in the medical record Use disposable equipment during client care Assign the client to a private room Post a sign in the client's room noting the allergy Communicate the allergy to the dietary department

Verify the allergy is documented in the medical record Post a sign in the client's room noting the allergy Communicate the allergy to the dietary department Rationale: When a client is newly admitted, it is essential that the nurse verify the client's allergies and ensure that they are appropriately documented. A sign noting the allergy should be placed in the client's room because other healthcare personnel who have not reviewed the chart should be made aware of the allergy. Finally, the allergy should be communicated to the dietary department because certain foods (bananas, kiwi, avocado) have the latex protein.

The nurse is caring for a 26-year-old patient who is unable to meet their nutritional needs by mouth. The interdisciplinary team decides it would be best to insert an NG tube for enteral feedings. After inserting the tube, the nurse knows that which of the following is the most accurate way to verify the placement of the tube? A. Aspiration of stomach contents B. pH verification of the aspirate C. Auscultation of air in the LUQ when injected into the tube D. Visualization on x-ray

Visualization on x-ray Rationale: Visualization on x-ray is the gold standard for verification of nasogastric tube placement. This allows the radiologist to visualize the tip of the tube in the stomach and recommend any changes in placement that may be needed, such as pulling the tube back or advancing further.

The nurse is caring for a client who recently had a partial gastrectomy. Which of the following medications should the nurse anticipate that the primary health care provider (PHCP) will order? A. Vitamin B12 B. Metoclopramide C. Sucralfate D. Hydroxyzine

Vitamin B12 Rationale: Procedures like a gastrectomy put the client at risk for pernicious anemia (B12 deficiency). It is quite common for a client to receive parenteral B12 replacement indefinitely.

The nurse is taking care of a client with encopresis. Which of the following statements correctly describe encopresis? A. Infrequent and hard to pass stools lasting greater than two weeks. B. Voluntary or involuntary fecal incontinence in children over the age of 4 who were previously toilet trained. C. Involuntary fecal incontinence in children over the age of 4 who were previously toilet trained. D. Inability to pass stool due to fecal impaction

Voluntary or involuntary fecal incontinence in children over the age of 4 who were previously toilet trained. Rationale: Voluntary or involuntary fecal incontinence in children over the age of 4 who were previously toilet trained is the correct definition of encopresis.

The nurse caring for a three-year-old with congestive heart failure recognizes which of the following as an early sign of digitalis toxicity? A. Bradypnea B. Tachycardia C. Vomiting D. Failure to thrive

Vomiting Rationale: The earliest sign of digitalis toxicity is vomiting. One episode, however, does not warrant discontinuing the medication. Digoxin increases the force of myocardial contraction, decreases conduction through the SA and AV nodes, and prolongs the refractory period of the AV node. The result is increased cardiac output and reduced heart rate. Therapeutic serum digoxin levels range from 0.5-2 ng/mL. Serum levels may be drawn 6-8 hours after a dose is administered, although they are usually drawn immediately before the next dose. In infants, the first symptoms of overdose are typically cardiac arrhythmias. Gastrointestinal symptoms (like vomiting) are some of the earliest signs.

appendectomy assessment findings considered a priority:

WBC Temp Compliance of post-op antibiotics Rationale: at risk for infection; monitor for fever

Hospitalization may affect or delay the progression of which physical development in a 1-year-old client? A. Walking B. Running C. Sitting D. Crawling

Walking Rationale: At the age of 1 year, children should be starting to walk. Hospitalization during this age could delay this stage of development.

Which of the following medications is contraindicated for a pregnant client? Select all that apply. Warfarin Finasteride Celecoxib Clonidine Transdermal Nicotine Clofazimine

Warfarin Finasteride Rationale: Warfarin (coumadin) has a pregnancy category X. It is associated with central nervous system defects, spontaneous abortion, stillbirth, prematurity, illness, and ocular defects at any time during pregnancy and fetal warfarin syndrome when given during the first trimester (Choice A). Finasteride also has a pregnancy category X, which has a high risk of causing permanent damage to the fetus (Choice B).

Which steps should the LPN reinforce with a patient regarding blood glucose monitoring at home? Select all that apply. Washing and drying hands before testing. Choosing a different puncture site for each test. Read the monitor's instructions as each monitor may have different requirements. Recording the glucose level and the date/time of the test. When using an alcohol wipe, make certain the alcohol is completely dry before lancing.

Washing and drying hands before testing. Choosing a different puncture site for each test. Read the monitor's instructions as each monitor may have different requirements. Recording the glucose level and the date/time of the test. When using an alcohol wipe, make certain the alcohol is completely dry before lancing. Rationale: Washing and drying hands before testing. Clean hands are essential to ensure that the blood sample is not contaminated. Washing hands also helps to remove any residue that could affect the reading. Choosing a different puncture site for each test. This helps prevent soreness or callus formation at one spot. It's recommended to rotate the puncture sites for each test. Read the monitor's instructions, as each monitor may have different requirements. It's always essential to follow the instructions provided by the manufacturer of your specific glucose meter and test strips, as they may have specific guidelines on whether or not to wipe away the first drop of blood. Recording the glucose level and the date/time of the test. Maintaining glucose readings and the date and time helps track trends and patterns in blood glucose levels. This is valuable information for healthcare providers when adjusting diet, exercise, or medication regimens.

The nurse is discussing varicose veins and at-home management with a group of clients. Which of the following should the nurse reinforce in the teaching session? A. When you are sitting, keep your legs lower than your heart B. Wear compression stockings during the day C. Participate in activities that have you stand for long periods D. Take a low-dose aspirin to prevent the development of new varicose veins

Wear compression stockings during the day Rationale: Compression stockings/hoses are effective because the external pressure promotes venous return. Compression hose combined with frequent position changes, daily walks, frequent position changes, and keeping the legs elevated to facilitate venous return is recommended.

The nurse is caring for a client admitted to the medical-surgical unit with cystic fibrosis Item 1 of 1 History And Physical 11-year-old female with cystic fibrosis was admitted with failure to thrive after losing 3 kilograms (6.6 pounds) over the past two weeks. This started following a hospitalization because of pneumonia, and the client could not regain the weight. The client's parents agreed to a short hospitalization to collaborate with a nutritionist, intravenous (IV) fluids, and other treatment interventions. On exam, the client is alert and completely oriented. The client has a sunken eye appearance, very dry and flaky skin. Brittle hair and nails were noted. Very thin appearance. No cardiac murmurs were noted, and some rhonchi in both lung fields. Productive cough noted with clear sputum. Bowel sounds were hyperactive in all four quadrants. The client reported no pain. The nurse reviews the physician's hi

Weigh the client daily (Indicated) Perform chest physiotherapy following meals (Not Indicated) Inquire about the client's dietary preferences (Indicated) Administer prescribed pancrealipase after meals (Not Indicated) Obtain a prescription for a multivitamin (Indicated) Encourage snacks that are low in sodium (Not Indicated) Plan to collaborate with a registered dietician (Indicated)

*NGN* The nurse cares for a 20-year-old with schizophrenia in the outpatient clinic Item 1 of 1 Nurses' Notes 4/10: The client was alert and completely oriented. The client had adequate eye contact. Affect was flattened, and he described his mood as 'okay.' Responses had some speech latency, and responses were quite limited. The speech had a normal rate, tone, and volume. Denied any hallucinations and reported no suicidal or homicidal ideations. No abnormal motor movements were observed. The client reports he just got employed at a local grocery store. He reports no changes in his living situation and wants a roommate to offset costs. The client reports taking his prescribed olanzapine 10 mg PO daily without missing any doses. He reports that the medication gives him a headache when he does not take it with food. The client was started on this medication six months ago. He states he has been 'thirsty' more often an

Weight Reports of thirst Rationale: Olanzapine is an atypical antipsychotic, and a concerning trend was noted with the client's increasing weight. Olanzapine may cause significant metabolic problems, including increased blood glucose, weight, and dyslipidemia. The other concerning factor is the client reporting thirst, which may suggest elevated blood glucose. The client's flattened affect and altered speech patterns are expected findings with schizophrenia.

The nurse is caring for a client diagnosed with hyperthyroidism. Which of the following signs and symptoms would be expected? Select all that apply. Weight loss Diarrhea Decreased appetite Insomnia Palpitations

Weight loss Diarrhea Insomnia Palpitations Rationale: Hyperthyroidism results in overactive thyroid, resulting in an increased amount of thyroid hormones. There is an increase in metabolism and, therefore, weight loss despite an increased appetite. Symptoms include increased hunger, thirst, diarrhea, insomnia, palpitations, and tremors. Weight loss despite increased appetite is a symptom of hyperthyroidism. Diarrhea is a symptom of hyperthyroidism. Insomnia is a symptom of hyperthyroidism. Increased thyroid hormones increase the heart rate. Clients may experience palpitations. Arrhythmias (atrial fibrillation) may follow.

The nurse is teaching a client newly diagnosed with osteoporosis. The nurse should recommend that the client perform which type of exercise? A. Stretching B. Stability and balance C. Cardiovascular D. Weight-bearing

Weight-bearing Rationale: Low-impact weight-bearing exercises are recommended for a client with osteoporosis because of their ability to increase bone density. Examples of this type of exercise include walking and elliptical training.

Which of the following are important points about fever in children that the LPN should review with a family who is being discharged home today? Select all that apply. When to seek medical attention for a fever above a certain temperature Appropriate methods for managing a child's fever at home Signs and symptoms that may indicate a serious underlying condition Importance of monitoring the child's hydration status Administration of antibiotics to treat the fever

When to seek medical attention for a fever above a certain temperature Appropriate methods for managing a child's fever at home Signs and symptoms that may indicate a serious underlying condition Importance of monitoring the child's hydration status Rationale: When to seek medical attention for fever above a specific temperature: It is essential for the LPN to educate the family about the particular temperature threshold at which they should seek medical attention for their child's fever. This helps ensure appropriate and timely medical intervention when necessary. Appropriate methods for managing a child's fever at home: LPNs should inform the family about proper measures to address a child's fever at home, such as providing adequate hydration, using over-the-counter fever-reducing medications following proper dosage instructions, and using appropriate cooling techniques (e.g., lukewarm sponge baths). Signs and symptoms that may indicate a severe underlying condition: The LPN should educate the family about the signs and symptoms that may indicate a more serious underlying condition accompanying the fever. This helps them recognize when further medical evaluation is needed, such as persistent high fever, severe headache, stiff neck, or changes in behavior. Importance of monitoring the child's hydration status: The LPN must emphasize the significance of monitoring the child's hydration when they have a fever. They should educate the family about signs of dehydration, such as decreased urine output, dry mouth, and lethargy, and encourage them to maintain adequate fluid intake.

Which of the following foods can the nurse recommend to parents of toddlers who have constipation? Select all that apply. Mac and Cheese Whole grains Whole milk Black beans Yogurt or kefir

Whole grains Black beans Yogurt or kefir Rationale: Whole grains are rich in fiber and are, therefore, an excellent choice for toddlers who have constipation. Black beans are high in fiber and are an excellent choice for toddlers who have constipation. Probiotic rich foods such as yogurt and kefir are good for constipation.

The nurse is evaluating her clients's lab results and notes that the potassium is 5.5 mEq/L(3.5-5 mEq/L). Which of the following EKG changes would be associated with hyperkalemia in a client? Select all that apply. Inverted T waves Widened QRS interval Tall, peaked T waves Prominent U-waves Prolonged PR interval

Widened QRS interval Tall, peaked T waves Prolonged PR interval Rationale: - Widened QRS interval is a very important EKG finding in hyperkalemia. Other EKG changes clients may experience when they are hyperkalemic include wide, flat P waves, a prolonged PR interval, a depressed ST segment, and tall, peaked T waves. - Tall, peaked T waves are a hallmark sign of hyperkalemia on an EKG. Remember this - it is a very common topic for NCLEX questions!! Hyperkalemia leads to serious arrhythmias and can progress to heart block, ventricular fibrillation, or even asystole if left untreated. - The PR interval, which reflects the time taken for the electrical impulse to travel from the sinoatrial (SA) node to the atrioventricular (AV) node, may become prolonged in hyperkalemia.

Following bariatric surgery, which of the following should the nurse give priority to preventing? A. Pain B. Depression C. Thrombophlebitis D. Wound infection

Wound infection Rationale: Wound infection is the most common complication among obese clients who have had surgery. This is mostly in part due to inadequate blood supply in the adipose tissue of obese clients.

Your client is receiving a non-steroidal anti-inflammatory drug (NSAID) in addition to a narcotic analgesic. The client wonders why an NSAID is necessary since the narcotic analgesic offers better pain relief. How would you respond to the client's question? A. I don't know and I suggest that you ask your doctor when you see her the next time. B. You are getting the NSAID because we are trying to wean you off the narcotic analgesic for moderate to severe pain. C. You are getting the NSAID, so the effects of the narcotic analgesic to combat your pain are more effective. D. You are getting the NSAID because it is a placebo, and it is proven to be effective for severe pain.

You are getting the NSAID, so the effects of the narcotic analgesic to combat your pain are more effective. Rationale: "You are getting the NSAID, so the effects of the narcotic analgesic to combat your pain are more effective" is an appropriate response to the client's query. An NSAID is an "adjuvant" medication used in combination with narcotic analgesics to treat moderate to severe pain. Adjuvant pain medications are used to enhance pain relief provided by other analgesics. The primary function of NSAIDs is to reduce inflammation. Therefore, NSAIDs are helpful in treating the pain caused by inflammation.

The nurse working in the emergency department triages a 29-year-old who states, "I am going to kill myself. They are coming for me!" Which of the following responses utilizes therapeutic communication? A. You are safe here; can you tell me more about what is happening? B. Please don't try to kill yourself; we will sedate you if we have to. C. Why would you kill yourself? D. Who is coming for you?

You are safe here; can you tell me more about what is happening? Rationale: This statement uses therapeutic communication by helping the client feel safe and asking open-ended questions to gather more information.

*NGN* 72-year-old male presents to the emergency department Item 5 of 6 Medications Bupropion 150 XL PO After receiving a consultation with psychiatry, the physician prescribed this client medications The nurse prepares to administer the prescribed bupropion. Which two (2) teaching points should the nurse reinforce? This medication may cause you to gain weight. You may notice a decreased libido while on this medicine. Have a diet that has a consistent intake of salt and water. You may have more energy with the medication. You will need ongoing laboratory work while on this medicine. Take this medication in the morning to prevent sleep problems.

You may have more energy with the medication. Take this medication in the morning to prevent sleep problems. Rationale: Bupropion is an atypical antidepressant that modulates norepinephrine and dopamine. This medication is activating and does provide the client with increased energy, therefore, decreasing their weight. Bupropion does not cause sexual side effects like other antidepressants. Considering that this medication is activating, it would be appropriate for the nurse to advise the client to take it in the morning and not in the evening to avoid worsening the insomnia. Laboratory work is not necessary while a client is taking bupropion, nor is maintaining a diet consistent in fluid and salt. This dietary recommendation is for lithium.

The nurse is caring for a seven-year-old client brought to the clinic by her parents Item 6 of 6 The nurse reinforces discharge teaching to the client Click to specify the information the nurse should reinforce? Select all that apply Your child may return to school once all the lesions have crusted. Warm baths with baking soda or oats may help with the itching. Contact the school to report your child's infection. Watch for signs of skin infection including swelling, drainage, and pain. To treat the fever, you may alternate between acetaminophen and aspirin.

Your child may return to school once all the lesions have crusted. Warm baths with baking soda or oats may help with the itching. Contact the school to report your child's infection. Watch for signs of skin infection including swelling, drainage, and pain. Rationale: Once all of the lesions have crusted over the child may resume attending school. The school should be notified of the infection to initiate appropriate notification to those who could be infected. Treatment of varicella is primarily symptomatic through antipyretics and antihistamines. Warm baths with baking soda or oats may provide additional relief from itching. Cellulitis is a complication of varicella caused by bacteria that enters the skin that is excessively itched.

The infection control nurse is collecting data on clients at risk for a urinary tract infection (UTI). Which client is at the greatest risk of developing a UTI? A client with A. a chronic indwelling urinary catheter receiving intravenous diuretics. B. diabetes mellitus who is receiving intravenous antibiotics for a wound infection. C. obesity being treated for urge incontinence. D. a history of frequent bladder infections.

a chronic indwelling urinary catheter receiving intravenous diuretics. Rationale: The most significant risk factor for a urinary tract infection is the presence of an indwelling urinary catheter. Bacteria may colonize the tip of the catheter within 48 hours of its placement. Thus, the nurse recognizes that having this invasive device is a key risk factor for UTI.

The nurse has collected a client's vital signs. The nurse notes that the client's apical pulse was 75 beats per minute, and the radial pulse was 69 beats per minute. The nurse should document this finding as A. a widened pulse pressure. B. a pulse deficit. C. pulsus paradoxus. D. an expected finding.

a pulse deficit. Rationale: A pulse deficit is a difference between the apical and peripheral pulses. This finding may signal that the client has a dysrhythmia, and the nurse should consider obtaining a 12-lead electrocardiogram and/or continuous telemetry monitoring.

The nurse cares for a 41-year-old female in the emergency department (ED) Item 5 of 6 Orders Admit to Med/Surg Activity as tolerated Initiate peripheral vascular access Oxycodone 5 mg PO q 6 hours, PRN pain Laboratory (aPTT, CBC, CMP, PT/INR) Warfarin 2 mg PO, Daily Heparin 5000 units intravenous (IV) bolus x1 Heparin infusion at 18 units/kg/hr; follow heparin protocol The nurse reviews the physician's orders and prescriptions Complete the sentences from the list of options below Prior to administering heparin, the nurse should obtain the client's (aPTT, complete metabolic panel (CMP), or PT/INR) and (blood type, weight, or height). While the client is receiving the prescribed heparin, the nurse will also need to monitor the client's (blood glucose, platelet count, or sodium level).

aPTT Weight Platelet count Rationale: Heparin infusions require an actual and accurate weight of the client. This information, along with the baseline labs, is necessary prior to administering heparin. Platelet count also needs to be monitored during the duration of the heparin infusion because of the potential adverse event of heparin-induced thrombocytopenia (HIT).

The nurse is caring for assigned clients. The nurse should initially A. administer prescribed antibiotics to a client with bacterial meningitis. B. reposition a client with chronic back pain who reports pain rated 6/10 on the Numerical Rating Scale. C. remove a nitroglycerin transdermal patch for a client with chronic angina. D. collect data on a client who had a coronary artery bypass grafting (CABG) three days ago and has a serum glucose of 135 mg/dL [70-110 mg/d].

administer prescribed antibiotics to a client with bacterial meningitis. Rationale: Bacterial meningitis has a high mortality rate and requires aggressive and prompt antibiotic treatment. The nurse should prioritize this client because this is a serious, acute concern that may result in death. Prescribed ceftriaxone and vancomycin are the preferred antibiotics in treating this pathogen.

The nurse is caring for a seven-year-old client brought to the clinic by her parents Item 2 of 6 Diagnosis Varicella Complete the following sentence by choosing from the list of options The primary healthcare provider diagnoses the client with varicella. The nurse understands that this virus is primarily spread through (aerosolized droplets, contaminated water, or surfaces of objects.) To primarily prevent the transmission of varicella, the nurse should advocate for (immunization, a prescription for valacyclovir, or screening those in the household.)

aerosolized droplets immunization Rationale: High concentrations of the varicella-zoster virus are found in the nasopharynx. The virus is primarily transmitted through aerosolized droplets. Primary prevention stems from education and immunization. To advocate for the primary prevention of varicella, the nurse should encourage immunization.

The nurse has received a prescription for bupropion for a client with major depressive disorder. Which of the following conditions in the client's medical history would be a contraindication to administering this medication? Select all that apply.

anorexia nervosa epilepsy Bupropion is an atypical antidepressant medication primarily indicated in major depressive disorder, seasonal affective disorder, smoking cessation, and antidepressant-induced smoking cessation. Bupropion is contraindicated if a client has conditions that may cause seizures, such as epilepsy. Bupropion tends to cause weight loss and is contraindicated for clients with anorexia nervosa or bulimia nervosa, as further weight loss would be detrimental in these conditions. Bupropion lowers the seizure threshold; if the client has a medical history of seizures, they should not receive this medication.

The nurse is teaching a course regarding bone marrow toxicity. The nurse should reinforce that the most serious form of bone marrow toxicity is A. aplastic anemia. B. leukocytosis. C. thrombocytopenia. D. granulocytosis.

aplastic anemia. Rationale: Aplastic anemia is the result of a hypersensitivity reaction and is often irreversible. It leads to pancytopenia, a severe decrease in all cell types: red blood cells, white blood cells, and platelets.

The nurse has attended a staff education program about disseminated intravascular coagulation (DIC). Which of the following clients is at risk for DIC? A client A. with iron deficiency anemia receiving parenteral iron sucrose infusion. B. being treated for gram-negative sepsis with intravenous antibiotics. C. with atrial fibrillation receiving prescribed rivaroxaban to reduce their risk for stroke. D. taking a daily aspirin to reduce their risk for acute coronary syndrome.

being treated for gram-negative sepsis with intravenous antibiotics. Rationale: A client with gram-negative sepsis faces an array of complications, including DIC. The release of the endotoxin by the bacteria may cause excessive activation of the clotting cascade, leading to exhaustion by the clotting factors. Clients with sepsis need to be monitored for DIC as this condition is life-threatening. The risk for DIC caused by gram-negative sepsis is even higher for those with hypothermia and acidosis because the coagulation factors' enzymatic functions are pH and temperature-dependent.

he nurse is caring for an adolescent with hemophilia who reports pain and joint bleeding after playing baseball. The nurse should prioritize the client's A. acute pain. B. bleeding. C. self-esteem. D. knowledge deficit.

bleeding. Rationale: Addressing the client's active bleeding is the priority, as the client may develop shock if it goes untreated. Bleeding associated with hemophilia is often found in the joint space or at the point of physical injury.

The licensed practical/vocational nurse (LPN/VN) reviews the function of a prescribed beta-blocker in the cardiovascular system. It would be appropriate for the nurse to state that beta-blockers Select all that apply. block catecholamines from binding to the beta receptors. reduce myocardial oxygen demand. increase cardiac contractility. increase cardiac output. prevent sodium and water resorption by inhibiting aldosterone secretion.

block catecholamines from binding to the beta receptors. reduce myocardial oxygen demand. Rationale: Beta-blockers decrease blood pressure by causing vasodilation of the vessels. They block catecholamines from the beta receptor sites found in the heart and lungs. Beta-blockers decrease the heart's workload through vasodilation and lowering the heart rate. This relaxation of the vasculature and reduction in heart rate will reduce the myocardial oxygen demand. This is why beta blockers (low doses) may be prescribed during an acute myocardial infarction and afterward.

The licensed practical/vocational (LPN/VN) nurse collects data on a client with Huntington's disease. Which of the following findings would be expected? Select all that apply. halitosis chorea hallucinations hematemesis weight loss

chorea hallucinations weight loss Rationale: The cardinal features of Huntington's disease include chorea (brief, involuntary movements involving the trunk, limbs, and face). Psychiatric symptoms such as depression, paranoia, delusions, and hallucinations are common with this disease. Weight loss is also a common finding caused by the excessive energy expended in abnormal movements.

*NGN* The clinic nurse is caring for a 38-year-old male Item 1 of 6 Nurses' Notes 1456 - 38-year-old male reports to the clinic for an annual physical examination and to establish care. The client reports no acute concerns but does admit to gaining a few pounds over the past several months. The client reports having decreased physical activity. He reports his dietary habits have changed because of his job, where he relies on fast food for breakfast and lunch. On assessment, the client is alert and completely oriented to person, place, and situation. The skin is warm and dry—patches of darkening and thickening of the skin around the skin folds. Lung sounds are clear; S1/S2 heart tones are auscultated. Peripheral pulses palpable, 2+. Bowel sounds are active in all quadrants. He denies any dysuria and reports his sex drive has decreased over the past several months. He reports occasional constipation, which causes

blood pressure waist size body mass index (BMI) dietary habits darkening patches in the skin folds stress Rationale: Concerning findings for this client include the client's high blood pressure (141/92 mm Hg). One reading cannot diagnose hypertension, but it is essential that this finding get addressed. The client's weight gain is concerning, as his BMI is 29, considered overweight (overweight is a BMI of 25 to 30; obesity is a BMI greater than 30). His increased waist size is concerning because abdominal obesity causes insulin resistance. Abdominal or central obesity is defined as 40 inches (102 cm) or greater for men or 35 inches (88 cm) or greater for women. The client's dietary habits are concerning because it was reported that he relies on fast food for most of his meals. Ultra-processed foods are directly linked to the development of cardiovascular disease and are a risk for colon cancer. Darkening patches in the skin folds is concerning because this could be acanthosis nigricans which occurs with insulin resistance. This may be a clinical sign suggesting diabetes mellitus. The client's stress level is concerning. Excessive and prolonged exposure to stress directly threatens a client's cardiovascular status. This is a modifiable risk factor for hypertension.

The licensed practical/vocational nurse (LPN/VN) is collecting data on a client who is pregnant. Which of the following findings would require follow-up? Select all that apply. third (S3) heart sound burning with urination dependent edema absence of menstruation breast tenderness decrease appetite

burning with urination decrease appetite Rationale: Urinary frequency and urgency are common during pregnancy because of hormonal changes, increased blood flow to the kidneys, blood volume, and glomerular filtration rate (GFR). Nocturia is also common because sodium and water are retained during the day and excreted overnight. Burning with urination is not expected and may require a prescribed urine analysis (UA) to rule out cystitis. A decrease in appetite is also not expected, as an increase in appetite is common because the caloric requirements of pregnancy significantly increase. The nurse should inquire if the client is experiencing excessive nausea, inhibiting her appetite.

The nurse works collaboratively with a pharmacist to identify clients at risk for lithium toxicity. The client at most significant risk for lithium toxicity is a client with A. asthma taking both long- and short-acting bronchodilators. B. chronic migraine headaches and was newly prescribed naproxen. C. hypertension newly prescribed clonidine transdermal patch. D. hypothyroidism and was recently prescribed levothyroxine.

chronic migraine headaches and was newly prescribed naproxen. Rationale: NSAIDs (naproxen, ibuprofen), ACE inhibitors (lisinopril, enalapril), and diuretics (furosemide, hydrochlorothiazide) should be avoided while a client is taking lithium. ACE inhibitors promote sodium wasting, and low levels of sodium precipitate lithium toxicity. NSAIDs reduce renal blood flow, cause lithium retention and raise its serum level to a potentially toxic range. The client with aches and pains should be recommended acetaminophen.

*NGN* The emergency department nurse is caring for a 27-year-old male client. Item 1 of 1 Nurses' Notes 1500: Client reports an 'excruciating' headache right behind his left eye. He says the pain is 'pulsating' and worse when lying down. He reports sensitivity to lights and sounds. He reports getting these headaches daily for the past two weeks at about the same time. On assessment, the client appears in moderate distress; he is alert, oriented, and has tearing in his left eye. His gait was steady as he paced around the exam room because having him sit down to obtain vital signs increased his pain. Lung sounds are clear; peripheral pulses were 2+. Normoactive bowel sounds. He reports his pain rated 9/10 on the Numerical Rating Scale. The client is demonstrating clinical manifestations consistent with cluster headache tension headache trigeminal neuralgia migraine headache

cluster headache Rationale: This client is experiencing a cluster headache. Cluster headaches have an abrupt onset, are more common in men, and are episodic. The pain for a cluster headache is usually localized behind the client's eye, which causes significant photosensitivity, tearing of the eye, and pulsating pain.

*NGN* The emergency department nurse is caring for a 22-year-old with altered mental status Item 1 of 1 Admission Notes 2330 - 22-year-old male client arrived at the emergency department (ED) with friends who were at a party and was observed snorting a white powder and started acting erratically. The client is hyper-alert, agitated, and only oriented to place on assessment. The client started shouting at staff during the assessment and struck a nurse with his fist. The primary healthcare provider (PHCP) was immediately notified of this incident. Vital Signs Temperature 98.0o F (37o C) Pulse 110/minute Respirations 16/minute Blood Pressure 155/96 mm Hg O2 saturation 96% on room air Complete the following sentences from the list of options Based on the client assessment, the client is likely intoxicated with (heroin, cocaine, or an opioid.) The nurse should immediately (restrain the client, obtain a prescription

cocaine restrain the client physical violence Rationale: The client is most likely intoxicated with cocaine based on the client's hyperarousal, hypertension, tachycardia, and agitation. Cocaine intoxication produces central nervous stimulation, making the client feel paranoid and act erratically. The nurse should restrain the client based on the client's physical violence. The client's physical violence seriously threatens the safety of those around him. The nurse has appropriate cause to restrain the client and then obtain an order from the PHCP within one hour of the initiation of restraints. The client is exhibiting hypertension and tachycardia; however, his physical violence is the priority to address as this creates an immediate safety concern.

The infection control nurse is responding to an outbreak of norovirus in the facility. The nurse should recommend that A. staff wears a surgical mask when providing client care. B. disposable utensils and dishware are used for meals. C. dietary staff wears a face shield when preparing client meals. D. commonly touched surfaces be disinfected with a bleach solution.

commonly touched surfaces be disinfected with a bleach solution. Rationale: Norovirus is a virus that is commonly implicated in gastroenteritis. The transmission of norovirus primarily occurs through surfaces contaminated with norovirus and then touching the mouth or other food items. Further, norovirus may be transmitted by having direct contact with another person who is infected or eating food or drinking liquids that are contaminated with norovirus. The most effective disinfectant for norovirus is bleach. It is also recommended that hand hygiene be performed with soap and water as it is superior to alcohol-based hand rubs. Norovirus requires contact precautions for a minimum of 48 hours after the symptoms resolve.

A nurse is reviewing prescriptions for assigned clients. Which prescriptions require follow-up with the primary healthcare provider? A client with Select all that apply. congestive heart failure prescribed diltiazem. hypertension prescribed clonidine. diabetes insipidus prescribed hydrocortisone. pulmonary emboli prescribed clopidogrel. atrial fibrillation prescribed amiodarone. bacterial cystitis prescribed valacyclovir.

congestive heart failure prescribed diltiazem. diabetes insipidus prescribed hydrocortisone. pulmonary emboli prescribed clopidogrel. bacterial cystitis prescribed valacyclovir. Rationale: A client with congestive heart failure should not be prescribed calcium channel blockers because of their negative inotropic effects, worsening heart failure. Further, hydrocortisone would be indicated to treat adrenal insufficiency, whereas vasopressin would be used for diabetes insipidus. Additionally, clopidogrel is an antiplatelet medication used to prevent stroke, where a client with a pulmonary embolism requires anticoagulants or thrombolytics. Finally, antibiotics such as ceftriaxone are indicated for bacterial cystitis, not antivirals such as valacyclovir.

indwelling urinary catheter education:

daily use of soap and water should be used around the urinary meatus Rationale: recommended for catheter care; clean procedure

*NGN* The nurse is caring for a 29-year-old female in the physician's office Item 2 of 6 Complete the following sentences by choosing from the list of options The client's neurological symptoms may be explained by (rapid gastric motility during meals, insufficient amount of insulin, or decreased levels of cobalamin) due to (reduced production of intrinsic factor, destruction of pancreatic beta cells, or rapid transit of hyperosmolar food material).

decreased levels of cobalamin reduced production of intrinsic factor Rationale: The client is experiencing manifestations associated with Vitamin B12 (cobalamin) deficiency. This is because the client's food bypasses the parietal cells on the stomach lining. The parietal cells secrete intrinsic factor which assists in the absorption of cobalamin. The client's manifestations are consistent with Vitamin B12 deficiency anemia.

The nurse is collecting data on a client with celiac disease. Which of the following findings would be expected? Select all that apply.

dehydration nausea and vomiting abdominal distention skin rash Rationale: In Celiac disease, clients have large amounts of diarrhea that put them at risk for dehydration. IV fluid administration will be a priority for this client. A client with Celiac disease may present with nausea and vomiting if they have not been following a gluten-free diet. Their body will not be able to absorb gluten, and therefore the consumption of it can cause nausea, vomiting, and diarrhea. Abdominal distention is an expected finding in a client with Celiac disease due to gluten intolerance and malabsorption. Their abdomen will not only be distended but uncomfortable and tender to palpation. A skin rash called dermatitis herpetiformis is a common celiac disease symptom characterized by itchy, blistering patches on the skin.

The nurse is collecting data on a client with celiac disease. Which of the following findings would be expected? Select all that apply. dehydration constipation nausea and vomiting abdominal distention skin rash

dehydration nausea and vomiting abdominal distention skin rash Rationale: In Celiac disease, clients have large amounts of diarrhea that put them at risk for dehydration. IV fluid administration will be a priority for this client. A client with Celiac disease may present with nausea and vomiting if they have not been following a gluten-free diet. Their body will not be able to absorb gluten, and therefore the consumption of it can cause nausea, vomiting, and diarrhea. Abdominal distention is an expected finding in a client with Celiac disease due to gluten intolerance and malabsorption. Their abdomen will not only be distended but uncomfortable and tender to palpation. A skin rash called dermatitis herpetiformis is a common celiac disease symptom characterized by itchy, blistering patches on the skin.

The nurse is caring for a client who developed a thyroid storm. The nurse should obtain a prescription for A. enalapril B. regular insulin C. levothyroxine D. dexamethasone

dexamethasone Rationale: A thyroid storm is a medical emergency and is a complication of hyperthyroidism. Manifestations of a thyroid storm include fever, tachycardia, hypertension, and cardiac dysrhythmias. Emergent treatments for a thyroid storm include prescribed dexamethasone (corticosteroids inhibit the peripheral conversion of T4 into T3), propranolol (reduce heart rate and blood pressure), and an antithyroid medication such as propylthiouracil.

The nurse has received an order to prepare a client for a water deprivation test. The nurse understands that this test is used to diagnose A. hyperthyroidism B. pheochromocytoma C. diabetes insipidus (DI) D. syndrome of inappropriate antidiuretic hormone (SIADH)

diabetes insipidus (DI) Rationale: DI can be divided into either neurogenic (central) or nephrogenic. The water deprivation test is used to help differentiate whether the DI is neurogenic or nephrogenic. In this test, the client is deprived of water for up to eight hours (they may still eat dry foods). Serial labs, including plasma and urine osmolality measurements, are obtained during that time. Additionally, the client's urine volume and weight are meticulously measured hourly. If the client's body weight should decrease, this supports the diagnosis of DI. At the end of the eight hours, a dose of desmopressin is administered. If there is an increase in urine osmolarity and a decrease in urine volume, it is considered central/neurogenic DI (because the problem responded to the DDAVP). If no response is observed after the DDAVP is administered, nephrogenic DI is likely.

The nurse in a community-based setting reinforces teach to clients about prostate cancer risk factors. Which of the following risk factors should the nurse share with the group? A. diet high in animal fat B. erectile dysfunction C. human immunodeficiency virus (HIV) D. human papillomavirus (HPV)

diet high in animal fat Rationale: A diet high in animal fat, especially red meat, is a strong risk factor for prostate cancer. Strong risk factors for prostate cancer include - African American ethnicity Increasing age, especially after age 40

The nurse is caring for a client diagnosed with atrial fibrillation. The nurse should anticipate a prescription for which of the following medications? Select all that apply.

diltiazem warfarin Rationale: Diltiazem is a rate-lowering calcium channel blocker used to manage atrial fibrillation. This medication assists in maintaining rate control. While not always indicated, an anticoagulant such as warfarin or rivaroxaban is used to manage atrial fibrillation, as this arrhythmia puts the patient at high risk for a stroke.

The nurse is educating a client about the newly prescribed oxymetazoline nasal spray. It would be appropriate for the nurse to instruct the client to A. sit upright for thirty minutes after taking this medication. B. do not use this medication for more than three days. C. change positions slowly while taking this medication. D. rinse your mouth out after taking this medication.

do not use this medication for more than three days. Rationale: Oxymetazoline is a nasal spray used to decrease congestion. This medication is administered intranasally and constricts blood vessels in the nasal passage. This medication should not be used for more than three days to prevent rebound congestion.

The license practical/vocational nurse (LPN/VN) is reinforcing teaching to a client prescribed iron supplementation. The nurse should instruct the client to Select all that apply. take the iron supplement with a meal rich in calcium. drink a glass of orange juice with your iron supplement. report any black stools to the physician. drink iron suspension with a straw. increase dietary fiber intake because of the potential for constipation.

drink a glass of orange juice with your iron supplement. drink iron suspension with a straw. increase dietary fiber intake because of the potential for constipation. Rationale: Orange juice is high in vitamin C, which will help increase iron absorption. Taking iron with vitamin C enhances its absorption. If the healthcare provider orders an oral suspension iron supplementation, the client should be instructed to drink it through a straw to avoid staining their teeth. Iron supplements commonly cause constipation, and increasing dietary fiber intake can help prevent or alleviate this side effect. Educating clients about the importance of a high-fiber diet is valid.

The nurse is assessing a male client taking prescribed risperidone. Which of the following findings would indicate the client is having an adverse effect? A. ptosis B. gingival hyperplasia C. polycythemia D. gynecomastia

gynecomastia Rationale: Risperidone is an atypical (second-generation) antipsychotic indicated in treating disorders such as schizophrenia, autism with behavioral disturbances, delusional disorder, and bipolar disorder. Risperidone is notorious for causing increased prolactin levels. This increase in prolactin levels may cause a client to develop gynecomastia and/or galactorrhea.

*NGN The clinic nurse is caring for a 38-year-old male Item 5 of 6 Physician Orders 12-lead electrocardiogram (ECG) atenolol 25 mg by mouth daily pitavastatin 1 mg by mouth daily fenofibrate 45 mg by mouth daily with food The nurse receives, reviews, and implements the physician orders Complete the sentences below by choosing from the list of options The nurse needs to obtain baseline (electrocardiogram, cardiac enzymes, or liver function tests) prior to administering atenolol because atenolol would be contraindicated if the client (had low liver enzymes, atrioventricular block or had an elevated troponin.) The nurse understands that the prescribed fenofibrate is intended to (lower blood pressure, reduce triglycerides, or decrease high density lipoproteins.) Prior to administering the prescribed pitavastatin, the nurse needs to obtain baseline (urine analysis, liver function tests, or c-reactive protein.)

electrocardiogram atrioventricular block reduce triglycerides liver function tests Rationale: Beta-blockers are contraindicated in atrioventricular blocks (AV blocks). An AV block is detected by obtaining a 12-lead electrocardiogram; thus, obtaining the ECG is essential before the client is discharged with the prescription. AV blocks occur when there is partial or complete interruption of impulse transmission from the atria to the ventricles. They are categorized in various degrees. If a client had an AV block and received a beta blocker, it could worsen the blockade. The client's triglycerides are dangerously elevated, and out of all the concerning findings, the high triglycerides put the client at serious risk for a vascular event (stroke or acute coronary syndrome). Fenofibrate is an adjunctive medication used to target high triglyceride levels with the intent to reduce them to a therapeutic level. Both fenofibrate and pitavastatin require the client to have a baseline liver function test. These medications may be hepatotoxic; thus, a baseline level and consistent monitoring are strongly recommended to prevent and promptly detect liver injury.

The nurse is preparing to measure the fundal height of a client at 16 gestational weeks. The nurse should prepare the client for this assessment by instructing the client to A. lay in a side-lying position with the knees bent. B. prepare for the insertion of an intravenous (IV) catheter. C. not to eat or drink two hours after this assessment. D. empty their bladder.

empty their bladder. Rationale: Measuring the fundal height is a painless and noninvasive way to evaluate fetal growth patterns and confirm gestational age. For this assessment, the client should empty their bladder to prevent elevation of the uterus.

The nurse is caring for a client who was prescribed lamotrigine. The nurse understands that this medication is intended to treat A. acute spinal shock. B. epilepsy. C. Parkinson's disease. D. multiple sclerosis.

epilepsy. Rationale: Lamotrigine is approved to treat epilepsy to prevent seizures. This medication is also indicated in maintaining bipolar disorder to provide mood stabilization. Lamotrigine is safe during pregnancy which makes this medication quite attractive for a client with bipolar disorder and epilepsy who wants to consider family planning.

The nurse is caring for a client requesting to leave against medical advice. The nurse barricades the client in their room because they feel that the client is not safe to go home. The nurse is demonstrating A. false imprisonment. B. malpractice. C. negligence. D. invasion of privacy.

false imprisonment. Rationale: Refusing to let a client leave against medical advice (AMA) and physically obstructing the client from leaving is a form of false imprisonment.

The nurse is reviewing a new prescription for amphotericin b. The nurse understands that this medication treats A. autoimmune infections. B. fungal infections. C. viral infections. D. bacterial infections.

fungal infections. Rationale: Amphotericin B is a powerful antifungal indicated in treating systemic fungal infections. This medication requires pre-medication with isotonic saline, diphenhydramine, and acetaminophen to help decrease the symptoms of fever, chills, and rigors associated with the infusion.

*NGN* The nurse is removing an indwelling urinary catheter Item 1 of 1 Nurses Note The removal of the client's indwelling urinary catheter was attempted. Perineal hygiene was performed before the removal. The urine collection bag was emptied with 450 mL of clear, straw-colored urine. 2 mL of fluid was removed during the deflation of the balloon. Resistance was felt when the tubing was removed, and the client reported discomfort. Drag words from the choices below to fill the blank in the following sentence Prior to attempting to remove the catheter again, the nurse should: 1. place a warm compress over the perineum 2. further deflate the catheter 3. cut the balloon inflation valve 4. position the client at 45 degrees

further deflate the catheter Rationale: The amount of fluid removed from the balloon (this secures the catheter in place inside of the bladder) was inadequate. 10 mL of fluid is typically used to inflate the catheter balloon to keep it secure inside the bladder. The nurse should further deflate the catheter balloon by passively allowing the fluid to fill the syringe. The nurse may gently pull back on the syringe plunger if this does not work. By removing the residual volume, the nurse should then remove the catheter.

*NGN* The clinic nurse is caring for a 38-year-old male Item 4 of 6 Physician Progress Notes 1355 - One-week follow-up appointment with a 38-year-old male. Labs show serious metabolic derangements, including high fasting blood glucose and hyperlipidemia. VS shows essential hypertension. Diagnosis - 1. essential hypertension 2. metabolic syndrome. Will perform additional diagnostics and follow up with the client in six weeks. The nurse reviews the physician's progress notes and formulates a care plan For each potential order, click to specify whether the potential order is indicated or not indicated for the client Note: Each row must have 1 response option selected glycated hemoglobin (a1c) test (Indicated or Not Indicated) antihypertensive medication (Indicated or Not Indicated) consultation with occupational therapy (Indicated or Not Indicated) consultation with dietician/nutritionist (Indicated or Not Indi

glycated hemoglobin (a1c) test (Indicated) antihypertensive medication (Indicated) consultation with occupational therapy (Not Indicated) consultation with dietician/nutritionist (Indicated) daily weights (Not Indicated) 12-lead electrocardiogram (ECG)(Indicated) serum cardiac enzymes(Not Indicated) Rationale: The physician has diagnosed the client with metabolic syndrome and essential hypertension. The client's fasting blood glucose level was elevated, and further testing is necessary with a glycated hemoglobin (a1c) test to either confirm or exclude diabetes mellitus Treatment for essential hypertension is a combination of lifestyle modifications and antihypertensives. The client has met the threshold for diagnosis through two visits showing hypertension and self-measurement with systolic blood pressure in the 150s. The client should confer with a dietician or nutritionist to correct the nutritional deficiencies contributing to the client's weight gain and increased blood pressure. Specifically, the client should be counseled on restricting sodium-rich and high-glycemic foods. A 12-lead electrocardiogram (ECG) is indicated because an ECG can determine any hypertension-related cardiac involvement. Left atrial and ventricular hypertrophy is the first ECG sign of heart disease resulting from hypertension. Left ventricular remodeling can be detected on the 12-lead ECG.

The nurse is caring for a client who sustained a stroke impacting the occipital lobe. Which of the following findings would support this diagnosis? A. homonymous hemianopia B. impaired proprioception C. expressive aphasia D. impulsivity

homonymous hemianopia Rationale: Visual disturbances are expected for a client with a stroke impacting the occipital lobe of the brain. The occipital lobe is the primary optical center of the brain. Homonymous hemianopia is a complete left or right visual field defect. The client may need to be taught to scan the room, and the nurse should place objects in the unaffected visual field.

The licensed practical/vocational nurse (LPN/VN) reviews newly prescribed medications for a client taking prescribed lithium. Which medication requires further follow-up by the LPN/VN? A. venlafaxine B. hydrochlorothiazide C. gabapentin D. losartan

hydrochlorothiazide Rationale: A client taking lithium should be instructed to avoid dehydration and hyponatremia. Lithium is a salt; when the client has decreased fluid volume, the drug accumulates and raises the lithium level. HCTZ is a thiazide diuretic and is contraindicated for a client taking lithium because of its ability to decrease fluid and sodium levels.

loop diuretics may cause:

hypokalemia hypocalcemia

The licensed practical vocational nurse (LPN/VN) assists a quality improvement nurse in planning initiatives to reduce fall risk factors in the acute care environment. Which of the following risk factors should the nurse recommend be addressed? Select all that apply.

inadequate client assessment communication failures dim lighting Rationale: The nurse should recommend addressing inadequate assessment because this is a significant risk factor for falls that may lead to client injury. The nurse can recommend a standardized fall risk assessment tool and monitor its execution. Communication failures between staff are a significant contributor to falls. The nurse should recommend addressing this by standardizing the handoff report and placing signals outside a client's room (a particular light, fall risk bands, or pictures outside of the client's room). The nurse can plausibly recommend the repair of dim environmental lighting, which is a risk factor for falls, especially when the client is ambulating within their room.

The nurse in the mental health unit is collecting data on a client with moderate anxiety. The nurse would anticipate which signs and symptoms to support this finding? Select all that apply.

increased pulse reports of headache narrowing of the perceptual field Rationale: Moderate anxiety is characterized by a client experiencing: Narrowing of the perceptual field Slightly scattered thought process The client can problem-solve and learn, although not at an optimal level Somatic symptoms such as headache, urinary urgency, and muscle tension Sympathetic symptoms such as an increased pulse, respiratory rate, palpitations, voice tremors, and shaking

The nurse is caring for a client with human immunodeficiency virus (HIV). It would be appropriate for the nurse to assign the client to a room with the client diagnosed with A. infectious mononucleosis. B. mycoplasma pneumonia. C. gastroenteritis (rotavirus). D. mumps (infectious parotitis).

infectious mononucleosis. Rationale: Infectious mononucleosis (IM) requires standard precautions. Disease transmission is spread by prolonged exposure to human saliva and is difficult to spread. Often IM is referred to as the 'kissing disease' because prolonged kissing may transmit this pathogen. It is appropriate to place a client with HIV in the same room as a client with IM. HIV requires standard precautions.

The licensed practical/vocational nurse (LPN/VN) is caring for a client with a tracheostomy. Which of the following items is essential to have at the bedside? A. air humidifier B. inner cannula C. nasal cannula oxygen D. tracheostomy brush

inner cannula Rationale: An inner cannula of the tracheostomy size and one smaller is necessary to keep at the bedside. This is essential in case the inner cannula becomes dislodged.

*NGN* The nurse is caring for a 29-year-old female in the physician's office Item 5 of 6 Physician's Orders Cyanocobalamin 1000 mcg intramuscular (IM) x 1 dose The nurse reviews the physician's orders Complete the sentences below from the list of options The nurse locates the client's vastus lateralis by injecting (into the middle third of the anterolateral aspect of the thigh, three finger widths below the acromion process, or just below the iliac crest on the side of the thigh.) and inject at (45 degrees, 90 degrees, or 15 degrees.)

into the middle third of the anterolateral aspect of the thigh 90 degrees Rationale: The appropriate anatomical landmark to administer an intramuscular (IM) injection into the client's vastus lateralis is locating the thigh's anterolateral aspect. It extends in an adult from a handbreadth above the knee to a handbreadth below the greater trochanter of the femur. The nurse should use the middle third for the injection. To help relax the muscle, ask the client to lie flat with the knee slightly flexed and foot externally rotated or to assume a sitting position. IM injections are administered at 90 degrees.

The nurse is caring for a child experiencing anaphylactic shock. The nurse should prioritize administering A. nasal cannula oxygen. B. intramuscular (IM) epinephrine. C. intravenous (IV) dexamethasone. D. intravenous (IV) diphenhydramine.

intramuscular (IM) epinephrine. Rationale: Anaphylactic shock is a life-threatening allergic reaction that can lead to rapid and severe airway constriction, hypotension, and other systemic symptoms. The first-line treatment of anaphylactic shock is IM epinephrine. The administration of this medication should not be delayed because the cascade of the shock can be alleviated by antagonizing histamine, which is in excess during anaphylactic shock. Epinephrine is preferred over diphenhydramine because epinephrine targets the histamine receptors in the airway.

phenylkentonuria complications:

irritability nausea and vomiting seizures

The nurse is caring for a group of assigned clients. The nurse should immediately follow up on the client who A. has a closed-chest drainage system and has redness at the insertion site. B. is receiving treatment for ulcerative colitis and has had three bloody stools in the past hour. C. is being treated for a concussion and reports a headache rated as 4 on a scale of 0 (no pain) to 10 (severe pain). D. is being treated for an ischemic stroke and has a blood pressure of 100/58 mm Hg.

is being treated for an ischemic stroke and has a blood pressure of 100/58 mm Hg. Rationale: The client with an ischemic stroke will require intense blood pressure monitoring because a low blood pressure will decrease cerebral perfusion, which is necessary for the unaffected areas of the brain. An optimal blood pressure for an ischemic stroke is 150/100 mm Hg to ensure cerebral perfusion. Allowing the blood pressure to be this high is considered permissive hypertension. Blood pressure lower than 150/100 mm Hg may cause further injury because of decreased cerebral perfusion. Likewise, the blood pressure should not exceed 185/110 mm Hg in an ischemic stroke because this may cause an extension of the stroke.

The nurse is caring for a group of children on the medical-surgical unit. The nurse should initially follow up on the child who A. is receiving treatment for Hirschsprung's disease and has a temperature of 101°F (38.3°C). B. has an indwelling urinary catheter and reports burning at the insertion site. C. has scant blood in their newly established ostomy pouch. D. has friends writing words on their fiberglass cast with different colored markers.

is receiving treatment for Hirschsprung's disease and has a temperature of 101°F (38.3°C). Rationale: A major complication of Hirschsprung's disease is the development of enterocolitis manifested by fever, abdominal distention, vomiting, and increased abdominal pain. Emergent intervention is necessary because the child may develop sepsis leading to septic shock.

best explains cystoc fibrosis:

it is an inherited disease causing excessive, thick mucus to build up in the body and causes blockages

patient with liver cirrhosis -assessment findings require follow up:

labored breathing respiratory rate (tachypnea) oral temp

The nurse is caring for a client receiving prescribed atomoxetine for attention deficit hyperactivity disorder (ADHD). Which clinical data should the nurse monitor while the client receives this medication? A. liver function tests B. complete blood count C. urine analysis D. fasting blood glucose

liver function tests Rationale: Atomoxetine is a non-stimulant treatment option for ADHD. This medication may adversely cause hepatic injury, and the nurse should monitor the client's liver function tests as they receive this medication. Atomoxetine is an attractive option for ADHD as it may not cause the nervousness, weight loss, and tics associated with stimulants.

*NGN* The nurse is caring for a 21-year-old in the emergency department Item 1 of 1 Nurses' Notes 1450 - Client was brought to the ED by his parents, where he is agitated and pacing within his room. His parents report that he has not taken his prescribed valproic acid for almost a week and has not eaten any food for the past two days. The client is disoriented on the exam and only states, 'he is the chosen one.' The client's speech is fast-paced and tangential. Physician Orders Admit the client to psychiatry 1 mg lorazepam intramuscular (IM) x one dose Initiate intravenous access Dextrose 5% in water (D5W) at 80 mL/hr Obtain valproic acid level Valproic acid 500 mg intravenous piggy-back (IVPB) For each prescribed medication below, click to specify the appropriate nursing action lorazepam (monitor liver function tests and complete blood count, monitor the client for hyperglycemia, or initiate fall precautions su

lorazepam (initiate fall precautions such as raising side rails) valproic acid (monitor liver function tests and complete blood count) Dextrose 5% in water (D5W) (monitor the client for hyperglycemia) Rationale: The client is experiencing acute mania because of not taking his prescribed mood stabilizer, valproic acid. The nurse should administer the lorazepam because it is a benzodiazepine and will have a sedative effect which may cause the client to fall. Thus, after administering lorazepam, the nurse should initiate fall precautions. The valproic acid administered intravenously will assist in breaking the mania. Valproic acid may cause hepatotoxicity and blood dyscrasias such as thrombocytopenia. These are essential monitoring parameters for the nurse. Dextrose 5% in water was prescribed for the client's inability to eat, a common feature of mania. Thus, the nurse should initiate the infusion of the Dextrose 5% in water as it contains some calories. D5W may raise the blood glucose, and if this fluid is used intermediate to long-term, the blood sugar should be monitored for hyperglycemia.

The nurse is reviewing the following prescriptions for assigned clients. The nurse should initially administer A. enoxaparin to a patient with a platelet count of 165,000 mm3. B. warfarin to a client with an international normalized ratio of 2.4 seconds. C. lorazepam to a client who had two seizures within the last hour. D. regular insulin to a client with a blood glucose of 285 mg/dl.

lorazepam to a client who had two seizures within the last hour. Rationale: For a client who had two seizures in the last hour, the likelihood of them having another seizure is high. The nurse should prioritize administering lorazepam to this client as this medication will have an inhibitory effect on the brain, therefore, decreasing seizure activity.

The nurse has just inserted an indwelling urinary catheter for a male client. The nurse plans on securing the catheter to the client's A. inner thigh. B. lower abdomen. C. outer thigh. D. medial thigh.

lower abdomen. Rationale: When securing an indwelling urinary catheter for a male, it is appropriate to anchor it to the lower abdomen or upper thigh. The catheter tubing should be secured to the lower abdomen or the upper thigh to prevent urethral injury.

*NGN* The nurse is caring for a 29-year-old female in the physician's office Item 3 of 6 Complete the sentence by choosing from the list of options The client is at highest risk for developing (pancreatitis, dumping syndrome, or pernicious anemia).

pernicious anemia Rationale: The client is at highest risk for developing pernicious anemia because of the reduction of intrinsic factor. Instric factor is essential in the absorption of cobalamin. The client's neurological manifestations and physical examination findings of the oral ulcers are consistent with classic pernicious anemia.

postmortem care preparation for viewing of the family:

place the client supine w/ a pillow under the head Rationale: to prevent the head from being discolored; elevate the head of the bed

Student nurse; client with left-sided weakness performing ADLs - action that requires intervention:

places the wheelchair as close to the bed as possible on the client's affected (weaker) side.

*NGN* The clinic nurse is caring for a 38-year-old male Item 3 of 6 Diagnostic Result Basic Metabolic Panel (fasting) Serum calcium 9.6 mg/dL (2.39 mmol/l) [9.0-10.5 mg/dL (2.24 - 2.61 mmol/l) Serum chloride 94 mEq/l (94 mmol/l) 90 to 110 mEq/L (90 -110 mmol/l) Serum glucose 126 mg/dL (6.99 mmol/l) 70-110 mg/dL (3.88 - 6.10 mmol/l) Serum potassium 4.2 mEq/l (4.2 mmol/l) 3.5 to 5.0 mEq/l (3.5 to 5.0 mmol/l) Serum sodium 136 mEq/l (136 mmol/l) 135 to 145 mEq/l (135 to 145 mmol/l) Serum creatinine 0.9 mg/dl (79.5 mcmol/l) 0.6 to 1.1 mg/dl (53 to 106 mcmol/l) Blood urea nitrogen 19 mg/dl (6.78 mmol/l) 10 to 20 mg/dl (3.57 to 7.14 mmol/l) Lipid Panel (fasting) Total cholesterol 235 mg/dl (6.07 mmol/l) [less than 200 mg/dl (<5.18 mmol/l)] High-density lipoprotein (HDL) 35 mg/dl (0.91 mmol/l) [more than 45 mm/dL (>0.75 mmol/L) for men; more than 55 mg/dL (>0.91 mmol/L) for women] Low-density lipoprotein (LDL) 135 mg/dl (

metabolic syndrome Rationale: This client has elevated blood pressure, high fasting blood glucose, abdominal obesity, and hyperlipidemia, which all conclude the client has metabolic syndrome. Metabolic syndrome is a collection of health impairments The clinical criteria for metabolic syndrome may vary, but in general, the following criteria are - Hypertension; decreased HDL-C; increased LDL-C; increased level of triglycerides; increased fasting blood glucose; abdominal obesity

The nurse is observing a client ambulate with crutches using the three-point gait. Which observation requires follow-up by the nurse? The client A. places the crutches 15 cm (6 inches) in front of and 15 cm (6 inches) to the side of each foot prior to walking. B. advances both crutches and the injured leg forward and then moves the non-injured leg. C. has the elbows flexed 30 degrees with the hands and arms supporting the body weight. D. moves a crutch at the same time as the opposing leg.

moves a crutch at the same time as the opposing leg. Rationale: This technique does not reflect the three-point gait. This gait pattern depicts the two-point gait, which requires at least partial weight bearing of both lower extremities. In the two-point gait, the left crutch and right leg move forward, followed by the right crutch and left leg.

The nurse is caring for a client who is receiving prescribed pregabalin. The client is experiencing the intended effect when they report less A. neuropathic pain. B. cravings for cigarettes. C. binge eating. D. depressive symptoms.

neuropathic pain. Rationale: Pregabalin is indicated in treating neuropathic pain, certain anxiety disorders, and focal seizures. This controlled substance is five times more potent compared to gabapentin.

A nurse is working in a busy medical-surgical unit and has received report on the following four clients. The nurse should first see the client with A. A client who underwent surgery yesterday and is complaining of incisional pain at a level of 8/10 B. new-onset atrial fibrillation who is complaining of chest pain. C. diabetes who has a foot wound that appears infected and an oral temperature of 100.4° F (38 ° C) D. advanced Alzheimer's disease who is confused and has had a recent fall.

new-onset atrial fibrillation who is complaining of chest pain. Rationale: New-onset atrial fibrillation and complaint of chest pain indicate a potentially serious cardiac issue that requires immediate attention. Chest pain can be a symptom of myocardial infarction or unstable angina, which requires prompt assessment and intervention to help prevent further complications. Therefore, this client should be the nurse's first priority.

The nurse supervises a new graduate care for a child immediately following surgery to correct an imperforate anus (anorectal malformation). Which action by the new graduate requires follow-up? The new graduate A. positions the child side-lying prone position with the hips elevated. B. obtains a rectal temperature. C. auscultates bowel sounds in all four abdominal quadrants. D. secures the indwelling urinary catheter bag to the frame of the bed.

obtains a rectal temperature. Rationale: Immediately following surgical repair of the imperforate anus, the surgical site is very fragile, and obtaining a rectal temperature may cause significant trauma to the suture line. This method of temperature is not recommended.

anticholinergic appropriate for urinary bladder urgency and incontinence:

oxybutynin Rationale: works as a bladder relaxant

The nurse is collecting data on a child admitted with varicella zoster (chickenpox). Which of the following assessment findings would be expected? A. linear cracks in the epidermis that extend into the dermis B. painful blisters filled with clear fluid C. nodules filled with either liquid or semisolid material D. elevated, plateau-like patches

painful blisters filled with clear fluid Rationale: Varicella-zoster (chickenpox) virus (VZV) is a condition that causes the individual to experience low-grade fever, malaise, and anorexia. Characteristically, the client with VZV develops a papule type of rash that transitions into vesicles filled with clear fluid. Once these vesicles rupture, it causes shallow erosions that start to heal. These vesicles are commonly painful and cause the client intense itching.

The nurse is collecting data on a client who has suspected Raynaud phenomenon/disease. Which of the following findings would support a diagnosis of Raynaud phenomenon/disease? A. unilateral swelling of the leg B. painful vasospasms C. crepitus of the joints D. claudication in feet and lower extremities

painful vasospasms Rationale: Raynaud phenomena is a condition causing painful vasospasms in response to emotional stress and cold temperatures. These painful vasospasms occur in the digits. Raynaud phenomena may be associated with autoimmune conditions such as systemic lupus erythematosus (SLE) and scleroderma.

The emergency department nurse is caring for a child who swallowed a toy. The child has trouble breathing but is conscious and not making any sounds. The nurse should immediately A. instruct the client to cough to clear the airway. B. apply a bag valve mask. C. perform abdominal thrusts. D. perform a blind finger sweep.

perform abdominal thrusts. Rationale: Performing abdominal thrusts should be the first action to relieve the obstruction. This action is essential because the child has trouble breathing and is not making any noise after swallowing a toy, which indicates choking. Since this client has an upper airway obstruction, performing a blind finger sweep is not recommended as it may cause the object to propel further down the airway. The nurse should only attempt a finger sweep if the object is visible.

The charge nurse must transfer a client from the maternity unit to the medical-surgical unit to make a bed available. It would be most appropriate for the nurse to transfer the client who is A. experiencing preterm contractions and requires continuous monitoring for potential preterm labor. B. postpartum following a cesarean section and needs close observation for surgical wound healing and pain management. C. admitted for pre-eclampsia and is experiencing a frontal headache. D. in her second trimester, undergoing evaluation for suspected gestational diabetes and requires a nonstress test.

postpartum following a cesarean section and needs close observation for surgical wound healing and pain management. Rationale: The most appropriate client to transfer from the maternity unit to the medical-surgical unit to make a bed available is the client who is postpartum after delivering her baby via cesarean section and needs close observation for surgical wound healing and pain management.

The licensed practical/vocational nurse (LPN/VN) is caring for a client receiving digoxin. When examining the client's laboratory data, the nurse should closely review the client's serum A. potassium. B. calcium. C. sodium. D. phosphorus.

potassium. Rationale: The nurse needs to monitor the client's potassium levels while taking digoxin. Decreased potassium levels (hypokalemia) may precipitate a client developing digoxin toxicity.

following clients require most rapid action to protect other clients in the ED from infection:

preganant woman with a blister-like rash on the face that is possibly having varicella

The nurse is caring for a seven-year-old client brought to the clinic by her parents Item 5 of 6 Orders Acetaminophen 325 mg PO x 1 dose Diphenhydramine 25 mg PO x 1 dose Discharge client home The nurse receives orders and prescriptions from the primary healthcare provider Complete the following sentence by choosing from the list of options The nurse understands that the acetaminophen is prescribed to treat the client's (pruritus, pyrexia, or skin lesions.) Additionally, the diphenhydramine has been prescribed to (decrease symptoms of pruritis, decrease the pyrexia, or prevent viral replication.)

pyrexia decrease symptoms of pruritus Rationale: Pyrexia is a common feature of varicella infections. The pyrexia (fever) can be mitigated with acetaminophen. Diphenhydramine is an antihistamine and can be used to mitigate symptoms of pruritus. This medication is intended to decrease the symptoms of pruritus (itching) and promote comfort.

The nurse is observing a student nurse feed a client requiring aspiration precautions. The nurse should intervene if the student A. asks the client to remain sitting upright for at least 30 to 60 minutes after a meal. B. reminds the client to tilt their head backward when eating and drinking. C. avoids mixing foods of different textures in the same mouthful. D. places salt and pepper on the client's food at their request.

reminds the client to tilt their head backward when eating and drinking. Rationale: This action by the student nurse requires intervention. The client should assume the chin-down position and be reminded not to tilt backward when eating or drinking. The client should be instructed to have their head turned and chin tucked to reduce the risk of aspiration. Reminding the client to tilt their head backward would increase their risk for aspiration.

requires droplet precautions:

rubella pharyngeal diphtheria

The nurse is observing infection control practices in the nursing unit. Which of the following findings require follow-up? A client with Select all that apply. H. pylori, placed on standard precautions. rotavirus provided a disposable blood pressure cuff. rubella and their door is kept closed. influenza ambulating in the hall with a surgical mask. Legionnaires' disease placed on contact precautions.

rubella and their door is kept closed. Legionnaires' disease placed on contact precautions. Rationale: These observations are inappropriate and require follow-up. The door should be closed in airborne isolation precautions, not droplet precautions. A client with rubella should be placed on droplet precautions. The minimum PPE required for droplet precautions is a surgical mask. Legionnaires' disease is not transmitted person-to-person but rather through infected water or soil. This bacterium requires standard precautions.

*NGN* The nurse is caring for a 23-year-old male in the psychiatry clinic Item 1 of 1 History And Physical Chief Complaint - 23-year-old male presents with his mother, who insists, 'he needs some help; all he does is work and play video games and doesn't socialize with anyone.' History of Present Illness - 23-year-old Caucasian male presents with his mother with reports of his asociality starting to impact his life. He reports that while in high school, he had a degree of anxiety about socializing with his peers. He thought that as the years passed, it would get better. He states his anxiety has declined, but he gets paranoid around individuals because they may want to 'do him wrong.' He cannot point to an example of maleficence caused by his friends. He states he doesn't have a problem with his self-esteem, but sometimes social situations are avoided because 'I can see ahead into the future, and I want to avoid

schizotypal personality disorder illogical thought content Rationale: The client is exhibiting classic manifestations of schizotypal personality disorder (SPD). SPD is characterized by Social avoidance Odd and eccentric behavior Magical thinking (illusions, ideas of reference) Eccentric attire Some paranoia Appropriate self-esteem (it is usually not impaired) Findings supporting the risk for SPD for this client include: the client's thought content is illogical and explains his social avoidance with magical reasoning. The client's reported ability to see into the future and use tarot cards is a classic finding, as some individuals with this condition may describe themselves as clairvoyant or psychic. Finally, the client reported an idea of reference regarding the quotes or lyrics he sees on social media as he states it pertains to him. The client's hair is bright green, and eccentric appearance/attire is a feature of this condition. The client's self-esteem was without impairment and is a key differential in avoidant personality disorder, where the client has feelings of negative self-worth. The client's job is also a supporting factor for SPD, as it is solitary and found in schizoid and schizotypal personality disorders. The preference to be alone is shared, but the magical and atypical thinking is exclusive to an individual with SPD.

The nurse receives a prescription for sevelamer. The nurse plans on administering this medication A. with the client's meals. B. immediately before hemodialysis. C. with a prescribed proton pump inhibitor (PPI). D. right before the client goes to bed.

with the client's meals. Rationale: Sevelamer is a phosphate binder indicated in the treatment of hyperphosphatemia associated with chronic kidney disease. This medication is purported to decrease serum phosphorus levels by binding to food. Thus, this medication is given with meals. Combined with a low phosphorus diet, the goal of this medication is to decrease serum phosphate levels.

*NGN* The nurse cares for a client six hours post-operative following a transurethral resection of a prostate (TURP) Item 1 of 1 Nurses' Notes The client was alert, oriented to person, place, time, and situation. The client has a three-way indwelling urinary catheter and is continuously irrigated with isotonic saline. Urine output is ketchup-like with medium to large clots. The client reports the need to urinate and reported pressure in the pelvic region described as spasms. Intake And Output Intake - Continuous bladder irrigation: 550 mL Output - Indwelling catheter: 975 mL Vital Signs Blood Pressure 100/60 mm Hg Temperature 98° F (36.7° C) Heart rate 106/min Respiratory rate 19 breaths per minute Oxygen saturation 95% on room air Drag words from the choices below to fill in the blank in the following sentence The client is demonstrating signs and symptoms of: 1. urinary catheter obstruction 2. hyponatremi

shock Rationale: Following a TURP procedure, clients should be monitored quite closely for hypovolemic shock, likely to occur within the first twenty-four hours post-operatively. An early manifestation of shock is tachycardia and restlessness. The client's vital signs show tachycardia and low blood pressure (not hypotensive, yet). The ketchup-like output supports the shock finding with medium to large clots. This type of output is concerning.

The nurse is caring for a client with Buerger's disease. The nurse plans on suggesting that the client receive a referral for A. occupational therapy. B. speech therapy. C. smoking cessation. D. group psychotherapy.

smoking cessation. Rationale: Arterial and venous blood flow impediments characterize Buerger's disease. This impediment is caused by inflammation and is significantly worsened by smoking. The nicotine causes vasoconstriction and worsens blood flow. A critical intervention for a client with this condition is discussing smoking cessation with this client.

The nurse cares for a 25-year-old female in the outpatient clinic Item 1 of 1 Nurses' Note 1130: 25-year-old female reporting to the clinic for a regularly scheduled follow-up. She is being treated for bipolar disorder with lamotrigine and valproic acid for bipolar I disorder. The client indicates that she 'doesn't feel better.' Over the past few weeks, she stated that 'she feels high, but yet feels low.' On assessment, the client is alert and oriented and has a constricted affect. She described her mood as 'whatever'. She denies suicidal ideations and hallucinations. She reports her sleep has been 'terrible' and that she 'cannot turn her mind off when she lays down.' Physician Orders Discontinue valproic acid Discontinue lamotrigine Start lithium 900 mg PO twice a day Follow-up visit in 3 weeks The nurse reviews the physician's orders and reinforces teaching regarding the prescribed lithium Complete the sentenc

sodium rich maintain a daily fluid intake of 2 to 3 liters a fine hand tremor thyroid panel Rationale: Lithium is a salt used in the treatment of bipolar disorder, and the client should be instructed to eat salty foods while maintaining a consistent fluid intake of between two to three liters a day. Dehydration and hyponatremia may precipitate lithium toxicity and should be avoided. Fine hand tremor is a common finding early in lithium therapy and typically resolves within one to two weeks. Hair loss is commonly seen with lithium along with weight gain because the sodium leads to fluid retention. Laboratory testing required during lithium therapy includes the thyroid panel because of the risk of hypothyroidism. The creatinine needs to be monitored because of the risk of diabetes insipidus and acute kidney injury. Finally, the lithium levels will need to be monitored.

cystic fibrosis s/sx:

steatorrhea meconium ileus salty sweat adventtious breath sounds

The nurse is triaging phone calls at a clinic. The nurse should initially follow-up with the client who reports A. decreased libido while receiving prescribed dutasteride. B. swelling in their right leg while receiving prescribed tamoxifen. C. hot flashes and night sweats while receiving prescribed letrozole. D. bone pain while receiving prescribed filgrastim.

swelling in their right leg while receiving prescribed tamoxifen. Rationale: Tamoxifen is a hormone antagonist. Specifically, it is an estrogen antagonist that prevents hormone receptor-positive breast cancer. For an unclear reason, Tamoxifen increases the likelihood of adverse clotting events such as deep vein thrombosis (DVT) and venous thromboembolism or pulmonary embolism (PE). If DVT is not quickly diagnosed and treated, the client may develop a PE that can be life-threatening. A client taking Tamoxifen who experiences manifestations of DVT (leg pain, edema) or PE (dyspnea, chest pain) should promptly report these symptoms.

The nurse is collecting data on a client experiencing serotonin syndrome. Which of the following findings would be expected? Select all that apply. tachycardia altered mental status somnolence anuria low blood pressure

tachycardia altered mental status Rationale: Tachycardia, altered mental status, increased blood pressure, altered mental status, hypertonia, and fever are manifestations of serotonin syndrome. Serotonin syndrome (or toxicity) is commonly caused by two serotonergic agents causing very high serotonin levels, which may lead to death in rare situations. The causes of serotonin syndrome primarily stem from the client being exposed to excessive serotonergic agents, including tramadol, SSRIs (citalopram, paroxetine, etc.), and MAOIs.

marked confidential d/t safety concerns - inappropriate action:

tell the client's mother he is okay when she calls to ask if he is still on the unit Rationale: no medical staff can give out any info which will violate confidentiality

The nurse is caring for the following assigned clients. The nurse should initially follow-up with the client who A. is repeatedly washing their hands. B. talking over others during group therapy. C. yelling and shouting at others. D. is voluntarily admitted and requesting discharge.

yelling and shouting at others. Rationale: The client yelling and shouting at other clients requires immediate intervention because this situation is hostile and warrants the nurse to de-escalate the situation before it intensifies.

The licensed practical/vocational nurse (LPN/VN) is participating in a quality improvement project to reduce urinary tract infections (UTIs) for older adult clients in long-term care. It would be appropriate for the nurse to recommend Select all that apply. the addition of more liquids to meal trays. standardizing the dosing times of prescribed diuretics. audio reminders for turning bed-bound clients. daily bathing using bath basins. a staff in-service on hand hygiene.

the addition of more liquids to meal trays. a staff in-service on hand hygiene. Rationale: A significant intervention the nurse can implement to reduce UTIs in older adults is offering more liquids. The older adult has a reduced thirst reflex, and this reduced reflex may cause the client to experience a fluid volume deficit, which is a major precipitating factor in the development of a UTI. Offering more fluids during mealtime and in-between meals is effective in mitigating UTIs. Hand hygiene is an effective intervention in curbing healthcare-acquired infections, including UTIs. This would be an appropriate recommendation.

phenytoin understanding:

therapeutic range is 10-20 mcg/mL and levels above can lead to toxicity

The nurse is caring for a client prescribed dutasteride. The nurse understands this medication had achieved its therapeutic effect when the client reports decreased symptoms of A. pyrosis. B. hypothyroidism. C. urinary retention. D. anxiety.

urinary retention. Rationale: Dutasteride works by inhibiting this enzyme 5-alpha reductase, which normally converts testosterone to 5-alpha dihydrotestosterone (DHT). DHT is a more potent form of testosterone and is the principal androgen responsible for stimulating prostatic growth. The growth of the prostate may cause BPH, therefore, causing overflow incontinence which is manifested as urinary retention. Decreased size of the prostate and less urinary retention is a therapeutic findings of this medication.

The nurse is instructing unlicensed assistive personnel (UAP) on how to modify activities of daily living for a client receiving a continuous infusion of heparin. The nurse should instruct the UAP to Select all that apply. use a soft-bristled toothbrush for oral care. use an electric razor when shaving. use a lift sheet when repositioning the client. use an emery board instead of nail clippers.

use a soft-bristled toothbrush for oral care. use an electric razor when shaving. use a lift sheet when repositioning the client. use an emery board instead of nail clippers. Rationale: This risk of bleeding is substantial for a client receiving a continuous infusion of heparin. The UAP should be instructed to perform oral care with a soft bristle toothbrush to prevent gingival bleeding. An electric razor is preferred over a traditional razor because of the decreased risk of trauma. A lift sheet should be used to reposition the client over sliding the client, reducing the risk of shearing injuries. Nail clippers may cause skin trauma. Thus, an emery board is preferred.

The nurse is caring for a client on heparin who has a new order to start warfarin. The nurse understands that this is because A. additional medication is needed. B. warfarin is more effective than heparin. C. warfarin is not effective until 2-3 days after it has been stated. D. heparin has a low molecular weight and is only effective for a short time.

warfarin is not effective until 2-3 days after it has been stated. Rationale: Unlike heparin, warfarin's anticoagulant activity can take 2-3 days to take effect. The client is commonly prescribed a warfarin-heparin bridge, where the client receives both medications. The heparin is discontinued once the international normalized ratio (INR) becomes therapeutic between 2-3.

The nurse is caring for a client with schizophrenia and has received a new prescription for clozapine. Prior to administering the first dose, the nurse plans on obtaining the client's A. weight. B. pulmonary function tests. C. urine analysis. D. visual acuity.

weight Rationale: Clozapine is an atypical (second-generation) antipsychotic indicated in treating schizophrenia. Clozapine causes the worst metabolic disturbances, including hyperglycemia, weight gain, and hyperlipidemia. These manifestations may drive the client into metabolic syndrome. The nurse must obtain a baseline weight to trend at future visits. Additional baseline data needed prior to starting clozapine include the client's neutrophil count, liver function tests, fasting blood glucose, and hemoglobin.

The LPN is caring for several clients with diabetes. The LPN should recognize that the clients at risk for hypoglycemia include which of the following? A client Select all that apply. who has diabetic ketoacidosis and is receiving continuous regular insulin intravenously. who is receiving methylprednisolone for an exacerbation of asthma. who has pancreatitis and is receiving total parenteral nutrition (TPN). who is nothing by mouth (NPO) status following a coronary artery bypass graft (CABG). who received six units of lispro insulin one hour ago and has not eaten.

who has diabetic ketoacidosis and is receiving continuous regular insulin intravenously. who received six units of lispro insulin one hour ago and has not eaten. Rationale: A client with diabetic ketoacidosis (DKA) receiving regular insulin intravenously is at significant risk for hypoglycemia because regular insulin (when given intravenously) peaks within fifteen to thirty minutes. This is why the client has their glucose taken every hour. Lispro insulin is rapid-acting, and if the client has not eaten within ten to fifteen minutes of getting the insulin, they run the risk of hypoglycemia.

The nurse reviews prescriptions for packed red blood cell (PRBC) transfusions. Which PRBC transfusion should the nurse question with the primary healthcare provider (PHCP)? A client A. with a febrile illness. B. with pulmonary edema. C. receiving mechanical ventilation. D. with a chest tube for a hemothorax.

with pulmonary edema. Rationale: A unit of PRBCs will add fluid volume, and if the client has pulmonary edema, the unit of blood should be questioned with the PHCP until the edema has resolved. Giving a unit of PRBCs may worsen pulmonary edema. Clients at risk for transfusion-associated circulatory overload (TACO) will need to receive their unit of PRBCs slower and may require diuretics after the blood has been administered.


Kaugnay na mga set ng pag-aaral

Fundamentals: ATI Practice Test A

View Set

Chapt 42- Musculoskeletal Trauma

View Set

Google Analytics Assessment Questions

View Set

Psych Stats: Learning Curve Questions

View Set